Está en la página 1de 432

CSUS Debate Aff K Toolkit

TP
2008 - 2009

AFF CRITIQUE TOOLBOX

***CEDE the political***...................................................................................................................10


cede the political.............................................................................................................................................10
cede the political: at the left is dead...............................................................................................................14
at performance: cede the political...................................................................................................................15
pragmatism.....................................................................................................................................................16
***Futurism Good***....................................................................................................................................17
futurism key to movements............................................................................................................................17
futurism key to crisis prevention....................................................................................................................19
futurism key to human survival......................................................................................................................20
at: any flaw with futurism...............................................................................................................................21
at: futurism/crises cause paralysis..................................................................................................................22
at: predictions wrong......................................................................................................................................23
at: media distortions.......................................................................................................................................24
at: futurism = no value to life.........................................................................................................................25
at: fear mongering by the state.......................................................................................................................26
at: futurism is statist........................................................................................................................................27
at: chaos inevitable.........................................................................................................................................28
***State***....................................................................................................................................................29
A2: State Bad..................................................................................................................................................29
A2: State Bad (Gender)..................................................................................................................................32
A2: State Bad (Environment).........................................................................................................................33
state good........................................................................................................................................................35
state good: checks capitalism.........................................................................................................................36
Realism Good.................................................................................................................................................37
Realism Good.................................................................................................................................................38
Reformism Good – Rorty...............................................................................................................................39
Reformism Good – Rorty – Impact................................................................................................................45
Political Vacuum Turn...................................................................................................................................46
Political Vacuum Turn...................................................................................................................................47
Micro-Politics Fail..........................................................................................................................................48
Micro-Politics Permutation............................................................................................................................49
Identity Politics Fail.......................................................................................................................................50
Realism Good.................................................................................................................................................51
Realism Good.................................................................................................................................................52
Realism Good.................................................................................................................................................53
*** Postmodernism ***.................................................................................................................................54
PoMo Bad General.........................................................................................................................................54
PoMo Bad General.........................................................................................................................................55
PoMo Bad General.........................................................................................................................................56
PoMo Bad General.........................................................................................................................................57
PoMo Bad General.........................................................................................................................................58
PoMo Bad – Public Sphere.............................................................................................................................59
PoMo Bad – Public Sphere.............................................................................................................................60
Ext – PoMo Jacks Public Engagement...........................................................................................................61
PoMo Bad – Best + Kellner............................................................................................................................62
PoMo Bad – Best + Kellner............................................................................................................................63

1/432
CSUS Debate Aff K Toolkit
TP
2008 - 2009

A2: Deconstruction.........................................................................................................................................64
at: deleuze and guattari...................................................................................................................................65
A2: Baudrillard...............................................................................................................................................66
A2: Baudrillard...............................................................................................................................................67
A2: Baudrillard...............................................................................................................................................68
at baudrillard: cede the political....................................................................................................................69
at baudrillard: Simulation..............................................................................................................................70
at baudrillard: Simulation..............................................................................................................................71
A2: Foucault...................................................................................................................................................72
A2: Foucault...................................................................................................................................................73
A2: Foucault...................................................................................................................................................74
at foucault: no impact.....................................................................................................................................75
at foucault: no impact.....................................................................................................................................76
at foucault: no impact.....................................................................................................................................77
at foucault: no impact (massacres).................................................................................................................78
at foucault: nazis unique.................................................................................................................................79
at foucault: aff good use of biopower.............................................................................................................80
at foucault: biopower good.............................................................................................................................81
at foucault: resistance solves impact..............................................................................................................82
at foucault: resistance solves impact..............................................................................................................83
at foucault: aff pre-req to alt...........................................................................................................................84
at foucault: cede the political..........................................................................................................................85
at foucault: geneaology...................................................................................................................................86
A2: Agamben..................................................................................................................................................87
at agamben: alternative fails.........................................................................................................................103
at agamben: link over simplified..................................................................................................................104
at agamben: nazis unique..............................................................................................................................105
at agamben: bare life....................................................................................................................................106
at agamben: the camp...................................................................................................................................107
at agamben: muselmann...............................................................................................................................108
***A2: Colonialism***................................................................................................................................109
at colonialism: US not an empire.................................................................................................................109
at colonialism: us not an empire...................................................................................................................110
at colonialism: US not an empire.................................................................................................................111
at colonialism: empire good.........................................................................................................................112
at empire: terrorism must be confronted......................................................................................................113
*** Ontology ***.........................................................................................................................................114
A2: Ontology................................................................................................................................................114
A2: Ontology................................................................................................................................................115
A2: Ontology................................................................................................................................................116
A2: Ontology................................................................................................................................................117
A2: Ontology................................................................................................................................................118
A2: Ontology................................................................................................................................................119
A2: Ontology................................................................................................................................................120
A2: Spanos....................................................................................................................................................121
A2: Spanos....................................................................................................................................................122
at spanos: cede the political..........................................................................................................................124
at spanos: no alternative...............................................................................................................................125
at spanos: no truth disempowering...............................................................................................................126
at spanos: humanism good............................................................................................................................127
at spanos: vietnam good...............................................................................................................................128
at spanos: vietnam good...............................................................................................................................129

2/432
CSUS Debate Aff K Toolkit
TP
2008 - 2009

A2: Heidegger..............................................................................................................................................130
A2: Heidegger..............................................................................................................................................131
A2: Heidegger..............................................................................................................................................132
A2: Heidegger..............................................................................................................................................133
A2: Hedegger................................................................................................................................................134
at heidegger: nazi..........................................................................................................................................135
at heidegger: nazi..........................................................................................................................................136
at heidegger: nazi..........................................................................................................................................137
at heidegger: humanism key to stop nazism.................................................................................................138
at heidegger: ethics too vague......................................................................................................................139
at heidegger: unconcealment bad.................................................................................................................140
at heidegger: paralysis..................................................................................................................................141
at heidegger: authoritarian............................................................................................................................142
at heidegger: no value to life........................................................................................................................143
at heidegger: no truth = nazism....................................................................................................................144
at heidegger: paralysis..................................................................................................................................145
at heidegger: calculations good....................................................................................................................146
at heidegger: dread of death bad...................................................................................................................147
at heidegger: permutation.............................................................................................................................148
at heidegger: being meaningless...................................................................................................................149
A2: heidegger: link over simplified..............................................................................................................150
A2: Dillon: calculations good......................................................................................................................151
Humanism Good...........................................................................................................................................152
Humanism Good...........................................................................................................................................153
Consequentialism good................................................................................................................................155
Consequentialism good................................................................................................................................156
Calculability good........................................................................................................................................157
Calculability good........................................................................................................................................158
Util Good......................................................................................................................................................159
Util Good......................................................................................................................................................160
***A2: rights K***......................................................................................................................................161
rights good....................................................................................................................................................161
rights good....................................................................................................................................................162
rights good....................................................................................................................................................163
rights good....................................................................................................................................................164
rights good: not monolithic..........................................................................................................................165
rights good: not western...............................................................................................................................166
rights good: check on statism.......................................................................................................................167
***Capitalism***.........................................................................................................................................168
A2: Capitalism..............................................................................................................................................168
A2: Capitalism..............................................................................................................................................169
A2: Capitalism..............................................................................................................................................170
A2: Zizek......................................................................................................................................................171
at lacan/zizek: “the act” fails........................................................................................................................191
at lacan/zizek: revolution fails......................................................................................................................199
at lacan/zizek: revolution fails......................................................................................................................200
at lacan/zizek: does not apply to aff.............................................................................................................201
at lacan/zizek: non-falsifiable.......................................................................................................................202
at lacan/zizek: non-falsifiable.......................................................................................................................203
at lacan/zizek: conservative politics.............................................................................................................204
at lacan/zizek: conservative politics.............................................................................................................205
at lacan/zizek: conservative politics.............................................................................................................206

3/432
CSUS Debate Aff K Toolkit
TP
2008 - 2009

at lacan/zizek: essentialism turn...................................................................................................................207


at lacan/zizek: essentialism turn...................................................................................................................208
at lacan/zizek: 9/11 arguments bad...............................................................................................................209
at lacan/zizek: no connection alt and ballot..................................................................................................210
A2: Empire (Hardt+Negri)...........................................................................................................................211
A2: Empire (Hardt+Negri)...........................................................................................................................212
A2: Empire (Hardt+Negri)...........................................................................................................................213
A2: Empire (Hardt+Negri)...........................................................................................................................214
at hardt/negri: alternative  terrorism.........................................................................................................215
at hardt/negri: alternative  terrorism.........................................................................................................216
at hardt/negri: alternative justifies holocaust................................................................................................217
at hardt/negri: globalization good.................................................................................................................218
at hardt/negri: capitalism good.....................................................................................................................219
at hardt/negri: at historical argument............................................................................................................220
at hardt/negri: no qualified data....................................................................................................................221
at hardt/negri: multitude fails.......................................................................................................................222
at hardt/negri: nation-state strong.................................................................................................................223
at hardt/negri: nation-state strong.................................................................................................................224
at hardt/negri: at biopower Impact...............................................................................................................225
at hardt/negri: alt fails...................................................................................................................................226
at hardt/negri: no multitude..........................................................................................................................227
at hardt/negri: no empire..............................................................................................................................228
at hardt/negri: hurts movements...................................................................................................................229
***A2: Language***...................................................................................................................................230
at langauge ks: cede the politcal...................................................................................................................230
at language k: generic...................................................................................................................................231
at language k: generic...................................................................................................................................232
at language ks: censorship hurst the left.......................................................................................................233
*** Representations ***...............................................................................................................................234
A2: Representations......................................................................................................................................234
A2: Representations......................................................................................................................................235
A2: Fear of Death.........................................................................................................................................236
A2: Fear of Death.........................................................................................................................................237
A2: Fear of Death.........................................................................................................................................238
at fear of death: mobilizes people/compassion.............................................................................................239
at fear of death: Fear Key to Value to Life...................................................................................................240
at fear of death: key to survival....................................................................................................................241
at fear of death: deterence good....................................................................................................................242
at fear of nukes: fear Key to Peace and Survival..........................................................................................243
at fear of nukes: Peace and Survival.............................................................................................................244
at nucelar numbing: plan solves impact.......................................................................................................245
at non-violence: could not solve the holocaust.............................................................................................246
at non-violence: Violence Key to Peace.......................................................................................................247
at cuomo: Negative Peace Key to Positive Peace........................................................................................248
threat construction: peace.............................................................................................................................249
threat construction: prevents escalation........................................................................................................250
threat construction: threats real....................................................................................................................251
threat construction: reps irrelevant...............................................................................................................252
A2: Love Alternative....................................................................................................................................253
A2: Localism (Nayar)...................................................................................................................................254
A2: Localism (Nayar)...................................................................................................................................255
at global/local: perm.....................................................................................................................................256

4/432
CSUS Debate Aff K Toolkit
TP
2008 - 2009

at global/local: global resistance key............................................................................................................257


A2: Disease K...............................................................................................................................................258
A2: Orientalism (Said).................................................................................................................................259
***Gender***..............................................................................................................................................260
at gender ir: no alt.........................................................................................................................................260
at gender ir: no link.......................................................................................................................................261
at gender ir: perm..........................................................................................................................................262
at gender ir: 3rd world feminism..................................................................................................................263
*** Ethics ***..............................................................................................................................................264
A2: Obligation To Other (Levinas)..............................................................................................................264
A2: Obligation To Other (Levinas)..............................................................................................................265
A2: Obligation To Other (Levinas)..............................................................................................................266
A2: Badiou....................................................................................................................................................267
A2: Badiou....................................................................................................................................................268
A2: Badiou....................................................................................................................................................269
A2: Badiou....................................................................................................................................................270
at badiou: ethics............................................................................................................................................271
at badiou: politics fail...................................................................................................................................272
at badiou: politics fail...................................................................................................................................273
at badiou: politics fail...................................................................................................................................274
at badiou: permutation..................................................................................................................................275
at badiou: alternative unworkable and communist.......................................................................................276
A2: Nietzsche...............................................................................................................................................277
A2: Nietzsche...............................................................................................................................................278
A2: Nietzsche...............................................................................................................................................280
A2: Nietzsche...............................................................................................................................................281
A2: Nietzsche...............................................................................................................................................282
A2: Nietzsche...............................................................................................................................................283
A2: Nietzsche...............................................................................................................................................285
A2: Nietzsche...............................................................................................................................................286
A2: Nietzsche...............................................................................................................................................287
A2: Nietzsche...............................................................................................................................................288
A2: Nietzsche...............................................................................................................................................289
A2: Nietzsche...............................................................................................................................................290
A2: Nietzsche...............................................................................................................................................291
*** Security ***...........................................................................................................................................292
A2: Security K..............................................................................................................................................292
A2: Security K..............................................................................................................................................293
A2: Security K..............................................................................................................................................294
A2: Security K..............................................................................................................................................295
***A2: Borders***......................................................................................................................................296
at borders: inevitable....................................................................................................................................296
at borders: solves war...................................................................................................................................297
at borders: africa...........................................................................................................................................298
***RANDOM CARDS***..........................................................................................................................299
at critiques of science...................................................................................................................................299
at said: no alternative....................................................................................................................................300
at butler: perm...............................................................................................................................................301
identity politics: cede the political................................................................................................................302
specific solvency outweighs the k link.........................................................................................................303
***08/09 TOPIC SPECIFIC***..................................................................................................................304
***Land K***..............................................................................................................................................305

5/432
CSUS Debate Aff K Toolkit
TP
2008 - 2009

SCIENCE GOOD.........................................................................................................................................306
PERM...........................................................................................................................................................307
PERM...........................................................................................................................................................308
PERM...........................................................................................................................................................309
***Famine K***..........................................................................................................................................310
PERM...........................................................................................................................................................311
Edkins Indict.................................................................................................................................................312
Repoliticization bad......................................................................................................................................313
Tech (s) Imperialism.....................................................................................................................................314
***Deep Eco K***.......................................................................................................................................315
AFF ANSWERS...........................................................................................................................................316
PERM – COTTON AFF...............................................................................................................................317
AFF ANSWERS...........................................................................................................................................318
AFF ANSWERS...........................................................................................................................................319
AFF ANSWERS...........................................................................................................................................320
AFF ANSWERS...........................................................................................................................................321
AFF A2: ECONOMY LINK........................................................................................................................322
AFF ANSWERS...........................................................................................................................................323
AFF ANSWERS...........................................................................................................................................324
*** Heidegger K***.....................................................................................................................................325
AFF ANSWERS: Perm................................................................................................................................326
Perm EXT.....................................................................................................................................................328
Aff – Management Good..............................................................................................................................329
Aff – Technology Good................................................................................................................................330
Aff – Technology Good................................................................................................................................331
Aff – Technology Good/AT: Tech Links.....................................................................................................332
Aff – Resource Mindset Good/AT: Resources Links...................................................................................333
Aff – Permutation Solves – Alternative Energy Key...................................................................................334
Aff – Permutation Solves – General.............................................................................................................335
Aff – Permutation Solves – General.............................................................................................................336
Aff – Permutation Solves – Technology......................................................................................................337
Aff – Permutation Solves – Technology......................................................................................................338
Aff – Permutation Solves/Ontology Not First..............................................................................................339
Aff – Technology/Calculation Permutation Solves......................................................................................340
Aff – Environmental Pragmatism Good.......................................................................................................341
Aff – Rejecting Management Bad/Impossible.............................................................................................342
Aff – Rejecting Technology Bad..................................................................................................................343
Aff – Alternative Doesn’t Solve...................................................................................................................345
Aff – Alternative Doesn’t Solve...................................................................................................................346
Aff – Alternative Doesn’t Solve...................................................................................................................347
Aff – Alternative Doesn’t Solve...................................................................................................................348
Aff – Ontology Doesn’t Come First – Pragmatism......................................................................................349
Aff – Ontology Doesn’t Come First – Violence..........................................................................................350
Aff – Ethics Before Ontology.......................................................................................................................351
Aff – Kritik Ethics Bad.................................................................................................................................352
Aff – Kritik Ethics Bad.................................................................................................................................353
Aff – Kritik Ethics Bad.................................................................................................................................354
Aff – Kritik Efhics Bad................................................................................................................................355
Aff – Impact Inevitable................................................................................................................................356
Aff – Impact Inevitable................................................................................................................................357
***Eco Fem K***........................................................................................................................................358
Connection of Nature and women is Bad.....................................................................................................359

6/432
CSUS Debate Aff K Toolkit
TP
2008 - 2009

Connection of Nature and women is Bad.....................................................................................................360


Connection of nature and women is Bad......................................................................................................361
Karen Warren Flawed...................................................................................................................................362
Perm Political action.....................................................................................................................................363
Ecofeminism excludes Women of Color......................................................................................................364
Ecofeminism excludes Women of Color......................................................................................................365
Ecofeminism excludes Women of Color......................................................................................................366
*** Bataille K***.........................................................................................................................................367
Bataille is too utopian...................................................................................................................................368
Bataille’s approach has no social importance..............................................................................................369
Bataille’s Methodology of Sacrifice is Flawed............................................................................................370
Discourse is Inaccessible and unpragmatic..................................................................................................371
Racial Realism Supersedes the K.................................................................................................................373
Black Feminism Supersedes Postmodern Kritik..........................................................................................374
Transgression is Socially Irresponsible; Compassion is Better Alt..............................................................375
Compassion Alternative -- AT: Sentimental and sickly..............................................................................376
Compassion Alternative – Transgression is Insufficient Experience...........................................................377
Transgression cannot recognize difference in mind-body experiences........................................................378
Gender K of Transgression vs. Compassion Alternatives............................................................................379
AT: Do Both Transgression and Compassion.............................................................................................380
** Cap K***.................................................................................................................................................381
Link Turn: Subsidies continue inequality.....................................................................................................382
AT: Link – Subsidies facilitate Neoliberalism.............................................................................................383
Alternative fails............................................................................................................................................384
Alternative Fails- Holloway.........................................................................................................................385
Alternative Fails- Total Rejection................................................................................................................386
Aff – Reform Possible/Key..........................................................................................................................387
A2: Captialism Imperialism/War..............................................................................................................388
A2: Capitalism Inequality/Economic Stratification.................................................................................389
A2: Capitalism Corporate Abuse/Poverty................................................................................................390
A2: Capitalism Starvation/Poverty...........................................................................................................391
A2: Capitalism Oppresses Working Class/Child Labor...............................................................................392
A2: Capitalism caused the great depression/trade wars...............................................................................393
at capitalism K: cede the political................................................................................................................394
at capitalism K: cede the political................................................................................................................395
at capitalism K: peace...................................................................................................................................396
at capitalism K: can be liberatory.................................................................................................................397
at capitalism K: no specifc alt = failure........................................................................................................398
Depictions of Capitalism as Bad..................................................................................................................399
Perm Solves/Reform Key (1/2)....................................................................................................................400
Perm Solves/Reform Key (2/2)....................................................................................................................401
Perm Solves- A2: State Bad.........................................................................................................................402
Reform Good (1/3).......................................................................................................................................403
Reform Good (2/3).......................................................................................................................................404
Reform Good (3/3).......................................................................................................................................405
Working W/n the System Key (1/2).............................................................................................................406
Working W/n the System Key (2/2).............................................................................................................407
Total Rejection of Capitalism Bad (1/2)......................................................................................................408
Total Rejection of Capitalism Bad (2/2)......................................................................................................409
Capitalism Inevitable....................................................................................................................................411
Capitalism Inevitable....................................................................................................................................412
Capitalism Inevitable- Markets....................................................................................................................413

7/432
CSUS Debate Aff K Toolkit
TP
2008 - 2009

Collapse Inevitable- Credit Crisis................................................................................................................414


Capitalism Inevitable/Good..........................................................................................................................415
Capitalism Not Root Cause..........................................................................................................................416
Capitalism Not Root Cause- No Free Markets.............................................................................................417
Capitalism Not Root Cause/Reformable......................................................................................................418
Capitalism is Reformable.............................................................................................................................419
Capitalism is Reformable.............................................................................................................................420
Capitalism is Human Nature........................................................................................................................421
Cap Good – Financial Independence............................................................................................................422
***Environmental Security K***................................................................................................................423
Affirmative Answers....................................................................................................................................424
Affirmative Answers....................................................................................................................................425
Affirmative Answers....................................................................................................................................426
Affirmative Answers....................................................................................................................................427
Affirmative Answers....................................................................................................................................428
Affirmative Answers....................................................................................................................................429
Affirmative Answers....................................................................................................................................430
Affirmative Answers....................................................................................................................................431
Affirmative Answers....................................................................................................................................432
Affirmative Answers....................................................................................................................................433

8/432
***CEDE the political***

cede the political

This failure to engage the political process turns the affirmative into spectators who are powerless to produce real
change.

Rorty 98 – professor emeritus of comparative literature and philosophy, by courtesy, at Stanford University (Richard,
“ACHIEVING OUR COUNTRY: Leftist Thought in Twentieth-Century America”, 1998, Pg. 7-9)

Such people find pride in American citizenship impossible, and vigorous participation in electoral politics pointless.
They associate American patriotism with an endorsement of atrocities: the importation of African slaves, the slaughter
of Native Americans, the rape of ancient forests, and the Viet nam War. Many of them think of national pride as
appropriate only for chauvinists: for the sort of American who rejoices that America can still orchestrate something like
the Gulf War, can still bring deadly force to bear whenever and wherever it chooses. When young intellectuals watch
John Wayne war movies after reading Heidegger, Foucault, Stephenson, or Silko, they often become convinced that
they live in a violent, inhuman, corrupt country. They begin to think of themselves as a saving remnant-as the happy
few who have the insight to see through nationalist rhetoric to the ghastly reality of contemporary America. But this
insight does not move them to formulate a legislative program, to join a political movement, or to share in a national
hope. The contrast between national hope and national self-mockery and self-disgust becomes vivid when one
compares novels like Snow Crash and Almanac of the Dead with socialist novels of the first half of the century-books
like The Jungle, An American Tragedy, and The Grapes of Wrath. The latter were written in the belief that the tone of
the Gettysburg Address was absolutely right, but that our country would have to transform itself in order to fulfill
Lincoln's hopes. Transformation would be needed because the rise of industrial capitalism had made the individualist
rhetoric of America's first century obsolete. The authors of these novels thought that this rhetoric should be replaced by
one in which America is destined to become the first cooperative commonwealth, the first classless society. This
America would be one in which income and wealth are equitably distributed, and in which the government ensures
equality of opportunity as well as individual liberty. This new, quasi-communitarian rhetoric was at the heart of the
Progressive Movement and the New Deal. It set the tone for the American Left during the first six decades of the
twentieth century. Walt Whitman and John Dewey, as we shall see, did a great deal to shape this rhetoric. The
difference between early twentieth-century leftist intellectuals and the majority of their contemporary counterparts is the
difference between agents and spectators. In the early decades of this century, when an intellectual stepped back from
his or her country's history and looked at it through skeptical eyes, the chances were that he or she was about to
propose a new political initiative . Henry Adams was, of course, the great exception-the great abstainer from ·politics.
But William James thought that Adams' diagnosis of the First Gilded Age as a symptom of irreversible moral and
political decline was merely perverse. James's pragmatist theory of truth was in part a reaction against the sort of de-
tached spectators hip which Adams affected. For James, disgust with American hypocrisy and self-deception was
pointless unless accompanied by an effort to give America reason to be proud of itself in the future. The kind of proto-
Heideggerian cultural pessimism which Adams cultivated seemed, to James, decadent and cowardly. "Democracy,"
James wrote, "is a kind of religion, and we are bound not to admit its failure. Faiths and utopias are the no blest
exercise of human reason, and no one with a spark of reason in him will sit down fatalistically before the croaker's
picture. "2
cede the political

The affirmative’s strategy is not political. Instead it is a strategy against politics. This undermines the possibility of
liberation.

Grossberg, 92 (Lawrence, Morris Davis Professor of Communication Studies at the University of North Carolina at
Chapel Hill, “We Gotta Get Out of this Place: Popular Conservatism and Postmodern Culture”, page 278-279)

Finally, the frontier itself is transformed. It is still partly defined by an attitude in which we are all implicated. In this
sense, the frontier is in everyone—and with it, the possibility of evil. But now its popular/resonance is rearticulated to
"activities" that have to be affectively and morally judged and policed. The enemy is not within people but in specific
activities that construct the frontier over in the image of the new conservatism. The frontier becomes a seductive
machine, seducing people not only into the need to invest, but ultimately into a series of temporary and mobile
investments which locate them within a popular conservatism. The frontier's articula tion by the logic of scandal marks a
real break with older conservatisms built on some notion of tradition. Here politics is not a solution to problems, but a
machine which organizes the population and its practices. What is on the "right" (in both senses) side of the frontier, on
the other side of politics, is a purely affective morality (ie., one which leaves no space within which specific actions can
be judged as anything other than scandalous). The new conservatism embodies, not a political rebellion but a rebellion
against politics. It makes politics into an other, located on the other side of the frontier. Anyone who actually talks about
serious problems and their solutions is a dreamer; anyone who celebrates the mood in which the problem is at once
terrifying and boring is a realist. It is no longer believing too strongly that is dangerous, but actually thinking that one is
supposed to make one's dreams come true. The failure of Earth Day cannot be explained by merely pointing to its
status as a feel-good media event, nor by pointing out the increasingly hypocritical appropriation of "green politics" by
corporate polluters. It is rather that ecology, like any "politics," has become a question of attitude and investment, as if
investing in the "correct" ideological beliefs, even demonstrating it, was an adequate construction of the political. Within
the new conservative articulation of the frontier, political positions only exist as entirely affective investments,
separated from any ability to act.
cede the political

Failure to engage in the political process will result in the takeover by the extreme right, leading to discrimination and
war worldwide

Rorty 98 – professor emeritus of comparative literature and philosophy, by courtesy, at Stanford University (Richard,
“ACHIEVING OUR COUNTRY: Leftist Thought in Twentieth-Century America”, 1998, pg. 89-94)
*WE DO NOT ENDORSE GENDERED LANGUAGE*

Many writers on socioeconomic policy have warned that the old industrialized democracies are heading into a Weimar-like period, one in which populist movements are
likely to overturn constitutional governments. Edward Luttwak, for example, has suggested that fascism may be the American future. The point of his book The
Endangered American Dream is that members of labor unions, and unorganized unskilled workers, will sooner or later realize that
their government is not even trying to prevent wages from sinking or to prevent jobs from being exported. Around the
same time, they will realize that suburban white-collar workers-them- selves desperately afraid of being downsized-are
not going to let themselves be taxed to provide social benefits for any one else. At that point, something will crack. The
nonsuburban electorate will decide that the system has failed and start looking around for a strongman to vote for-someone
willing to assure them that, once he is elected, the smug bureaucrats, tricky lawyers, overpaid bond salesmen, and postmodernist
professors will no longer be calling the shots. A scenario like that of Sinclair Lewis’ novel It Can’t Happen Here may then be played out. For once
such a strongman takes office, nobody can predict what will happen. In 1932, most of the predictions made about what
would happen if Hindenburg named Hitler chancellor were wildly overoptimistic . One thing that is very likely to happen is that the
gains made in the past forty years by black and brown Americans, and by homosexuals, will be wiped out. Jocular
contempt for women will come back into fashion. The words "nigger" and "kike" will once again be heard in the
workplace. All the sadism which the academic Left has tried to make unaccept able to its students will come flooding back. All
the resentment which badly educated Americans feel about having their manners dictated to them by college graduates will find an outlet. But such a renewal of sadism
will not alter the effects of selfishness. For
after my imagined strongman takes charge, he will quickly make his peace with the
international superrich, just as Hitler made his with the German industrialists. He will invoke the glorious memory of the
Gulf War to provoke military adventures which will generate short-term prosperity. He will be a disaster for the country
and the world. People will wonder why there was so little resistance to his evitable rise. Where, they will ask, was the American Left? Why was it only
rightists like Buchanan who spoke to the workers about the consequences of globalization? Why could not the Left channel the mounting rage of
the newly dispossessed? It is often said that we Americans, at the end of the twentieth century, no longer have a Left. Since nobody denies the
existence of what I have called the cultural Left, this amounts to an admission that that Left is unable to engage in national politics. It is not the sort of Left which
can be asked to deal with the consequences of globalization. To get the country to deal with those consequences, the present cultural Left would have to
transform itself by opening relations with the residue of the old reformist Left, and in particular with the labor unions. It would have to talk much more about
money, even at the cost of talking less about stigma. I have two suggestions about how to effect this transition. The first is that the Left
should put a moratorium on theory. It should try to kick its philosophy habit . The second is that the Left should try to mobilize what remains of our pride in
being Americans. It should ask the public to consider how the country of Lincoln and Whitman might be achieved. In support of my first suggestion, let me cite a passage
from Dewey's Reconstruction in Philosophy in which he expresses his exasperation with the sort of sterile debate now going on under the rubric of "individualism versus
communitarianism." Dewey thought that all discussions which took this dichotomy seriously suffer from a common defect. They are all committed to the logic of general
notions under which specific situations are to be brought. What we want is light upon this or that group of individuals, this or that concrete human being, this or that special
institution or social arrangement. For such a logic of inquiry, the traditionally accepted logic substitutes discussion of the meaning of concepts and their dialectical
relationships with one another. Dewey was right to be exasperated by sociopolitical theory conducted at this level of abstraction. He was wrong when he went on
to say that ascending to this level is typically a rightist maneuver, one which supplies "the apparatus for intellectual justifications of the established order. "9 For
such ascents are now more common on the Left than on the Right. The contemporary academic Left seems to think that the higher your level of abstraction, the
more subversive of the established order you can be. The more sweeping and novel your conceptual apparatus, the more radical your criti que. When one of
today's academic leftists says that some topic has been "inadequately theorized," you can be pretty certain that he or she is going to drag in either philosophy of lan -
guage, or Lacanian psychoanalysis, or some neo-Marxist version of economic determinism. Theorists of the Left think that dissolving political agents into plays of
differential subjectivity, or political initiatives into pursuits of Lacan's impossible object of desire, helps to subvert the established order. Such subversion, they say, is
accomplished by "problematizing familiar concepts." Recent attempts to subvert social institutions by problematizing concepts have produced a few very good books.
They have also produced many thousands of books which represent scholastic philosophizing at its worst. The authors of these purportedly "subversive" books honestly
believe that they are serving human liberty. But it is almost impossible to clamber back down from their books to a level of abstraction on which one might discuss the
merits of a law, a treaty, a candidate, or a political strategy. Even though what these authors "theorize" is often something very concrete and near at hand-a current TV
show, a media celebrity, a recent scandal-they offer the most abstract and barren explanations imaginable. These
futile attempts to philosophize one's
way into political relevance are a symptom of what happens when a Left retreats from activism and adopts a
spectatorial approach to the problems of its country. Disengagement from practice pro duces theoretical hallucinations.
These result in an intellectual environment which is, as Mark Edmundson says in his book Nightmare on Main Street, Gothic. The cultural Left is haunted by
ubiquitous specters, the most frightening of which is called "power." This is the name of what Edmundson calls Foucault's "haunting agency, which is everywhere
and nowhere, as evanescent and insistent as a resourceful spook."10

cede the political


Institutional approaches are the only way to avoid the collapse of all movements and effectively challenge the flawed
state policies.

Grossberg, 92 (Lawrence, Morris Davis Professor of Communication Studies at the University of North Carolina at
Chapel Hill, “We Gotta Get Out of this Place: Popular Conservatism and Postmodern Culture”, page 388-389)

The demand for moral and ideological purity often results in the rejection of any hierarchy or organization. The
question-can the master's tools be used to tear down the master's house?-ignores both the contingency of the relation
between such tools and the master's power and, even more importantly, the fact that there may be no other tools
available. Institutionalization is seen as a repressive impurity within the body politic rather than as a strategic and
tactical, even empowering, necessity . It sometimes seems as if every progressive organization is condemned to
recapitulate the same arguments and crisis, often leading to their collapse. 54 For example, Minkowitz has described a
crisis in Act Up over the need for efficiency and organization, professionalization and even hierarchy,55 as if these
inherently contradicted its commitment to democracy. This is particularly unfortunate since Act Up, whatever its
limitations, has proven itself an effective and imaginative political strategist. The problems are obviously magnified with
success, as membership, finances and activities grow. This refusal of efficient operation and the moment of
organization is intimately connected with the Left's appropriation and privileging of the local (as the site of democracy
and resistance). This is yet another reason why structures of alliance are inadequate, since they often assume that an
effective movement can be organized and sustained without such structuring. The Left needs to recognize the
necessity of institutionalization and of systems of hierarchy, without falling back into its own authoritarianism. It needs
to find reasonably democratic structures of institutionalization, even if they are impure and compromised.

The desire for pure politics undermines a litany of meaningful possibilities at overcoming domination.
Grossberg, 92 (Lawrence, Morris Davis Professor of Communication Studies at the University of North Carolina at
Chapel Hill, “We Gotta Get Out of this Place: Popular Conservatism and Postmodern Culture”, page 396)

Above all, rethinking the possibility of a Left politics will require a new model of
intellectual and political authority which does not begin by confidently judging every
investment, every practice, every articulation and every individual. It will have to
measure both intellectual and political progress by movement within the fragile and
contradictory realities of people's lives, desires, fears and commit ments, and not by
some idealized utopia nor by its own theoretical criteria. It will offer a moral and
progressive politics which refuses to "police" everyday life and to define a structure
of "proper" and appropriate behaviors and attitudes. An impure politics—certainly,
without the myth of a perfect reflexivity which can guarantee its authority (for
authority is not an intellectual prize). A contaminated politics, never innocent, rooted
in the organization of distance and densities through which all of us move together and
apart, sometimes hesitatingly, at other times recklessly. A politics that attempts to
move people, perhaps just a little at first, in a different direction. But a politics
nonetheless, one which speaks with a certain authority, as limited and frail as the lives
of those who speak it. It will have to be a politics articulated by and for people who
are inevitably implicated in the contemporary crisis of authority and whose lives have
been shaped by it. A politics for and by people who live in the contemporary world of popular tastes,
and who are caught in the disciplined mobilization of everyday life. A politics for people who are
never innocent and whose hopes are always partly defined by the very powers and
inequalities they oppose. A modest politics that struggles to effect real change,
that enters into the often boring challenges of strategy and compromise. An impure
politics fighting for high stakes.

cede the political: at the left is dead


Progressive momentum can be built upon with big tent, practical politics—operating at only the level of ideology dooms
the movement.
Wilson, 2000 – Editor and Publisher of Illinois Academe – 2000 (John K. Wilson, “How the Left can Win Arguments and
Influence People” p. 5- 6)

The trend toward progressive attitudes among Americans has only accelerated. Today, Americans advocate gender equality on
a level unthinkable at the time I was born, an era when airline stewardess were fired when they turned thirty, got married, or gained fifteen pounds.
Today, racial equality is an ideal widely accepted, even if the reality falls short. Today, equality for gays and lesbians is a
politically viable possibility, a remarkable leap for an issue that was virtually invisible at the time of the Stonewall riot. Today, environmental
awareness and the enormous number of people who recycle would have been unimaginable to the small group of
activists who gathered to celebrate the first Earth Day. Even though the American people have been moving to the left
on a number of important issues, the two major political parties have shifted to the right. The left's revival requires both
the recognition of the disadvantages it faces and a willingness to fight against those barriers while making use of the
advantages that progressives have over the right. The biggest advantage that the left holds is that it doesn't have to be afraid of
speaking the truth to the public. Conservatives, despite their assertions of public support, must always be wary of dealing too openly with Americans.
That is, every idea on the right must be carefully vetted to ensure the proper spin control. Even "radical" ideas such as Steve Forbes's flat tax must
conceal the extent of tax cuts for the rich under the disguise of a universal tax reduction. This book argues that progressives need to reshape
their arguments and their policy proposals to increase their influence over American politics. It also contends that the left
need not sell its soul or jettison its diverse constituents in order to succeed. Rather than moderation, I urge a new kind of tactical radicalism. Rather
than a monolithic left focused on class or labor or postmodernism or whatever the pet ideological project of the day is, I
advocate a big-tent left capable of mobilizing all its people. Progressives already have the hearts and minds of the
American people. What the left lacks is a political movement to translate that popularity into political action. What the
left needs is a rhetorical framework and political plan of action to turn the progressive potential in America into a
political force.

Progressive ideals are alive and well in the US.


Wilson, 2000 – Editor and Publisher of Illinois Academe – 2000 (John K. Wilson, “How the Left can Win Arguments and
Influence People” p. 2- 4)

The thesis of this book is that a majority of Americans now believe (or could easily be persuaded to believe) in many
progressive ideas, even though the power of the progressive movement itself in mainstream politics has largely
disintegrated. In reality, progressives are nearly everywhere, with the possible exception of corporate boardrooms, the
White House, and Bob Jones University. Progressives look like everyone else, although they appear to be a little more
forlorn than most. Unfortunately, the progressive views of the American majority do not translate into political power.
Progressives cannot sit back and await the rising masses to thrust the left into power. Rather, progressives need to give their potential
supporters a reason to be politically active and intellectually interested in the ideas of the left. If you relied on just the mass media in America or on election results, you would have to conclude that
this is a conservative nation. We hear about polls declaring that the American people demand lower taxes, smaller government, the elimination of welfare, the mass execution of criminals, and daily
pledges of allegiance to the free market. We see Republicans in charge of Congress, successfully pursuing their goals of putting a prison on every corner and lowering taxes on the wealthy in order to
allow economic prosperity to trickle down to everyone else. America must be conservative. It seems logical, doesn't it? If the Republicans hold political power and the "liberal" Democrats are following
their lead, this must mean that the majority of Americans share the values of the right. If the "liberal" media agree with this assessment, then it's surely an established fact: progressivism as a mass
. Progressivism as an ideology is a powerful force in the American psyche. From environmentalism to
movement is dead in America
feminism to racial equality, Americans believe deeply in progressive ideas. All these ideologies were minority
movements just a generation ago; now, however, open opposition to them is considered political suicide in most of the
country. Why, then, does a progressive political movement seem so unthinkable? In a political system controlled by the principle of "one dollar, one vote," these progressive views lose out to the
more economically powerful ideas held by the conservative status quo. These progressive ideas end up being ignored by mega-media corporations controlled by the same wealthy forces. This book
is not an attempt to establish a philosophy of the left. Like any political movement, the left has many different philosophies driving its members. Leftists are concerned about civil rights, gay rights,
women's rights, poverty, homelessness, education, imprisonment, empowerment, and much more. Leftists believe in liberalism, Marxism, libertarianism, Christianity, and a wide range of other
ideologies. Trying to find a common intellectual ground for everything is impossible, since not every leftist can possibly share the same belief in every issue and in what the top priorities should be.
Even trying to define what a leftist is seems to be a difficult task, especially since most of the people who believe in leftist ideas may be unwilling to accept the label. This book is, instead, a guide for
political rhetoric and strategic action, a sometimes helpful, sometimes annoying attempt to help the left overcome its own flaws and seek out ways to reach and convince a larger audience about
progressive ideas. This is a self-help book for leftists looking for ways to convince the world that what they believe is correct. This book is also a road map showing how the left can turn the public
: over the past several decades, American political attitudes have
debate to issues they can win. This book originates from a puzzling paradox
become dramatically more progressive. Movements for civil rights, women's equality, and environmental protection,
once promoted by a radical fringe, are now fully embraced by the mainstream.

at performance: cede the political

Faith in performance is naïve and fails to reshape politics


Rothberg & Valente 97 Molly Anne Rothenberg, Assoc. Prof English @ Tulane and Joseph Valente, Assis. Prof
English @ U. Ill, Feb. 1997, College Literature, v. 25, Iss. 1, “Performative chic,” p pq

The recent vogue for performativity, particularly in gender and postcolonial studies, suggests that
the desire for political potency has displaced the demand for critical rigor.l Because Judith Butler
bears the primary responsibility for investing performativity with its present critical cachet, her work
furnishes a convenient site for exposing the flawed theoretical formulations and the hollow political
claims advanced under the banner of performativity. We have undertaken this critique not solely in the
interests of clarifying performativity's theoretical stakes: in our view, the appropriation of
performativity for purposes to which it is completely unsuited has misdirected crucial activist
energies, not only squandering resources but even endangering those naive enough to act on
performativity's (false) political promise. It is reasonable to expect any practical political
discourse to essay an analysis which links its proposed actions with their supposed effects,
appraising the fruits of specific political labors before their seeds are sown. Only by means of
such an assessment can any political program persuade us to undertake some tasks and forgo
others. Butler proceeds accordingly: "The task is not whether to repeat, but how to repeat or, indeed to
repeat, and through a radical proliferation of gender, to displace the very gender norms that enable
repetition itself' (Gender Trouble 148). Here, at the conclusion to Gender Trouble, she makes good her
promise that subjects can intervene meaningfully, politically, in the signification system which
iteratively constitutes them. The political "task" we face requires that we choose "how to repeat" gender
norms in such a way as to displace them. According to her final chapter, "The Politics of Parody," the
way to displace gender norms is through the deliberate performance of drag as gender parody.
pragmatism

A critical mass of small reforms is the only way that a radical left agenda is possible.

Wilson, 2000 – Editor and Publisher of Illinois Academe – 2000 (John K. Wilson, “How the Left can Win Arguments and
Influence People” p. 121- 123)

Progressives need to be pragmatic in order to be powerful. However, pragmatism shouldn't be confused with
Clintonian centrism and the abandonment of all substance. Pragmatists have principles, too. The difference between a
pragmatic progressive and a foolish one is the willingness to pick the right fights and fight in the right way to
accomplish these same goals. The current failure of progressivism in America is due to the structure of American
politics and media, not because of a wrong turn that the movement took somewhere along the way. What the left
needs is not a "better" ideology but a tactical adaptation to the obstacles it faces in the contemporary political scene. A
pragmatic progressivism does not sacrifice its ideals but simply communicates them better to the larger public . The words we
use shape how people respond to our ideas. It’s tempting to offer the standard advice that progressives should present their ideas in the most palatable form. But palatable to whom? The media
managers and pedestrian pundits who are the intellectual gatekeepers won't accept these ideas. By the time progressives transform their ideas into the political baby food necessary for inclusion in
current debates, it barely seems to be worth the effort. Leftists need to seize the dominant political rhetoric, even though it may be conservative in its goals, and turn it in a progressive direction.
Progressives need to use the antitax ideology to demand tax cuts for the poor. Progressives need to use the antigovernment and antiwelfare ideology to demand the end of corporate welfare.
Progressives need to translate every important issue into the language that is permissible in the mainstream. Something will inevitably be lost in the translation. But the political soul underlying these
The left does not need to abandon its progressive views in order to be
progressive ideas can be preserved and brought to the public's attention.
popular. The left only needs to abandon some of its failed strategies and become as savvy as the conservatives are at
manipulating the press and the politicians. The language of progressive needs to become more mainstream, but the
ideas must remain radical. In an age of soulless politicians and spineless ideologies, the left has the virtue of integrity.
Until progressives become less self-satisfied with the knowledge that they're right and more determined to convince
everyone else of this fact, opportunities for political change will not be forthcoming. Progressives have also been
hampered by a revolutionary instinct among some leftist groups. According to some left wingers, incremental progress
is worthless---that is, nothing short of a radical change in government will mean anything to them. Indeed, for most
radical left wingers, liberal reforms are a threat to the movement, since they reduce the desire for more extreme
changes. What the revolutionaries fail to realize is that progressive achievements can build on one another. If anything
approaching a political revolution actually happens in America, it will be due to a succession of popular, effective,
progressive reforms. A popular uprising in the ballot box is possible only if the left can change its political assumptions
about smaller, specific issues.
***Futurism Good***

futurism key to movements

Debates by non-government action about future crises are critical to social movements—distopian visions are mobilizing
transnational movements that are effectively pressuring governments into preventing everything from nuclear annihilation to
slowing the spread of AIDS.
Kurasawa, 04 (Professor of Sociology, York University of Toronto, Fuyuki, Constellations Volume 11, No 4, 2004).

In the twenty-first century, the lines of political cleavage are being drawn along those of competing dystopian visions . Indeed, one of the
notable features of recent public discourse and socio-political struggle is their negationist hue, for they are devoted as much to the prevention of disaster as to the realization of the good, less to what ought to be than
what could but must not be. The debates that preceded the war in Iraq provide a vivid illustration of this tendency, as both camps rhetorically invoked incommensurable catastrophic scenarios to make their respective
cases. And as many analysts have noted, the multinational antiwar protests culminating on February 15, 2003 marked the first time that a mass movement was able to mobilize substantial numbers of people
dedicated to averting war before it had actually broken out. More generally, given past experiences and awareness of what might occur in the future, given the cries of ‘never again’ (the Second World War, the
Holocaust, Bhopal, Rwanda, etc.) and ‘not ever’ (e.g., nuclear or ecological apocalypse, human cloning) that are emanating from different parts of the world, the avoidance of crises is seemingly on everyone’s lips –
and everyone’s conscience. From the United Nations and regional multilateral organizations to states, from non-governmental organizations to transnational social movements, the determination to prevent the
actualization of potential cataclysms has become a new imperative in world affairs. Allowing past disasters to reoccur and unprecedented calamities to unfold is now widely seen as unbearable when, in the process,
the suffering of future generations is callously tolerated and our survival is being irresponsibly jeopardized. Hence, we need to pay attention to what a widely circulated report by the International Commission on
Rather than
Intervention and State Sovereignty identifies as a burgeoning “culture of prevention,”3 a dynamic that carries major, albeit still poorly understood, normative and political implications.
bemoaning the contemporary preeminence of a dystopian imaginary, I am claiming that it can enable a novel form of
transnational socio-political action, a manifestation of globalization from below that can be termed preventive foresight. We
should not reduce the latter to a formal principle regulating international relations or an ensemble of policy prescriptions for
official players on the world stage, since it is, just as significantly, a mode of ethico-political practice enacted by participants in the
emerging realm of global civil society. In other words, what I want to underscore is the work of farsightedness, the social processes through which civic associations are simultaneously
constituting and putting into practice a sense of responsibility for the future by attempting to prevent global catastrophes. Although the labor of preventive foresight takes place in varying political and socio-cultural
settings – and with different degrees of institutional support and access to symbolic and material resources – it is underpinned by three distinctive features: dialogism, publicity, and transnationalism. In the first
instance, preventive foresight is an intersubjective or dialogical process of address, recognition, and response between two parties in global civil society: the ‘warners,’ who anticipate and send out word of possible
, the
perils, and the audiences being warned, those who heed their interlocutors’ messages by demanding that governments and/or international organizations take measures to steer away from disaster. Secondly
work of farsightedness derives its effectiveness and legitimacy from public debate and deliberation. This is not to say that a fully
fledged global public sphere is already in existence, since transnational “strong publics” with decisional power in the formal-
institutional realm are currently embryonic at best. Rather, in this context, publicity signifies that “weak publics” with distinct yet
occasionally overlapping constituencies are coalescing around struggles to avoid specific global catastrophes.4 Hence, despite
having little direct decision-making capacity, the environmental and peace movements, humanitarian NGOs, and other similar
globally-oriented civic associations are becoming significant actors involved in public opinion formation. Groups like these are
active in disseminating information and alerting citizens about looming catastrophes, lobbying states and multilateral
organizations from the ‘inside’ and pressuring them from the ‘outside,’ as well as fostering public participation in debates about
the future. This brings us to the transnational character of preventive foresight, which is most explicit in the now commonplace
observation that we live in an interdependent world because of the globalization of the perils that humankind faces (nuclear
annihilation, global warming, terrorism, genocide, AIDS and SARS epidemics, and so on); individuals and groups from far-flung
parts of the planet are being brought together into “risk communities” that transcend geographical borders.5 Moreover, due to
dense media and information flows, knowledge of impeding catastrophes can instantaneously reach the four corners of the earth
– sometimes well before individuals in one place experience the actual consequences of a crisis originating in another. My
contention is that civic associations are engaging in dialogical, public, and transnational forms of ethico-political action that
contribute to the creation of a fledgling global civil society existing ‘below’ the official and institutionalized architecture of
international relations. The work of preventive foresight consists of forging ties between citizens; participating in the circulation of
flows of claims, images, and information across borders; promoting an ethos of farsighted cosmopolitanism; and forming and
mobilizing weak publics that debate and struggle against possible catastrophes. Over the past few decades, states and
international organizations have frequently been content to follow the lead of globally- minded civil society actors, who have been
instrumental in placing on the public agenda a host of pivotal issues (such as nuclear war, ecological pollution, species
extinction, genetic engineering, and mass human rights violations).
futurism ket to movements

Debates from non-governmental actors about futurism and crises aversion have been critical to forming movements addressing
pollution, genocide, AIDS, racism, and war. Decades of social activism have been motivated by futurism—this history
overwhelms their non-empirical theory.

Kurasawa, 04 (Professor of Sociology, York University of Toronto, Fuyuki, Constellations Volume 11, No 4, 2004).

Societies emerging from the horrors and devastation of two world wars came to recognize that certain dangers (principally wars
of aggression, genocide, crimes against humanity, and nuclear armageddon) needed to be averted at all costs. The international
community thereby devised a number of institutional responses, such as the Charter giving birth to the United Nations, the
signing of the Universal Declaration of Human Rights, and the UN Convention on the Prevention and Punishment of the Crime of
Genocide. However, by paralyzing the United Nations system and fuelling a nuclear arms race, the onset and escalation of the
Cold War rendered the institutional sphere largely ineffective. In response to this paralysis came the nuclear disarmament and
peace movements, which were spurred on by the terrifying realization that human beings had devised the means for their own
annihilation and that the two geopolitical blocs were pursuing an exterminist logic; given that human survival could no longer be
entrusted to governments or multilateral institutions, citizens had to organize themselves to tackle the problem head-on. In the
1970s and 1980s, widely circulated reports from the Club of Rome and the Brundtland Commission combined with environmental
activism brought another global threat to public attention, the prospect of ecological ruin caused by a rampant industrialism that
mercilessly depleted the earth’s resources and polluted it at an unsustainably destructive pace. Yet it is since the end of the Cold
War that the idea of prevention has truly come into its own in both the formally and informally organized domains of global
governance. The dissolution of the bipolar stalemate between East and West opened the door to greater inter-state
coordination and collaboration, perhaps most significantly at the United Nations Security Council.9 The creation of supranational
judicial institutions (e.g., the International Criminal Tribunal for the former Yugoslavia and the International Criminal Court) are
also signal achievements of the post-Cold War world order, for they may well have a latent deterrence effect despite the fact that
they are designed to prosecute crimes against humanity ex post facto. The Rome Treaty establishing the International Criminal
Court is itself part of an expanding infrastructure of multinational conferences and agreements that has come into being over the
past decade or so; governments and NGOs have participated in large-scale, UN-sponsored summits that have yielded
agreements or declarations incorporating strong preventive language: the Rio Summit on the environment, the Kyoto Protocol on
climate change, the International Treaty to Ban Landmines, and, of most relevance for our purposes, the Declaration on the
Responsibilities of the Present Generations Towards Future Generations.10 Furthermore, the unfolding of a process of
globalization from below has meant that certain civil society organizations are increasingly vocal in demanding that governments,
multilateral institutions and transnational corporations take preventive action or cease to engage in activities and support policies
that imperil humankind. In addition, farsightedness has become a priority in world affairs due to the appearance of new global
threats and the resurgence of ‘older’ ones. Virulent forms of ethno-racial nationalism and religious fundamentalism that had
mostly been kept in check or bottled up during the Cold War have reasserted themselves in ways that are now all-too-familiar –
civil warfare, genocide, ‘ethnic cleansing,’ and global terrorism. And if nuclear mutually assured destruction has come to pass,
other dangers are filling the vacuum: climate change, AIDS and other diseases (BSE, SARS, etc.), as well as previously
unheralded genomic perils (genetically modified organisms, human cloning). Collective remembrance of past atrocities and
disasters has galvanized some sectors of public opinion and made the international community’s unwillingness to adequately
intervene before and during the genocides in the ex-Yugoslavia and Rwanda, or to take remedial steps in the case of the
spiraling African and Asian AIDS pandemics, appear particularly glaring.
futurism key to crisis prevention

Scenario planning is critical in a world where annihilation is a possibility—addressing problems now greatly enhances our ability
to avert global catastrophe.

Kurasawa, 04 (Professor of Sociology, York University of Toronto, Fuyuki, Constellations Volume 11, No 4, 2004).

Independently of this contractualist justification, global civil society actors are putting forth a number of arguments countering
temporal myopia on rational grounds. They make the case that no generation, and no part of the world, is immune from
catastrophe. Complacency and parochialism are deeply flawed in that even if we earn a temporary reprieve, our children and
grandchildren will likely not be so fortunate unless steps are taken today. Similarly, though it might be possible to minimize or
contain the risks and harms of actions to faraway places over the short-term, parrying the eventual blowback or spillover effect is
improbable. In fact, as I argued in the previous section, all but the smallest and most isolated of crises are rapidly becoming
globalized due to the existence of transnational circuits of ideas, images, people, and commodities. Regardless of where they
live, our descendants will increasingly be subjected to the impact of environmental degradation, the spread of epidemics, gross
North-South socioeconomic inequalities, refugee flows, civil wars, and genocides. What may have previously appeared to be
temporally and spatially remote risks are ‘coming home to roost’ in ever faster cycles. In a word, then, procrastination makes little
sense for three principal reasons: it exponentially raises the costs of eventual future action; it reduces preventive options; and it
erodes their effectiveness. With the foreclosing of long-range alternatives, later generations may be left with a single course of
action, namely, that of merely reacting to large-scale emergencies as they arise. We need only think of how it gradually becomes
more difficult to control climate change, let alone reverse it, or to halt mass atrocities once they are underway. Preventive
foresight is grounded in the opposite logic, whereby the decision to work through perils today greatly enhances both the
subsequent room for maneuver and the chances of success. Humanitarian, environmental, and techno-scientific activists have
convincingly shown that we cannot afford not to engage in preventive labor. Moreover, I would contend that farsighted
cosmopolitanism is not as remote or idealistic a prospect as it appears to some, for as Falk writes, “[g]lobal justice between
temporal communities, however, actually seems to be increasing, as evidenced by various expressions of greater sensitivity to
past injustices and future dangers.”36 Global civil society may well be helping a new generational self-conception take root,
according to which we view ourselves as the provisional caretakers of our planetary commons. Out of our sense of responsibility
for the well-being of those who will follow us, we come to be more concerned about the here and now.
futurism key to human survival

Futurism is key to human survival. Debates amongst citizens are the only way to reign in the excesses of statism.

Kurasawa, 04 (Professor of Sociology, York University of Toronto, Fuyuki, Constellations Volume 11, No 4, 2004).

In recent years, the rise of a dystopian imaginary has accompanied damning assessments and widespread recognition of the
international community’s repeated failures to adequately intervene in a number of largely preventable disasters (from the
genocides in the ex-Yugoslavia, Rwanda, and East Timor to climate change and the spiraling AIDS pandemics in parts of sub-
Saharan Africa and Asia). Social movements, NGOs, diasporic groups, and concerned citizens are not mincing words in their
criticisms of the United Nations system and its member-states, and thus beginning to shift the discursive and moral terrain in
world affairs. As a result, the callousness implicit in disregarding the future has been exposed as a threat to the survival of
humanity and its natural surroundings. The Realpolitik of national self-interest and the neoliberal logic of the market will
undoubtedly continue to assert themselves, yet demands for farsightedness are increasingly reining them in. Though
governments, multilateral institutions, and transnational corporations will probably never completely modify the presentist
assumptions underlying their modes of operation, they are, at the very least, finding themselves compelled to account for
egregious instances of short-sightedness and rhetorically commit themselves to taking corrective steps. What may seem like a
modest development at first glance would have been unimaginable even a few decades ago, indicating the extent to which we
have moved toward a culture of prevention. A new imperative has come into being, that of preventive foresight.
at: any flaw with futurism

Any problem that they identify about futurism will only be worse in a world where we give up. Either others will decide for us or
we will be overwhelmed by crises. Futurism may have flaws but scenario planning by citizens is the best hope that we have.

Kurasawa, 04 (Professor of Sociology, York University of Toronto, Fuyuki, Constellations Volume 11, No 4, 2004).

None of this is to disavow the international community’s rather patchy record of avoiding foreseeable calamities over the last
decades, or to minimize the difficulties of implementing the kinds of global institutional reforms described above and the perils of
historical contingency, presentist indifference toward the future, or alarmism and resignation. To my mind, however, this is all the
more reason to pay attention to the work of preventive foresight in global civil society, through which civic associations can build
up the latter’s coordination mechanisms and institutional leverage, cultivate and mobilize public opinion in distant parts of the
world, and compel political leaders and national and transnational governance structures to implement certain policies. While
seeking to prevent cataclysms from worsening or, better yet, from occurring in the first place, these sorts of initiatives can and
must remain consistent with a vision of a just world order. Furthermore, the labor of farsightedness supports an autonomous view
of the future, according to which we are the creators of the field of possibilities within which our successors will dwell. The current
socio-political order, with all its short-term biases, is neither natural nor necessary. Accordingly, informed public participation in
deliberative processes makes a socially self-instituting future possible, through the involvement of groups and individuals active
in domestic and supranational public spaces; prevention is a public practice, and a public responsibility. To believe otherwise is, I
would argue, to leave the path clear for a series of alternatives that heteronomously compromise the well-being of those who will
come after us. We would thereby effectively abandon the future to the vagaries of history (‘let it unfold as it may’), the
technocratic or instrumental will of official institutions (‘let others decide for us’), or to gambles about the time-lags of risks (‘let
our progeny deal with their realization’). But, as I have tried to show here, this will not and cannot be accepted. Engaging in
autonomous preventive struggles, then, remains our best hope. A farsighted cosmopolitanism that aims to avert crises while
working toward the realization of precaution and global justice represents a compelling ethico-political project, for we will not
inherit a better future. It must be made, starting with us, in the here and now.
at: futurism/crises cause paralysis

Crisis scenarios do not cause paralysis—historically, the most effective social movements have used distopian imagery to
compel action.

Kurasawa, 04 (Professor of Sociology, York University of Toronto, Fuyuki, Constellations Volume 11, No 4, 2004).

Returning to the point I made at the beginning of this paper, the significance of foresight is a direct outcome of the transition
toward a dystopian imaginary (or what Sontag has called “the imagination of disaster”).11 Huxley’s Brave New World and
Orwell’s Nineteen Eighty-Four, two groundbreaking dystopian novels of the first half of the twentieth century, remain as influential
as ever in framing public discourse and understanding current techno-scientific dangers, while recent paradigmatic cultural
artifacts – films like The Matrix and novels like Atwood’s Oryx and Crake – reflect and give shape to this catastrophic
sensibility.12 And yet dystopianism need not imply despondency, paralysis, or fear. Quite the opposite, in fact, since the
pervasiveness of a dystopian imaginary can help notions of historical contingency and fallibilism gain traction against their
determinist and absolutist counterparts. Once we recognize that the future is uncertain and that any course of action produces
both unintended and unexpected consequences, the responsibility to face up to potential disasters and intervene before they
strike becomes compelling. From another angle, dystopianism lies at the core of politics in a global civil society where groups
mobilize their own nightmare scenarios (‘Frankenfoods’ and a lifeless planet for environmentalists, totalitarian patriarchy of the
sort depicted in Atwood’s Handmaid’s Tale for Western feminism, McWorld and a global neoliberal oligarchy for the alternative
globalization movement, etc.). Such scenarios can act as catalysts for public debate and socio-political action, spurring citizens’
involvement in the work of preventive foresight.
at: predictions wrong

Just because we cannot predict the future with total certainty does not mean that we cannot make educated guesses. And,
scenario planning is key to making responsible choices. We are obligated to take care of the planet if we have a significant role
to play.

Kurasawa, 04 (Professor of Sociology, York University of Toronto, Fuyuki, Constellations Volume 11, No 4, 2004).

A radically postmodern line of thinking, for instance, would lead us to believe that it is pointless, perhaps even harmful, to strive
for farsightedness in light of the aforementioned crisis of conventional paradigms of historical analysis. If, contra teleological
models, history has no intrinsic meaning, direction, or endpoint to be discovered through human reason, and if, contra scientistic
futurism, prospective trends cannot be predicted without error, then the abyss of chronological inscrutability supposedly opens up
at our feet. The future appears to be unknowable, an outcome of chance. Therefore, rather than embarking upon grandiose
speculation about what may occur, we should adopt a pragmatism that abandons itself to the twists and turns of history; let us be
content to formulate ad hoc responses to emergencies as they arise. While this argument has the merit of underscoring the
fallibilistic nature of all predictive schemes, it conflates the necessary recognition of the contingency of history with unwarranted
assertions about the latter’s total opacity and indeterminacy. Acknowledging the fact that the future cannot be known with
absolute certainty does not imply abandoning the task of trying to understand what is brewing on the horizon and to prepare for
crises already coming into their own. In fact, the incorporation of the principle of fallibility into the work of prevention means that
we must be ever more vigilant for warning signs of disaster and for responses that provoke unintended or unexpected
consequences (a point to which I will return in the final section of this paper). In addition, from a normative point of view, the
acceptance of historical contingency and of the self-limiting character of farsightedness places the duty of preventing catastrophe
squarely on the shoulders of present generations. The future no longer appears to be a metaphysical creature of destiny or of the
cunning of reason, nor can it be sloughed off to pure randomness. It becomes, instead, a result of human action shaped by
decisions in the present – including, of course, trying to anticipate and prepare for possible and avoidable sources of harm to our
successors. Combining a sense of analytical contingency toward the future and ethical responsibility for it, the idea of early
warning is making its way into preventive action on the global stage.

Scenario planning is no longer the product of sterile government number crunching. Debates amongst citizens about future
crises have created a global early warning next work that has both been proven relatively accurate and able to influence
government action.

Kurasawa, 04 (Professor of Sociology, York University of Toronto, Fuyuki, Constellations Volume 11, No 4, 2004).

Despite the fact that not all humanitarian, technoscientific, and environmental disasters can be predicted in advance, the
multiplication of independent sources of knowledge and detection mechanisms enables us to foresee many of them before it is
too late. Indeed, in recent years, global civil society’s capacity for early warning has dramatically increased, in no small part due to the
impressive number of NGOs that include catastrophe prevention at the heart of their mandates.17 These organizations are often the first to detect signs of trouble, to dispatch investigative or fact-finding missions,
and to warn the international community about impending dangers; to wit, the lead role of environmental groups in sounding the alarm about global warming and species depletion or of humanitarian agencies
What has come into being, then, is a
regarding the AIDS crisis in sub-Saharan Africa, frequently months or even years before Western governments or multilateral institutions followed suit.
loose-knit network of watchdog groups that is acquiring finely tuned antennae to pinpoint indicators of forthcoming or already
unfolding crises. This network of ‘early warners’ are working to publicize potential and actual emergencies by locating indicators
of danger into larger catastrophic patterns of interpretation, culturally meaningful chains of events whose implications become
discernable for decision-makers and ordinary citizens (‘this is why you should care’).18 Civic associations can thus invest
perilous situations with urgency and importance, transforming climate change from an apparently mild and distant possibility to an irreversible and grave threat to human survival, and
genocide from a supposedly isolated aberration to an affront to our common humanity. The growing public significance of preventive message in global affairs is part and parcel of what Ignatieff has termed an
“advocacy revolution,”19 since threatened populations and allied organizations are acting as early warning beacons that educate citizens about certain perils and appeal for action on the part of states and multilateral
institutions. Global civil society players have devised a host of ‘naming and shaming’ strategies and high-profile information campaigns to this effect, including press conferences, petitions, mass marches, and
0 The advocacy revolution is
boycotts, and spectacular stunts that denounce bureaucratic inertia, the reckless pursuit of profit, or the preponderance of national interests in world affairs.2
having both ‘trickle-down’ and ‘trickle-up’ effects, establishing audiences of constituents and ordinary citizens conversant with
some of the great challenges facing humanity as well as putting pressure on official institutions to be proactive in their long-term
planning and shorter-term responses.

at: media distortions


They are right that the media is not perfect. But, the proliferation of different types of media makes government cover-ups very
difficult and can dramatically shape public opinion. And, there is a healthy skepticism of the media that mobilizes citizens to
question further.

Kurasawa, 04 (Professor of Sociology, York University of Toronto, Fuyuki, Constellations Volume 11, No 4, 2004).

None of this would be possible without the existence of global media, whose speed and range make it possible for reports of an
unfolding or upcoming disaster to reach viewers or readers in most parts of the world almost instantaneously. Despite the highly
selective character of what is deemed newsworthy and state and commercial influence on what is broadcast, several recent
attempts to hide evidence of acts of mass violence (Tiananmen Square, East Timor, Chechnya, etc.) and crises (e.g., during the
Chernobyl nuclear accident in the Soviet Union or the SARS outbreak in China) have failed; few things now entirely escape from
the satellite camera, the cellular telephone, or the notebook computer. And although the internet may never become the populist
panacea technological determinists have been heralding for years, it remains a key device through which concerned citizens and
activists can share and spread information. While media coverage almost always follows a crisis rather than preceding it, the
broadcast of shocking images and testimonies can nevertheless shame governments and international organizations into taking
immediate steps. The ‘CNN or BBC effect,’ to which we should now add the ‘Al-Jazeera effect,’ is a surprisingly powerful force in
impacting world public opinion, as the now notorious Abu Ghraib prison photographs remind us. The possibility that the threat of
media exposure may dissuade individuals and groups from enacting genocidal plans or reckless gambles with our future is one
of the lynchpins of prevention in our information-saturated age. Are forewarnings of disasters being heard? The mobilization of
official intervention and popular interest has certainly been mixed, yet global civil society is having some success in cultivating
audiences and advocates coalescing around specific perils (mass human rights violations, ecological devastation, genetic
engineering, epidemics, and so on). After Bhopal and Chernobyl, after ‘mad cow disease’ and the war in Iraq, citizens are
scrutinizing, questioning and even contesting official expertise in risk assessment more than ever before.21 Hence, in a world
where early warnings of cataclysms are often available, pleading ignorance or helplessness to anticipate what may come in the
future becomes less and less plausible.
at: futurism = no value to life

Short-sightedness is what makes life disposable—the techno-strategic logic that they indict is at its worst when we refuse to
consider long-term consequences. Future orientation is the only way to make better decisions.

Kurasawa, 04 (Professor of Sociology, York University of Toronto, Fuyuki, Constellations Volume 11, No 4, 2004).

At another level, instrumental-strategic forms of thought and action, so pervasive in modern societies because institutionally
entrenched in the state and the market, are rarely compatible with the demands of farsightedness. The calculation of the most
technically efficient means to attain a particular bureaucratic or corporate objective, and the subsequent relentless pursuit of it,
intrinsically exclude broader questions of long-term prospects or negative side-effects. What matters is the maximization of
profits or national self-interest with the least effort, and as rapidly as possible. Growing risks and perils are transferred to future
generations through a series of trade-offs: economic growth versus environmental protection, innovation versus safety, instant
gratification versus future well-being. What can be done in the face of short-sightedness? Cosmopolitanism provides some of the
clues to an answer, thanks to its formulation of a universal duty of care for humankind that transcends all geographical and
socio-cultural borders. I want to expand the notion of cosmopolitan universalism in a temporal direction, so that it can become
applicable to future generations and thereby nourish a vibrant culture of prevention. Consequently, we need to begin thinking
about a farsighted cosmopolitanism, a chrono-cosmopolitics that takes seriously a sense of “intergenerational solidarity” toward
human beings who will live in our wake as much as those living amidst us today. But for a farsighted cosmopolitanism to take
root in global civil society, the latter must adopt a thicker regulative principle of care for the future than the one currently in vogue (which amounts to little
more than an afterthought of the nondescript ‘don’t forget later generations’ ilk). Hans Jonas’s “imperative of responsibility” is valuable precisely because it prescribes an ethico-political relationship to the future
consonant with the work of farsightedness.27 Fully appreciating Jonas’s position requires that we grasp the rupture it establishes with the presentist assumptions imbedded in the intentionalist tradition of Western
ethics. In brief, intentionalism can be explained by reference to its best-known formulation, the Kantian categorical imperative, according to which the moral worth of a deed depends upon whether the a priori
“principle of the will” or “volition” of the person performing it – that is, his or her intention – should become a universal law.28 Ex post facto evaluation of an act’s outcomes, and of whether they correspond to the initial
intention, is peripheral to moral judgment. A variant of this logic is found in Weber’s discussion of the “ethic of absolute ends,” the “passionate devotion to a cause” elevating the realization of a vision of the world
above all other considerations; conviction without the restraint of caution and prudence is intensely presentist.29 By contrast, Jonas’s strong consequentialism takes a cue from Weber’s “ethic of responsibility,” which
stipulates that we must carefully ponder the potential impacts of our actions and assume responsibility for them – even for the incidence of unexpected and unintended results. Neither the contingency of outcomes
, consequentialism reconnects what intentionalism prefers to
nor the retrospective nature of certain moral judgments exempts an act from normative evaluation. On the contrary
keep distinct: the moral worth of ends partly depends upon the means selected to attain them (and vice versa), while the
correspondence between intentions and results is crucial. At the same time, Jonas goes further than Weber in breaking with
presentism by advocating an “ethic of long-range responsibility” that refuses to accept the future’s indeterminacy, gesturing
instead toward a practice of farsighted preparation for crises that could occur.30 From a consequentialist perspective, then,
intergenerational solidarity would consist of striving to prevent our endeavors from causing large-scale human suffering and
damage to the natural world over time. Jonas reformulates the categorical imperative along these lines: “Act so that the effects of
your action are compatible with the permanence of genuine human life,” or “Act so that the effects of your action are not
destructive of the future possibility of such life.”31 What we find here, I would hold, is a substantive and future-oriented ethos on
the basis of which civic associations can enact the work of preventive foresight.
at: fear mongering by the state

Debate is the antidote to state fear mongering—scenario planning by informed groups can counter-act official misinformation.
And, the alternative is that the governments will continue to scare us but we will be too apolitical and ill informed to counter act
lies.

Kurasawa, 04 (Professor of Sociology, York University of Toronto, Fuyuki, Constellations Volume 11, No 4, 2004).

State and market institutions may seek to produce a culture of fear by deliberately stretching interpretations of reality beyond the
limits of the plausible so as to exaggerate the prospects of impending catastrophes, or yet again, by intentionally promoting
certain prognoses over others for instrumental purposes. Accordingly, regressive dystopias can operate as Trojan horses
advancing political agendas or commercial interests that would otherwise be susceptible to public scrutiny and opposition.
Instances of this kind of manipulation of the dystopian imaginary are plentiful: the invasion of Iraq in the name of fighting
terrorism and an imminent threat of use of ‘weapons of mass destruction’; the severe curtailing of American civil liberties amidst
fears of a collapse of ‘homeland security’; the neoliberal dismantling of the welfare state as the only remedy for an ideologically
constructed fiscal crisis; the conservative expansion of policing and incarceration due to supposedly spiraling crime waves; and
so forth. Alarmism constructs and codes the future in particular ways, producing or reinforcing certain crisis narratives, belief
structures, and rhetorical conventions. As much as alarmist ideas beget a culture of fear, the reverse is no less true. If fear-
mongering is a misappropriation of preventive foresight, resignation about the future represents a problematic outgrowth of the
popular acknowledgment of global perils. Some believe that the world to come is so uncertain and dangerous that we should not
attempt to modify the course of history; the future will look after itself for better or worse, regardless of what we do or wish. One
version of this argument consists in a complacent optimism perceiving the future as fated to be better than either the past or the
present. Frequently accompanying it is a self-deluding denial of what is plausible (‘the world will not be so bad after all’), or a
naively Panglossian pragmatism (‘things will work themselves out in spite of everything, because humankind always finds ways
to survive’).37 Much more common, however, is the opposite reaction, a fatalistic pessimism reconciled to the idea that the future
will be necessarily worse than what preceded it. This is sustained by a tragic chronological framework according to which
humanity is doomed to decay, or a cyclical one of the endless repetition of the mistakes of the past. On top of their dubious
assessments of what is to come, alarmism and resignation would, if widely accepted, undermine a viable practice of
farsightedness. Indeed, both of them encourage public disengagement from deliberation about scenarios for the future, a
process that appears to be dangerous, pointless, or unnecessary. The resulting ‘depublicization’ of debate leaves dominant
groups and institutions (the state, the market, techno-science) in charge of sorting out the future for the rest of us, thus effectively
producing a heteronomous social order. How, then, can we support a democratic process of prevention from below? The
answer, I think, lies in cultivating the public capacity for critical judgment and deliberation, so that participants in global civil
society subject all claims about potential catastrophes to examination, evaluation, and contestation.
at: futurism is statist

Debates amongst citizens about government policy are proof that futurism is not statist—it is able to mobilize citizens to demand
change and imagine alternative political futures.

Kurasawa, 04 (Professor of Sociology, York University of Toronto, Fuyuki, Constellations Volume 11, No 4, 2004).

NGOs and social movements active in global civil society have drawn upon the moral imagination in similar ways, introducing
dystopian scenarios less as prophecies than as rhetorical devices that act as ‘wake-up calls.’ Dystopias are thrust into public
spaces to jolt citizens out of their complacency and awaken their concern for those who will follow them. Such tropes are
intended to be controversial, their contested character fostering public deliberation about the potential cataclysms facing
humankind, the means of addressing them, and the unintended and unexpected consequences flowing from present-day trends.
In helping us to imagine the strengths and weaknesses of different positions towards the future, then, the dystopian imaginary
crystallizes many of the great issues of the day. Amplifying and extrapolating what could be the long-term consequences of
current tendencies, public discourse can thereby clarify the future’s seeming opaqueness. Likewise, fostering a dystopian moral
imagination has a specifically critical function, for the disquiet it provokes about the prospects of later generations is designed to
make us radically question the ‘self-evidentness’ of the existing social order.34 If we imagine ourselves in the place of our
descendants, the takenfor- granted shortsightedness of our institutionalized ways of thinking and acting becomes problematic.
Indifference toward the future is neither necessary nor inevitable, but can be – and indeed ought to be – changed. Aside from the
moral imagination, and given that the idea of gambling with humanity’s future or failing to minimize its possible sources of
suffering is logically unsustainable, the appeal to reason represents another main trigger of intergenerational solidarity.
at: chaos inevitable

Chaos is not inevitable—careful future planning has been enormously effective. Medical research, humanitarian law, and
environmental regulations are just a few areas where futurism has prevented enormous suffering. Debates amongst citizens are
key to assessing probability and effectively planning.

Kurasawa, 04 (Professor of Sociology, York University of Toronto, Fuyuki, Constellations Volume 11, No 4, 2004).

Moreover, keeping in mind the sobering lessons of the past century cannot but make us wary about humankind’s supposedly
unlimited ability for problemsolving or discovering solutions in time to avert calamities. In fact, the historical track-record of last-
minute, technical ‘quick-fixes’ is hardly reassuring. What’s more, most of the serious perils that we face today (e.g., nuclear
waste, climate change, global terrorism, genocide and civil war) demand complex, sustained, long-term strategies of planning,
coordination, and execution. On the other hand, an examination of fatalism makes it readily apparent that the idea that
humankind is doomed from the outset puts off any attempt to minimize risks for our successors, essentially condemning them to
face cataclysms unprepared. An a priori pessimism is also unsustainable given the fact that long-term preventive action has had
(and will continue to have) appreciable beneficial effects; the examples of medical research, the welfare state, international
humanitarian law, as well as strict environmental regulations in some countries stand out among many others. The evaluative
framework proposed above should not be restricted to the critique of misappropriations of farsightedness, since it can equally
support public deliberation with a reconstructive intent, that is, democratic discussion and debate about a future that human
beings would freely self-determine. Inverting Foucault’s Nietzschean metaphor, we can think of genealogies of the future that
could perform a farsighted mapping out of the possible ways of organizing social life. They are, in other words, interventions into
the present intended to facilitate global civil society’s participation in shaping the field of possibilities of what is to come. Once
competing dystopian visions are filtered out on the basis of their analytical credibility, ethical commitments, and political
underpinnings and consequences, groups and individuals can assess the remaining legitimate catastrophic scenarios through
the lens of genealogical mappings of the future. Hence, our first duty consists in addressing the present-day causes of eventual
perils, ensuring that the paths we decide upon do not contract the range of options available for our posterity.42 Just as
importantly, the practice of genealogically inspired farsightedness nurtures the project of an autonomous future, one that is
socially self-instituting. In so doing, we can acknowledge that the future is a human creation instead of the product of
metaphysical and extra-social forces (god, nature, destiny, etc.), and begin to reflect upon and deliberate about the kind of
legacy we want to leave for those who will follow us. Participants in global civil society can then take – and in many instances
have already taken – a further step by committing themselves to socio-political struggles forging a world order that, aside from
not jeopardizing human and environmental survival, is designed to rectify the sources of transnational injustice that will continue
to inflict needless suffering upon future generations if left unchallenged.
***State***

A2: State Bad

Debating about the state does not mean capitulating to it --- discussing government policy
creates critical understanding that facilitates resistance against its worst abuses
Donovan and Larkin ’06 (Clair and Phil, Australian National University, Politics, Vol. 26, No. 1)
We do not suggest that political science should merely fall into line with the government
instrumentalism that we have identified, becoming a 'slave social science' (see Donovan, 2005). But, we
maintain that political scientists should be able to engage with practical politics on their own terms and should
be able to provide research output that is of value to practitioners. It is because of its focus on understanding,
explanation, conceptualisation and classification that political science has the potential to
contribute more to practical politics, and more successfully. As Brian Barry notes, 'Granting (for the
sake of argument) that [students of politics] have some methods that enable us to improve on the deliverances of
untutored common sense or political journalism, what good do they do? The answer to that question is: not much.
But if we change the question and ask what good they could do, I believe that it is possible to justify a more positive
answer' (Bany, 2004, p. 22). A clear understanding of how institutions and individuals interact or how
different institutions interact with each other can provide clear and useful insights that practitioners can
successfully use, making - or perhaps remaking - a political science that 'directs research efforts to good
questions and enables incremental improvements to be made' (ibid., 19). In this sense, political
science already has the raw material to make this contribution, but it chooses not to utilise it in
this way: no doubt, in part, because academics are motivated to present their findings to other
academics and not the practitioners within the institutions they study.

Change outside the state is temporary --- only engaging institutions produces lasting remedies
Milbrath ’96 (Lester W., Professor Emeritus of Political Science and Sociology – SUNY Buffalo, Building Sustainable Societies, Ed.
Pirages, p. 289)

In some respects personal change cannot be separated from societal change. Societal transformation will not be successful
without change at the personal level; such change is a necessary but not sufficient step on the route to sustainability. People
hoping to live sustainably must adopt new beliefs, new values, new lifestyles, and new worldview. But lasting personal change is
unlikely without simultaneous transformation of the socioeconomic/political system in which people function. Persons may
solemnly resolve to change, but that resolve is likely to weaken as they perform day-today within a system reinforcing different
beliefs and values. Change agents typically are met with denial and great resistance. Reluctance to challenge mainstream society
is the major reason most efforts emphasizing education to bring about change are ineffective. If societal transformation must be
speedy, and most of us believe it must, pleading with individuals to change is not likely to be effective.
A2: State Bad

State power is flexible and open to reorientation


Krause and Williams ‘97
(Keith, Professor of Political Science – York U., and Michael, Professor of Political Science – U Southern Main,
Critical Security Studies: Concepts and Cases, p. xvi)

Many of the chapters in this volume thus retain


a concern with the centrality of the state as a locus not only
of obligation but of effective political action. In the realm of organized violence, states also
remain the preeminent actors. The task of a critical approach is not to deny the centrality of the
state in this realm but, rather, to understand more fully its structures, dynamics, and possibilities
for reorientation. From a critical perspective, state action is flexible and capable of reorientation,
and analyzing state policy need not therefore be tantamount to embracing the statist assumptions of
orthodox conceptions. To exclude a focus on state action from a critical perspective on the grounds that
it plays inevitably within the rules of existing conceptions simply reverses the error of
essentializing the state. Moreover, it loses the possibility of influencing what remains the most
structurally capable actor in contemporary world politics.

Only debating state policies can avert nuclear conflict --- this doesn’t mean accepting the system
Spanier ’90 (John, PhD – Yale and Teacher – U Florida, Games Nations Play, p. 115)

Whether the observer personally approves of the "logic of behavior" that a particular framework seems to
suggest is not the point. It is one thing to say, as done here, that the state system condemns each state to
be continually concerned with its power relative to that of other states, which, in an anarchical system, it regards as
potential aggressors. It is quite another thing to approve morally of power politics . The utility of the state-
system framework is simply that is points to the "essence" of state behavior. It does not pretend to account for all factors, such as
moral norms, that motivate states. As a necessarily simplified version of reality, it clarifies what most basically concerns and
drives states and what kinds of behavior can be expected. We, as observers, may deplore that behavior and the anarchical system
that produces it and we may wish that international politics were not as conflictual and violent as the twentieth century has
already amply demonstrated. We may prefer a system other that one in which states are so committed to advancing their own
national interests and protecting their sovereignty. Nevertheless, however much we may deplore the current
system and prefer a more peaceful and harmonious world, we must first understand the contemporary
one if we are to learn how to "manage" it and avoid the catastrophe of a nuclear war.
A2: State Bad

Ignoring the state is politically disastrous --- only opposition to specific institutions can
meaningfully challenge domination
Grossberg ’92 (Lawrence, Professor of Communication – U Illinois, We Gotta Get Out of This Place, p. 390-1)

But this would mean that the


Left could not remain outside of the systems of governance. It has sometimes to work with,
against and with in bureaucratic systems of governance. Consider the case of Amnesty
International, an immensely effective organization when its major strategy was (similar to that of the Right) exerting
pressure directly on the bureaucracies of specific governments. In recent years (marked by the recent rock tour), it has
apparently redirected its energy and resources, seeking new members (who may not be committed to actually doing anything;
membership becomes little more than a statement of ideological support for a position that few
are likely to oppose) and public visibility. In stark contrast, the most effective struggle of the Left in recent
times has been the dramatic (and, one hopes continuing) dismantling of apartheid in South Africa. It was accomplished
by mobilizing popular pressure on the institutions and bureaucracies of economic and governmental
institutions, and it depended upon a highly sophisticated organizational structure. The Left too often thinks that it
can end racism and sexism and classism by changing people’s attitude and everyday practices
(e.g. the 1990 Black boycott of Korean stores in New York). Unfortunately, while such struggles may be
extremely visible, they are often less effective than attempts to move the institutions (e.g. banks, taxing
structures, distributors) which have put the economic relations of bleack an immigrant populations in place and which
condition people’s everyday practices. The Left needs institutions which can operate within the system of
governance, understanding that such institutions are the mediating structures by which power is actively
realized. It is often by directing opposition against specific institutions that power can be challenged.
The Left assumed for some time now that, since it has so little access to the apparatuses of agency, its only alternative is to seek a public voice in the media through
tactical protests. The Left does in fact need more visibility, but it also needs greater access to the entire range of apparatuses of decision making power. Otherwise
the Left has nothing but its own self-righteousness. It is not individuals who have produced starvation and the other social
disgraces of our world, although it is individuals who must take responsibility for eliminating them. But to do so, they must act with organizations, and within the
systems of organizations which in fact have the capacity (as well as responsibility) to fight them. Without such organizations, the only models of political
commitment are self-interest and charity. Charity suggests that we act on behalf of others who cannot act on their own behalf. But we are all precariously caught in
the circuits of global capitalism, and everyone’s position is increasingly precarious and uncertain. It will not take much to change the position of any individual in the
United States, as the experience of many of the homeless, the elderly and the “fallen” middle class demonstrates. Nor are there any guarantees about the future of any
single nation. We can imagine ourselves involved in a politics where acting for another is always acting for oneself as well, a politics in which everyone struggles
with the resources they have to make their lives (and the world) better, since the two are so intimately tied together! For example, we need to think of affirmation
action as in everyone’s best interests, because of the possibilities it opens. We need to think with what Axelos has described as a “planetary thought” which “would
be a coherent thought—but not a rationalizing and ‘rationalist’ inflection; it would be a fragmentary thought of the open totality—for what we can grasp are
fragments unveiled on the horizon of the totality. Such a politics will not begin by distinguishing between the local and the global (and certainly not by valorizing
Resistance is always a
one over the other) for the ways in which the former are incorporated into the latter preclude the luxury of such choices.
local struggle, even when (as in parts of the ecology movement) it is imagined to connect into its global
structures of articulation: Think globally, act locally. Opposition is predicated precisely on locating the points of articulation between them, the
points at which the global becomes local, and the local opens up onto the global. Since the meaning of these terms has to be understood in the context of any
Fight locally because that is the scene
particular struggle, one is always acting both globally and locally: Think globally, act appropriately!
of action, but aim for the global because that is the scene of agency. “Local struggles directly target national and
international axioms, at the precise point of their insertion into the field of immanence.” This requires the imagination and construction of forms of unity,
.
commonality and social agency which do not deny differences Without such commonality, politics is too easily reduced to a question of individual rights (i.e., in the
terms of classical utility theory); difference ends up “trumping” politics, bringing it to an end. The struggle against the disciplined mobilization of everyday life can
only be built on affective commonalities, a shared “responsible yearning: a yearning out towards something more and something better than this and this place now.”
The Left, after all, is defined by its common commitment to principles of justice, equality and democracy (although these might conflict) in economic, political and
cultural life. It is based on the hope, perhaps even the illusion, that such things are possible. The construction of an affective commonality attempts to mobilize
people in a common struggle, despite the fact that they have no common identity or character, recognizing that they are the only force capable of providing a new
historical and oppositional agency. It strives to organize minorities into a new majority.
A2: State Bad (Gender)

The state is not inherently patriarchal –- reformism is a more effective way to challenge patriarchy
Rhode ’94 (Deborah L., Professor of Law – Stanford, 107 Harv. L. Rev. 1181, April, Lexis)

Neither can the state be understood solely as an instrument of men's interests. As a threshold matter, what
constitutes those interests is not self-evident, as MacKinnon's own illustrations suggest. If, for example, policies liberalizing
abortion serve male objectives by enhancing access to female sexuality, policies curtailing abortion presumably also serve male
objectives by reducing female autonomy. n23 In effect, patriarchal frameworks verge on tautology. Almost any
gender-related policy can be seen as either directly serving men's immediate interests, or as compromising
short-term concerns in the service of broader, long-term goals, such as "normalizing" the system and stabilizing power
relations. A framework that can characterize all state interventions as directly or indirectly patriarchal
offers little practical guidance in challenging the conditions it condemns. And if women are not a homogenous
group with unitary concerns, surely the same is true of men. Moreover, if the state is best understood as a network of institutions
with complex, sometimes competing agendas, then the patriarchal model of single-minded instrumentalism seems highly
implausible. It is difficult to dismiss all the anti-discrimination initiatives of the last quarter century as purely counter-
revolutionary strategies. And it is precisely these initiatives, with their appeal to "male" norms of "objectivity and the
impersonality of procedure, that [have created] [*1186] leverage for the representation of women's interests." n24 Cross-
cultural research also suggests that the status of women is positively correlated with a strong state,
which is scarcely the relationship that patriarchal frameworks imply. n25 While the "tyrannies" of public
and private dependence are plainly related, many feminists challenge the claim that they are the same. As Carole Pateman notes,
women do not "live with the state and are better able to make collective struggle against institutions than individuals." n26 To
advance that struggle, feminists need more concrete and contextual accounts of state institutions than patriarchal frameworks
have supplied. Lumping together police, welfare workers, and Pentagon officials as agents of a
unitary patriarchal structure does more to obscure than to advance analysis. What seems necessary is a
contextual approach that can account for greater complexities in women's relationships with governing institutions. Yet despite
their limitations, patriarchal theories underscore an insight that generally informs feminist theorizing. As Part II reflects,
governmental institutions are implicated in the most fundamental structures of sex-based inequality and in the strategies
necessary to address it. [Continues] These tensions within the women's movement are, of course, by no means unique. For any
subordinate group, the state is a primary source of both repression and assistance in the struggle for
equality. These constituencies cannot be "for" or "against" state involvement in any categorical sense. The questions are always
what forms of involvement, to what ends, and who makes these decisions. From some feminist perspectives, liberalism has failed
to respond adequately to those questions because of deeper difficulties. In part, the problem stems from undue faith in formal
rights. The priority granted to individual entitlements undermines the public's sense of collective responsibility. This critique has
attracted its own share of criticism from within as well as from outside the feminist community. As many left feminists, including
critical race theorists, have noted, rights-based claims have played a crucial role in advancing group as well
as individual interests. n32 Such
claims can express desires not only for autonomy, but also for participation in
the struggles that shape women's collective existence. The priority that state institutions place on rights is not in
itself problematic. The central difficulty is the limited scope and inadequate enforcement of currently recognized entitlements.
Since rights-oriented campaigns can advance as well as restrict political struggle, evaluation of their strategic value demands
historically-situated contextual analysis.
A2: State Bad (Environment)

The immediacy of environmental degradation makes state action essential --- anti-statist critiques
fail and reproduce violence
Eckersley ‘04 (Robyn, Department of Political Science, University of Melbourne, THE GREEN STATE: RETHINKING
DEMOCRACY AND SOVEREIGNTY, p. 90-93)
It might be tempting to conclude from this general critique that states are part of the problem rather than
the solution to ecological degradation. With its roots in the peace and antinuclear movements, the green movement
has long been critical of the coercive modality of state power—including the state-military-industrial complex—and might therefore be
understandably skeptical toward the very possibility of reforming or transforming states into more
democratic and ecologically responsive structures of government. The notion that the state might come to represent an ecological savior and
Yet such an anti-statist posture cannot withstand
trustee appears both fanciful and dangerous rather than empowering.
critical scrutiny from a critical ecological perspective. The problem seems to be that while states have been
associated with violence, insecurity, bureaucratic domination, injustice, and ecological degradation, there is
no reason to assume that any alternatives we might imagine or develop will necessarily be free of, or less burdened by,
such problems. As Hedley Bull warns, violence, insecurity, injustice, and ecological degradation pre-date the
state system, and we cannot rule out the possibility that they are likely to survive the demise of the state system, regardless
of what new political structures may arise. ‘9 Now it could be plausibly argued that these problems might be lessened under a more
democratic and possibly decentralized global political architecture (as bioregionalists and other green decentralists have argued). However, there
is no basis upon which to assume that they will be lessened any more than under a more deeply democratized state system. Given the
of many ecological problems (e.g., global warming), building on the state
seriousness and urgency
governance structures that already exist seems to be a more fruitful path to take than any attempt to
move beyond or around states in the quest for environmental sustainability.20 Moreover, as a matter of principle, it can be argued that environmental
benefits are public goods that ought best be managed by democratically organized public power and not by private power.2l Such an approach is consistent with
critical theory’s concern to work creatively with current historical practices and associated understandings rather than fashion utopias that have no purchase on such
practices aid understandings. In short, there is more mileage to be gained by enlisting and creatively developing the existing norms, rules, and practices of state
governance in ways that make state power more democratically and ecologically accountable than designing a new architecture of global governance de novo (a
daunting and despairing proposition). Skeptics should take heart from the fact that the organized coercive power of democratic states is not a totally untamed power,
insofar as such power must be exercised according to the rule of law and principles of democratic oversight. This is not to deny that state power can sometimes be
seriously abused (e.g., by the police or national intelligence agencies). Rather, it is merely to argue that such powers are not unlimited and beyond democratic control
and redress. The focus of critical ecological attention should therefore be on how effective this control and redress has been, and how it might be strengthened. The
same argument may be extended to the bureaucratic arm of the state. In liberal democratic states, with the gradual enlargement, specialization, and depersonalization
of state administrative power have also come legal norms and procedures that limit such power according to the principle of democratic accountability. As Gianfranco
Poggi has observed, at the same time as the political power of the state has become more extensive in terms of its subject matter and reach, so too have claims for
public participation in the exercise of this power widened.22 This is also to acknowledge the considerable scope for further, more deep-seated democratic oversight.
Indeed, it is possible to point to a raft of new ecological discursive designs that have already emerged as partial antidotes to the technocratic dimensions of the
administrative state, such as community right-to-know legislation, community environmental monitoring and reporting, third-party litigation rights, environmental and
technology impact assessment, statutory policy advisory committees, citizens’ juries, consensus conferences, and public environmental inquiries. Each of these
initiatives may be understood as attempts to confront both public and private power with its consequences, to widen the range of voices and perspectives in state
administration, to expose or prevent problem displacement, and/or to ensure that the sites of economic, social, and political power that create and/or are responsible
for ecological risks are made answerable to all those who may suffer the consequences. This is precisely where an ongoing green critical focus on the state can remain
productive. Insofar as any agency of the state (military, police, or environmental protection agencies) is no longer properly accountable to citizens (whether directly
and/or via the executive or the parliament), then the democratic state is failing its citizens. Seen in this ligh t, the green critique of the
administrative state should be understood not as a critique of the state per se but rather a critique of
illegitimate power. It is a power that is no longer properly accountable to citizens according to the ideals of liberal democracy. The ultimate
challenge for critical political ecologists should not be simply to bring liberal democratic practice into alignment with liberal democratic ideals
(although this would be a good start) but to outline a distinctively green set of regulative ideals, and a green democratic constitutional state that is
The concern should not be the mere fact that
less exclusionary and more public spirited than the liberal democratic state.
states exercise power but rather how this power can be made more accountable and hence more
legitimate.
state good

Turning away from the state prevents mobilization for good causes.
Goble 98 (Paul, Publisher of RFE/RL, “THE CONSEQUENCES OF DEPOLITICIZATION,” Radio Free Europe, October 12,
1998, http://www.friends-partners.org/friends/news/omri/1998/10/981012I.html(opt,mozilla,unix,english,,new), accessed July
07)

First, as people turn away from the state as the source of support, they inevitably care less about what the state does and are
less willing to take action to assert their views. That means that neither the state nor the opposition can mobilize them to take
action for or against anything. As a result, the opposition cannot easily get large numbers of people to demonstrate even if the
opposition is taking positions that polls suggest most people agree with. And the government cannot draw on popular support
even when it may be doing things that the people have said they want. That means that the size of demonstrations for or against
anything or anyone are an increasingly poor indicator of what the people want or do not want the state to do. Second, precisely
because people are focusing on their private lives and taking responsibility for them, they are likely to become increasingly upset
when the state attempts to intervene in their lives even for the most benign purposes, particularly if it does so in an ineffective
manner. Such attitudes, widespread in many countries and important in limiting the power of state institutions, nonetheless pose
a particular danger to countries making the transition from communism to democracy. While those views help promote the
dismantling of the old state, they also virtually preclude the emergence of a new and efficient one. As a result, these countries
are often likely to find themselves without the effective state institutions that modern societies and economies require if they are
to be well regulated. And third, countries with depoliticized populations are especially at risk when they face a crisis. The
governments cannot count on support because people no longer expect the governments to be able to deliver.
state good: checks capitalism

The state is necessary to check the free market.


Kamiya 97 (Gary, Executive Editor, “Smashing the State,” Salon.com, The Brainwave Project, January 20, 1997,
http://www.salon.com/jan97/state2970120.html )

Perhaps the most depressing thing about libertarianism is its almost unconscious aversion to the notion that in a representative
democracy, we are the government. Of course, our democracy is plagued with big-money corruption and a thousand other
problems, but when a significant percentage of people begin to think of government as "them," democracy itself is in trouble.
There is a discomforting family resemblance between libertarianism and the militia movement. The libertarian insistence on
seeing government as a malevolent or at best obstructionist external force fails to acknowledge its organic, changing nature.
Government does, of course, set policy and attempt to dictate the course of events, but much of what it does is respond to, and
referee, conflicts in society. Far from being a reified Other, government exists precisely to grapple -- through the instrument of
law -- with issues that individuals cannot resolve by themselves. The libertarian failure to recognize the flexibility of law gives a
scholastic, how-many-angels-can-dance-on-the-head-of-a-pin quality to many of its arguments. When property rights clash with
environmental rights, for example, who adjudicates? Government does, through law: No libertarian solution would produce a
different framework. Government will not resolve those problems to the liking of all interested parties -- but neither would any
other process. We have big government in large part because we live in an enormously complex society -- because we have big
problems. Libertarians are fond of saying the regulatory welfare state is somehow a continuation of despotic power -- as if there
were a historical thread running between the Sun King and Sweden's social democracy. This tendentious view, verging on
paranoia, is not only ahistorical, it ignores the role modern governments play in moderating corporate power.
Realism Good

Realism cannot be simply rejected – it is a permanent part of the thinking of foreign policy
elites
Guzzini ‘98 (Stefano, Prof – Central European U, Realism in International Relations and International Political
Economy, p. 22)

Therefore, in a third step, this chapter also claims that it


is impossible just to heap realism onto the dustbin of
history and start anew. This is a non-option. Although realism as a strictly causal theory has been a
disappointment, various realist assumptions are well alive in the minds of many practitioners and
observers of international affairs. Although it does not correspond to a theory which helps us to understand a real
world with objective laws, it is a world-view which suggests thoughts about it, and which permeates our daily
language for making sense of it. Realism has been a rich, albeit very contestable, reservoir of
lessons of the past, of metaphors and historical analogies, which, in the hands of its most gifted representatives, have been
proposed, at times imposed, and reproduced as guides to a common understanding of international affairs. Realism is alive
in the collective memory and self-understanding of our (i.e. Western) foreign policy elite and
public, whether educated or not. Hence, we cannot but deal with it. For this reason, forgetting realism is
also questionable. Of course, academic observers should not bow to the whims of daily politics But staying at distance, or
being critical, does not mean that they should lose the capacity to understand the language of those who make significant
decisions, not only in government, but also in firms, NGOs, and other institutions. To the contrary this understanding as
it is a prerequisite for
increasingly varied as it may be, is a prerequisite for their very profession. More particularly,
opposing the more irresponsible claims made in the name, although not always necessarily in
the spirit, of realism. This short-term conflict makes transition to their alternative impossible --– only realism
can provide a pragmatic bridge Murray ‘97 (Alastair J.H, Prof Political Theory, U Edinburgh, Reconstructing
Realism: Between Power Politics and Cosmopolitian Ethics, p. 194) Given that, in the absence of
a resolution of such difficulties, longer-term objectives are liable to be unachievable, realism
would seem to offer a more effective strategy of transition than reflectivism itself. Whereas, in
constructivism, such strategies are divorced from an awareness of the immediate problems which obstruct such efforts, and, in
critical theoretical perspectives, they are divorced from the current realities of international politics altogether, realism's
emphasis on first addressing the immediate obstacles to development ensures that it at least
generates strategies which offer us a tangible path to follow. If these strategies perhaps lack the
visionary appeal of reflectivist proposals, emphasising simply the necessity of a restrained, moderate diplomacy in order to
ameliorate conflicts between states, to foster a degree of mutual understanding in international relations, and, ultimately, to
develop a sense of community which might underlie a more comprehensive international society, they at least seek to take
advantage of the possibilities of reform in the current international system without jeopardising
the possibilities of order. Realism's gradualist reformism, the careful tending of what it regards as an
essentially organic process, ultimately suggests the basis for a more sustainable strategy for reform
than reflectivist perspectives, however dramatic, can offer.
Realism Good

Abandoning security fails -–- all that will happen is that non-realist will be removed from
office
Kavka ‘87 (Gregory S., Prof – UC Irvine, Moral Paradoxes of Nuclear Deterrence, p. 86-87)

The lesson of the kidney case seems to be that one can, at most, actively impose substantially lesser risks or harms
on other innocent people to protect oneself. Can this lesson be applied to national as well as individual self-defense?
One might contend that it cannot be, appealing for support to the hallowed ought-implies-can principle.
According to that principle agents, including nations, can only be obligated to act in ways they are capable of
acting. But, it may be suggested , nations are literally incapable of refraining from taking steps
believed to be necessary for national defense, even if these impose horrible risks or harms
on outside innocents. For any government that failed to undertake the requisite defensive
actions (e.g., any government that abandoned nuclear deterrence) would be quickly ousted
and replaced by a government willing to under take them.
Reformism Good – Rorty

The alternative values theory over practice, meaning it can never solve.
Rorty 98[Richard, PhD in Philosophy from Yale, Achieving Our Country: Leftist Thought in Twentieth-Century America pg 36]

This leads them to step back from their country and, as they say, "theorize" it. It leads them to do what Henry
Adams did: to give cultural politics preference over real politics, and to mock the very idea that
democratic institutions might once again be made to serve social justice. It leads them to prefer
knowledge to hope.
I see this preference as a turn away from secularism and pragmatism-as an attempt to do precisely what Dewey and Whitman thought should not be done: namely, to
the leftists who are most concerned
see the American adventure within a fixed frame of reference, a frame supplied by theory. Paradoxically,
not to "totalize," and who insist that everything be seen as the play of discursive differences
rather than in the old metaphysics-of-presence way, are also the most eager to theorize, to
become spectators rather than agents.37 But that is helping yourself with one hand to what you
push away with the other. The further you get from Greek metaphysics, Dewey urged, the less anxious you should be to find a frame within which to
fit an ongoing historical process.
This retreat from secularism and pragmatism to theory has accompanied a revival of ineffability.
We are told over and over again that Lacan has shown human desire to be inherently unsatisfiable, that Derrida has shown meaning to be undecidable, that Lyotard
has shown commensuration between oppressed and oppressors to be impossible, and that events such as the Holocaust or the massacre of the original Americans are
Hopelessness has become fashionable on the Left-principled, theorized, philosophical
unrepresentable.
hopelessness. The Whitrnanesque hope which lifted the hearts of the American Left before the 1960s is now thought to have been a symptom of a naive
"humanism."
I see this preference for knowledge over hope as repeating the move made by leftist intellectuals who, earlier in the century, got their Hegelianism from Marx rather
than Dewey. Marx thought we should be scientific rather than merely utopian-that we should interpret the historical events of our day within a larger theory. Dewey
did not. He thought one had to view these events as the protocols of social experiments whose outcomes are unpredictable.
This Left still wants to put
The Foucauldian Left represents an unfortunate regression to the Marxist obsession with scientific rigor.
historical events in a theoretical context. It exaggerates the importance of philosophy for politics,
and wastes its energy on sophisticated theoretical analyses of the significance of current events.
But Foucauldian theoretical sophistication is even more useless to leftist politics than was Engels' dialectical materialism. Engels at least had an eschatology.
. Because they regard liberal reformist initiatives as symptoms of a
Foucauldians do not even have that
discredited liberal "humanism," they have little interest in designing new social experiments.
This distrust of humanism, with its retreat from practice to theory, is the sort of failure of nerve which leads people to abandon secularism for a belief in sin, and in
Delbanco's "fixed standard by which deviance from the truth can be measured and denounced." It leads them to look for a frame of
reference outside the process of experimentation and decision that is an individual or a national
life. Grand theorieseschatologies like Hegel's or Marx's, inverted eschatologies like Heidegger's, and rationalizations of hopelessness like Foucault's and Lacan's-
satisfy the urges that theology used to satisfy. These are urges which Dewey hoped Americans might cease to feel. Dewey wanted Americans to share a civic religion
that substituted utopian striving for claims to theological knowledge.
Reformism Good – Rorty

The alternative provides no method of implementation and overlooks political solutions


Rorty 98 [Richard, PhD in Philosophy from Yale, Achieving Our Country: Leftist Thought in Twentieth-Century America pg 76]
With this partial substitution of Freud for Marx as a source of social theory, sadism rather than selfishness has become the principal target of the Left. The heirs of the
. Many members of this Left specialize in what they
New Left of the Sixties have created, within the academy, a cultural Left
call the "politics of difference" or "of identity" or "of recognition." This cultural Left thinks more
about stigma than about money, more about deep and hidden psychosexual motivations than
about shallow and evident greed.
This shift of attention came at the same time that intellectuals began to lose interest in the labor unions, partly as a result of resentment over the union members' failure
to back George McGovern over Richard Nixon in 1972. Simultaneously, the leftist ferment which had been centered, before the Sixties, in the social science
departments of the colleges and the universities moved into the literature departments. The study of philosophy-mostly apocalyptic French and German philosophy-
replaced that of political economy as an essential preparation for participation in leftist initiatives.

The new cultural Left which has resulted from these changes has few ties to what remains of the pre-Sixties reformist Left. That
saving remnant consists largely of labor lawyers and labor organizers, congressional staffers, lowlevel bureaucrats hoping to rescue the welfare state from the
Republicans, journalists, social workers, and people who work for foundations. These are the people who worry about the way in which the practices of the National
Labor Relations Board changed under the Reagan administration, about the details of alternative proposals for universal health care, about budgetary constraints on
This residual reformist Left
Head Start and daycare programs, and about the reversion of welfare programs to state and local governments.
thinks more about laws that need to be passed than about a culture that needs to be changed.
The difference between this residual Left and the academic Left is the difference between the people who read books like Thomas Geoghegan's Which
Side Are You On?-a brilliant explanation of how unions get busted-and people who read Fredric Jameson's Postmodemism, or The Cultural Logic of Late Capitalism.
operates on a level of abstraction too high to encourage any particular
The latter is an equally brilliant book, but it
political initiative. After reading Geoghegan, you have views on some of the things which need to be done. After reading Jameson, you have
views on practically everything except what needs to be done.

The academic, cultural Left approves-in a rather distant and lofty way--of the activities of these
surviving reformists. But it retains a conviction which solidified in the late Sixties. It thinks that the system, and not just the laws, must be changed.
Reformism is not good enough. Because the very vocabulary of liberal politics is infected with dubious presuppositions which need to be exposed, the first task of the
Left must be, just as Confucius said, the rectification of names. The concern to do what the Sixties called "naming the system" takes precedence overreforming the
laws.
Reformism Good – Rorty

The alternative is too abstract – it creates a spectatorial approach rather than an activist one
Rorty 98[Richard, PhD in Philosophy from Yale, Achieving Our Country: Leftist Thought in Twentieth-Century America pg 91]

the
It is often said that we Americans, at the end of the twentieth century, no longer have a Left. Since nobody denies the existence of what I have called
cultural Left, this amounts to an admission that that Left is unable to engage in national politics. It is not the sort of
Left which can be asked to deal with the consequences of globalization. To get the country to deal with those
consequences, the present cultural Left would have to transform itself by opening relations with the residue of the old reformist Left, and in particular with the labor
unions. It would have to talk much more about money, even at the cost of talking less about stigma.
I have two suggestions about how to effect this transition.
The first is that the Left should put a moratorium on theory. It should try to kick its philosophy habit. The second is that the Left should
try to mobilize what remains of our pride in being Americans. It should ask the public to consider how the country of 'Lincoln and Whitman might be achieved.
the sort of sterile
In support of my first suggestion, let me cite a passage from Dewey's Reconstruction in Philosophy in which he expresses his exasperation with
debate now going on under the rubric of "individualism versus communitarianism." Dewey thought that all
discussions which took this dichotomy seriously suffer from a common defect. They are all committed to the
logic of general notions under which specific situations are to be brought. What we want is light upon this or that group of individuals, this or that concrete human
being, this or that special institution or social arrangement. For such a logic of inquiry, the traditionally accepted logic substitutes discussion of the meaning of
concepts and their dialectical relationships with one another.
Dewey was right to be exasperated by sociopolitical theory conducted at this level of abstraction. He was wrong when he went on to say that ascending to this level is
typically a rightist maneuver, one which supplies "the apparatus for intellectual justifications of the established order. "9 For such ascents are now more common on
The contemporary academic Left seems to think that the higher your level of
the Left than on the Right.
abstraction, the more subversive of the established order you can be. The more sweeping and
novel your conceptual apparatus, the more radical your critique.
When one of today' s academic leftists says that some topic has been "inadequately theorized," you can be pretty certain that he or she is going to drag in either
Theorists of the Left think that
philosophy of language, or Lacanian psychoanalysis, or some neo-Marxist version of economic determinism.
dissolving political agents into plays of differential subjectivity, or political initiatives into
pursuits of Lacan's impossible object of desire, helps to subvert the established order. Such
subversion, they say, is accomplished by "problematizing familiar concepts."
Recent attempts to subvert social institutions by problematizing concepts have produced a few very good books. They have also produced many thousands of books
which represent scholastic philosophizing at its worst. The authors of these purportedly "subversive" books honestly believe that they are serving human liberty. But it
is almost impossible to clamber back down from their books to a level of abstraction on which one might discuss the merits of a law, a treaty, a candidate, or a
Even though what these authors "theorize" is often something very concrete and near
political strategy.
at hand-a current TV show, a media celebrity, a recent scandal-they offer the most abstract and barren explanations
imaginable.
These futile attempts to philosophize one's way into political relevance are a symptom of what
happens when a Left retreats from activism and adopts a spectatorial approach to the problems of
its country. Disengagement from practice produces theoretical hallucinations. These result in an intellectual
environment which is, as Mark Edmundson says in his book Nightmare on Main Street, Gothic. The cultural Left is haunted by
ubiquitous specters, the most frightening of which is called "power." This is the name of what
Edmundson calls Foucault's "haunting agency, which is everywhere and nowhere, as evanescent
and insistent as a resourceful spook."
Reformism Good – Rorty

Even if their criticism is right, we need to keep the alternative private, while still taking public
political action
Rorty 98[Richard, PhD in Philosophy from Yale, Achieving Our Country: Leftist Thought in Twentieth-Century America pg 96]
I have argued in various books that the philosophers most often cited by cultural leftists-Nietzsche, Heidegger,
Foucault, and Derrida-are largely right in their criticisms of Enlightenment rationalism. I have argued further that
traditional liberalism and traditional humanism are entirely compatible with such criticisms. We can still be
old-fashioned reformist liberals even if, like Dewey, we give up the correspondence theory of truth and start
treating moral and scientific beliefs as tools for achieving greater human happiness, rather than as representations of
the intrinsic nature of reality. We can be this kind of liberal even after we turn our backs on Descartes, linguistify
subjectivity, and see everything around us and within us as one more replaceable social construction.

But I have also urged that insofar as these antimetaphysical, anti-Cartesian philosophers offer a quasi-religious form
of spiritual pathos, they should be relegated to private life and not taken as guides to political deliberation. The
notion of "infinite responsibility," formulated by Emmanuel Levinas and sometimes deployed by Derrida-as well as
Derrida's own frequent discoveries of impossibility, unreachability, and unrepresentabflity-e-may be useful to some
of us in our individual quests for private perfection. When we take up our public responsibilities, however, the
infinite and the unrepresentable are merely nuisances. Thinking of our responsibilities in these terms is as
much of a stumbling-block to effective political organization as is the sense of sin. Emphasizing the
impossibility of meaning, or of justice, as Derrida sometimes does, is a temptation to Gothicize--to view democratic
politics as ineffectual, because unable to cope with preternatural forces.

Whitman and Dewey, I have argued, gave us all the romance, and all the spiritual uplift, we Americans need to go
about our public business. As Edmundson remarks, we should not allow Emerson, who was a precursor of both
Whitman and Dewey, to be displaced by Poe, who was a precursor of Lacan. For purposes of thinking about how
to achieve our country, we do not need to worry about the correspondence theory of truth, the grounds of
normativity, the impossibility of justice, or the infinite distance which separates us from the other. For those
purposes, we can give both religion and philosophy a pass. We can just get on with trying to solve what Dewey
called "the problems of men. "
Reformism Good – Rorty

Only political action can revive the Left –- the plan is a key step to building alliances
Rorty 98[Richard, PhD in Philosophy from Yale, Achieving Our Country: Leftist Thought in Twentieth-Century America pg 98]

The cultural Left often seems convinced that the nation-state is obsolete, and that there is
therefore no point in attempting to revive national politics. The trouble with this claim is that the
government of our nation-state will be, for the foreseeable future, the only agent capable of making
any real difference in the amount of selfishness and sadism inflicted on Americans. It is no comfort
to those in danger of being immiserated by globalization to be told that, since national
governments are now irrelevant, we must think up a replacement for such governments. The
cosmopolitan super-rich do not think any replacements are needed, and they are likely to prevail.
Bill Readings was right to say that "the nation-state [has ceased] to be the elemental unit of capitalism," but it
remains the entity which makes decisions about social benefits, and thus about social justice.12 The current leftist
habit of taking the long view and looking beyond nationhood to a global polity is as useless as was faith in Marx's
philosophy of history, for which it has become a substitute. Both are equally irrelevant to the question of how to
prevent the reemergence of hereditary castes, or of how to prevent right-wing populists from taking advantage of
resentment at that reemergence. When we think about these latter questions, we begin to realize that one of the
essential transformations which the cultural Left will have to undergo is the shedding of its semiconscious anti-
Americanism, which it carried over from the rage of the late Sixties. This Left will have to stop thinking up
ever more abstract and abusive names for "the system" and start trying to construct inspiring images
of the country. Only by doing so can it begin to form alliances with people outside the academy-
and, specifically, with the labor unions. Outside the academy, Americans still want to feel patriotic. They still want
to feel part of a nation which can take control of its destiny and make itself a better place. If the Left forms no
such alliances, it will never have any effect on the laws of the United States. To form them will
require the cultural Left to forget about Baudrillard' s account of America as Disneyland-as a country of
simulacra-and to start proposing changes in the laws of a real country, inhabited by real people who
are enduring unnecessary suffering, much of which can be cured by governmental action. 13 Nothing
would do more to resurrect the American Left than agreement on a concrete political platform, a
People's Charter, a list of specific reforms. The existence of such a listendlessly reprinted and debated, equally
familiar to professors and production workers, imprinted on the memory both of professional people and of those
who clean the professionals' toilets-might revitalize leftist politics. 14 --1 The problems which can be cured
by governmental action, and which such a list would canvass, are mostly those that stem from
selfishness rather than sadism. But to bring about such cures it would help if the Left would
change the tone in which it now discusses sadism. The pre-Sixties reformist Left, insofar as it concerned
itself with oppressed minorities, did so by proclaiming that all of us-black, white, and brown-are Americans, and
that we should respect one another as such. This strategy gave rise to the "platoon" movies, which showed
Americans of various ethnic backgrounds fighting and dying side by side. By contrast, the contemporary
cultural Left urges that America should not be a melting-pot, because we need to respect one
another in our differences. This Left wants to preserve otherness rather than ignore it.
Reformism Good – Rorty

Current reformism will solve the criticism – we should stick with concrete solutions like the plan
Rorty 98[Richard, PhD in Philosophy from Yale, Achieving Our Country: Leftist Thought in Twentieth-Century America pg 102]

Edmundson, Delbanco, and other cultural commentators have remarked that the contemporary United States is filled
with visions of demons and angels. Stephen King and Tony Kushner have helped form a national collective
unconscious which is "Gothic" in Edmundson's sense. It produces dreams not of political reforms but of
inexplicable, magical transformations. The cultural Left has contributed to the formation of this
politically useless unconscious not only by adopting "power" as the name of an invisible,
ubiquitous, and malevolent presence, but by adopting ideals which nobody is yet able to imagine
being actualized. Among these ideals are participatory democracy and the end of capitalism. Power will pass
to the people, the Sixties Left ,believed, only when decisions are made by all those who may be
affected by their results. This means, for example, that economic decisions will be made by stakeholders rather
than by shareholders, and that entrepreneurship and markets will cease to play their present role. When they do,
capitalism as we know it will have ended, and something new will have taken its place. But what this new thing will
be, nobody knows. The Sixties did not ask how the various groups of stakeholders were to reach a consensus about
when to remodel a factory rather than build a new one, what prices to pay for raw materials, and the like. Sixties
leftists skipped lightly over all the questions which had been raised by the experience of nonmarket economies in the
so-called socialist countries. They seemed to be suggesting that once we were rid of both bureaucrats and
entrepreneurs, "the people" would know how to handle competition from steel mills or textile factories in the
developing world, price hikes on imported oil, and so on. But they never told us how "the people" would learn how
to do this. The cultural Left still skips over such questions. Doing so is a consequence of its
preference for talking about "the system" rather than about specific social practices and specific
changes in those practices. The rhetoric of this Left remains revolutionary rather than reformist
and pragmatic. Its insouciant use of terms like "late capitalism" suggests that we can just wait for
capitalism to collapse, rather than figuring out what, in the absence of markets, will set prices
and regulate distribution. The voting public, the public which must be won over if the Left is to emerge from the academy into the
public square, sensibly wants to be told the details. It wants to know how things are going to work after markets are put behind us. It wants to
know how participatory democracy is supposed to function. The cultural Left offers no answers to such demands for
further information, but until it confronts them it will not be able to be a political Left. The
public, sensibly, has no interest in getting rid of capitalism until it is offered details about the
alternatives. Nor should it be interested in participatory democracy-the liberation of the people from the power of the technocrats-until it is
told how deliberative assemblies will acquire the same know-how which only the technocrats presently possess. Even someone like myself,
whose admiration for John Dewey is almost unlimited, cannot take seriously his defense of participatory democracy against Walter Lippmann's
insistence on the need for expertise. IS The cultural Left has a vision of an America in which the white patriarchs have stopped voting and have
left all the voting to be done by members of previously victimized groups, people who have somehow come into possession of more foresight and
imagination than the selfish suburbanites. These formerly oppressed and newly powerful people are expected to be as angelic as the straight white
males were diabolical. If I shared this expectation, I too would want to live under this new dispensation. Since I see no reason to share it, I think
that the Left should get back into the business of piecemeal reform within the framework of a market economy. This was the business the
Someday, perhaps, cumulative piecemeal reforms
American Left was in during the first two-thirds of the century.
will be found to have brought about revolutionary change. Such reforms might someday produce a presently
unimaginable nonmarket economy, and much more widely distributed powers of decisionmaking. They might also, given similar reforms in other
countries, bring about an international federation, a world government. In such a new world, American national pride would become as quaint as
we should not let the abstractly described best
pride in being from Nebraska or Kazakhstan or Sicily. But in the meantime,
be the enemy of the better. We should not let speculation about a totally changed system, and a
totally different way of thinking about human life and human affairs, replace step-by-step reform
of the system we presently have.
Reformism Good – Rorty – Impact

A collapse of the Reformist Left means a return to discrimination and inevitable war
Rorty 98[Richard, PhD in Philosophy from Yale, Achieving Our Country: Leftist Thought in Twentieth-Century America pg 87]
If the formation of hereditary castes continues unimpeded, and if
the pressures of globalization create such castes not
only in the United States but in all the old democracies, we shall end up in an Orwellian world. In such a world,
there may be no supernational analogue of Big Brother, or any official creed analogous to Ingsoc. But there will be an analogue of the Inner Party-namely, the
international, cosmopolitan super-rich. They will make all the important decisions. The analogue of Orwell's Outer Party will be educated, comfortably off,
cosmopolitan professionals-Lind's "overclass," the people like you and me.
The job of people like us will be to make sure that the decisions made by the Inner Party are carried out smoothly and efficiently. It will be in the interest of the
international superrich to keep our class relatively prosperous and happy. For they need people who can pretend to be the political class of each of the individual
nation-states. For the sake of keeping the proles quiet, the super-rich will have to keep up the pretense that national politics might someday make a difference. Since
economic decisions are their prerogative, they will encourage politicians, of both the Left and the Right, to specialize in cultural issues," The aim will be to keep the
minds of the proles elsewhere--to keep the bottom 75 percent of Americans and the bottom 95 percent of the world's population busy with ethnic and religious
hostilities, and with debates about sexual mores. If the proles can be distracted from their own despair by media-created pseudo-events, including the occasional brief
and bloody war, the super-rich will have little to fear.
Contemplation of this possible world invites two responses from the Left. The first is to insist
that the inequalities between nations need to be mitigated-and, in particular, that the Northern Hemisphere must share its
wealth with the Southern. The second is to insist that the primary responsibility of each democratic nation-state is to its own least advantaged citizens. These two
responses obviously conflict with each other. In particular, the first response suggests that the old democracies should open their borders, whereas the second suggests
that they should close them. 8
The first response comes naturally to academic leftists, who have always been internationally minded. The second response comes
naturally to members of trade unions, and to the marginally employed people who can most easily be recruited into right-wing populist movements. Union members in
the United States have watched factory after factory close, only to reopen in Slovenia, Thailand, or Mexico. It is no wonder that they see the result of international free
trade as prosperity for managers and stockholders, a better standard of living for workers in developing countries, and a very much worse standard of living for
American workers. It would be no wonder if they saw the American leftist intelligentsia as on the side of the managers and stockholders-as sharing the same class
interests. For we intellectuals, who are mostly academics, are ourselves quite well insulated, at least in the short run, from the effects of globalization. To make things
we often seem more interested in the workers of the developing world than in the fate of our
worse,
fellow citizens.
Many writers on socioeconomic policy have warned that the old industrialized democracies are heading into a Weimar-like period, one in which
populist movements are likely to overturn constitutional governments. Edward Luttwak, for example, has suggested that fascism may be the
American future. The point of his book The Endangered American Dream is that members of labor unions, and unorganized unskilled workers,
will sooner or later realize that their government is not even trying to prevent wages from sinking or to prevent jobs from being exported. Around
the same time, they will realize that suburban white-collar workers-them-selves desperately afraid of being downsized-are not going to let
themselves be taxed to provide social benefits for anyone else.
At that point, something
will crack. The nonsuburban electorate will decide that the system has failed and start
looking around for a strongman to vote for--someone willing to assure them that, once he is elected, the smug bureaucrats,
tricky lawyers, overpaid bond salesmen, and postmodernist professors will no longer be calling the shots. A scenario like that of Sinclair Lewis'
novel It Can't Happen Here may then be played out.
For once such a strongman takes office, nobody can predict
what will happen. In 1932, most of the predictions made about what would happen if Hindenburg named Hitler chancellor were wildly
overoptimistic. One thing that is very likely to happen is that the gains made in the past forty years by
black and brown Americans, and by homosexuals, will be wiped out. Jocular contempt for
women will come back into fashion. The words "nigger" and "kike" will once again be heard in the workplace. All the
sadism which the academic Left has tried to make unacceptable to its students will come
flooding back. All the resentment which badly educated Americans feel about having their
manners dictated to them by college graduates will find an outlet. But such a renewal of sadism will not alter the
effects of selfishness. For after my imagined strongman takes charge, he will quickly make his peace with
the international superrich, just as Hitler made his with the German industrialists. He will invoke
the glorious memory of the Gulf War to provoke military adventures which will generate short-
term prosperity. He will be a disaster for the country and the world. People will wonder why
there was so little resistance to his evitable rise. Where, they will ask, was the American Left? Why was it
only rightists like Buchanan who spoke to the workers about the consequences of globalization? Why could not the Left channel the mounting
rage of the newly dispossessed?
Political Vacuum Turn

Abdicating political engagement creates a vacuum that will be filled by violent elites
Cook ’92 (Anthony, Associate Professor, Georgetown Law, New England LR, Spring, 26 New Eng.L. Rev. 751)

The effect of deconstructing the power of the author to impose a fixed meaning on the text or offer a continuous
narrative is both debilitating and liberating. It is debilitating in that any attempt to say what should be done
within even our insular Foucaultian preoccupations may be oppositionalized and deconstructed as an illegitimate
privileging of one term, value, perspective or narrative over another. The struggle over meaning might continue ad
infinitum. That is, if a deconstructionist is theoretically consistent and sees deconstruction not as a political tool but
as a philosophical orientation, political action is impossible, because such action requires a degree of
closure that deconstruction, as a theoretical matter, does not permit. Moreover, the approach is debilitating because
deconstruction without material rootedness, without goals and vision, creates a political and spiritual void into
which the socially real power we theoretically deconstruct steps and steps on the disempowered and
dispossessed.  [*762]  To those dying from AIDS, stifled by poverty, dehumanized by sexism and racism, crippled
by drugs and brutalized by the many forms of physical, political and economic violence that characterizes our
narcissistic culture, power hardly seems a matter of illegitimate theoretical privileging. When vision,
social theory and political struggle do not accompany critique, the void will be filled by the rich, the
powerful and the charismatic, those who influence us through their eloquence, prestige, wealth and power.

Their alternative grants tacit support to neo-liberal violence --- political engagement is
necessary to check statist abuses
Barbrook ’97 (Dr. Richard, School of Westminster, Nettime, “More Provocations”, 6-5, http://ww.nettime.org/Lists-
Archives/nettime-l-9706/msg00034.html)
we
I thought that this position is clear from my remarks about the ultra-left posturing of the 'zero-work' demand. In Europe,
have real social problems of deprivation and poverty which, in part, can only be solved by state action. This does
not make me a statist, but rather an anti-anti-statist. By opposing such intervention because they
are carried out by the state, anarchists are tacitly lining up with the neo-liberals. Even worse, refusing even to vote for
the left, they acquiese to rule by neo-liberal parties. I deeply admire direct action movements. I was a radio pirate and we provide server space
for anti-roads and environmental movements. However, this doesn't mean that I support political abstentionism or, even worse, the mystical
nonsense produced by Hakim Bey. It
is great for artists and others to adopt a marginality as a life style choice, but
most of the people who are economically and socially marginalised were never given any choice. They are
excluded from society as a result of deliberate policies of deregulation, privatisation and welfare cutbacks carried
out by neo-liberal governments. During the '70s, I was a pro-situ punk rocker until Thatcher got elected. Then we learnt
the hard way that voting did change things and lots of people suffered if state power was withdrawn from
certain areas of our life, such as welfare and employment. Anarchism can be a fun artistic pose. However,
human suffering is not.
Political Vacuum Turn

Only concrete action can prevent mass suffering


Ling ‘01(LHM, Professor, The New School, New York, Post-Colonial International Relations: Conquest and Desire Between Asia and
the West)
Without concrete action for change, postmodernism's `dissident voices' have remained bracketed,
disconnected, not really real. In maintaining `a critical distance' or `position offshore' from which to `see the possibility
of change' (Shapiro, 1992: 49), the postmodern critic brushed off too conveniently the immediate cries of
those who know they are burning in the hells of exploitation, racism, sexism, starvation, civil war, and
the like but who have few means or strategies to deal with them. What hope do they have of
overthrowing the shackles of sovereignty without a program of action? After all, asked Mark Neufeld,
`What is political without partisanship?' (Neufeld, 1994: 31). In not answering these questions, postmodernists recycled,
despite their avowals to the contrary, the same sovereign outcome as (neo)realism: that is,
discourse divorced from practice, analysis from policy, deconstruction from reconstruction,
particulars from universals, and critical theory from problem-solving. Dissident international relations could
not accommodate an interactive, articulating, self-generative Other. Its exclusive focus on the Western Self ensured, instead, (neo)realism's
sovereignty by relegating the Other to a familiar, subordinate identity: that is, as a mute, passive reflection of the West or utopian projection of
the West's dissatisfaction with itself. Critique
became romanticized into a totalizing affair - especially for
those who must bear the brunt of its repercussions. bell hooks asked, appropriately: `[s]hould we not be
suspicious of postmodern critiques of the "subject" when they surface at a historical moment when many subjugated people feel
themselves coming to voice for the first time?' (hooks, 1990: 28) Without this recognition, postmodernists ended up
marginalizing, silencing, and exiling precisely those who are `the greatest victims of the West's
essentialist conceits (the excolonials and neocolonials, Blacks, women, and so forth)' (Krishna, 1993:
405). Worse yet, added Roger Spegele, dissidence as offshore observation has `freed us from the
recognition that we have a moral obligation to do anything about it' (Spegele, 1992: 174).
Micro-Politics Fail

Micropolitics fail – they prevent coalition building that is essential to achieving real goals, this
ensures groups continual existence on the margins of power and eventual cooptation by dominant
power structures
Best and Kellner 02 prof phil @ UT el paso prof phil @ UCLA (Steven, Doug, “Postmodern Politics and the Battle for the Future”
http://www.gseis.ucla.edu/faculty/kellner/Illumina%20Folder/kell28.htm)

The emphasis on local struggles and micropower, cultural politics which redefine the political, and attempts to
develop political forms relevant to the problems and developments of the contemporary age is extremely valuable,
but there are also certain limitations to the dominant forms of postmodern politics. While an emphasis on
micropolitics and local struggles can be a healthy substitute for excessively utopian and ambitious
political projects, one should not lose sight that key sources of political power and oppression are
precisely the big targets aimed at by modern theory, including capital, the state, imperialism, and
patriarchy. Taking on such major targets involves coalitions and multi-front struggle, often
requiring a politics of alliance and solidarity that cuts across group identifications to mobilize
sufficient power to struggle against, say, the evils of capitalism or the state.
Thus, while today we need the expansion of localized cultural practices, they attain their real significance only
within the struggle for the transformation of society as a whole. Without this systemic emphasis, cultural and
identity politics remain confined to the margins of society and are in danger of degenerating into
narcissism, hedonism, aestheticism, or personal therapy, where they pose no danger and are
immediately coopted by the culture industries. In such cases, the political is merely the personal, and the original
intentions of the 1960s goal to broaden the political field are inverted and perverted. Just as economic and political demands have their referent in
subjectivity in everyday life, so these cultural and existential issues find their ultimate meaning in the demand for a new society and mode of
production
Micro-Politics Permutation

The permutation solves – a combination of macro and micropolitics is best


Best and Kellner 02 prof phil @ UT el paso prof phil @ UCLA (Steven, Doug, “Postmodern Politics and the Battle for the Future”
http://www.gseis.ucla.edu/faculty/kellner/Illumina%20Folder/kell28.htm)

Yet we would insist that it


is not a question of micro vs macropolitics, as if it were an either/or proposition,
but rather both dimensions are important for the struggles of the present and future.[15] Likewise,
we would argue that we need to combine the most affirmative and negative perspectives, embodying
Marcuse's declaration that critical social theory should be both more negative and utopian in reference to the status
quo.[16] There are certainly many things to be depressed about is in the negative and cynical
postmodernism of a Baudrillard, yet without a positive political vision merely citing the negative
might lead to apathy and depression that only benefits the existing order. For a dialectical politics,
however, positive vision of what could be is articulated in conjunction with critical analysis of what is in a
multioptic perspective that focuses on the forces of domination as well as possibilities of emancipation
While postmodern politics and theory tend to polarize into either the extremely negative or excessively
affirmative, key forms of postmodern literature have a more dialectical vision. Indeed, some of the more
interesting forms of postmodern critique today are found in fictional genres such as cyberpunk and magical realism. Cyberpunk, a subgenre
within science fiction, brings science fiction down to earth, focusing not on the intergalactic battles in the distant future, but the social problems
facing people on earth in the present.[17] Cyberpunk writers such as Bruce Sterling and William Gibson offer an unflinching look at a grim social
reality characterized by transnational capitalist domination, Social Darwinist cultural settings, radical environmental ruination, and the implosion
of the body and technology, such that humans become more and more machine like and machines increasingly become like human beings. Yet
cyberpunk novels foreground this nightmare world in order to warn us that it is an immanent possibility for the near future, in order to awaken
readers to a critical reflection on technology and social control, and to offer hope for alternative uses of technology and modes of social life.
Similarly, magical realism examines the wreckage of centuries of European colonialism, but also maintains a positive outlook, one that embraces
the strength and creativity of the human spirit, social solidarity, and spiritual and political transcendence. Like cyberpunk novels, magical realism
incorporate various aesthetic forms and conventions in an eclectic mixture that fuses postmodernism with social critique and models of resistance.
But it is also a mistake, we believe, to ground one's politics in either modern or postmodern
theory alone. Against one-sided positions, we advocate a version of reconstructive postmodernism
that we call a politics of alliance and solidarity that builds on both modern and postmodern
traditions. Unlike Laclau and Mouffe who believe that postmodern theory basically provides a basis for a new
politics, and who tend to reject the Enlightenment per se, we believe that the Enlightenment continues to
provide resources for political struggle today and are skeptical whether postmodern theory alone can
provide sufficient assets for an emancipatory new politics. Yet the Enlightenment has its blindspots and
dark sides (such as its relentless pursuit of the domination of nature, and naive belief in "progress," so we believe
that aspects of the postmodern critique of Enlightenment are valid and force us to rethink and reconstruct
Enlightenment philosophy for the present age. And while we agree with Habermas that a reconstruction of the
Enlightenment and modernity are in order, unlike Habermas we believe that postmodern theory has
important contributions to make to this project)
Identity Politics Fail

Identity politics fail – it forces a splinter into sub-groups, preventing any possible positive change
Best and Kellner 02 prof phil @ UT el paso prof phil @ UCLA (Steven, Doug, “Postmodern Politics and the Battle for the Future”
http://www.gseis.ucla.edu/faculty/kellner/Illumina%20Folder/kell28.htm)

Various forms of postmodern politics have been liberatory in breaking away from the abstract and
ideological universalism of the Enlightenment and the reductionist class politics of Marxism, but they tend to be
insular and fragmenting, focusing solely on the experiences and political issues of a given group,
even splintering further into distinct subgroups such as divide the feminist community. Identity
politics are often structured around simplistic binary oppositions such as Us vs. Them and Good vs.
Bad that pit people against one another, making alliances, consensus, and compromise difficult or impossible.
This has been the case, for example, with tendencies within radical feminism and ecofeminism
which reproduce essentialism by stigmatizing men and "male rationality" while exalting women
as the bearers of peaceful and loving value and as being "closer to nature."[18] Elements in the black
nationalist liberation movement in the 1960s and the early politics of Malcolm X were exclusionist and racist,
literally demonizing white people as an evil and inferior race. Similarly, the sexual politics of some gay and
lesbian groups tend to exclusively focus on their own interests, while the mainstream environmental
movement is notorious for resisting alliances with people of color and grass roots movements.[19]
Even though each group needs to assert their identity as aggressively as possible, postmodern
identity politics should avoid falling into seriality and sheer fragmentation. These struggles, though
independent of one another, should be articulated within counterhegemonic alliances, and attack power formations
on both the micro- and macro-levels. Not all universalistic appeals are ideological in the sense criticized by Marx;
there are common grounds of experience, common concerns, and common forms of oppression that different groups
share which should be articulated -- concerns such as the degradation of the environment and common forms of
oppression that stem from capitalist exploitation and alienated labor
Realism Good

Abandoning realism risks a transition to fascism


Mearsheimer ‘95John J., professor of political science at the University of Chicago. “The False Promise of International Institutions.”
International Security, Vol. 19, No. 3., Winter 94/95.

Nevertheless, critical theorists occasionally point to particular factors that might lead to changes in
international relations discourse. In such cases, however, they usually end up arguing that changes in the
material world drive changes in discourse. For example, when Ashley makes surmises about the future of
realism, he claims that "a crucial issue is whether or not changing historical conditions have
disabled longstanding realist rituals of power." Specifically, he asks whether "developments in late
capitalist society," like the "fiscal crisis of the state," and the "internationalization of capital," coupled with "the
presence of vastly destructive and highly automated nuclear arsenals [has] deprived statesmen of the latitude for
competent performance of realist rituals of power?" (157) Similarly, Cox argues that fundamental change occurs
when there is a "disjuncture" between "the stock of ideas people have about the nature of the world and the practical
problems that challenge them." He then writes, "So me of us think the erstwhile dominant mental construct of
neorealism is inadequate to confront the challenges of global politics today." (158)
It would be understandable if realists made such arguments, since they believe there is an
objective reality that largely determines which discourse will be dominant. Critical theorists,
however, emphasize that the world is socially constructed, and not shaped in fundamental ways
by objective factors. Anarchy, after all, is what we make of it. Yet when critical theorists attempt to explain why
realism may be losing its hegemonic position, they too point to objective factors as the ultimate cause of change.
Discourse, so it appears, turns out not to be determinative, but mainly a reflection of developments in the objective
world. In short, it seems that when critical theorists who study international politics offer glimpses of their thinking
about the causes of change in the real world, they make arguments that directly contradict their own theory, but
which appear to be compatible with the theory they are challenging. (159)
There is another problem with the application of critical theory to international relations. Although critical
theorists hope to replace realism with a discourse that emphasizes harmony and peace, critical
theory per se emphasizes that it is impossible to know the future. Critical theory according to its
own logic, can be used to undermine realism and produce change, but it cannot serve as the basis
for predicting which discourse will replace realism, because the theory says little about the
direction change takes. In fact, Cox argues that although "utopian expectations may be an element in
stimulating people to act...such expectations are almost never realized in practice." (160)
Thus, in a sense, the communitarian discourse championed by critical theorists is wishful thinking,
not an outcome linked to the theory itself. Indeed, critical theory cannot guarantee that the new
discourse will not be more malignant than the discourse it replaces. Nothing in the theory
guarantees, for example, that a fascist discourse far more violent than realism will not emerge as
the new hegemonic discourse.
Realism Good

Only realism has empirical support – other theories are simply not supported in practice
Mearsheimer ’95 John J., professor of political science at the University of Chicago. “The False Promise of International Institutions.”
International Security, Vol. 19, No. 3., Winter 94/95.

Three points are in order regarding the critical theorists' interpretation of history. First ,
one cannot help but be
struck by the sheer continuity of realist behavior in the critical theorists' own account of the past.
Seven centuries of security competition and war represents an impressive span of time,
especially when you consider the tremendous political and economic changes that have taken
place across the world during that lengthy period. Realism is obviously a human software
package with deep-seated appeal, although critical theorists do not explain its attraction.
Second, a close look at the international politics of the feudal era reveals scant support for the
claims of critical theorists: Markus Fischer has done a detailed study of that period, and he finds "that feudal
discourse was indeed distinct, prescribing unity, functional cooperation, sharing, and lawfulness." (166) More
importantly, however, he also finds "that while feudal actors observed these norms for the most part on the level
of form, they in essence behaved like modern states." Specifically, they "strove for exclusive
territorial control, protected themselves by military means, subjugated each other, balanced
against power, formed alliances and spheres of influence, and resolved their conflicts by the use
and threat of force." (167) Realism, not critical theory, appears best to explain international
politics in the five centuries of the feudal era.
Third, there are good reasons to doubt that the demise of the Cold War means that the millennium
is here. It is true that the great powers have been rather tame in their behavior towards each other
over the past five years. But that is usually the case after great-power wars. Moreover, although
the Cold War ended in 1989, the Cold War order that it spawned is taking much longer to
collapse, which makes it difficult to determine what kind of order or disorder will, replace it. For
example, Russian troops remained in Germany until mid-1994, seriously impinging on German
sovereignty, and the United States still maintains a substantial military presence in Germany. Five
years is much too short a period to determine whether international relations has been
fundamentally transformed by the end of the Cold War, especially given that the "old" order of
realist discourse has been in place for at least twelve centuries.
Realism Good

Trying to infuse states with ethical norms is hopeless – the anarchic international system
will prevent them from going along
Mearsheimer ‘95
John J., professor of political science at the University of Chicago. “The False Promise of International Institutions.” International Security, Vol.
19, No. 3., Winter 94/95.

For critical theorists, the key to achieving a "postmodern international system" is to alter state
identity radically, or more specifically, to transform how states think-about themselves and their
relationship with other states. (139) In the jargon of the theory, "intersubjective understandings and
expectations" matter greatly. (140) In practice, this means that states must stop thinking of themselves
as solitary egoists, and instead develop a powerful communitarian ethos. (141) Critical theorists
aim to create an international system characterized not by anarchy, but by community. States
must stop thinking of themselves as separate and exclusive--i.e., sovereign--actors, and instead
see themselves as mutually conditioned parts of a larger whole. (142) States, or more precisely,
their inhabitants and leaders, should be made to care about concepts like "rectitude," "rights,"
and "obligations." In short, they should have a powerful sense of responsibility to the broader international
community.
*** Postmodernism ***

PoMo Bad General

Postmodernism collapses into meaninglessness when we attempt to apply pomo to real


world problems. Their relativism has value has value only in an intellectual vacuum.
Christopher Butler, Sept. 5, 2K5
[“The Postmodern Vacuum,” http://christopherbutler.wordpress.com/2005/09/05/the-postmodern-vacuum/, The Invisible Things, Articles in
Apologetics, accessed 3-10-2007, JT]

According to postmodern theorist Jean-François Lyotard, postmodernism is characterized by its ‘incredulity


toward metanarratives.’ The late literary theorist and philosopher Jacques Derrida concurred, and argued that knowledge
could not maintain integrity without invoking an ‘original utterance,’ the logos. The idea that man would be given or have
access to the ‘original utterance’ is thus called logocentrism. Critics of postmodern thought would rightly agree and point out that
without being grounded in an objective standard, postmodernism can masquerade as philosophy
without having to account for the logical disparities that so clearly exist when such a system
attempts to be practically applied. The Christian, regardless of his philosophical or logical capabilities in argument,
should be expectedly and unabashedly logocentric. The Word of God, as preserved in the Bible is the first and last word- the
source of the metanarrative from which we presume objective morality, elitism of ideas, and the convictions of the individual.
This is fundamental to Christian theology and has clear and logical implications upon forming a Christian worldview. As
observed by a Wikipedia author, ‘Many of these critiques attack, specifically, the perceived "abandonment of objective truth" as
being the crucial unacceptable feature of the post-modern condition.’ This is absolutely the case, for there can be no
compatibility between a philosophy reliant upon the reality of objective truth and a philosophy that elevates agnosticism to its
objective truth (essentially, to state that truth does not exist and defend such a statement as being true).
We have all encountered postmodernism in its popular form of relativism, an attitude reflected in
a phrase at this point cliché: ‘Truth is relative.’ Yet it is such a pronouncement which pulls the carpet from
underneath itself! If it is so, then that must include the statement itself, thereby invalidating it. If it is not so, then such a statement
is meaningless in its inability to adequately reflect reality. One who claims that truth is relative speaks into an
intellectual vacuum; he can neither generate worthwhile response, nor express anything truly
meaningful. This is the bankruptcy of postmodern thought, and yet Alan Bloom, in his book, The Closing of
the American Mind, affirms the state of things as he writes, ‘Every professor on a university campus today can be absolutely sure
of one thing: that almost every student coming in for an education is confident that truth is not absolute but relative.’

Postmodernism is a coping mechanism for global capitalism


Steve Mizrach, 2K7
[“Talking pomo: An analysis of the postmodern movement,” http://www.fiu.edu/~mizrachs/pomo.html, accessed 3-10-2007, JT]

postmodernism is a diversionary
Critics of postmodernism come mainly from the Marxist camp. They feel that
tactic, the last ditch of a late capitalism in the process of dying. They dislike fervently the way that
postmodern aesthetics rejects socialist realism - and, for that matter, epistemological realism.
They often point out how semiotics and the postmodern idea that everything is image and nothing is
substance are used cynically by advertising agencies - which, unable to sell us real goods of real
production, can now only sell us images of satisfaction and packaged happiness.
PoMo Bad General

The critique locks us in an endless cycle of debate and prevents any action
Jarvis in ’02 Darryl S.L., “International Relations and the ‘third debate,’” Praeger Publishers, copyright 2002

Debate is generally considered to be at the heart of intellectual life. It is the dynamic that allows scholars to
locate themselves intellectually within their field of study, to express opinions within a communal environment, and,
in the best of all possible worlds, to function as a catalyst for the generation of new insights and ideas. Without
debate, intellectual life would wither away.
The study of international politics is no different in this regard. It flourishes when debate flourishes and
is all the better for a plurality of ideas that are freely expressed and critically explored. Often, however, debates
are cast in such a way that there appear to be only two options: a right or wrong answer dependent
on the side of the fence where one is sitting. The science/relativism issue is a good example. Neorealists cast
relativists in a negative light, painting them as an evil nemesis only too willing to destroy the fabric of
academic life. In contrast, relativists flaunt their antipathy toward science and adopt relativism as a
way of gaining maximal leverage against their opponents. Importantly, however, despite their
antagonism, the relativists entrap themselves in a hermeneutic logic, failing to transcend the cyclic
structure of tile debate. Criticism toward neorealism becomes simply opposition to neorealism.
In this case, debate becomes bogged down in a cycle of affirmation and repudiation as scholars
line up according to the side that best captures their loyalties and beliefs.
The lesson to be learned from this is that extreme viewpoints generate, and tend to perpetuate, equally
extreme discourses. The absolute faith that both sides have in the "truth" of their own views
makes it impossible for them to see any merit in the views of others, fostering the sort of radical
repudiationism we see in the work of Justine Rosenberg and Jim George, among others. What these theorists fail
to understand is that the way they frame their criticisms also determines how others respond to them
and suggests that transcending the cyclic logic so indicative of contemporary theoretical debates
in International Relations requires considerably more precision, elegance, and ingenuity than
displayed by George.
Unfortunately, this oppositional logic
informs much of the debate about realism, creating a kind of
epistemological dualism that entraps theorists in a(n) seemingly inescapable cycle of affirmation
and repudiation. The strengths and weaknesses of each approach become mostly superfluous to the process, and
tribal loyalty to one creed over another becomes the nexus around which debate and discourse are
conducted. Ideological battle replaces theoretical discourse with each side painting its opponent
in the worst possible light while reinforcing the legitimacy of its own position by discrediting
that of the opponent(s). There are, then, limits to what can be accomplished with criticism,
highlighting that ideologies and political positions are not held solely as a result of their logic or elegance.
PoMo Bad General

The alt fails, must engage in political structures to solve practical problems
Jarvis in ‘02
Darryl S.L., “International Relations and the ‘third debate,’” Praeger Publishers, copyright 2002

But whatever one says about radical human agency and of its prospects for liberation, in practice emancipation
will involve some degree of social engineering: inequalities have to be corrected, wrongs made
right, and injustices corrected. The agents of global change, whoever they might be, will have to
force some individuals and groups to do their bidding. In the end, legislative reform and the forced
direction of groups and individuals are unavoidable realities. Moreover, in situations where
entrenched cultural and historical values collide, and this is a likely possibility from a theory that seeks
to "help others speak for themselves," we might reasonably expect some degree of violence and
have to tolerate it. George, however, refuses to explore these probabilities. Do the advocates of
postmodern values, for example, take up arms against those who are unwilling to let "others" speak
for themselves? If they do not, then their case has no real teeth. But if they do, they must, at
some stage, sanction the use of force. This is a conundrum endemic to the theoretical architecture
of postmodernism, and one George fails to tackle, indeed is reticent even to acknowledge. Clearly, however,
George wants to defend the proposition that his "new world order" will be less dangerous than the
new/old one of George Bush senior, the Clinton administration, or of George Bush junior and the realists. But,
again, he fails to demonstrate how his version of postmodernism can prevent the intrusion and
corruption of its schema by violence or else justify the use of violence in pursuit of those ends he
otherwise champions. He does neither.
PoMo Bad General

The critique fails; it ignores the reality and permanence of political institutions. At best,
their alt is wishful thinking.
Jarvis in ‘02
Darryl S.L., “International Relations and the ‘third debate,’” Praeger Publishers, copyright 2002

Moreover, why should we assume that states and individuals want to listen and will listen to what the marginalized
and the oppressed have to say? There is precious little evidence to suggest that "listening" is something the advanced
capitalist countries do very well at all. Indeed, one of the allegations so forcefully alleged by Muslim
fundamentalists as justification for the terrorist attacks of September 11 is precisely that the West, and America in
particular, are deaf to the disenfranchised and impoverished in the world. Certainly, there are agencies and
individuals who are sensitive to the needs of the "marginalized" and who champion institutional forums where
indigenous voices can be heard. But on even the most optimistic reckoning, such forums and institutions represent
the exception, not the rule, and remain in the minority if not dwarfed by those institutions that represent Western,
first world interests. To be sure, this is realist power-political image of the current configuration of the global polity, but one apparently, and
ironically, endorsed by George if only because it speaks to the realities of the marginalized, the imposed silences, and the multitude of
oppressions on which George founds his call for a postmodern ethic.
Recognizing such realities, however, does not explain George's penchant for ignoring them entirely, especially in terms of the structural rigidities
they pose for meaningful reform. Indeed, George's desire to move to a new "space beyond International Relations" smacks of
wishful idealism, ignoring the current configuration of global political relations and power distribution; of the
incessant ideological power of hyperindividualism, consumerism, advertising, Hollywood images, and fashion
icons; and of the innate power bestowed on the (institutional) barons of global finance, trade, and transnational
production. George seems to have little appreciation of the structural impediments such institutions pose for radical
change of the type he so fiercely advocates. Revolutionary change of the kind desired by George ignores that fact
that many individuals are not disposed to concerns beyond their family, friends, and daily work lives. And
institutional, structural transformation requires organized effort, mass popular support, and dogged single
mindedness if societal norms are to be challenged, institutional reform enacted, consumer tastes altered, and political
sensibilities reformed. Convincing Nike that there is something intrinsically wrong with paying Indonesian workers
a few dollars a week to manufacture shoes for the global market requires considerably more effort than postmodern
platitudes and/or moral indignation. The cycle of wealth creation and distribution that sees Michael Jordan receive
multimillion dollar contracts to inspire consumer demand for Nike products, while the foot soldiers in the factory
eke out a meager existence producing these same products is not easily, or realistically challenged by
pronouncements of moving beyond International Relations to a new, nicer, gentler nirvana.

The critique prevents action


Jarvis in ‘02
Darryl S.L., “International Relations and the ‘third debate,’” Praeger Publishers, copyright 2002
In any period of political or intellectual upheaval it is commonplace to hear pronouncements of the death of the old and the birth
of the new, of the clear superiority of the latest ideology, moral code, revolutionary government, philosophical system, or theory.
Western intellectual history is littered with such examples. The need for a "new beginning" is a constant refrain in intellectual life,
and the logic behind such calls is often seductive and compelling. The past is a repository of flawed ideas, dangerous moral and
social codes, superstitions and illusions, and a millstone around the neck of the present. To continue to privilege and idealize the
past not only distorts present thinking and retards practice, but hampers our ability to deal effectively with current and future
political problems. By casting off the chains of the past, we will come to see the present in a new light and look forward to a
better, brighter future. At the heart of views of this sort is an overwhelming desire for liberation and freedom. But reality is rarely
so clear-cut.
PoMo Bad General

Critique is contradictory and replicates same problems it attempts to solve.


Jarvis in ‘02
Darryl S.L., “International Relations and the ‘third debate,’” Praeger Publishers, copyright 2002

First, why should one accept the central tenet of postmodernism that all cognitive and ethical claims are merely
made in relation to incommensurable metanarratives? The mere fact of disagreement among narratives or
worldviews shows only that different people have held different beliefs; it does not show that all such beliefs are
equally valid or that none has any better foundation than another. Reason and evidence, in fact, better support the
claims of modem science than of premodern folk wisdom. The standard postmodernist reply is that the "reason and
evidence" appealed to here are only those of the modern, scientific metanarrative and that there are no universal
standards of reason and truth to which one can appeal beyond that particular point of view. But the sort of truth
claims and privileging moves that postmodernists condemn in their opponents are claims and
moves they make themselves. Indeed, it is hard to see how they or anyone else can avoid presupposing such
notions of truth and its implications. Thus, so far as postmodernism implies the advent of a postmodern
historical epoch characterized by a certain attitude to what has gone before. it relies inescapably
on a metanarrative of its own. In its contempt for, and reduction of, all claims of truth and justice
to claims of interest and power, that metanarrative is more thoroughly totalizing than most
Enlightenment views--with the possible exception of Marxism, its intellectual precursor in this regard. The very
notion of the "unmasking" of power presupposes a conception of truth, namely, that there truly is
something to be unmasked. In short, postmodernism displays a deeply self-contradictory
character that it shares with other forms of relativism: it relativizes all claims except its own.
PoMo Bad – Public Sphere

Their alternative crushes social engagement in the public sphere


Boggs ‘97(Carl, Professor of Political Science – National University, Theory & Society 26, December, p. 766-68)

Postmodernism and its offshoots (poststructuralism, semiotics, difference feminism, etc.) have indeed reshaped much of academia,
including such disciplines as sociology, history, literature, film, and communications. More than that, the theory (if that is the correct label for some- thing so diffuse)
amounts to a kind of anti-paradigm paradigm, which often refocuses debates around defining motifs of the post-Fordist order: commodification of culture, the media
spectacle, proliferation of images and symbols, fragmentation of identities, the dispersion of local movements, and loss of faith in conventional political ideologies
and organizations. So far as all this is concerned, post-modernism can be viewed as marking a rather healthy break with the past.50 The problem is that the main thrust
of postmodernism so devalues the common realm of power, governance, and economy that the dynamics of social and
institutional life vanish from sight. Where the reality of corporate, state, and military power wind up vanishing within a post-
modern amorphousness, the very effort to analyze social forces and locate agencies or strategies of change becomes impossible.
In its reaction against the comprehensive historical scope of Marxism, the micro approach dismisses in toto macropolitics and
with it any conceivable modern project of radical transformation. An extreme ``micro'' focus is most visible in such theorists
as Baudrillard who, as Steven Best and Douglas Kellner put it, in effect ``announce the end of the political project in the end of
history and society''51 – a stance that replicates the logic of a profoundly depoliticized culture. Postmodern theory has been
interpreted as a current fully in sync with the mood of political defeat that has overcome the left in most industrialized countries
since the early 1980s.52 It is hardly coincidental that postmodernism grew into an academic fashion in the wake of failed hopes
after the sixties and the later decline of popular movements in the face of a rising conservative hegemony. The crisis of Marxism and the
disintegration of Communist regimes in Eastern Europe further intensifed feelings of resignation on the left. The new middle strata that was the backbone of the new
left and new social movements turned in larger numbers toward careers and more affluent lifestyles. Radicalism, where it persisted to any extent, took on the veneer of
an ``aesthetic pose.'' Thus, at a time of mounting pessimism and retreat, the rhetorical question posed by Alex Callinicos scarcely demands an answer: ``What political
subject does the idea of a postmodern epoch help constitute?''53 By the 1990s even the discussion of political subjectivity or agency among leftist academics seemed
rather passe¨ .54 In politics as in the cultural and intellectual realm, a postmodern fascination with indeterminacy, ambiguity, and chaos easily
supports a drift toward cynicism and passivity; the subject becomes powerless to change either itself or society. Further, the
pretentious, jargon-filled, and often indecipherable discourse of postmodernism reinforces the most faddish tendencies in
academia. Endless (and often pointless) attempts to deconstruct texts and narratives readily become a facade behind which
professional scholars justify their own retreat from political commitment. In Russell Jacoby's words: ``At the end of the radical theorizing
project is a surprise: a celebration of academic hierarchy, professions, and success. Never has so much criticism yielded so much affirmation. From Foucault the
professor learned that power and institutions saturate everything. Power is universal; complicity with power is universal, and this means university practices and
malpractices are no better or worse than anything else.''55 While multiple sites of power and resistance need to be more clearly theorized than in the past, and while
Marxian fixation on class struggle, the primacy of capital-labor relations, and social totality has lost its rationale, the extreme postmodern assault on macro
institutions severs the connection between critique and action. Moreover, to the extent that postmodernism embraces a notion of
subjectivity that is decentered and fragmented, the very idea of citizenship gets obscured. As Philip Wexler argues, the social,
legal, and political requirements of citizenship were historically founded upon universal norms of democracy, freedom, and
equality, but postmodernism, which blurs everything and dissolves politics into the sphere of culture and everyday life, destroys
this foundation. Once the subject melts into a murky cultural diffuseness, into a world of images and spectacles, the elements of
citizenship simply evaporate.56 Various democratic ideals may be kept alive within the official ideology, mainly to legitimate
the electoral ritual, but they fail to resonate with the times. As Wexler concludes: ``For now, citizen- ship will remain the
appropriate sign of post-modernism and semiotic society – a restored sign artifact that may be recycled and used so long as it
does not disturb contemporary society's profound need for superficiality.''57 In the splintered, discontinuous world inhabited by
Baurdrillard, Foucault, and kindred theorists, social bonds are weakened and the link between personal life and the public sphere
is fractured. Where truth, language, and ideology are perpetually contested, nothing is settled or taken for granted. While this
ethos corresponds well to an era in which emphasis is placed on local knowledge and identity movements, it is a depoliticizing
ethos insofar as it blurs or dismisses macro forms of economic and political power. Where the state is either ignored or broken
down into a mosaic of localized and partial entities, politics too winds up obliterated. Symbols and images become far more
important than concrete struggles involving rival claims to power, economic interests, and visions of a better society.58 In a
social order where symbols and images dominate mass consciousness, the splintering of local identities coincides with the decline
of political opposition. Corporate colonization is left only feebly challenged by the proliferation of local groups, by the
celebration of diversity and multiculturalism that has entered into American public discourse since the 1980s. Dispersed
identities, however constructed, are easily assimilated into the sphere of the all-powerful commodity, which coincides with the
spread of anti-political sentiment. As communities assume what Zygmunt Bauman calls an ``imaginary'' character,59 identities
become detached from the public sphere, and politics is allowed to descend into a spectacle. Hence the eclipse of the collective
subject and the atrophy of political language that defines so much postmodern theorizing is now linked more and more to the
stubborn reality of corporate domination.
PoMo Bad – Public Sphere

Extinction results ---- only robust engagement can check elite influences that intensify all
violence
Boggs ‘97(Carl, Professor of Political Science – National University, Theory & Society 26, December, p. 773-4)

The decline of the public sphere in late twentieth-century America poses a series of great dilemmas and
challenges. Many ideological currents scrutinized here ^ localism, metaphysics, spontaneism, post- modernism, Deep
Ecology – intersect with and reinforce each other. While these currents have deep origins in popular movements of the 1960s and
1970s, they remain very much alive in the 1990s. Despite their different outlooks and trajectories, they all share one thing
in common: a depoliticized expression of struggles to combat and overcome alienation. The false
sense of empowerment that comes with such mesmerizing impulses is accompanied by a loss of public
engagement, an erosion of citizenship and a depleted capacity of individuals in large groups to work
for social change. As this ideological quagmire worsens, urgent problems that are destroying the fabric of
American society will go unsolved – perhaps even unrecognized – only to fester more ominously into the
future. And such problems (ecological crisis, poverty, urban decay, spread of infectious diseases,
technological displacement of workers) cannot be understood outside the larger social and global context of
internationalized markets, finance, and communications. Paradoxically, the widespread retreat from politics,
often inspired by localist sentiment, comes at a time when agendas that ignore or side- step these global
realities will, more than ever, be reduced to impotence. In his commentary on the state of citizenship today, Wolin refers to
the increasing sublimation and dilution of politics, as larger numbers of people turn away from public concerns toward private
ones. By diluting the life of common involvements, we negate the very idea of politics as a source
of public ideals and visions.74 In the meantime, the fate of the world hangs in the balance. The unyielding truth
is that, even as the ethos of anti-politics becomes more compelling and even fashionable in the United States, it is the
vagaries of political power that will continue to decide the fate of human societies. This last point
demands further elaboration. The shrinkage of politics hardly means that corporate colonization will be
less of a reality, that social hierarchies will somehow disappear, or that gigantic state and military
structures will lose their hold over people's lives. Far from it: the space abdicated by a broad citizenry,
well-informed and ready to participate at many levels, can in fact be filled by authoritarian and reactionary elites – an
already familiar dynamic in many lesser- developed countries. The fragmentation and chaos of a Hobbesian world, not
very far removed from the rampant individualism, social Darwinism, and civic violence that have been so much a part of the
American landscape, could be the prelude to a powerful Leviathan designed to impose order in the
face of disunity and atomized retreat. In this way the eclipse of politics might set the stage for a
reassertion of politics in more virulent guise – or it might help further rationalize the existing power structure. In either case,
the state would likely become what Hobbes anticipated: the embodiment of those universal, collective interests that had vanished
from civil society.75
Ext – PoMo Jacks Public Engagement

Postmodernism cements depoliticization and cements corporate colonization


Boggs, 2000 (Carl, “The End of Politics, Corporate Power and the Deline of the Public Sphere” pg 220-221)

In the splintered, discontinuous universe theorized by Foucault, Baudrillard, Richard Rorty, and kindred writers,
social coherence is weakened and the linkage between personal and public, micro and macro, and
local and global is fractured-a devastating turn for politics, especially radical politics, in an age of
globalization. Further, where truth, language, and social referents are so tenaciously contested as to dissolve into
limitless interpretations of what constitutes basic social trends in the world, where nothing is ever settled, no strategy
for change is even thinkable. While this perspective may correspond well to a milieu in which greater attention is
devoted (perhaps rightly) to local knowledge and concerns, it is nonetheless depoliticizing insofar as it tends
to obscure what in fact needs to be retheorized-that is, macro levels of economic and political
power. Where the state system, for example, is devalued or broken down into a mosaic of dispersed and partial
entities, politics too ends up obliterated. Oddly enough for a discourse that embodies such radical pretensions, the
whole realm of symbols and images-central to what Norman Denzin describes as the "cinematic age"30-becomes far
more important than concrete struggles around rival claims to power, economic interests, and visions of a better
society. Despite its critical and oppositional language, therefore, postmodernism is actually system-
reproducing in its celebration of fragmented, localized, and (occasionally) privatized discourses; it fits the
imperatives of corporate colonization, partly because it reflects an ethos of public disintegration and
partly because, in its extreme formulations, it gives rise to a disempowering nihilism.
PoMo Bad – Best + Kellner

Exclusive emphasis on postmodern micro-politics misses the key sources of power --- only
multi-faceted coalitional politics prevent degeneration into narcissism and defeat
Best and Kellner ‘01(Steven, Prof Philosophy, UT El Paso and Douglass, Philosophy Chair, Postmodern Politics and the Battle
for the Future, Democracy and Nature: The International Journal of Inclusive Democracy, Vol. 7, No. 1)
The emphasis on local struggles and micropower, cultural politics which redefine the political, and attempts to develop political
forms relevant to the problems and developments of the contemporary age is extremely valuable, but there are also certain
limitations to the dominant forms of postmodern politics. While an emphasis on micropolitics and local struggles can be a healthy
substitute for excessively utopian and ambitious political projects, one should not lose sight that key sources of political power
and oppression are precisely the big targets aimed at by modern theory, including capital, the state, imperialism, and patriarchy.
Taking on such major targets involves coalitions and multi-front struggle, often requiring a politics of alliance and solidarity
that cuts across group identifications to mobilize sufficient power to struggle against, say, the evils of capitalism or the state. Thus,
while today we need the expansion of localized cultural practices, they attain their real significance only within the struggle for the transformation of society as a
whole. Without this systemic emphasis, cultural and identity politics remain confined to the margins of society and are in danger of
degenerating into narcissism, hedonism, aestheticism, or personal therapy, where they pose no danger and are immediately
coopted by the culture industries. In such cases, the political is merely the personal, and the original intentions of the 1960s goal to broaden the political
field are inverted and perverted. Just as economic and political demands have their referent in subjectivity in everyday life, so these cultural and existential issues find
their ultimate meaning in the demand for a new society and mode of production. Yet we would insist that it is not a question of micro vs macropolitics, as if it were an
either/or proposition, but rather both dimensions are important for the struggles of the present and future.[15] Likewise, we would argue that we need to combine the
most affirmative and negative perspectives, embodying Marcuse's declaration that critical social theory should be both more negative and utopian in reference to the
status quo.[16] There are certainly many things to be depressed about is in the negative and cynical postmodernism of a Baudrillard,
yet without a positive political vision merely citing the negative might lead to apathy and depression that only benefits the
existing order. For a dialectical politics, however, positive vision of what could be is articulated in conjunction with critical
analysis of what is in a multioptic perspective that focuses on the forces of domination as well as possibilities of emancipation. While postmodern politics and
theory tend to polarize into either the extremely negative or excessively affirmative, key forms of postmodern literature have a more dialectical vision. Indeed, some
of the more interesting forms of postmodern critique today are found in fictional genres such as cyberpunk and magical realism. Cyberpunk, a subgenre within science
fiction, brings science fiction down to earth, focusing not on the intergalactic battles in the distant future, but the social problems facing people on earth in the present.
[17] Cyberpunk writers such as Bruce Sterling and William Gibson offer an unflinching look at a grim social reality characterized by transnational capitalist
domination, Social Darwinist cultural settings, radical environmental ruination, and the implosion of the body and technology, such that humans become more and
more machine like and machines increasingly become like human beings. Yet cyberpunk novels foreground this nightmare world in order to warn us that it is an
immanent possibility for the near future, in order to awaken readers to a critical reflection on technology and social control, and to offer hope for alternative uses of
technology and modes of social life. Similarly, magical realism examines the wreckage of centuries of European colonialism, but also maintains a positive outlook,
one that embraces the strength and creativity of the human spirit, social solidarity, and spiritual and political transcendence. Like cyberpunk novels, magical realism
incorporate various aesthetic forms and conventions in an eclectic mixture that fuses postmodernism with social critique and models of resistance. But it is also a
mistake, we believe, to ground one's politics in either modern or postmodern theory alone. Against one-sided positions, we
advocate a version of reconstructive postmodernism that we call a politics of alliance and solidarity that builds on both modern
and postmodern traditions. Unlike Laclau and Mouffe who believe that postmodern theory basically provides a basis for a new politics, and who tend to reject
the Enlightenment per se, we believe that the Enlightenment continues to provide resources for political struggle today and are
skeptical whether postmodern theory alone can provide sufficient assets for an emancipatory new politics. Yet the Enlightenment
has its blindspots and dark sides (such as its relentless pursuit of the domination of nature, and naive belief in "progress," so we believe that
aspects of the postmodern critique of Enlightenment are valid and force us to rethink and reconstruct Enlightenment philosophy for the present
age. And while we agree with Habermas that a reconstruction of the Enlightenment and modernity are in order, unlike Habermas we believe that
postmodern theory has important contributions to make to this project. Various forms of postmodern politics have been liberatory in breaking
away from the abstract and ideological universalism of the Enlightenment and the reductionist class politics of Marxism ,
but they tend to be insular and
fragmenting, focusing solely on the experiences and political issues of a given group, even splintering further into distinct
subgroups such as divide the feminist community. Identity politics are often structured around simplistic binary oppositions such
as Us vs. Them and Good vs. Bad that pit people against one another, making alliances, consensus, and compromise difficult or
impossible. This has been the case, for example, with tendencies within radical feminism and ecofeminism which reproduce
essentialism by stigmatizing men and "male rationality" while exalting women as the bearers of peaceful and loving value and as being "closer to nature."[18]
Elements in the black nationalist liberation movement in the 1960s and the early politics of Malcolm X were exclusionist and racist, literally demonizing white people
as an evil and inferior race. Similarly, the sexual politics of some gay and lesbian groups tend to exclusively focus on their own interests, while the mainstream
environmental movement is notorious for resisting alliances with people of color and grass roots movements.[19] Even though each group needs to assert their identity
as aggressively as possible, postmodern identity politics should avoid falling into seriality and sheer fragmentation. These struggles,
though independent of one another, should be articulated within counterhegemonic alliances, and attack power formations on
both the micro- and macro-levels. Not all universalistic appeals are ideological in the sense criticized by Marx; there are
common grounds of experience, common concerns, and common forms of oppression that different groups share which should be articulated --
concerns such as the degradation of the environment and common forms of oppression that stem from capitalist exploitation and
alienated labor.
PoMo Bad – Best + Kellner

Postmodernism draws distinctions between different social identities; this legitimates the formation
of hierarchies of power, allowing for class privilege
Kellner prof phil @ UCLA 1998 (Douglas, “Boundaries and Borderlines: Reflections on Jean Baudrillard and
Critical Theory” http://www.uta.edu/huma/illuminations/kell2.htm)

Since contemporary capitalism was producing in effect a new metaphysics and new ideology, philosophical critique
defined as the critique of ideology became an integral part of its social theory. In this context, it is relevant to note
that one of the generally overlooked functions of ideology is to draw false boundaries within such domains as sex,
race, and class, so as to construct ideological divisions between men and women, the "better classes" and "the lower
classes," whites and peoples of color, etc. Ideology constructs divisions between proper and improper behavior,
while constructing a hierarchy within each of these domains which justifies the domination of one sex, race, and
class over others by virtue of its alleged superiority, or the natural order of things. For example, women are said to
be by nature passive, domestic, submissive, etc., and their proper domain is thus deemed to be the private sphere, the
home, while the public sphere was reserved for, allegedly, more active, rational, and domineering men.
In these ideological operations we see abstraction at work: ideologies which legitimate the superiority of men over
women, or of capitalism over other social systems, so as to attempt to justify the privileges of the ruling classes or
strata, -- such patriarchal capitalist ideologies abstract from the injustices, inequities, and suffering produced by
patriarchal capitalism, such as the glaring inequities of power and wealth within a supposedly egalitarian society.
Thus I believe that abstraction is fundamentally related to the key features of ideology such as legitimation,
domination, and mystification, and that the drawing of boundaries (between allegedly inferior and superior systems,
groups, policies, values, etc.) also plays a fundamental role in this process.[1] Boundary maintainence (between men
and women, capitalists and workers, whites and non-whites, Americans and the rest of the world, capitalism and
communism, etc. etc.) serves the interests of social domination, as well as the functions of legitimation and
mystification of social reality. Thus I am proposing that the "distortion," "mystification," "masking," and other
occluding functions usually associated with ideology are related to a certain sort of abstraction and to a specific type
of ideological boundaries.
A2: Deconstruction

Deconstruction creates a political void that will be filled by elites, locking in oppression
Cook ’92 (Anthony, Associate Professor, Georgetown Law, New England LR, Spring, 26 New Eng.L. Rev. 751)

The effect of deconstructing the power of the author to impose a fixed meaning on the text or offer a continuous
narrative is both debilitating and liberating. It is debilitating in that any attempt to say what should be done
within even our insular Foucaultian preoccupations may be oppositionalized and deconstructed as an illegitimate
privileging of one term, value, perspective or narrative over another. The struggle over meaning might continue ad
infinitum. That is, if a deconstructionist is theoretically consistent and sees deconstruction not as a political tool but
as a philosophical orientation, political action is impossible, because such action requires a degree of
closure that deconstruction, as a theoretical matter, does not permit. Moreover, the approach is debilitating because
deconstruction without material rootedness, without goals and vision, creates a political and spiritual void into
which the socially real power we theoretically deconstruct steps and steps on the disempowered and
dispossessed.  [*762]  To those dying from AIDS, stifled by poverty, dehumanized by sexism and racism, crippled
by drugs and brutalized by the many forms of physical, political and economic violence that characterizes our
narcissistic culture, power hardly seems a matter of illegitimate theoretical privileging. When vision,
social theory and political struggle do not accompany critique, the void will be filled by the rich, the
powerful and the charismatic, those who influence us through their eloquence, prestige, wealth and power.
at: deleuze and guattari

The negative’s conception of the alternative and its need to function alone ensures its failure—Deleuze and Guattari
opposed total opposition of the state and the nomad—treating the nomad as a complete outsider is narcissistic and
impossible.

Mann, 95 (Professor of English at Pomona, Paul, “Stupid Undergrounds,” PostModern Culture 5:3, Project MUSE)

Intellectual economics guarantees that even the most powerful and challenging work cannot protect itself from the order of
fashion. Becoming-fashion, becoming-commodity, becoming-ruin. Such instant, indeed retroactive ruins, are the virtual
landscape of the stupid underground. The exits and lines of flight pursued by Deleuze and Guattari are being shut down and
rerouted by the very people who would take them most seriously. By now, any given work from the stupid underground's critical
apparatus is liable to be tricked out with smooth spaces, war-machines, n - 1s, planes of consistency, plateaus and
deterritorializations, strewn about like tattoos on the stupid body without organs. The nomad is already succumbing to the
rousseauism and orientalism that were always invested in his figure; whatever Deleuze and Guattari intended for him, he is
reduced to being a romantic outlaw, to a position opposite the State, in the sort of dialectical operation Deleuze most despised.
And the rhizome is becoming just another stupid subterranean figure. It is perhaps true that Deleuze and Guattari did not
adequately protect their thought from this dialectical reconfiguration (one is reminded of Breton's indictment against Rimbaud for
not having prevented, in advance, Claudel's recuperation of him as a proper Catholic), but no vigilance would have sufficed in
any case. The work of Deleuze and Guattari is evidence that, in real time, virtual models and maps close off the very exits they
indicate. The problem is in part that rhizomes, lines of flight, smooth spaces, BwOs, etc., are at one and the same time
theoretical-political devices of the highest critical order and merely fantasmatic, delirious, narcissistic models for writing, and thus
perhaps an instance of the all-too-proper blurring of the distinction between criticism and fantasy. In Deleuze-speak, the stupid
underground would be mapped not as a margin surrounding a fixed point, not as a fixed site determined strictly by its relation or
opposition to some more or less hegemonic formation, but as an intensive, n-dimensional intersection of rhizomatic plateaus.
Nomadology and rhizomatics conceive such a "space" (if one only had the proverbial nickel for every time that word is used as a
critical metaphor, without the slightest reflection on what might be involved in rendering the conceptual in spatial terms) as a
liquid, colloidal suspension, often retrievable by one or another techno-metaphorical zoning (e.g., "cyberspace"). What is at
stake, however, is not only the topological verisimilitude of the model but the fantastic possibility of nonlinear passage, of multiple
simultaneous accesses and exits, of infinite fractal lines occupying finite social space. In the strictest sense, stupid philosophy.
Nomad thought is prosthetic, the experience of virtual exhilaration in modalities already mapped and dominated by nomad,
rhizomatic capital (the political philosophy of the stupid underground: capital is more radical than any of its critiques, but one can
always pretend otherwise). It is this very fantasy, this very narcissistic wish to see oneself projected past the frontier into new
spaces, that abandons one to this economy, that seals these spaces within an order of critical fantasy that has long since been
overdeveloped, entirely reterritorialized in advance. To pursue nomadology or rhizomatics as such is already to have lost the
game. Nothing is more crucial to philosophy than escaping the dialectic and no project is more hopeless; the stupid-critical
underground is the curved space in which this opposition turns back on itself.
A2: Baudrillard

Baudrillard leaves the masses to collapse, only engaging in macro-politics solves


Best prof phil @ UT el paso and Kellner prof phil @ UCLA 2k2 (Steven, Doug, “Postmodern Politics and the
Battle for the

Future” http://www.gseis.ucla.edu/faculty/kellner/Illumina%20Folder/kell28.htm)
A postmodern politics begins to take shape during the 1960s, when numerous new political groups and struggles emerged. The development of a new postmodern
politics is strongly informed by the vicissitudes of social movements in France, the United States, and elsewhere, as well as by emerging postmodern theories. The
utopian visions of modern politics proved, in this context, difficult to sustain and were either rejected in favor of cynicism, nihilism, and, in some cases, a turn to the
The modern emphasis on collective struggle,
right, or were dramatically recast and scaled down to more "modest" proportions.
solidarity, and alliance politics gave way to extreme fragmentation, as the "movement" of the
1960s splintered into various competing struggles for rights and liberties. The previous emphasis
on transforming the public sphere and institutions of domination gave way to new emphases on
culture, personal identity, and everyday life, as macropolitics were replaced by the micropolitics of local
transformation and subjectivity. In the aftermath of the 1960s, novel and conflicting conceptions of postmodern politics emerged .
Postmodern politics thus take a variety of forms and would include the anti-politics of
Baudrillard and his followers, who exhibit a cynical, despairing rejection of the belief in
emancipatory social transformation, as well as a variety of efforts to create a new or reconstructed politics. On the
extreme and apolitical position of a Baudrillard, we are stranded at the end of history, paralyzed and frozen,
as the masses collapse into inertia and indifference, and simulacra and technology triumph over
agency. Thus, from Baudrillard's perspective, all we can do is "accommodate ourselves to the time left to
us."
A2: Baudrillard

Baudrillard’s radical theory is incapable solving the problem it critiques, only totalizing
solutions are capable of breaking down capitalist structures that are the root cause
Kellner prof phil @ UCLA 1998 (Douglas, “Boundaries and Borderlines: Reflections on Jean Baudrillard and
Critical Theory” http://www.uta.edu/huma/illuminations/kell2.htm)

Against Foucault, Lyotard, and others who reject macro-theory, the category of totality, or meta-narratives , I would argue that precisely
now we need such totalizing theories to attempt to capture the new totalizations being undertaken by capitalism in the realm of
consumption, the media, information, etc. Now, more than ever, we need macro-theories that will attempt to cognitively map the
context of the new forms of social develoment and the relationships between spheres like the economy, culture, education, politics, etc.
Furthermore, unlike Mark Poster (forthcoming) and others, I believe that it is a mistake to sever the mode of information from the mode of
production, and believe that there continues to be "determination in the last instance" by the economic in the current stage of capitalism. Thus I would propose that
the new social conditions, new technological developments, and new political challenges should be conceptualized in terms of a
theory of techno-capitalism rather than postmodernism. With Fredric Jameson (1984), I would propose that we are currently in a
new configuration of capitalism where postmodernism can be read as the cultural logic of capital but where the hegemony of
capital is still the fundamental principle of social organization and where capital attempts to control ever more domains of life. I
would, however, agree with those who claim that we need to rethink the problematics of radical politics, of socialism or even radical
social transformation or emancipation, in the light of the new social conditions and challenges -- though I shall not address this issue here Yet against a radical
implosive postmodernism such as one finds in Baudrillard -- and Arthur Kroker's and David Cook's The Postmodern Scene
(1986) is an even more extreme case -- I would argue for the need to draw boundaries, or conceptual distinctions, and to make what Marx calls
"rational abstractions" rather than leaping into the delirious postmodern implosion of all boundaries, abstractions, and distinctions in the
vertiginous flux of the hyperreal. As Wittgenstein and Derrida attacked metaphysical abstractions which dissolved differences in unifying
schemes, we should undertake to criticize ideological-metaphysical abstractions yet should also draw distinctions which make connections and
which conceptualize important differences. As Marx put it in his introduction to the Grundrisse: "It might seem, therefore, that in order to talk
about production at all we must either pursue the process of historic development through its different phases, or declare beforehand that we are
dealing with a specific historical epoch such as e.g. modern bourgeois production, which is indeed our particular theme. However, all epochs of
production have certain common traits, common characteristics. Production in general is an abstraction, but a rational abstraction insofar as it
really brings out and fixes a common element" (i.e. in different modes of production). Yet Marx goes on to insist that: "Still, this general
category, this common element sifted out by comparison, is itself segmented many times over and splits into different determinations. Some
determinations belong to all epochs, others only to a few" (Marx 1976, p. 106). Thus, while there are general "determinations valid for production
as such," and so for all epochs, we must not, he warns, obscure "their essential differences" (Marx 1976, p. 101). Consequently, for Marx
"rational abstraction" fixes a "common element" that plays a constitutive role in various situations and contexts. The "concrete" in this analysis is
itself a product of many determinations, many relations, and "rational abstraction" thus designates specific determinations in a multiple and multi-
dimensional relational chain. "Bad abstraction" is thus overcome by situating abstractions back into a specific set of differential relations,
contextualizing one's concepts and analyses within a set of historically specific and complex social relations. This is what, I maintain, we need to
do in the postmodernism debate: we should grasp the differences between the old and the new stages of society (or art,
philosophy, etc.), and the continuities between the previous and new stage of society -- a continuity constituted precisely by the continuing
primacy of capitalist relations of production in the current organization of society.[4] Thus, against postmodernists who celebrate the radically "new" -- and
rupture, discontinuity, and difference, -- I would argue that we need to characterize both the continuities and the discontinuities in
the historical process and that this involves both pointing to ruptures and breaks in recent history as well as continuities.
Consequently, while New French Theory has attempted to cross the borderline and to chart out the terrain of the new, their claims
for an absolute break between modernity and postmodernity are not always convincing. Although we may be living within a
borderline, or transitional space, between the modern and the postmodern, and may be entering a terrain where old modes of thought and language are not always useful , it seems at
this point in time that in many ways, New French Theory is itself flawed and not of much use in helping us to understand and
resolve many of the crucial theoretical and political problems that we currently face (i.e. moving beyond the current age of conservative
hegemony, learning to use and live with new technologies in ways that will enhance human life, and understanding and dealing with a wide range of social problems
from technological unemployment to AIDS). Thus while we clearly need new theories and politics to understand the conflicts, problems, and
developments of the contemporary era I believe that we need new concatenations of Marxism, Critical Theory, and New French
theory to solve the theoretical and political problems which confront us today.
A2: Baudrillard

Baudrillard is wrong --- his theory rests on untenable premises and ignores the important
role that capitalism plays in the creation of the media
Kellner prof phil @ UCLA 1998 (Douglas, “Boundaries and Borderlines: Reflections on Jean Baudrillard and
Critical Theory” http://www.uta.edu/huma/illuminations/kell2.htm)
It is here, I suggest, on Baudrillard's borderline between the modern and the postmodern, that a critical interrogation of his thought and politics
should begin. On this point, almost every discussion of Baudrillard in English seems to presuppose that he is right, that we
are in something like a postmodern condition, that we have left modernity behind and are in a qualitatively new society where the old categories
and old distinctions no longer hold. Such a vision rests, I believe, partly on wishful thinking and partly on a desire to
differentiate oneself from old-fashioned traditionalists, while positioning oneself as avant-garde. Stronger, in my
forthcoming book on Baudrillard, I shall that his postmodern social theory rests on some shaky theoretical premises,
especially concerning the role of the media, cybernetics and design, and representation and social reproduction in
the contemporary world (Kellner 1989b). Living on the Borderline Consequently, in confronting the differences between Baudrillard and the now
classical Critical Theory, the following issues arise: is Baudrillard correct that we have entered a postmodern society, or are we still stuck in a more stream-lined and
advanced version of the old capitalist society? Are the fundamental boundaries within social theory (between classes, forces and relations of production, Left and
Right, domination and emancipation, representations and reality, etc.) still intact and effective, or have they been superseded and imploded by contemporary social
developments? What is the status of representation, social critique, emancipation, and socialism in the allegedly postmodern world? In short, are Marxism and Critical
Theory still viable enterprises, or have their assumptions and positions been vitiated by contemporary social developments? My own position is that if Marxism and
Critical Theory want to continue to be relevant to the theoretical and political concerns of the present age, they must address the issues advanced by the postmodern
critical social theory today must attempt to theorize the new social
challenge to previous traditions of social theory. This means that
conditions and phenomena analyzed by the postmodernists, and must demonstrate that their categories and theories
continue to be applicable and illuminating in theorizing the new social conditions. This in turn requires rethinking such enterprises
as Marxism and Critical Theory in terms of the new issues posed and the new challenges advanced by the current configurations of the media, consumer and
information societies; by cybernetics and design; by the restructuring of labor and production; by the new configurations of class; and by the new modes of the
colonization of everyday life. The responses of those identified with Critical Theory to New French Theory and the postmodernism debate so far, however, have been
highly defensive and not particularly productive. Habermas has tended to interpret postmodern thinkers under the sign of irrationalism, and has himself continued to
defend modernity and rationalism without always successfully addressing the critiques of modernity, rationalism, and his own work advanced by the postmodernists
and New French Theory (Habermas 1987). Most recent articles on postmodernism and New French Theory in Telos -- which has consistently championed certain
versions of neo-Marxism and Critical Theory in the U.S. over the past two decades -- are primarily hostile, dismissive, and not particularly illuminating (Berman 1984
and Wolin 1984). Their mode of reception is primarily an Adornesque absolute negation rather than a Benjaminian redemptive hermeneutic which would attempt to
appropriate or redeem what is valuable and useful in New French Theory. Indeed, I would maintain that Critical Theory has so far rejected New French Theory
precisely at those points where its own classical theories are most in need of revision and development: i.e. in attempting to theorize new social conditions and
phenomena like the consumer society, media, information, computerization, etc. The classical Critical Theory of the consumer society tends to downplay the
importance of sign-value and the semiological dimension, while its media theories and ideology critique of popular culture often underemphasized the importance of
form, of codes, of the nature and structure of media themselves --precisely the focus of the best of the New French Theory (Kellner 1989a and 1989b). And, finally,
New French Theory has focused on such new phenomena as cybernetics, computerization, the information society, etc. that have appeared since the classical texts of
The
Critical Theory were produced which Critical Theory today must deal with if it is not to become irrelevant to the current problems of the present age.
attempts of New French Theory, however, to conceptualize these new phenomena in terms of a "post," and often
anti-Marxian discourse and framework, however, are highly problematical as is their frequent denunciation of
macro-social theory in favor of micro theory and politics (this is particularly true of Lyotard, Deleuze and Guattari,
and others). It is my view that New French Theorists like Baudrillard, Lyotard and Foucault have made a serious
theoretical and political mistake in severing their work from the Marxian critique of capitalism precisely at a point
when the logic of capital has been playing an increasingly important role in structuring the new stage of society
which I conceptualize as a new stage of capitalism -- capitalism as techno-capital (Kellner 1989a). Indeed, I would
argue that Marxian categories are of central importance precisely in analyzing the phenomena focused on by
Baudrillard and New French Theory: the consumer society, the media, information, computers, etc. For it is
capitalism that is determining what sort of media, information, computers, etc. are being produced and distributed
precisely according to their logic and interests. That is, in techno-capitalist societies, information, as Herbert Schiller
and others have shown, is being more and more commodified, accessible only to those who can pay for it and who
have access to it. Education itself is becoming more and more commodified as computers become more essential to
the process of education, while more and more domains of knowledge and information are commodified and
transmitted through computers (I'm thinking both of computer learning programs which force consumers to buy programs to learn typing,
math, history, foreign languages, etc. as well as modem-programs and firms like Compu-Serve which make access an abundance of information,
entertainment, networking, etc. via computer for those who can afford to pay its per minute information prices).
at baudrillard: cede the political

Baudrillard is just a fashionable source of cynicism—not a political strategy.


Rojek 93 (Chris, Deputy Director, Theory, Culture & Society Centre , Professor of Sociology and Culture at Nottingham Trent
University, Forget Buadrillard? Edited by Chris Rojek, pgs 109)

His lacerating nihilism, his readiness to prick any cause, his devotion to experience for experience s sake, are all recurring tropes
of at least one type of modernism. To be sure, modernism is a multi-faceted concept. Rather than speak of the project of
modernism it is perhaps more accurate to speak o projects of modernism. These projects work around a central dichotomy:
reflecting the order of things and exposing the funda mental disorder of things. In the political realm the keynote projects designed
to reflect the order of things have been (a) providing a theory of liberal democracy which legitimates the operation of he market;
(b) the socialist critiques of capitalism and the plan for the reconstruction of society; and (c) the feminist transformation of the
male order of things. These are all constructive projects. They either aim to give shape to people's lives or they seek to replace
the easing set of politico-economic conditions with a state of affairs that is judged to be superior on rational or moral grounds.
Baudrillard it might be said, traces the dispersal of these projects He relishes being the imp of the perverse, the ruthless
exponent of the disorder of things His work exposes the posturing and circularities of constructive arguments. But in doing this
Baudrillard is not acting as the harbinger of a new postmodern state of affairs. Rather he is treading the well worn paths of one
type of modernist sceptism and excess – a path which has no other destiny than repletion. His message of ‘no future’ does not
transcend the political dilemma of modernism, it exemplifies it.

Baudrillard’s alternative fails to confront real world politics.

Best & Kellner, 98 Department of Philosophy at University of Texas-El Paso, 1998 [Steven & Douglas,
http://www2.cddc.vt.edu/illuminations/kell28.htm, “Postmodern Politics and the Battle for the Future”]

In the aftermath of the 1960s, novel and conflicting conceptions of postmodern politics emerged. Postmodern politics thus take a
variety of forms and would include the anti-politics of Baudrillard and his followers, who exhibit a cynical, despairing rejection of
the belief in emancipatory social transformation, as well as a variety of efforts to create a new or reconstructed politics. On the
extreme and apolitical position of a Baudrillard, we are stranded at the end of history, paralyzed and frozen, as the masses
collapse into inertia and indifference, and simulacra and technology triumph over agency. Thus, from Baudrillard's perspective,
all we can do is "accommodate ourselves to the time left to us." [3]
at baudrillard: Simulation

We do, in fact, know the difference between simulation and reality—the media plays a healthy role in the public sphere.

March, 95 James Marsh, Professor of Philosophy, Fordham University, 95, Critique, Action, and Liberation, pp. 292-293

Such an account, however, is as one-sided or perhaps even more one-sided than that of naive modernism. We
note a residual idealism that does not take into account socioeconomic realities already pointed out such as the
corporate nature of media, their role in achieving and legitimating profit, and their function of manufacturing
consent. In such a postmodernist account is a reduction of everything to image or symbol that misses the
relationship of these to realities such as corporations seeking profit, impoverished workers in these
corporations, or peasants in Third-World countries trying to conduct elections. Postmodernism does not
adequately distinguish here between a reduction of reality to image and a mediation of reality by image. A
media idealism exists rooted in the influence of structuralism and poststructuralism and doing insufficient
justice to concrete human experience, judgment, and free interaction in the world.4 It is also paradoxical or
contradictory to say it really is true that nothing is really true, that everything is illusory or imaginary.
Postmodemism makes judgments that implicitly deny the reduction of reality to image. For example, Poster and
Baudrillard do want to say that we really are in a new age that is informational and postindustrial. Again, to say
that everything is imploded into media images is akin logically to the Cartesian claim that everything is or might
be a dream. What happens is that dream or image is absolutized or generalized to the point that its original
meaning lying in its contrast to natural, human, and social reality is lost. We can discuss Disneyland as
reprehensible because we know the difference between Disneyland and the larger, enveloping reality of
Southern California and the United States.5 We can note also that postmodernism misses the reality of the
accumulation-legitimation tension in late capitalism in general and in communicative media in particular. This
tension takes different forms in different times. In the United States in the 1960s and 1970s, for example, social,
economic, and political reality occasionally manifested itself in the media in such a way that the electorate
responded critically to corporate and political policies. Coverage of the Vietnam war, for example, did help
turn people against the war. In the 1980s, by contrast, the emphasis shifted more toward accumulation in the
decade dominated by the “great communicator.” Even here, however, the majority remained opposed to
Reagan’s policies while voting for Reagan. Human and social reality, while being influenced by and represented
by the media, transcended them and remained resistant to them.6 To the extent that postmodernists are critical of
the role media play, we can ask the question about the normative adequacy of such a critique. Why, in the
absence of normative conceptions of rationality and freedom, should media dominance be taken as bad rather
than good? Also, the most relevant contrasting, normatively structured alternative to the media is that of the
“public sphere,” in which the imperatives of free, democratic, nonmanipulable communicative action are
institutionalized. Such a public sphere has been present in western democracies since the nineteenth century but
has suffered erosion in the twentieth century as capitalism has more and more taken over the media and
commercialized them. Even now the public sphere remains normatively binding and really operative
through institutionalizing the ideals of free, full, public expression and discussion; ideal, legal requirements
taking such forms as public service programs, public broadcasting, and provision for alternative media; and
social movements acting and discoursing in and outside of universities in print, in demonstrations and forms of
resistance, and on media such as movies, television, and radio.7
at baudrillard: Simulation

Relegating human suffering to the realm of simulation is just nihilism, crushing politics.
Kellner, 89 Phil. Chair @ UCLA, 1989, Jean Baudrillard, p. 107-8, Douglas

Yet does the sort of symbolic exchange which Baudrillard advocates really provide a solution to the question of
death? Baudrillard’s notion of symbolic exchange between life and death and his ultimate embrace of nihilism
(see 4.4) is probably his most un-Nietzschean moment, the instant in which his thought radically devalues life
and focuses with a fascinated gaze on that which is most terrible — death. In a popular French reading of
Nietzsche, his ‘transvaluation of values’ demanded negation of all repressive and life- negating values in favor of
affirmation of life, joy and happiness. This ‘philosophy of value’ valorized life over death and derived its values
from phenomena which enhanced, refined and nurtured human life. In Baudrillard, by contrast, life does not
exist as an autonomous source of value, and the body exists only as ‘the caarnality of signs,’ as a mode of
display of signification. His sign fetishism erases all materialjty from the body and social life, and makes
possible a fascinated aestheticized fetishism of signs as the primary ontological reality. This way of seeing
erases suffering, disease, pain and the horror of death from the body and social life and replaces it with
the play of signs — Baudrillard’s alternative. Politics too is reduced to a play of signs, and the ways in
which different politics alleviate or intensify human suffering disappears from the Baudrillardian
universe. Consequently Baudrillard’s theory spirals into a fascination with signs which leads him to
embrace certain privileged forms of sign culture and to reject others (that is, the theoretical signs of
modernity such as meaning, truth, the social, power and so on) and to pay less and less attention to materiality
(that is, to needs, desire, suffering and so on) a trajectory will ultimately lead him to embrace nihilism (see
4.4).
A2: Foucault

Foucault’s rejection of meta-solutions dooms the alternative, it prevents the formation of new
totalizing theories that are essential to breaking down the capitalist structures that are the root
cause of the problem
Kellner prof phil @ UCLA 1998 (Douglas, “Boundaries and Borderlines: Reflections on Jean Baudrillard and
Critical Theory” http://www.uta.edu/huma/illuminations/kell2.htm)
Against Foucault, Lyotard, and others who reject macro-theory, the category of totality, or meta-narratives, I would
argue that precisely now we need such totalizing theories to attempt to capture the new totalizations being
undertaken by capitalism in the realm of consumption, the media, information, etc. Now, more than ever, we need
macro-theories that will attempt to cognitively map the context of the new forms of social develoment and the
relationships between spheres like the economy, culture, education, politics, etc. Furthermore, unlike Mark Poster
(forthcoming) and others, I believe that it is a mistake to sever the mode of information from the mode of
production, and believe that there continues to be "determination in the last instance" by the economic in the current
stage of capitalism. Thus I would propose that the new social conditions, new technological developments, and new
political challenges should be conceptualized in terms of a theory of techno-capitalism rather than postmodernism.
With Fredric Jameson (1984), I would propose that we are currently in a new configuration of capitalism where
postmodernism can be read as the cultural logic of capital but where the hegemony of capital is still the fundamental
principle of social organization and where capital attempts to control ever more domains of life. I would, however,
agree with those who claim that we need to rethink the problematics of radical politics, of socialism or even radical
social transformation or emancipation, in the light of the new social conditions and challenges -- though I shall not
address this issue here

Biopower in contemporary society is an expression of the enhancement of life, not the power to kill
Ojakangus Helsinki Collegium for Advanced Studies, 2k5 (Mika, “Impossible Dialogue on Bio-power” http://www.foucault-studies.com/no2/ojakangas1.pdf)
In  fact,  the  history  of  modern  Western  societies  would  be  quite  incomprehensible  without  taking  into  account  that  there  exists  a  form  o  power  which
refrains  from  killing  but  which  nevertheless  is  capable  of  directing  people’s  lives.  The  effectiveness  of  bio‐power  can  be  seen  lying
precisely  in  that  it  refrains  and  withdraws  before  every  demand  of  killing,  even  though  these  demands
would  derive  from  the  demand  of  justice.  In  bio‐political  societies,  according  to  Foucault,  capital
punishment  could  not  be  maintained  except  by  invoking  less  the  enormity  of  the  crime  itself  than  the
monstrosity  of  the  criminal:  “One  had  the  right  to  kill  those  who  represented  a  kind  of  biological  danger
to  others.”112  However,  given  that  the  “right  to  kill”  is  precisely  a  sovereign  right,  it  can  be  argued  that  the  bio‐political  societies  analyzed  by
Foucault  were  not  entirely  bio‐political.  Perhaps,  there neither  has  been  nor  can  be  a  society  that  is  entirely  bio‐political.  Nevertheless,  the  fact  is
that  present‐day  European  societies  have  abolished  capital  punishment.  In  them,  there  are  no  longer  exceptions.  It  is
the  very  “right  to  kill”  that  has  been  called  into  question.  However,  it  is  not  called  into  question  because  of  enlightened  moral
sentiments,  but  rather  because  of  the  deployment  of  bio‐political  thinking  and  practice.    For  all  these  reasons,  Agamben’s  thesis,
according  to  which  the  concentration  camp  is  the  fundamental  bio‐political  paradigm  of  the  West,  has  to
be  corrected.113  The  bio‐political  paradigm  of  the  West  is  not  the  concentration  camp,  but,  rather,  the
present‐day  welfare  society  and,  instead  of  homo  sacer,  the  paradigmatic  figure  of  the  bio‐political  society
can  be  seen,  for  example,  in  the  middle‐class  Swedish  social‐democrat.  Although  this  figure  is  an  object  –
and  a  product  –  of  the  huge  bio‐political  machinery,  it  does  not  mean  that  he  is  permitted  to  kill  without
committing  homicide.  Actually,  the  fact  that  he  eventually  dies,  seems  to  be  his  greatest  “crime”  against
the  machinery.  (In  bio‐political  societies,  death  is  not  only  “something  to  be  hidden  away,”  but,  also,  as  Foucault  stresses,  the
most  “shameful  thing  of  all”.114)  Therefore,  he  is  not  exposed  to  an  unconditional  threat  of  death,  but  rather  to
an  unconditional  retreat  of  all  dying.  In  fact,  the  bio‐political  machinery  does  not  want  to  threaten  him,
but  to  encourage  him,  with  all  its  material  and  spiritual  capacities,  to  live  healthily,  to  live  long  and  to  live  happily  –  even  when,  in
biological  terms,  he  “should  have  been  dead  long  ago”.115  This  is  because  bio‐power  is  not  bloody  power  over  bare  life  for  its
own  sake  but  pure  power  over  all  life  for  the  sake  of  the  living.  It  is  not  power  but  the  living,  the
condition  of  all  life  –  individual  as  well  as  collective  that  is  the  measure  of  the  success  of  bio‐power.    
A2: Foucault

Engaging liberalism is essential to breaking down biopower, it promotes a form of


rationality that limits state power
Lacombe Criminology Simon Fraser U 1996 (Danny, “Reforming Foucault: A Critique of the Social Control
Thesis” The British Journal of Sociology, Vol. 47, No. 2 Jstor

The nature of the relation between the individual and the political order concerned Foucault in his studies of 'bio-
power' and 'bio-politics'. In this work, he implicitly negates his earlier claims that rights in the West were unequivocally linked to the sovereign
(1980b, 1988, 199 1). Foucault introduced the notion of 'bio-power' in his work on sexuality to designate the proliferation of a technology of
power-knowledge primarily concerned with life. Bio-power was a mechanism that took charge of life by 'investing the body,
health, modes of subsistence and habitation, living conditions, the whole space of existence' (Foucault 1980b: 14344, emphasis added). The
notion of bio-power is useful for our understanding of the phenomenon of resistance because while it represents a
totalizing or universal mechanism -one that interpellates the subject as a member of a population - it also contains
the seed for a counter-power or a counter-politics because that mechanism individualizes the subject of a population.
It is this aspect of bio-power, its simultaneous totalizing and individual-izing tendencies, that is of importance in understanding the strategies by which individual
the forces that
subjects can claim the right to self-determination. Foucault explains that against this [bio-]power that was still new in the nineteenth century,
resisted relied for support on the very thing it invested, that is, on life and man as a living being . Since the last century, the
great struggles that have challenged the general system of power were not guided by the belief in a return to former rights, or by the age-old dream of a cycle of time
or a Golden Age. (. . .) [Wlhat was demanded and what served as an objective was life, understood as the basic needs, man's concrete essence, the realization of his
potential, a plentitude of the possible. Whether or not it was Utopia that was wanted is of little importance; what we have seen has been a very real process of struggle;
life as a political object was in a sense taken at face value and turned back against the system that was bent on controlling it. It was life more than the law that became
the issue of political struggles, even if the latter were formulated through affirmations concerningrights. The 'right' tolife, to one's body, to health, to happiness, to the
satisfaction of needs, and beyond all the oppressions or 'alienations,' the 'right' to rediscover what one is and all that one can be, this 'right' (. . .) was the political
response to all these new procedures of power which did not derive, either, from the traditional right of sovereignty. (Foucault 1980b: 144-5) If
life,
understood here as 'man's concrete essence', is affirmed through rights claims, then, like Foucault we can no longer
conceive law as necessarily linked to the sovereign. It must be linked to a different political rationality, one I believe, in
which human rights are at the centre. While Foucault never specifically addressed the question of human rights, his lectures on 'bio-politics' (at
the College de France between 1978 and 1979) suggest that struggles for life and for self-determination are to be understood in
the context of liberalism. In his lectures, he explores the relation between bio-power -the mechanisms taking charge
of life -and the emergence of bio-politics, by which he means the way in which a rationalization was attempted, dating from the eighteenth century, for the
problems posed to governmental practice by the phenomena specific to an ensemble of living beings: health, hygiene, birthrate, longevity, races . . .(198 1 :353)
Foucault's statement is significant because it suggests that we cannot dissociate the problems posed by the question
of population (bio-power) from the political rationality within which they emerged, liberalism. Far from conceiving
it as a political theory or a representation of society, Foucault understands liberalism as an 'art of government', that
is, as a particular practice, activity and rationality used to administer, shape, and direct the conduct of people (1981 :
358). As a rationality of government - a 'governmentality' -liberalism, towards the beginning of the eighteenth century, breaks from reason of state (la
raison d'e'tat) which since the sixteenth century had sought to 'justify the growing exercise of government' (Foucault 198 1 :354). What distinguishes
liberalism from reason of state as an art of government is that for liberalism 'there is always too much government'
(Foucault 1981: 354-5). In fact, far from being organized around the principle of a strong state, liberalism upholds
the principle of maximal economy with minimal government (Foucault 1981: 354). The question of liberalism, that
of 'too much governing,' regulates itself, according to Foucault, 'by means of a continuing reflection' (1 98 1: 354).
The idea of reflexivity here is significant because it refers to a mechanism of self-critique, and self-limitation,
inherent in liberalism. Foucault claims that Liberalism (. . .) constitutes - and this is the reason both for its polymorphous character and for its recurrences - an
instrument for the criticism of reality. Liberalism criticizes an earlier functioning government from which one tries to escape; it
examines an actual practice of government that one attempts to reform and to rationalize by a fundamental analysis; it criticizes a practice of government to which one
is opposed and whose abuses one wishes to curb. As a result of this, one can discover liberalism under different but simultaneous forms, both as a schema for the
regulation of governmental practice and as a theme for sometimes radical opposition to such practice. (Foucault 198 1 : 356) What
allows liberalism to
oppose state power, then, is not the principle of sovereignty or the idea of a natural right external to the state; rather
it is a rationality, a governmentality of life that takes on 'the character of a challenge' (Foucault 1981 :353). People
resist the conditions under which they live, they make claims for or against the state, because they have been
submitted to government. In other words, the political technologies that seek to render us governable as a population
(bio-power and bio-politics) simultaneously make possible the critique of these same technologies.'
A2: Foucault

Foucault’s relativism dooms his project, it forces the alternative into a morally bankrupt
mode of thinking – the only ethical action is to take action to increase access to public
health
Maesena, Netherlands Institute for the Social Sciences and Nijhuis Municipal Department of Public Health, 1999
(Laurent J G van der, Harry G J, “Continuing the debate on the philosophy of modern public health: social quality as
a point of reference” http://jech.bmj.com/cgi/content/full/54/2/134?ck=nck)
ETHICS AND SOCIAL QUALITY

Foucault's conceptual scheme is highly stimulating for analysing propositions and points of view
on ethical questions. The position taken by Petersen and Lupton however also implies a form of
relativism. The world is appreciated as totally contingent and dependent on the structure of human minds. Claims
for an objective reality are judged as arrogant. Doyal and Gough commented implicitly on Foucault's
perspective by stating the consistent relativist one who regards the whole of social life as a "construction", each aspect of which
has no more or less veracity than the other enters a moral wasteland into which few have feared to tread.78 Roy
Bhaskar's conclusion concerning the epistemic fallacy can be applied to Peterson's and Lupton's study as well.
Statements about being can be reduced to or analysed in terms of statements about knowledge
that is, that ontological questions can always be transposed into epistemological terms. This
results in the systematic dissolution of the idea of a world independent of but investigated by
science.79 From our perspective, society is to be conceived both as the totality of conditions of
human actions and as the result of human actions. For a valid comprehension of society as a
subject of public heath, the multi-layered mechanisms that can change these human actions and
strategies need to be analysed. The challenge of our modern enterprise of public health is to
improve social conditions related to health. In doing so, we are in fact meeting our values and
norms, in other words, the ethical questions of public health. The social quality theory tries to respond to this
challenge. In our opinion, modern public health, based on a theory of which we have attempted to outline some
principles, has to play an important part in this moral endeavour.
at foucault: no impact

Foucault says that power is not inherently evil—it is only a problem when it turns into domination.

Foucault, quoted in an interview published in 97 (Michel, philosopher, professor and chairman of the History of Systems of
Thought @ the College de France, Ethics Subjectivity and Truth, Essential Works of Foucault 1954-1984, Vol. 1, Ed. Paul
Rabinow, 1997, p. 298-299)

Power is not evil. Power is games of strategy. We all know that power is not evil! For example, let us take sexual or amorous
relationships: to wield power over the other in a sort of open-ended strategic game where the situation may be reversed is not
evil; it’s a part of love, of passion and sexual pleasure. And let us take, as another example, something that has often been
rightly criticized—the pedagogical institution. I see nothing wrong in the practice of a person who, knowing more than others in a
specific game of truth, tells those others what to do , teaches them, and transmits knowledge and techniques to them. The
problem in such practices where power— which is not in itself a bad thing— must inevitably come into play in knowing how to
avoid the kind of domination effects where a kid is subjected to the arbitrary and unnecessary authority of a teacher, or a student
put under the thumb of a professor who abuses [their] authority. I believe that this problem must be framed in terms of rules of
law, rational techniques of government and ethos, practices of the self and of freedom.

All health policies are not the same—biopower within a democratic context are radically different than their fascism examples.

Dickinson 04 - Associate Professor, History Ph.D., U.C. Berkeley - 2004 (Edward Ross, “Biopolitics, Fascism, Democracy: Some
Reflections on Our Discourse About “Modernity,” Central European History, vol. 37, no. 1, 1–48)

In the Weimar model, then, the rights of the individual, guaranteed formally by the constitution and substantively by the welfare
system, were the central element of the dominant program for the management of social problems. Almost no one in this period
advocated expanding social provision out of the goodness of their hearts. This was a strategy of social management, of social
engineering. The mainstream of social reform in Germany believed that guaranteeing basic social rights— the substantive or
positive freedom of all citizens — was the best way to turn people into power, prosperity, and profit. In that sense, the democratic
welfare state was— and is — democratic not despite of its pursuit of biopower, but because of it. The contrast with the Nazi state
is clear. National Socialism aimed to construct a system of social and population policy founded on the concept of individual
duties, on the ubiquitous and total power of the state , and on the systematic absorption of every citizen by organizations that could implant that power at every level of their lives —
in political and associational life, in the family, in the workplace, and in leisure activities. In the welfarist vision of Weimar progressives, the task of the state was to create an institutional framework that would give
individuals the wherewithal to integrate themselves successfully into the national society, economy, and polity. The Nazis aimed, instead, to give the state the wherewithal to do with every citizen what it willed. And
where Weimar welfare advocates understood themselves to be constructing a system of knowledge and institutions that would manage social problems, the Nazis fundamentally sought to abolish just that system by
eradicating — by finding a “final solution” to — social problems. Again, as Peukert pointed out, many advocates of a rights-based welfare structure were open to the idea that “stubborn” cases might be legitimate
targets for sterilization; the right to health could easily be redefined as primarily a duty to be healthy, for example. But the
difference between a strategy of social management built on the rights of the citizen and a system of racial policy built on the
total power of the state is not merely a semantic one; such differences had very profound political implications, and established
quite different constraints. The rights-based strategy was actually not very compatible with exclusionary and coercive policies; it
relied too heavily on the cooperation of its targets and of armies of volunteers, it was too embedded in a democratic institutional
structure and civil society, it lacked powerful legal and institutional instruments of coercion, and its rhetorical structure was too
heavily slanted toward inclusion and tolerance.
at foucault: no impact

Even if they are right that our policy is biopolitical, the fact that it is carried out by a democratic state makes it profoundly
different.

Dickinson 04 - Associate Professor, History Ph.D., U.C. Berkeley - 2004 (Edward Ross, “Biopolitics, Fascism, Democracy: Some
Reflections on Our Discourse About “Modernity,” Central European History, vol. 37, no. 1, 1–48)

In short, the continuities between early twentieth-century biopolitical discourse and the practices of the welfare state in our own
time are unmistakable. Both are instances of the “disciplinary society” and of biopolitical, regulatory, social-engineering
modernity, and they share that genealogy with more authoritarian states, including the National Socialist state, but also fascist
Italy, for example. And it is certainly fruitful to view them from this very broad perspective. But that analysis can easily become
superficial and misleading, because it obfuscates the profoundly different strategic and local dynamics of power in the two kinds
of regimes. Clearly the democratic welfare state is not only formally but also substantively quite different from totalitarianism.
Above all, again, it has nowhere developed the fateful, radicalizing dynamic that characterized National Socialism (or for that
matter Stalinism), the psychotic logic that leads from economistic population management to mass murder. Again, there is
always the potential for such a discursive regime to generate coercive policies. In those cases in which the regime of rights does
not successfully produce “health,” such a system can —and historically does— create compulsory programs to enforce it. But
again, there are political and policy potentials and constraints in such a structuring of biopolitics that are very different from those
of National Socialist Germany. Democratic biopolitical regimes require, enable, and incite a degree of self-direction and
participation that is functionally incompatible with authoritarian or totalitarian structures. And this pursuit of biopolitical ends
through a regime of democratic citizenship does appear, historically, to have imposed increasingly narrow limits on coercive
policies, and to have generated a “logic” or imperative of increasing liberalization. Despite limitations imposed by political context
and the slow pace of discursive change, I think this is the unmistakable message of the really very impressive waves of
legislative and welfare reforms in the 1920s or the 1970s in Germany.90 Of course it is not yet clear whether this is an
irreversible dynamic of such systems. Nevertheless, such regimes are characterized by sufficient degrees of autonomy (and of
the potential for its expansion) for sufficient numbers of people that I think it becomes useful to conceive of them as productive of
a strategic configuration of power relations that might fruitfully be analyzed as a condition of “liberty,” just as much as they are
productive of constraint, oppression, or manipulation. At the very least, totalitarianism cannot be the sole orientation point for our
understanding of biopolitics, the only end point of the logic of social engineering.
at foucault: no impact

Biopower does not expand authority over the body when it is deployed by a government that also respects rights.

Dickinson 04 - Associate Professor, History Ph.D., U.C. Berkeley - 2004 (Edward Ross, “Biopolitics, Fascism,
Democracy: Some Reflections on Our Discourse About “Modernity,” Central European History, vol. 37, no. 1, 1–48)

At its simplest, this view of the politics of expertise and professionalization is certainly plausible. Historically speaking, however,
the further conjecture that this “micropolitical” dynamic creates authoritarian, totalitarian, or homicidal potentials at the level of the
state does not seem very tenable. Historically, it appears that the greatest advocates of political democracy —in Germany left
liberals and Social Democrats —have been also the greatest advocates of every kind of biopolitical social engineering, from
public health and welfare programs through social insurance to city planning and, yes, even eugenics.102 The state they built
has intervened in social relations to an (until recently) ever-growing degree; professionalization has run ever more rampant in
Western societies; the production of scientistic and technocratic expert knowledge has proceeded at an ever more frenetic pace.
And yet, from the perspective of the first years of the millennium, the second half of the twentieth century appears to be the great
age of democracy in precisely those societies where these processes have been most in evidence. What is more, the
interventionist state has steadily expanded both the rights and the resources of virtually every citizen — including those who
were stigmatized and persecuted as biologically defective under National Socialism. Perhaps these processes have created an
ever more restrictive “iron cage” of rationality in European societies. But if so, it seems clear that there is no necessary
correlation between rationalization and authoritarian politics; the opposite seems in fact to be at least equally true.
Biopower doesn’t lead to tyranny
at foucault: no impact (massacres)

Biopower does not make massacres vital—a specific form of violent sovereignty is also required.

Ojakangas, 05 - PhD in Social Science and Academy research fellow @ the Helsinki Collegium for Advanced Studies @
University of Helsinki – 2005 (Mika, “The Impossible Dialogue on Biopower: Foucault and Agamben,” May 2005, Foucault
Studies, No. 2, http://www.foucault-studies.com/no2/ojakangas1.pdf)

Admittedly, in the era of biopolitics, as Foucault writes,  even “massacres  have  become  vital.”  This  is  not the case, however,
because violence is hidden  in  the foundation of biopolitics, as Agamben  believes. Although the twentieth century
thanatopolitics is  the “reverse  of  biopolitics”, it should not be understood, according to Foucault, as “the  effect, the result, or
the logical consequence” of biopolitical rationality. Rather, it should be understood,  as he suggests, as an outcome of the
“demonic combination” of the sovereign power and biopower, of “the city-citizen game and the shepherd-flock game” or as I
would like to put it, of patria potestas (father’s unconditional power of life and death over his son) and cura maternal (mother’s 
unconditional  duty  to  take  care  of  her  children). Although massacres can be carried out in the name of care, they do not
follow from the logic of biopower for which death is the “object of taboo”. They follow from the  logic  of  sovereign  power,  which 
legitimates  killing  by  whatever arguments it chooses, be it God, Nature, or life.  

Biopower does not cause racism or massacres—it is only when it is in the context of a violent or racist government that it is
dangerous.

Ojakangas, 05 - PhD in Social Science and Academy research fellow @ the Helsinki Collegium for Advanced Studies @
University of Helsinki – 2005 (Mika, “The Impossible Dialogue on Biopower: Foucault and Agamben,” May 2005, Foucault
Studies, No. 2, http://wlt-studies.com/no2/ojakangas1.pdf)

It is the logic of racism, according to Foucault, that makes killing acceptable in modern biopolitical societies. This is
not to say, however, that biopolitical societies are necessarily more racist than other societies. It is to say that in the
era of biopolitics, only racism, because it is a determination immanent to life, can “justify the murderous function of
the State”.89 However, racism can only justify killing – killing that does not follow from the logic of biopower but from
the logic of the sovereign power. Racism is, in other words, the only way the sovereign power, the right to kill, can be maintained in biopolitical societies: “Racism is bound up with
workings of a State that is obliged to use race, the elimination of races and the purification of the race, to exercise its sovereign power.”90 Racism is, in other words, a discourse – “quite
compatible”91 with biopolitics – through which biopower can be most smoothly transformed into the form of sovereign power. Such transformation, however, changes everything. A biopolitical society
that wishes to “exercise the old sovereign right to kill”, even in the name of race, ceases to be a mere biopolitical society, practicing merely biopolitics. It becomes a “demonic combination” of
sovereign power and biopower, exercising sovereign means for biopolitical ends. In its most monstrous form, it becomes the Third Reich. For this reason, I cannot subscribe to Agamben’s thesis,
according to which biopolitics is absolutized in the Third Reich.93 To be sure, the Third Reich used biopolitical means – it was a state in which “insurance and reassurance were universal”94 –
What distinguishes the Third
and aimed for biopolitical ends in order to improve the living conditions of the German people -- but so did many other nations in the 1930s.
Reich from those other nations is the fact that, alongside its biopolitical apparatus, it erected a massive machinery of
death. It became a society that “unleashed murderous power, or in other words, the old sovereign right to take life”
throughout the “entire social body”, as Foucault puts it.95 It is not, therefore, biopolitics that was absolutized in the
Third Reich – as a matter of fact, biopolitical measures in the Nazi Germany were, although harsh, relatively modest
in scale compared to some present day welfare states – but rather the sovereign power : “This power to kill, which ran through the entire
social body of Nazi society, was first manifested when the power to take life, the power of life and death, was granted not only to the State but to a whole series of individuals, to a considerable
number of people (such as the SA, the SS, and so on). Ultimately, everyone in the Nazi State had the power of life and death over his or her neighbours, if only because of the practice of
The only thing that the Third Reich actually
informing, which effectively meant doing away with the people next door, or having them done away with.96”
absolutizes is, in other words, the sovereignty of power and therefore, the nakedness of bare life – at least if
sovereignty is defined in the Agambenian manner: “The sovereign is the one with respect to whom all men are
potentially homines sacri, and homo sacer is the one with respect to whom all men act as sovereigns.”97
at foucault: nazis unique

Nazi biopolitics were unique.

Dickinson 04 - Associate Professor, History Ph.D., U.C. Berkeley - 2004 (Edward Ross, “Biopolitics, Fascism, Democracy: Some
Reflections on Our Discourse About “Modernity,” Central European History, vol. 37, no. 1, 1–48)

Again, Peukert was very aware that he was writing the history of only one kind of modernity, and that the most destructive
potentials of modern social engineering discourse were only to be realized in a very specific historical context. The “Final
Solution” was, as he remarked, “one among other possible outcomes of the crisis of modern civilization,” and one possible only
in the context of the concatenation of economic, social, and political disasters through which Germany passed in the two
decades before 1933. The fact that Nazism was “one of the pathological developmental forms of modernity does not imply that
barbarism is the inevitable logical outcome of modernization,” which also created “opportunities for human emancipation.” And
yet, again, the history that Peukert actually wrote was the history of disaster— a disaster that, frequently, does seem at least
highly likely. The “fatal racist dynamic in the human and social sciences,” which consists in their assignment of greater or lesser
value to human characteristics, does “inevitably become fixated on the utopian dream of the gradual elimination of death,” which
is “unfailingly” frustrated by lived reality. In periods of fiscal crisis the frustration of these “fantasies of omnipotence” generates a
concern with “identifying, segregating, and disposing of ” those judged less valuable.68 In the most detailed exposition of his
analysis, Grenzen der Sozialdisziplinierung, Peukert argues that, given the “totalitarian claim to validity” of bourgeois norms, only
the two “strategies of pedagogical normalization or eugenic exclusion” were open to middle-class social reformers; when the one
failed only the other remained. Yet the failure of pedagogical normalization was preprogrammed into the collision between
middle-class “utopias of order” and the “life-worlds” of the working class, which were rendered disorderly by the logic of industrial
capitalism.69 Again, in Peukert’s model it seems to me that it is really only a matter of time and circumstance before the
fundamentally and necessarily murderous potential of modernity is unleashed.
at foucault: aff good use of biopower

Biopower is a description of our era—it is neither inherently good, nor bad. Our specific context is more important than their
sweeping generalization.

Dickinson 04 - Associate Professor, History Ph.D., U.C. Berkeley - 2004 (Edward Ross, “Biopolitics, Fascism, Democracy: Some
Reflections on Our Discourse About “Modernity,” Central European History, vol. 37, no. 1, 1–48)

This notion is not at all at odds with the core of Foucauldian (and Peukertian) theory. Democratic welfare states are regimes of
power/knowledge no less than early twentieth-century totalitarian states; these systems are not “opposites,” in the sense that
they are two alternative ways of organizing the same thing. But they are two very different ways of organizing it. The concept
“power” should not be read as a universal stifling night of oppression, manipulation, and entrapment, in which all political and
social orders are grey, are essentially or effectively “the same.” Power is a set of social relations, in which individuals and groups
have varying degrees of autonomy and effective subjectivity. And discourse is, as Foucault argued, “tactically polyvalent.”
Discursive elements (like the various elements of biopolitics) can be combined in different ways to form parts of quite different
strategies (like totalitarianism or the democratic welfare state); they cannot be assigned to one place in a structure, but rather
circulate. The varying possible constellations of power in modern societies create “multiple modernities,” modern societies with
quite radically differing potentials.91
at foucault: biopower good

Biopower is also positive—such as the dramatic decrease in infant mortality.

Dickinson 04 - Associate Professor, History Ph.D., U.C. Berkeley - 2004 (Edward Ross, “Biopolitics, Fascism,
Democracy: Some Reflections on Our Discourse About “Modernity,” Central European History, vol. 37, no. 1, 1–48)

Of course, at the most simple-minded level, it seems to me that an assessment of the potentials of modernity that ignores the
ways in which biopolitics has made life tangibly better is somehow deeply flawed. To give just one example, infant mortality in
Germany in 1900 was just over 20 percent; or, in other words, one in five children died before reaching the age of one year. By
1913, it was 15 percent; and by 1929 (when average real purchasing power was not significantly higher than in 1913) it was only
9.7 percent.93 The expansion of infant health programs— an enormously ambitious, bureaucratic, medicalizing, and sometimes
intrusive, social engineering project— had a great deal to do with that change. It would be bizarre to write a history of biopolitical
modernity that ruled out an appreciation for how absolutely wonderful and astonishing this achievement— and any number of
others like it — really was. There was a reason for the “Machbarkeitswahn” of the early twentieth century: many marvelous things
were in fact becoming machbar. In that sense, it is not really accurate to call it a “Wahn” (delusion, craziness) at all; nor is it
accurate to focus only on the “inevitable” frustration of “delusions” of power. Even in the late 1920s, many social engineers could
and did look with great satisfaction on the changes they genuinely had the power to accomplish.
at foucault: resistance solves impact

Even if they win that our health policy turns to the dark side of biopolitics, their impact will still be prevented by localized
resistance.

Dickinson 04 - Associate Professor, History Ph.D., U.C. Berkeley - 2004 (Edward Ross, “Biopolitics, Fascism,
Democracy: Some Reflections on Our Discourse About “Modernity,” Central European History, vol. 37, no. 1, 1–48)

In the current literature, it seems that biopolitics is almost always acting on (or attempting to act on) people; it is almost never
something they do. This kind of model is not very realistic. This is not how societies work. The example of the attempt to create a
eugenic counseling system in Prussia should be instructive in this respect. Here public health and eugenics experts—
technocrats— tried to impart their sense of eugenic crisis and their optimism about the possibility of creating a better “race” to the
public; and they successfully mobilized the resources of the state in support of their vision. And yet, what emerged quite quickly
from this effort was in fact a system of public contraceptive advice — or family planning. It is not so easy to impose technocratic
ambitions on the public, particularly in a democratic state; and “on the ground,” at the level of interactions with actual persons
and social groups, public policy often takes on a life of its own, at least partially independent of the fantasies of technocrats. This
is of course a point that Foucault makes with particular clarity. The power of discourse is not the power of manipulative elites,
which control it and impose it from above. Manipulative elites always face resistance, often effective, resistance. More important,
the power of discourse lies precisely in its ability to set the terms for such struggles, to define what they are about, as much as
what their outcomes are. As Foucault put it, power— including the power to manage life —“comes from everywhere.”105
Biomedical knowledge was not the property only of technocrats, and it could be used to achieve ends that had little to do with
their social-engineering schemes.106 Modern biopolitics is a multifaceted world of discourse and practice elaborated and put into
practice at multiple levels throughout modern societies.

As Foucault argues, power is fluid. Biopower has created new freedoms as well as new oppressions—context is key.

Dickinson 04 - Associate Professor, History Ph.D., U.C. Berkeley - 2004 (Edward Ross, “Biopolitics, Fascism,
Democracy: Some Reflections on Our Discourse About “Modernity,” Central European History, vol. 37, no. 1, 1–48)

Uncoupling “technocracy” from “discourse” is not yet enough, however. We should also be alive to the ways in which new social
practices, institutions, and knowledge generated new choices — a limited range of them, constrained by all kinds of discursive
and social frameworks, but nonetheless historically new and significant. Modern biopolitics did create, in a real sense, not only
new constraints but also new degrees of freedom— new levers that increased people’s power to move their own worlds, to
shape their own lives. Our understanding of modern biopolitics will be more realistic and more fruitful if we reconceptualize its
development as a complex process in which the implications of those new choices were negotiated out in the social and
discursive context. Again, in the early twentieth century many more conservative biopolitical “experts” devoted much of their
energy precisely to trying— without any discernable success— to control those new degrees of freedom. For most social liberals
and Social Democrats, however, those new choices were a potential source of greater social efficiency and social dynamism.
State policy reflected the constant negotiation and tension between these perspectives. Nor should we stop at a reexamination of
knowledge and technology. It might make sense, too, to reexamine the process of institution-building, the elaboration of the
practices and institutions of biopolitics. No doubt the creation of public and private social welfare institutions created instruments
for the study, manipulation, or control of individuals and groups. But it also generated opportunities for self-organization and
participation by social groups of all kinds.
at foucault: resistance solves impact

Their K oversimplifies—biopower is not a one-way street—it produces equivalent resistances that check the impact.

Campbell, 98 - professor of international politics at the University of Newcastle - 1998 (David, “Writing Security: United States
Foreign Policy and the Politics of Identity,” pg. 204-205)

The political possibilities enabled by this permanent provocation of power and freedom can be specified in more detail by
thinking in terms of the predominance of the “bio-power” discussed above. In this sense, because the governmental practices of
biopolitics in Western nations have been increasingly directed toward modes of being and forms of life — such that sexual
conduct has become an object of concern, individual health has been figured as a domain of discipline, and the family has been
transformed into an instrument of government— the ongoing agonism between those practices and the free dom they seek to
contain means that individuals have articulated a series of counterdemands drawn from those new fields of concern. For
example, as the state continues to prosecute people according to sexual orientation, human rights activists have proclaimed the
right of gays to enter into formal marriages, adopt children, and receive the same health and insurance benefits granted to their
straight counterparts. These claims are a consequence of the permanent provoca tion of power and freedom in biopolitics, and
stand as testament to the “strategic reversibility” of power relations: if the terms of governmental practices can be made into focal
points for resistances, then the “history of government as the ‘conduct of conduct’ is interwoven with the history of dissenting
‘counterconducts.”’39 Indeed, the emergence of the state as the major articulation of “the political” has involved an unceasing
agonism between those in office and those they rule. State intervention in everyday life has long incited popular col lective action,
the result of which has been both resistance to the state and new claims upon the state. In particular, “the core of what we now
call ‘citizenship’ consists of multiple bargains hammered out by rulers and ruled in the course of their struggles over the means of
state action, especially the making of war.” In more recent times, constituencies associated with women’s, youth, ecological, and
peace movements (among others) have also issued claims on society. These resistances are evidence that the break with the
discursive/nondiscursive dichotomy central to the logic of interpretation undergirding this analysis is (to put it in conventional
terms) not only theoretically licensed; it is empirically warranted. Indeed, expanding the interpretive imagination so as to enlarge
the categories through which we understand the constitution of “the political” has been a necessary precondition for making
sense of Foreign Policy’s concern for the ethical borders of identity in America. Accordingly, there are manifest political
implications that flow from theorizing identity. As Judith Butler concluded: “The deconstruction of identity is not the deconstruction
of politics; rather, it establishes as political the very terms through which identity is articulated.”
at foucault: aff pre-req to alt

Foucault’s concept of resistance is only possible in a world without violence—the aff is a pre-requisite for the alternative.

Bevir, 99 – Department of Political Science @ University of Newcastle – 1999 (Mark, “Foucault and Critique:
Deploying Agency against Autonomy, Political Theory, Volume 27 No. 1, Page 65 February 1999, JSTOR)

Perhaps we might say, therefore, that power or pastoral-power recognizes the value of the subject as an agent, whereas
violence or discipline attempts to extinguish the capacity of the subject for agency. Although Foucault, of course, never describes
things in quite these terms, he does come remarkably close to doing so. In particular, he defines violence, in contrast to power,
as aiming at domination or as a physical constraint that denies the ability of the other to act: “where the determining factors
saturate the whole there is no relationship of power,” rather “it is a question of a physical relationship of constraint.”27 Similarly,
he defines power, in contrast to violence, as able to come into play only where people have a capacity to act, perhaps even a
capacity to act freely: “power is exercised over free subjects, and only insofar as they are free,” by which “we mean individual or
collective subjects who are faced with a field of possibilities in which several ways of behaving, several reactions and diverse
comportments may be realized.”28 If we thus accept that power always treats the subject as an agent, whereas violence always
attempts to extinguish the capacity of the subject for agency, we can see why Foucault’s later work on power emphasises that
power, unlike violence, necessarily entails a capacity for resistance. To treat someone as an agent, one has to recognise that
they can do other than one wishes—they can resist. Power can exist only where people have a capacity to act freely, and so only
where they can resist that power. Perhaps, therefore, we should define as violent any relationship—whether overtly violent or not
—in which an individual has his action determined for him. Violence manifests itself in any relationship between individuals,
groups, or societies in which one denies the agency of the others by seeking to define for them actions they must perform.
Power, in contrast, appears in any relationship—although no overtly violent relationship could meet the following requirement—in
which an individual does not have his action determined for him. Power manifests itself whenever individuals, groups, or
societies act as influences on the agency of the subject without attempting to determine the particular actions the subject
performs. Here a rejection of autonomy implies that power is ineliminable, while a defence of agency implies that power need not
degenerate into violence. Foucault’s final work on the nature of governmentality suggests, therefore, that society need not
consist solely of the forms of discipline he had analysed earlier. Society might include an arena in which free individuals attempt
only to influence one another. I hope my discussion of Foucault’s theory of governmentality has pointed to the way in which a
distinction between violence and power might provide us with normative resources for social criticism absent from his earlier
work. Provided we are willing to grant that the capacity for agency has ethical value—and this seems reasonable enough—we
will denounce violent social relations and champion instead a society based on a more benign power.
at foucault: cede the political

Endless investigation of power makes real struggles against oppression impossible.

Hicks, 03- Professor and chair of philosophy at Queens College of the CUNY (Steven V., “Nietzsche, Heidegger, and Foucault:
Nihilism and Beyond,” Foucault and Heidegger: Critical Encounters, Ed. Alan Milchman and Alan Rosenberg, p. 109, Questia)

Hence, the only “ethico-political choice” we have, one that Foucault thinks we must make every day, is simply to determine which
of the many insidious forms of power is “the main danger” and then to engage in an activity of resistance in the “nexus” of
opposing forces. 72 “Unending action is required to combat ubiquitous peril.” 73 But this ceaseless Foucauldian “recoil” from the
ubiquitous power perils of “normalization” precludes, or so it would seem, formulating any defensible alternative position or
successor ideals. And if Nietzsche is correct in claiming that the only prevailing human ideal to date has been the ascetic ideal,
then even Foucauldian resistance will continue to work in service of this ideal, at least under one of its guises, viz., the nihilism of
negativity. Certainly Foucault's distancing of himself from all ideological commitments, his recoiling from all traditional values by
which we know and judge, his holding at bay all conventional answers that press themselves upon us, and his keeping in play
the “twists” and “recoils” that question our usual concepts and habitual patterns of behavior, all seem a close approximation, in
the ethicopolitical sphere, to the idealization of asceticism.

Critiques of power are so localized that they prevent coalition from forming that could genuinely fight oppression.

Cook, 92- Associate Professor at Georgetown Law School (Anthony E., “A Diversity of Influence: Reflections on Postmodernism,
Spring, 26 New Eng.L. Rev. 751, Lexis)

Several things trouble me about Foucault's approach. First, he nurtures in many ways an unhealthy insularity that fails to connect
localized struggle to other localized struggles and to modes of oppression like classism, racism, sexism, and homophobia that
transcend their localized articulation within this particular law school, that particular law firm, within this particular church or that
particular factory. I note among some followers of Foucault an unhealthy propensity to rely on rich, thick, ethnographic type
descriptions of power relations playing themselves out in these localized laboratories of social conflict. This reliance on detailed
description and its concomitant deemphasis of explanation begins, ironically, to look like a regressive positivism which purports
to sever the descriptive from the normative, the is from the ought and law from morality and politics. Unless we are to be trapped
in this Foucaultian moment of postmodern insularity, we must resist the temptation to sever description from explanation.
Instead, our objective should be to explain what we describe in light of a vision embracing values that we make explicit in
struggle. These values should act as magnets that link our particularized struggles to other struggles and more global critiques of
power. In other words, we must not, as Foucault seems all too willing to do, forsake the possibility of more universal narratives
that, while tempered by postmodern insights, attempt to say and do something about the oppressive world in which we live.
Second, Foucault's emphasis on the techniques and discourses of knowledge that constitute the human subject often
diminishes, if not abrogates, the role of human agency. Agency is of tremendous importance in any theory of oppression,
because individuals are not simply constituted by systems of knowledge but also constitute hegemonic and counter-hegemonic
systems of knowledge as well. Critical theory must pay attention to the ways in which oppressed people not only are victimized
by ideologies of oppression but the ways they craft from these ideologies and discourses counter-hegemonic weapons of
liberation.
at foucault: geneaology

Foucaultian genealogy is trapped in a double bind: its extreme relativism either undercuts its political usefulness or a new master
discourse is produced.

Habermas, 87- Permanent Visiting Professor at Northwestern (Jürgen, The Philosophical Discourse of Modernity, p. 279)

Foucault's historiography can evade relativism as little as it can this acute presentism. His investigations are caught ex actly in the
self-referentiality that was supposed to be excluded by a naturalistic treatment of the problematic of validity. Genealogical
historiography is supposed to make the practices of power, precisely in their discourse-constituting achievement, accessible to
an empirical analysis. From this perspective, not only are truth claims confined to the discourses within which they arise; they
exhaust their entire significance in the functional contribution they make to the self-maintenance of a given totality of discourse.
That is to say, the meaning of validity claims consists in the power effects they have. On the other hand, this basic assumption of
the theory of power is self-referential; if it is correct, it must destroy the foundations of the research inspired by it as well. But if
the truth claims that Foucault himself raises for his genealogy of knowledge were in fact illusory and amounted to no more than
the effects that this theory is capable of releasing within the circle of its adherents, then the entire undertaking of a critical
unmasking of the human sciences would lose its point. Foucault pursues genealogical historiography with the serious intent of
getting a science underway that is superior to the mismanaged human sciences. If, then, its superiority cannot be expressed in
the fact that something more convincing enters in place of the convicted pseudo-sciences, if its superiority were only to be
expressed in the effect of its suppressing the hitherto dominant scientific discourse in fact, Foucault's theory would exhaust itself
in the politics of theory, and indeed in setting theoretical-political goals that would overburden the capacities of even so heroic a
one-man enterprise. Foucault is aware of this. Consequently, he would like to single out his genealogy from all the rest of the
human sciences in a manner that is reconcilable with the fundamental assumptions of his own theory. To this end, he turns
genealogical historiography upon itself; the difference that can establish its preeminence above all the other human sciences is
to be demonstrated in the history of its own emergence.
A2: Agamben

Sovereignty must be used strategically –-- critique can be simultaneous


Lombardi ’96 (Mark Owen, Associate Prof Political Science – Tampa, Perspectives on Third-World Sovereignty, p. 161)

Sovereignty is in our collective minds. What we look at, the way we look at it and what we expect to see must
be altered. This is the call for international scholars and actors. The assumptions of the paradigm will dictate
the solution and approaches considered. Yet, a mere call to change this structure of the system does little except
activate reactionary impulses and intellectual retrenchment. Questioning the very precepts of sovereignty, as has
been done in many instances, does not in and of itself address the problems and issues so critical to transnational
relations. That is why theoretical changes and paradigm shifts must be coterminous with applicative studies.
One does not and should not precede the other. We cannot wait until we have a neat self-contained and accurate
theory of transnational relations before we launch into studies of Third-World issues and problem-solving. If we
wait we will never address the latter and arguably most important issue-area: the welfare and quality of life for
the human race.

Agamben’s criticism fetishizes biopolitics, using it as an excuse for action or instrument for
confronting it blocking critical thought
Virno, 2002 Paolo, (“General intellect, exodus, multitude,” in Archipelago number 54, June,
http://www.neuralyte.org/~joey/generation-online/p/fpvirno2.htm)

Agamben is a thinker of great value but also, in my opinion, a thinker with no political vocation.
Then, when Agamben speaks of the biopolitical he has the tendency to transform it into an
ontological category with value already since the archaic Roman right. And, in this, in my opinion,
he is very wrong-headed. The problem is, I believe, that the biopolitical is only an effect derived
from the concept of labor-power. When there is a commodity that is called labor-power it is
already implicitly government over life. Agamben says, on the other hand, that labor-power is only
one of the aspects of the biopolitical; I say the contrary: over all because labor power is a
paradoxical commodity, because it is not a real commodity like a book or a bottle of water,
but rather is simply the potential to produce. As soon as this potential is transformed into a
commodity, then, it is necessary to govern the living body that maintains this potential, that
contains this potential. Toni (Negri) and Michael (Hardt), on the other hand, use biopolitics in a
historically determined sense, basing it on Foucault, but Foucault spoke in few pages of the
biopolitical - in relation to the birth of liberalism - that Foucault is not a sufficient base for
founding a discourse over the biopolitical and my apprehension, my fear, is that the
biopolitical can be transformed into a word that hides, covers problems instead of being an
instrument for confronting them. A fetish word, an "open doors" word, a word with an
exclamation point, a word that carries the risk of blocking critical thought instead of helping
it. Then, my fear is of fetish words in politics because it seems like the cries of a child that is
afraid of the dark..., the child that says "mama, mama!", "biopolitics, biopolitics!". I don't
negate that there can be a serious content in the term, however I see that the use of the term
biopolitics sometimes is a consolatory use, like the cry of a child, when what serves us are, in
all cases, instruments of work and not propaganda words.
A2: Agamben

Agamben’s representation of the muselmann belittles the experiences of the actual


experiences of the victims in concentration and extermination camps.
Mesnard in 2004 (Phillipe, Totalitarian Movement and Political Religions, V5, Issue 1, “The Political Philosophy of Giorgio Agamben:
A Critical Evaluation” pp. 139-157)

In his exploration of Auschwitz and his interpretation of Levi’s writings, Agamben postulates that the notion of
testimony can be understood only from the viewpoint of the ‘dead witnesses’ perfectly epitomised by the
‘muselmann’. The term ‘muselmann’ was used in Auschwitz to refer to deportees who suffered from clinical
exhaustion and multiple, chronic illnesses and came to embody, in the eyes of fellow deportees, what man, subjected
to extreme brutality and deprivation and on the verge of death, could become. The ‘muselmann’ was the incarnation
of their own fate. Using this figure, Agamben develops the following paradox: since any testimony is, in essence,
impossible, and since, therefore, the reality of the death camps is inaccessible, only the impossible figure of the
‘muselmann’ can point to the ethical reality of Auschwitz. Thus, radicalising Levi’s words, Agamben turns the
‘muselmann’ into the ‘integral witness’. Such a contention is naturally reminiscent of the thought discussed earlier:
the ‘muselmann’ becomes the ‘epiphanic’ incarnation of what testifies to the existence of the human dimension in
man as revealed in Auschwitz, its only resemblance to man being its cadaveric appearance.

To decipher the meaning of Agamben’s ‘muselmann’, it is necessary to look into the polysemic dimension of this
word, which evidently transcends the concentration camp universe. From a purely lexical point of view, first of all,
the word ‘muselmann’ is utterly foreign to the Polish language and to the numerous other languages that were
spoken in Auschwitz. From a current affairs, contextual point of view ( Remnants of Auschwitz was published in
1998), the Muslim, who is linked to political Islam and Palestine, can be seen as the antagonist of the Jew, who is
conversely linked to modern Israel and Zionism. 25 From an imaginary point of view, the ‘muselmann’ is
reminiscent of the numerous texts and pictorial representation of the suffering body of Christ. 26 Moreover,
Agamben’s theory neglects the complexity of the concentration camp reality, and the crucial differences between the
functioning of camps on the one hand, and the extermination centres on the other. While Agamben can on no
account whatsoever be linked to or accused of negationist views, he must be criticised for his abusive use of the
victims’ representations, a trend which has become fairly general. Thus, the figure of the ‘muselmann’ is a faithful
epitome of the ‘screen victim’ syndrome frequently found in mediadriven humanitarian operations: the figure tends
to hide the real victims and blur our understanding of what actually happened.
A2: Agamben

RIGHTS ARE NOT NECESSARILY OR MERELY A PART OF THE STATE;


RATHER, AS PRACTICES ON THE PART O HUMAN BEINGS INTERACTING
WITHIN THE SOCIAL FORMS.

Daly, 2004. (Frances, Research fellow in the Philosophy Department at the Australian National University, “The
non-citizen and the concept of human rights”, Borderlands E-Journal).

A significant part of Agamben's rejection of rights is based on the belief that rights necessarily involve processes by
which values are made eternal. The argument as to whether there is a problem with the idea of eternal metajuridical
values able to be inscribed within rights is interesting for a number of reasons. Firstly, there is the problem of what
particular values we find inscribed, and by what process these values have been arrived at; secondly, there are
certainly difficulties at issue with an understanding of what is eternal within such values. But such arguments often
proceed on the basis of misplaced assumptions. Instead of indicating the actual nature of the problem with particular
metajuridical values, or indeed indicating what, for example, a construction of an eternalized, homogeneous
substrate would mean for the idea of a social contract or rights, it is presumed that, despite whatever it is that
constitutes their content, it is such values themselves that are at fault. Agamben elides all difference by assuming
that right has only judgment, calculation and control as its outcome, and that the basis of right is its place within the
structure of the State. And yet right is not necessarily or merely a part of the State; rather it is
better understood as practices on the part of human beings interacting within social forms
(of which the State may or may not be a part). The problem would appear to be that not only
are we no clearer as to the actual problems involved in such values, but we are also left
without a basis for the critique of the intention of right. And, not surprisingly, we are also
without any basis for considering the productive content of these values. Legal positivism
assumes or sets out the basis for rights within a normative framework of the State that merely
takes for granted judicial postulates of the inalienability of rights, the basis of rights in property
and assumptions that people are in fundamental accord on matters of right. It is unable to
imagine a realm of freedom against the State. But within rights, I would argue, we can detect
unsatisfied demands that have nothing to do with essentialist assumptions about 'man' or
'citizen'. These demands are concerned with an understanding of human freedom in
relation to values of solidarity, justice and the overcoming of alienation; they are historical
and contingent, shifting and alive, and are not about a fixed, static, generic essence of the
person, or some ahistorical or superhistorical immutable totality. What it is to be human is
open and changeable, although not without determinations, commonalities and shared
properties that can emerge at various times. Simply because we would want to challenge a
distorted, limited or perhaps unappealing view of what it is to be human, does not mean
that we are unable to say anything about what it is to be a creative, suffering, desiring
being. Somewhat strangely, Agamben's argument is ultimately more concerned with the problem of contradictions
within the theory and practice of rights and with attendant illusions that arise from these contradictions than with a
critique of content or with an examination of a new potentiality that might emerge out of what he takes to be our
present vacuousness. Such contradictions and illusions certainly do exist in relation to right, although as far as
↓Card Continues ↓
↑Card continued ↑

attitudes to the law are concerned I believe that a Slöterdijkean 'cynical reason' probably more
accurately describes the matter. The assumption that any understanding of human values is a
reductionist, eternalizing essentialism has become one of the banalities of much contemporary
theorizing, but even in more considered forms it often fails to come to terms with what it is
attempting to criticize. For example, if there is a problem with making right the depository of
eternal values, this is hardly because we have arrived at some content that would forevermore
allow us to express a sense of justice in common. Rather, it is because what becomes 'eternalized'
(or, more correctly, what merely congeals under certain circumstances and is able to be
reformulated for these changing circumstances) is a view of what it means to be human in terms
of an ability to possess. Thus, what is frequently taken to be the eternal nature of right is,
unfortunately, anything but the idea of communal principles that would provide some natural
standard, however derived, for justice. Instead, what becomes solidified, and, more importantly,
reified, is a positivity of existing conditions (such as the right to possess, exclude and alienate)
through which doctrines of the rights of the individual are determined. This, I want to argue, is at
the basis of the juridical objectification separating ethics or justice from law. It is this reification
of law rather than the eternalization of values that is of significance to a critique of rights. With
the rise of individualism in the seventeenth and eighteenth centuries, the idea of natural rights of
the individual, of liberty, fraternity, and equality of the individual – of 'inalienable' rights and
normative ideals – was quite clearly conceived in terms of the citoyen. What persisted of a sense
of natural justice for all, whose standard had been derived from various sources - in nature, God,
a view of reason or human nature - was undoubtedly distorted by a sense of individualism
defined in terms of possession and property rights. But this sense was not completely
extinguished. It is certainly on the basis of a realm of legal positivism and its doctrines of
positive law, a realm which assumes that no element of law or right pre-exists an act of the State,
that some of the basic contradictions that Agamben highlights are likely to emerge. For it is the
State that institutes types of validity for its laws on the basis of procedure rather than any sense
of morality or principles of justice. But there are other pathways to rights, other forms in which
principles of justice have been derived and enacted. And if this is the case, why must we then
necessarily conclude from a critique of legal positivism that there can be no ethical basis to
rights?
A2: Aganben

Rather than kritiking the possible illusions that may arise from the existence of rights, it is
more urgent that we kritik the absence of sense of justice.
Daly, 2004. (Frances, Research fellow in the Philosophy Department at the Australian National University, “The
non-citizen and the concept of human rights”, Borderlands E-Journal).

If we are to understand the real function of rights in the modern State, as Agamben wishes to do, as rights of the
citizen serving the interests of the nation-State, then we need to understand why a separation between human and
citizen rights emerged, and what relation the distinction between these rights has to the propositions of an ethical or
just life. This, I want to argue, involves understanding an inheritance that brings with it illusions and aporias, and, at
the same time, a theoretical heritage that has engaged with certain ideals and intentions that reveal an anticipation of
what is right and just. An ahistorical disdain for legal action is merely the obverse of the process of fetishizing
legality. Much theory that merely substitutes the idea of the static essence of the person to explain the consequence
of good and evil in the world with an equally static, invariant view of authority and the State is, I would argue,
ultimately eternalizing such concepts. Undoubtedly, some sort of move beyond categories
underscoring divisions within the ways people are entitled to live their lives is necessary. But
much of the power of any such critique must depend upon the manner in which the context of
this life – the possible experience of acting in the world, or 'form-of-life' - is itself understood. In
the absence of any such context, what tends to emerge is a return to the problem of rights
reduced to a division of form and content, rather than the overturning of this very problematic.
Only in this case, because the content is seen to fall short of the abstraction of, for example, a
"whatever singularity", the form is wholly discarded. More importantly, by revisiting this
problem via a dismissal of the context of rights, and more specifically of the possibility of traces
of the intention towards human dignity, a rich heritage of critique is sidelined. Moreover, a
separation of law and ethics in rights is just as much the separation of ethics from law, the latter
an entirely necessary basis for an autonomous, ethical life beyond the juridical relations of the
world of goods. For ethics to be completely subsumed by the law would hardly be a desirable
thing; the law carries with it exploitative, ideological relations that are not a part of a viable
ethics. Nor would it be in any way likely, given that what we are dealing with in relation to
current, existing law is a form of decision-making as the ideological playing out of a conflict of
wills upon the assumption that the State is all of us and that the will of the State must ultimately,
justly, prevail. This is not the basis of a radical equality or solidarity. What we have instead is the
separation of society from the ideals of the ethicality of a subject, partly, but not exclusively, the
outcome of an Enlightenment abstract rationalism that insists upon a narrow calculation for its
judgments. It is this absence of a sense of real justice that most needs to be subjected to
critique today, not the possible illusions which might arise from the existence of rights.
Agamben sees the necessity of this separation but he attributes it to a lack of ambiguity inherent within law. He also
nervously empties any sense of being human of anything that is not simply potentiality, as if right and ethics have
nothing to do with human possibility. The concept of potentiality has much that is worthwhile – there is a radical
uncompletedness within our being human - but it needs what I refer to as "an ethos of the imagination" that is able to
imagine an emancipatory ethics, and that is grounded in a basic responsibility for the other (Daly, 2001: passim) to
be of use in critical theorizing.
A2: Agamben

Agamben’s kritik fails, another state formation will always rise instead wehsould use the
state in strategic instances for responsibility of human needs.
Passavant, 2007 (Paul A, Political Theory 2007; 35; 147, “The Contradictory State of Giorgi Agamben,”
http://ptx.sagepub.com/cgi/content/abstract/35/2/147, Pg25-26)

Finally, Agamben indicates, through the example of the apostle Paul and the remnant of those who faithfully adhere to messianic law , the
possibility of active political subjects adequate to the challenge of state sovereignty. This argument,
however, contradicts his earlier positions embracing potentiality over the acts emblematic of sovereign decisions, and an experience of being
by relying on a determinate situation to create the conditions of possibility
beyond any idea of law. It also,
for a successful speech act, occludes the forms of power needed to maintain this situation against
other ontological possibilities much as his first theory of passage beyond the state of integrated spectacle did. This argument also
begs the question of how this messianic community might relate to that which remains other to its situation. That is, Agamben must
address the very questions that his ontological approach to state sovereignty intended to avoid—
questions of power and otherness. In sum, Agamben remains haunted by the very problems that motivated not
only his critique of the state but also his attempt to remove this inquiry from political philosophy to “first” philosophy. 43
At the end of Agamben’s theory of the state, politics remains. There are four implications of this critique for political theory and
the state. First, the modern state is poorly understood as transcendent, unitary, and sovereign. The “state” encompasses a variety of
institutions, many of which predate modernity.44 The Foucauldian understanding of government, I suggested, is the practice by which
articulations between these institutions are forged—and non-state institutions are joined to this chain—and they are mobilized toward various
purposes. The plural nature of this ensemble is precisely what gives extension to the modern state.45 Second, if we
treat the state as an ensemble of institutions, then the concept of a state of emergency is poorly suited to
understanding our political present. Agamben rightly criticizes the USA PATRIOT Act in State of Exception. This law, like most laws
that are passed in an ongoing legal system, amends a variety of other laws and sits on a foundation created by these other laws, such as the
Antiterrorism and Effective Death Penalty Act of 1996. The Antiterrorism Act created the possibility of attributing guilt by association since it
criminalized the provision of material support for organizations that the administration deems “terrorist”—provisions that the USA PATRIOT
Act builds upon.46 From this perspective, current policies are less “exceptional,” unfortunately, and more a continuing development of a national
security state apparatus that has been built through legislation like the National Security Act of 1947, through discourse, and through the creation
of stakeholders (the military-industrial complex).47 In other words, another state formation is struggling to emerge through the
ruin of liberal democracy in the United States, and this emergence (and ruin) is hastened by those who seek to
enhance surveillance and presidential powers, while diminishing the power of courts and legislative oversight as a
response to September 11, 2001.48 Third, any social formation is constituted by elements of both contingency and
determination. By emphasizing pure potentiality, Agamben misses this and either cherishes the excessive quality of
pure potentiality to the neglect of the exigent needs of the present, or neglects how the active political subjects he
does defend are embedded within finite commitments that necessarily persevere through the foreclosure of other
possibilities. Some contemporary political theorists concerned with injustice and the lack of democracy also emphasize contingency, excess, and potentiality
over determination, finitude, and acts.49 These theorists correctly seek to disrupt oppressive patterns. Since politics—hence political change—would not be possible
under conditions of absolute determination, emphasizing contingency or excess makes sense. Yet reflection upon the retraction of certain state services from places
like the Bronx during the late 1970s permits us to see how neither justice nor democracy is served by excessive economic duress or violence. Not only are these
contingencies unjust, but also their incapacitating effects prevent democratic practices of government where the latter necessarily presupposes some collective
capacity to direct and achieve collective purposes. State
actions that mitigate chaos, economic inequality, and violence, then,
potentially contribute to the improved justice of outcomes and democracy. Political theorists must temper celebrating
contingency with a simultaneous consideration of the complicated relation that determination has to democratic purposes.5 Fourth, the state’s
institutions are among the few with the capacity to respond to the exigency of human needs identified by political
theorists. These actions will necessarily be finite and less than wholly adequate, but responsibility may lie on the side of
acknowledging these limitations and seeking to redress what is lacking in state action rather than calling for pure
potentiality and an end to the state. We may conclude that claims to justice or democracy based on the wish to rid ourselves of the state once and for all
are like George W. Bush claiming to be an environmentalist because he has proposed converting all of our cars so that they will run on hydrogen.51 Meanwhile, in the
here and now, there are urgent claims that demand finite acts that by definition will be both divisive and less than what a situation demands.52 In the end, the
state remains. Let us defend this state of due process and equal protection against its ruinous other.
A2: Agamben

ALT – DN SOLVE
Agamben is problematic because he offers little political vocation, his shallow analysis of
biopolitics prevents the alternative for solving
Virno, 2002. (Paolo, linguistics professor, “INTERVIEW WITH PAOLO VIRNO”, <http://www.generationonline.
org/p/fpvirno2.htm>.)

Agamben is a problem. Agamben is a thinker of great value but also, in my opinion, a thinker
with no
political vocation. Then, when Agamben speaks of the biopolitical he has the tendency to
transform it into an ontological category with value already since the archaic Roman right. And,
in this, in my opinion, he is very wrong-headed. The problem is, I believe, that the biopolitical is
only an effect derived from the concept of labor-power. When there is a commodity that is called
labor-power it is already implicitly government over life. Agamben says, on the other hand, that
labor-power is only one of the aspects of the biopolitical; I say the contrary: over all because
labor power is a paradoxical commodity, because it is not a real commodity like a book or a
bottle of water, but rather is simply the potential to produce. As soon as this potential is
transformed into a commodity, then, it is necessary to govern the living body that
maintains this potential, that contains this potential. Toni (Negri) and Michael (Hardt), on the
other hand, use biopolitics in a historically determined sense, basing it on Foucault, but Foucault
spoke in few pages of the biopolitical - in relation to the birth of liberalism - that Foucault is not
a sufficient base for founding a discourse over the biopolitical and my apprehension, my fear, is
that the biopolitical can be transformed into a word that hides, covers problems instead of being
an instrument for confronting them. A fetish word, an "open doors" word, a word with an
exclamation point, a word that carries the risk of blocking critical thought instead of helping it.
Then, my fear is of fetish words in politics because it seems like the cries of a child that is afraid
of the dark..., the child that says "mama, mama!", "biopolitics, biopolitics!". I don't negate that
there can be a serious content in the term, however I see that the use of the term biopolitics
sometimes is a consolatory use, like the cry of a child, when what serves us are, in all cases,
instruments of work and not propaganda words.
A2: Agamben

ALT – DN SOLVE
Although Agamben says that we need a new and coherent form ontology of potentiality,
this already exists; his calls for this new alternative are merely false.
Casarino and Negri, 2001. (Cesare, Associate rofessor in the Department of Cultural Studies and Comparative Literature at the
University of Minnesota, and Antonio, Political Philosophy Professor at Padua University, founded Potere Operaio (Worker Power) Group,
leading member of the Autonomia Operaia. Taught at the Université de Vincennes (Paris-VIII) and the Collège International de Philosophie, “It’s
a Powerful Life: A Conversation on Contemporary Philosophy”, Project Muse, p.178-180.)

This claim regarding Agamben’s failure to think the constitution of the political has made me think of the sentence
with which he ends his discussion of your arguments about constituent power. Let me read it to you. He writes:
“Until a new and coherent ontology of potentiality (beyond the steps that have been made in this direction by
Spinoza, Schelling, Nietzsche, and Heidegger) has replaced the ontology founded on the primacy of the act and
its relation to potentiality, a political theory exempt from the aporias of sovereignty remains unthinkable.”13
On the one hand, I completely agree with what he is saying here, and, on the other hand, I would like to take all this
in different directions from the ones I think he pursues. I do share his concerns regarding that dominant ontological
tradition that is willing and able to conceive of potentiality only as mere means to that all-important end that is the
act—thereby not only subordinating potentials to acts but also failing to understand and indeed to think potentiality
tout court (even though, as Agamben rightly points out, the foundational text of this tradition—namely, Aristotle’s
Metaphysics—is far more complex and astute with respect to the question of potentiality than such a tradition has
often dared to admit). Such concerns lead him in the end to attempt to produce a concept of potentiality without
making recourse to the mediating passage or transformation from potentiality to act—that is, to conceive of
potentiality no longer in relation to the act, and, indeed, to think potentiality at once without any relation and without
any act whatsoever.14 (And, clearly, this is also tantamount to producing a concept of means without end.) Whereas
I agree that it is necessary to banish this mediating relation from any thought of potentiality— a relation that in the
end has always had the effect of enslaving potentiality to act—I also feel that by getting rid of the relation one does
not somehow get rid of the act too, that it is one thing to dispense with this relation and quite another to imply that
the whole question of actualization will also vanish into thin air or become irrelevant once such a relation has been
Wnally dispensed with. On the contrary! Doing without this relation should lead to a radical and global
reconceptualization of both potentiality and act as immanent to each other, that is, as distinct yet indiscernible from
each other. It is only by rethinking both at once in such a way that a “new and coherent ontology of potentiality” can
at all come into being. And this is why, whereas Agamben suggests in the above sentence that such an ontology
is yet to come, I think that it already exists in some form, and that, in particular, Deleuze took important steps in
this direction. If we understand Deleuze’s deployment of the dyad of “virtual” and “actual” as one of his ways of
posing the question of the relation between potentiality and act, for example, we can see that Deleuze does not
dispense with the actual just because it has only too often been used to suppress and indeed repress the virtual, and
does not theorize the virtual in isolation from the actual. In Deleuze, the virtual and the actual form an immanent
circuit, in the sense that each of the two is the obverse side of the other—and hence the actual always has virtual
facets, always leads parallel virtual lives, and vice versa.15 The virtual and the actual, thus, are two different ways
of apprehending the very same thing. Importantly, this also means that the actualization of the virtual never
constitutes an impoverishment or mortiWcation of the virtual, because such an actualization always produces in its
turn still other virtual realities. In Agamben, on the other hand, one often gets the feeling that potentiality always
pulls back at the last moment from realizing itself in the act precisely because he understands such a realization to
constitute nothing other than the depletion and death of potentiality: it’s as if potentiality, by realizing itself in the
act, would be relegated to playing the role of a haunting yet fossilized presence within the act, not unlike a mummy
within the sarcophagus, or, better yet, within a pyramid. And yet, having said all this, I also think that there are
elements in Agamben’s work that point in different directions, which might be more reconcilable with Deleuze’s
positions on this matter (I am thinking, for example, of that beautiful chapter on the question of halos in Agamben’s
The Coming Community, about which I have commented elsewhere.)16
A2: Agamben

ALT – DN = ACTION
Agamben’s acceptance of Schmitt’s central theme regarding political judgment make it
impossible for him to make a real alternative which he can contest.
Norris, 2003. (Andrew, Political Science Professor at Penn, “The Exemplary Exception”, Radical Philosophy,
<http://www.radicalphilosophy.com/default.asp?channel_id=2188&editorial_id=13097>.)

Such claims are difficult for political philosophy to address, as they undermine so many of its guiding assumptions.
Instead of asking us to construct and evaluate different plans of action, Agamben asks us to evaluate the
metaphysical structure and implications of the activity of politics as such. Instead of asking us to consider the true or
proper nature of political identity, Agamben asks us to consider a threshold state of the non-identical, the liminal.
And far from bringing concepts such as rights, authority, public interest, liberty or equality more clearly into view,
Agamben operates at a level of abstraction at which such concepts blur into their opposites. He takes this approach
because, like Arendt, he believes that claims to justice can only be made if one understands the ground of the
political upon which both justice and injustice stand. If Foucault's goal was 'to make the cultural unconscious
apparent',4 Agamben's is that of bringing to expression the metaphysics that our history has thus far only shown. He
argues that, properly understood, what that history shows us is that politics is the truly fundamental structure of
Western metaphysics insofar as it occupies the threshold on which the relation between the living being and the
logos is realized. In the 'politicization' of bare life - the metaphysical task par excellence - the humanity of living
man is decided [si decide].É There is politics because man is the living being who, in language, separates and
opposes himself to his own bare life and, at the same time, maintains himself in relation to that bare life in an
inclusive exclusion.5 What is perhaps both most intriguing and most problematic about Agamben's work is that -
unlike, say, that of Philippe Lacoue-Labarthe and Jean-Luc Nancy - it brings these claims about metaphysics into
dialogue with a specific set of quite concrete examples, including refugee camps, hospital wards, death rows and
military camps. All of these are sites where, on Agamben's account, one can perceive the metaphysical negation that
allows for the affirmation of distinctively human life: bare life, nuda vita.
One way to evaluate Agamben's claims is to consider how well they help us to describe and understand such
examples.6 Another is to ask whether Agamben's claims are intelligible on their own account - to see, that is,
whether they open themselves up to an immanent critique. This approach has a number of advantages, chief among
which is that it does not demand that we simply choose whether to accept or reject Agamben's approach in a global
way. Instead such an approach allows us to be open to a radically different way of thinking about politics and
political philosophy while at the same time maintaining some critical distance from it. In what follows I want to
pursue this option by way of considering Agamben's appropriation of the early decisionist political theory of Carl
Schmitt. I will argue that Agamben's acceptance of Schmitt's central claims regarding political judgment make it
impossible for him to weave together his suggestive reading of examples from philosophy and political history into a
mode of political thought that fulfils his own ambition of 'returning thought to its practical calling'.7
Agamben's project hinges upon the paradigmatic status of the camp. But on his own account, there is an
isomorphism between the exception and the example or paradigm. Given his acceptance of Schmitt's analysis of the
former as the product of the sovereign decision, this makes Agamben's evaluation of the camp as 'the fundamental
biopolitical paradigm of the West' into a sovereign decision beyond the regulation of rule or reason. As this casts his
readers as either subject or enemy, it is hard to imagine how the politics it might produce will serve as a real
alternative to that which it contests.
A2: Agamben

ALT - OVERSIMPLIFIES
Agamben’s attempt to apply his theory to actual reality oversimplifies the situation. The
conception of the muselmann, the dead witness, in concentration camps belittles the
resistance that did occur.
Mesnard in 2004 (Phillipe, Totalitarian Movement and Political Religions, V5, Issue 1, “The
Political Philosophy of Giorgio Agamben: A Critical Evaluation” pp. 139-157)

At a seminar on Saint Paul held in Paris in 1999, Agamben advocated what appeared to be a relevant
distinction between time, the time of the end and the end in itself. In other words, he urged his audience not to
confuse eschatology with messianism. However, as soon as time is no longer conceived of in its transcendental
dimension and is envisaged as a materialised, immanent notion, and as soon as Agamben seeks to apply his
theory not only to texts but also to historical reality and concentration camps, his thought starts to
oversimplify reality itself. The most blatant example of oversimplification is the figure of the ‘muselmann’,
who compresses and distorts the complexity of the camp and, in the first instance, the complexity of the
multiple temporalities which co-existed within the camp; such temporalities simply cannot be reduced to the
mere image of emaciated bodies recycled into a rhetorical figure.
Wolfgang Sofski, whose book L’Organisation de la terreur was a source of inspiration to Agamben, 28 offers a
seminal categorisation of the different temporalities generated by the camp. On the one hand, he identifies a
planned temporality detached from the past and from the future and deprived of any intimate continuity, in
which human beings were brutally dispossessed of both their own personalities and of all spirituality. The
crushing monotony of this relationship between time and existence, repeated time and again through the
same cycle, was nonetheless disrupted by sudden, unexpected avalanches of violence. This temporality slowly
led to the degradation of the self under the yolk of the camp’s organisation and hierarchy, from the kapos to the SS.
On the other hand, Sofski identifies rival, precarious temporalities generated by the various forms of
resistance (passive or active, individual or collective), and the numerous instances of corruption and
irregularity. One could also add to this temporality the idea of temporary salvation as well as the fatal events which
took place in special places, such as the Revier (infirmary). Even if Sofski could be criticised for nearly turning the
notion of ‘absolute power’ into a myth, for speaking of ‘the’ camp and ‘the’ SS, his rigorous research is seminal in
that it acknowledges the heterogeneity of the camp system. The same cannot be said of Agamben, whose quest
leads him to ‘find what he is looking for’ in terms of truth, essence and paradigm.
A2: Agamben

ALT - OVERSIMPLIFIES
Agamben’s simplistic theories eliminate the actual complexities of the historical facts of the
situations he chooses to analyze.
Mesnard in 2004 (Phillipe, Totalitarian Movement and Political Religions, V5, Issue 1, “The
Political Philosophy of Giorgio Agamben: A Critical Evaluation” pp. 139-157)

Agamben’s denigration of the Sonderkommando on the rather simplistic basis that they
were an integral part of the bipolar victim/ henchman scheme, precludes any
understanding of the ‘grey areas’. His blindness to the many ambiguities of human life, as
found in his desire to seek in the ‘muselmann’ an impossible, pure witness, reveals the
biaises of this philosopher. He refuses to investigate rationality, and erects a rhetorical
edifice which is aimed at the sublime and, in filigrane, reflects his attraction for
irrationality; he is fascinated by a type of essentialist monocausalism; he radicalises
Heidegger’s ontology. These are some of the features of a philosophy permeated by strong
theological motives, evenif its expression is to a certain extent secularised.
Agamben could have decided to restrict his research to the margins of the dual problematic
of the National Socialist camp system and Judeocide, to revolve around it as he had done until
Bartleby ou la creation. In 1995, however, perhaps deeply traumatised by the genocides
which took place in Rwanda and in the former Yugoslavia, by the numerous
commemorations of the Judeocide and possibly also by the electoral breakthrough, into the
political landscape of his own country, of the Movimento Sociale Italiano under its new guise
as the ‘National Alliance’, Agamben decided to move toward political philosophy. He started
to cenceptualise twentieth-century violence. His irrepressible tendency to reduce the
complexity of reality down to its essentialist nature, however, raises a question which
transcends Agamben himself, namely the question of the inadequate linkage between, on
the one hand, the long philosophical tradition to which Agamben is indebted, and, on the
other, modern and contemporary history. For as long as philosophy remains a prisoner of the
onto-theology which permeates it, this question will remain formulated as follows: can
philosophy investigate the concepts of violence and time without losing its way through an
essential quest which distracts, and sometimes totally isolates it from the sociopolitical issues of
our time?
A2: Agamben

AFF AT: ALT – THEORY FLAWED


Agamben fails to look at the concentration camps in terms of the Third Reich. In
constructing the camp as space outside of the law, he ignores the fact that the historical
occurrences were not considered the norm, which presents a fundamental flaw in his
theory. His “all or nothing” perspective when it comes to the forms of life reproduce the
terror upon which the Third Reich was built.
Mesnard in 2004 (Phillipe, Totalitarian Movement and Political Religions, V5, Issue 1, “The
Political Philosophy of Giorgio Agamben: A Critical Evaluation” pp. 139-157)

This deliberate lack of an historical outlook is the source of other errata and
misinterpretations by Agamben. For instance, his interpretation of politics in terms of all or
nothing stems from a specific period characterised by the unrivalled rule of a state of
terror, which has become the sole articulation between the law and the norm. It is a period
in which the law is no more than its own ideological falsification (the rule of racial laws and
criminalisation), and the norm has assumed the caricatured appearance of the law. The
repressive Nazi regime, built upon the SS apparatus, strengthened by its administrative and legal institutions and
cemented by an overwhelming propaganda machine, steered German society through a radical period which was
exacerbated by the war and the ‘total war’. Exceptional procedures were widespread throughout the Reich, but no
one even tried to pretend that these were the norm.34 By striving to locate in ‘the camp’
what he calls the ‘very paradigm of political space’, Agamben erects an insurmountable
frontier around concentration camps which become, in fact, isolated from their
surrounding society, and turns them into an exclusive ‘outside’. Consequently, Agamben
fails to envisage the global system which surrounded the camps, and included numerous
social and economic interfaces present on the entire territory of the Third Reich.
Agamben’s attempt to locate a paradigm in ‘the’ camp is seriously flawed, because any
paradigm must be conceived and constructed from the viewpoint of a whole society. In an echo
to a remark made by Martin Broszat to Saul Friedländer, even ‘Auschwitz’ cannot account for the vast apparatus, established on a European
level, which gave birth to the concentration and extermination systems.
Agamben’s vision of the camp as an absolute
space is also flawed: camps were in fact the sordid, random conjunction of selective,
unrivalled power and absolute relativity.
Let us make another observation here. It could be argued that the camps were structures governed by extremely tight, interwoven
sets of rules, interdictions and laws (albeit arbitrary and useless ones), structures in which, therefore, the ‘all is possible’ assertion
was difficult to enact. After all, were camps not places where potentialities were so restricted than
the very act of creating a potentiality (by drawing, writing, creating or just surviving) was in
itself an act of resistance? The ‘all is possible’ was the fate of the most prisoners, but only in the sense that the ‘all’ could reduce
them to nothing with the minimal of delay. The ‘all or nothing’ logic favoured by Agamben fails to grasp
and envisage the entire range of potentialities. In fact, he reproduces and makes his own
the very logic upon which terror is built, a logic which from his early writings carries the
idea of the dominated man crushed by the omnipotent ‘all’. This logic is incarnated in the
‘muselmann’: it reveals a purist thought in which politics is envisaged under the exclusive sign of a paradigmatic absolute which
discredits any territory which is not political in its essence. This radical vision of politics, also shared by French philosopher Jean-Luc Nancy, can
be challenged through a citation by Jacques Rancière, who argues that politics ‘n’est jamais pure, jamais fondée sur une essence propre de la
communauté et de la loi’.35
A2: Agamben

AFF AT: LAW PART OF SOVEREIGN


Agamben’s assumption of the notion of law is incorrect. It does not have to be an order for
the sovereign. Rather, it can be an idea of a ‘rule’.
Hussain, 2000. (Nasser, Department of history, University of California, Berkeley, “Thresholds: Sovereignty and
the Sacred”, Law and Society Association, University of Massachusetts, Lexis Nexis.)

It is fairly safe to say that not only has the concept of sovereignty been undertheorized but that,
just as importantly, since the end of ancient regime monarchies and the rise and consolidation of
liberalconstitutional states, the need for such theorization has been considered doubtful. The
figure of the sovereign has been relegated to a repertoire of archaic images: the prerogative of
kings and the ritualistic majesty of despots and absolutist [*499] monarchs. Although the
question of sovereign power is not entirely absent from contemporary scholarship, increasingly
this question belongs either to strictly historical issues of the "king's two bodies" and the like or
to the legal theoretical problems of the distribution of sovereign power within a normative rule-
bound framework. A clear and well-known example of the latter is H.L.A. Hart's The Concept of
Law (1961).
Written in response to a positivist theory inherited from John Austin, which defines law as a
"command of the sovereign," Hart's "fresh response" in The Concept of Law aims to move
definitions of law away from notions of "orders, obedience, habits and threats" and toward "the
idea of a rule," without which, Hart insists, "we cannot hope to elucidate even the most
elementary forms of law" (Hart 1961: 78). Hart's "concept" of law is a complex combination of
primary rules of obligation with secondary rules, the latter which are found in more mature legal
systems and confer powers and stipulate procedures. This picture is generally well known and
we need not dwell on its intricacies here. What is important for our purposes is the way in which
sovereignty in Hart's schema is reduced entirely to a framework of rules. These rules, Hart
contends, are not just descriptive of the sovereign and those who obey him but are fundamental
and constitutive.
In a significant passage in The Concept of Law, Hart attempts to show how the notion of
sovereign orders virtually disappears in the rule-bound format of a modern electoral democracy.
Framing the explanation in the vocabulary of a historical Bildung, a developmental schema that
unself-consciously subtends much of the text, Hart argues that in the case in which the sovereign
is identifiable with a single person, it may be possible to concede that the rules of governance
(for instance, the requirement that orders must be declared and signed by the monarch) exist in a
descriptive mode. But in the more disseminated form of the electorate - indeed, in the case of
procedures that members of a society must follow in order to function as an electorate in the first
place - rules "cannot themselves have the status of orders issued by the sovereign, for nothing
can count as orders issued by the sovereign unless the rules already exist and have been
followed" (pp. 74-75). Such a circularity of logic and process effectively occludes the possibility
of action outside the circle.
A2: Agamben

AFF AT: AGAMBEN CREDIBLE


Agamben contradicts himself in his works.
Passavant, 2007. (Paul, A., “The Contradictory State of Giorgio Agamben”, Sage Journals Online.)

I argue that Giorgio Agamben employs two, contradictory theories of the state in his works.
Earlier works, such as The Coming Community and Means without End, suggest that the state
today functions as an aspect of the society of the spectacle where spectacle is the logical
extension of the commodity form under late capitalism. This part of Agamben's work attributes a
determined character to the state and a determining power to the economic forces of capitalism
that conditions particular forms of the state. Later work, such as Homo Sacer: Sovereign Power
and Bare Life and State of Exception, are preoccupied with the logic of juridical sovereignty and
the increased frequency of states of emergency. This part of Agamben's work attributes a
determining strength to the state under current conditions. Although his earlier work provides a
more coherent narrative of how it is possible to move from contemporary society to ideal
community, it does not provide the theory of political action necessary to overcome the power of
the state he describes when he theorizes the state in Homo Sacer and State of Exception. None of
the three possibilities of political action present in his later works provides passage beyond
state sovereignty without violating his philosophical commitments.
A2: Agamben

AFF AT: WHATEVER BEING


In order to be political, one must be at risk; the ‘whatever being’ is simply too
homogeneous a concept, closing off meaningful politics.
Wilson, 2006. (Matthew, W., Department of Geography, University of Washington, “Life at risk: interrogating
the political status of queering bodies”, p. 14-15.)

To be political, our bodies must also be vulnerable. This statement, Butler’s argument in
Precarious Life, calls our attention to identification and disidentification – precisely the paradox
Agamben is confronted with at the end of Homo Sacer. And yet, to be relevant, visible, subject,
and alive, we must engage in political projects that claim political status as such. The FDA and
its epidemiological evidence must be (and will continue to be) pressured to recognize and resist
the impulse to abjectify the gay male donor. The contradiction that I pose (following Agamben)
of critiquing and pressuring is to precisely engage in projects which (again) produce homo sacer
– the new threshold of difference. The STD guide represents a different perspective. While also
complicit in abjectifing queer bodies as at-risk bodies needing greater responsibility, the STD
guide is a project that seems less discriminatory at the surface. I argue, however, that these two
projects have similar biopolitical mechanisms; in each, which bodies matter is re-scripted
through discourse and in each, the unraveling body is the central figure of the narrative. The
queer body is exactly this unraveling body, caught up in a what Lee Edelman calls a “death
drive”: a term for “what the queer, in the order of the social, is called forth to figure: the
negativity opposed to every form of social viability” (2004: 9). ‘Life at risk’ and unraveling
bodies is queer by opposition to the social. And so, somewhat inconclusively, I return to the
question I posed earlier: Is presuming a life beyond (or not at) risk simply too risky? In other
words, is returning to a universal notion of ‘being’ – what Agamben (1993 [1990]) calls
“whatever” being, in The Coming Community – simply too homogeneous a concept, risking
the loss of exclusion that queer politics are somewhat founded upon? While to be political (and
therefore biopoliticized) is to also already be bare life and precarious life, resistance somehow
outside of the political is largely inconceivable. Further exploration of these thresholds of
(post)modern life is necessary – how are they constituted, and through what spaces – such that
the mechanisms of displacement and dissection could be undone (or at least that the adverse
effects of which could be lessened). Being mindful of the contradictions of such projects, as this
paper has attempted to present, is equally as necessary.
A2: Agamben

AFF AT: WHATEVER BEING


Agamben’s ‘whatever being’ is not possible at more than a personal lever. Agamben’s
impact will never happen that perpetual warfare will collapse the government ending the
state as an institution.
Cmiel, 1996. (Kenneth, Professor of Cultural History at Iowa, “The Fate of the Nation and the Withering of the
State”, American Literary History, Spring, p. 196, JSTOR)

If community cannot be a closed thing, if it is forever open to the potentially new, then the dream
of a national community is simply impossible. In Agamben's community, the idea of something
being "un- American" makes no sense, for there is no defining essence in a "whatever
singularity." Yet Agamben is also aware that capitalism and the state will continue. Indeed, he
recognizes that after the fall of Communism, they are sweeping the globe. Politics, in the future,
Agamben argues, will not be community building but the perpetual project of communities
against the state, "a struggle between the State and the non-State (humanity), an insurmountable
disjunction between whatever singularity and the State organization" (84). I doubt Agamben's
new community is actually coming. It remains far from clear that communities without
identities are emerging anywhere except in the febrile imaginations of a few philosophers. It
is not that I dislike the dream. It is for me the most attractive dream there is. It is that I am
skeptical that such "whatever singularities" are possible on more than the level of personal
behavior. Politics is too clunky for such subtlety. Even the new social movements seem far
more down-to-earth and prone to defining themselves than Agamben's theorizing. Politics,
alas, demands more leaden language. Still, the image of the state fighting communities is one
worth pondering. Its distance from earlier welfare state thinking could not be more dramatic.
Instead of the state embodying the will of the nation, we have a picture of numerous
communities at war with the state. It is, and I say this with no relish, a far more plausible picture
of our emerging politics than Walzer's happy pluralism. Just think of insurance companies,
Perotistas, and gay and lesbian activists-all communities distrustful of the state, all committed to
struggling with the state. Agamben does not ask what this perpetual warfare will do to
government. Like Walzer, he assumes that the state will trudge on as before. Yet if this
warfare between humanity and the state is constant, is it not plausible to surmise that
hostility to the state will become permanent? With the fiction that the state embodies the
nation's will dying, who will defend the state? Who will keep it from becoming the recipient
of increasing rancor and from being permanently wobbly? Isn't that a good way of
understanding recent politics in the US? And as for Agamben's own Italy the past decade has
revealed a public far more disgusted with the state than even in America.
at agamben: alternative fails

Agamben’s “coming community” is too weak to be sustainable.

Gordon 04 (Andrew, Lee and Juliet Folger Fund Professor of History at Harvard University, “Review Study: Rethinking Area
Studies, Once More,” Journal Of Japanese Studies, Vol. 30, No. 2, 2004, pg. 424-425)

Okada draws on Giorgio Agamben to argue for “singularities to form a community without affirming an identity.” Such a
community would be premised on a belief “that humans co-belong without any representable condition of belonging” (p. 200).
This is an ambitious but doomed quest. The sort of community here envisioned is devoid of the emotional attachments that
reinforce strong communities in real life. The sad part—and here I agree with Okada entirely— is that these emotions so easily
rest on feelings of exclusion or essentialist notions of identity; the sadder part is that I don’t see how the community he seeks
could generate loyalties sufficient to allow its survival.
at agamben: link over simplified

Agamben’s biopower is over-simplified and prevents us from confronting specific political circumstances.

Virno 02 (Paolo, PhD and Italian philosopher, “General intellect, exodus, multitude,” Archipelago No. 54, June 2002,
http://www.generation-online.org/p/fpvirno2.htm)
Agamben is a thinker of great value but also, in my opinion, a thinker with no political vocation. Then, when Agamben speaks of
the biopolitical he has the tendency to transform it into an ontological category with value already since the
archaic Roman right. And, in this, in my opinion, he is very wrong-headed. The problem is, I believe, that the biopolitical is only
an effect derived from the concept of labor-power. When there is a commodity that is called labor-power it is already implicitly
government over life. Agamben says, on the other hand, that labor-power is only one of the aspects of the biopolitical; I say the
contrary: over all because labor power is a paradoxical commodity, because it is not a real commodity like a book or a bottle of
water, but rather is simply the potential to produce. As soon as this potential is transformed into a commodity, then, it is
necessary to govern the living body that maintains this potential, that contains this potential. Toni (Negri) and Michael (Hardt), on
the other hand, use biopolitics in a historically determined sense, basing it on Foucault, but Foucault spoke in few pages of the
biopolitical - in relation to the birth of liberalism - that Foucault is not a sufficient base for founding a discourse over the
biopolitical and my apprehension, my fear, is that the biopolitical can be transformed into a word that hides, covers problems
instead of being an instrument for confronting them. A fetish word, an "open doors" word, a word with an exclamation point, a
word that carries the risk of blocking critical thought instead of helping it. Then, my fear is of fetish words in politics because it
seems like the cries of a child that is afraid of the dark..., the child that says "mama, mama!", "biopolitics, biopolitics!". I don't
negate that there can be a serious content in the term, however I see that the use of the term biopolitics sometimes is a
consolatory use, like the cry of a child, when what serves us are, in all cases, instruments of work and not propaganda words.
at agamben: nazis unique

Not all politics turn to Nazism—modern power structures are incredibly diverse.

Rabinow & Rose 03 (Paul, Professor of Anthropology at UC Berkeley, Nikolas, Professor of Sociology @ the London School of
Economics, “Thoughts On The Concept of Biopower Today,” December 10, 2003, http://www.lse.ac.uk/collections/sociology/
pdf/RabinowandRose-BiopowerToday03.pdf, accessed July 07, pg. 8-9)

The interpretation of contemporary biopolitics as the politics of a state modeled on the figure of the sovereign suits the twentieth
century absolutisms of the Nazis and Stalin. But we need a more nuanced account of sovereign power to analyze contemporary
rationalities or technologies of politics. Since these authors take their concept and point of reference from Foucault, it is worth
contrasting their postulate of a origin and beneficiary of biopower to Foucaultís remarks on sovereignty as a form of power whose
diagram, but not principle, is the figure of the sovereign ruler. Its characteristic is indeed ultimately a mode of power which relies
on the right to take life. However, with the exception of certain ‘paroxysmal’ moments, this is a mode of power whose activation
can only be sporadic and non-continuous. The totalization of sovereign power as a mode of ordering daily life would be too
costly, and indeed the very excesses of the exercise of this power seek to compensate for its sporadic nature. Sovereignty, in
this sense, is precisely a diagram of a form of power not a description of its implementation. Certainly some forms of colonial
power sought to operationalize it, but in the face of its economic and governmental costs, colonial statecraft was largely to take a
different form. The two megalomaniac State forms of the twentieth century also sought to actualize it, as have some others in
their wake: Albania under Hoxha, North Korea. But no historian of pre-modern forms of control could fail to notice the
dependence of sovereign rule in its non-paroxysmal form on a fine web of customary conventions, reciprocal obligations, and the
like, in a word, a moral economy whose complexity and scope far exceeds the extravagance displays of the sovereign.
Sovereign power is at one and the same time an element in this moral economy and an attempt to master it.

Not all biopolitics bring about genocide—it trivializes Nazism to say that all enactments of the state of exception are equivalent.

Rabinow & Rose 03 (Paul, Professor of Anthropology at UC Berkeley, Nikolas, Professor of Sociology @ the London School of
Economics, “Thoughts On The Concept of Biopower Today,” December 10, 2003, http://www.lse.ac.uk/collections/sociology/
pdf/RabinowandRose-BiopowerToday03.pdf, pg. 8-9)

Agamben takes seriously Adorno’s challenge “how is it possible to think after Auschwitz?” But for that very reason, it is to
trivialize Auschwitz to apply Schmitt’s concept of the state of exception and Foucault’s analysis of biopower to every instance
where living beings enter the scope of regulation, control and government. The power to command under threat of death is
exercised by States and their surrogates in multiple instances, in micro forms and in geopolitical relations. But this is not to say
that this form of power commands backed up by the ultimate threat of death is the guarantee or underpinning principle of all
forms of biopower in contemporary liberal societies. Unlike Agamben, we do not think that : the jurist the doctor, the scientist, the
expert, the priest depend for their power over life upon an alliance with the State (1998: 122). Nor is it useful to use this single
diagram to analyze every contemporary instance of thanato-politics from Rwanda to the epidemic of AIDS deaths across Africa.
Surely the essence of critical thought must be its capacity to make distinctions that can facilitate judgment and action.
at agamben: bare life

The concept of bare life over-determines the power of the state—theories that emphasize resistance are more powerful.

Cesarino & Negri 04 (Cesare, associate professor of cultural studies, Antonio, professor emeritus @ the Collège International de
Philosophie, “It’s a Powerful Life: A Conversation on Contemporary Philosophy,” Cultural Critique, Vol. 57, Spring 2004, pg.
172-173)
I believe Giorgio is writing a sequel to Homo Sacer, and I feel that this new work will be resolutive for his thought—in the sense that he will be forced in it to resolve and find a way out of the ambiguity that has
qualified his understanding of naked life so far. He already attempted something of the sort in his recent book on Saint Paul, but I think this attempt largely failed: as usual, this book is extremely learned and elegant;
it remains, however, somewhat trapped within Pauline exegesis, rather than constituting a full-fledged attempt to reconstruct naked life as a potentiality for exodus, to rethink naked life fundamentally in terms of
the concept of naked life is not an impossible, unfeasible one. I believe it is possible to push the image of power to
exodus. I believe that
the point at which a defenseless human being [un povero Cristo] is crushed, to conceive of that extreme point at which power
tries to eliminate that ultimate resistance that is the sheer attempt to keep oneself alive. From a logical standpoint, it is possible
to think all this: the naked bodies of the people in the camps, for example, can lead one precisely in this direction. But this is also
the point at which this concept turns into ideology: to conceive of the relation between power and life in such a way actually ends
up bolstering and reinforcing ideology. Agamben, in effect, is saying that such is the nature of power: in the final instance, power
reduces each and every human being to such a state of powerlessness. But this is absolutely not true! On the contrary: the
historical process takes place and is produced thanks to a continuous constitution and construction, which undoubtedly confronts
the limit over and over again—but this is an extraordinarily rich limit, in which desires expand, and in which life becomes
increasingly fuller. Of course it is possible to conceive of the limit as absolute pow-erlessness, especially when it has been
actually enacted and enforced in such a way so many times. And yet, isn't such a conception of the limit precisely what the limit
looks like from the standpoint of constituted power as well as from the standpoint of those who have already been totally
annihilated by such a power—which is, of course, one and the same standpoint? Isn't this the story about power that power itself
would like us to believe in and reiterate? Isn't it far more politically useful to conceive of this limit from the standpoint of those
who are not yet or not completely crushed by power, from the standpoint of those still struggling to overcome such a limit, from
the standpoint of the process of constitution, from the standpoint of power [potenza]?
at agamben: the camp

Suggesting that the camp is everywhere is silly—government power may be expansive but it does not always produce corpses—
Nazism was unique.

Levi & Rothberg 03 (Neil, Professor of English @ Drew University, Michael, Professor of English @ the University of Sydney,
“Auschwitz and the Remnants of Theory: Towards an Ethics of the Borderlands,” (11: 1/2), 2003, pg.30-31)

At the same time, Agamben's formulations strike us as problematic and inadequate in several respects. First, by restructuring the
"zone of the human" to conform to the condition of the Muselmann, Agamben removes the figure of the Muselmann from the
context-the camps-in which he or she is "produced ." The Muselmann becomes an isolated figure floating, like a Giacometti
sculpture, in an otherwise apparently empty abstract space that Agamben calls "humanity." The Muselmann is meant to bear a
certain truth about the nature of ethics "after Auschwitz," but is it not important when trying to articulate such an ethics to reflect
on what Auschwitz was?4 Surely such an account should attend to the historical, legal, and political conditions that led to the
development of the camp system, including the kinds of features that Zygmunt Bauman focuses on in Modernity and the
Holocaust - such as a massive, morally indifferent bureaucratic apparatus that dehumanized its "objects" and distanced its
agents from a sense of responsibility for their actions, as well as the obsessive hatred of the Jews that Saul Friedländer has
recently dubbed "redemptive antisemitism."5 If the Muselmann would not have existed without these factors, shouldn't an ethics
focused upon this figure also take account of them? Interestingly enough, in Homo Sacer Agamben himself argues that "the
camp" is the "nomos" (definitive political element) of the modern. In remarking that "[w]hat happened in the camps so exceeds
the juridical concept of crime that the specific juridico-political structure in which those events took place is often simply omitted
from consideration" (1998, 166), Agamben could be preparing a critique of what is omitted from Remnants of Auschwitz. Homo
Sacer argues that the camp is the space where the state of exception becomes normal and where "whether or not atrocities are
committed depends not on law but on the civility and ethical sense of the police who temporarily act as sovereign" (1998, 174).
This line of argument produces an antinomy in the Agamben oeuvre: for the Agamben of Homo Sacer a camp is a camp if
anything is possible within it, no matter whether or not it actually produces Muselmänner and corpses, while for the Agamben of
Remnants of Auschwitz the important fact about the Muselmann is simply that such a figure happened, not where and how he
became possible. What links the positions of his two works is a level of abstraction that deliberately brackets features of each
paradigm ordinarily understood as essential: for the camp, figures such as the Muselmann; for the Muselmann, the conditions of
the camp. Both moves permit Agamben to dismantle the boundary between the Nazi camps and the modern world . We have
already seen this in relation to the Muselmann, in the wake of whose existence all previously existing moral concepts must be
revised. It can be seen also in the examples of modern camps Agamben offers, including, "[t]he soccer stadium in Bari into which
the Italian police in 1991 provisionally herded all illegal Albanian immigrants," the zones d'attentes in French international airports
where foreigners requesting refugee status are held, and even, he suggests in an earlier version of the essay, gated
communities in the USA (1998, 174).6 At such moments Agamben seems to be suggesting that Auschwitz is potentially
everywhere, a suggestion that ends up eliding the specific challenges posed both by the Muselmann and the camp system.
at agamben: muselmann

Agamben’s claim that the Muselmann is the ‘complete witness’ undermines the historical importance of other positions within
Auschwitz. This is the equivalent of denying that other survivors had authentic experiences and must be rejected.

Levi & Rothberg 03 (Neil, Professor of English @ Drew University, Michael, Professor of English @ the University of Sydney,
“Auschwitz and the Remnants of Theory: Towards an Ethics of the Borderlands,” (11: 1/2), 2003, pg.31-32)

We would also identify a second problem with Agamben's approach: the grounds for Agamben's selection of the Muselmann as
the "complete witness" are not clear. Ethics after Auschwitz must take account of the Muselmann, but that does not justify
transforming him into a fetish, the sole site of the truth of the camps. If Levi's own testimony is on his own account unrepresentative, that surely does not mean that it has
no truth content. The fact that Levi himself distrusts the testimony of, say, former members of the Sonderkommando (the camp inmates who were forced, under threat of death, to operate the crematoria) is no reason
to disqualify such testimony out of hand. The power of Claude Lanzmann's astonishing film Shoah derives in no small part from the testimony of a former "crematorium raven" (P. Levi 60). Despite his attempt to
develop a complex theory of testimony premised on the relationship between the Muselmann and the surviving witness, Agamben ultimately homogenizes the site of witness by polarizing those positions. While there
is warrant for such a reading in Levi's texts (e.g., Levi's notion of "the drowned and the saved"), those texts also include the hypothesis of "the gray zone," a zone of ethical uncertainty in which figures such as the
Sonderkommando are paradigmatic. In fact, testimony from the gray zone may prove as illuminating about the ethical challenges of the Nazi genocide as that derived from an understanding of Levi's paradox. Despite
the serious reservations expressed by Levi about the testimonies of figures who were forced into the most terrible complicity with the Nazis, such testimonies have been shown to be of great value in understanding
.7 In what remains one of the most profound attempts to "think" the
the Nazi genocide, and, indeed, in making clear the need for theoretical innovation in order to do so
Nazi genocide, historian and social theorist Dan Diner proposes that Nazi action can be most effectively illuminated from the
perspective of the gray zone, and particularly that of the Judenräte - the Jewish councils who ran the ghettos and were charged
to make decisions about who would be allowed to work and who would be sent to the camps (130-137). The councils negotiated
on the assumption that the Nazis were rational - specifically, that they would not want to exterminate a productive labor source
while at war. The Nazis utilized this assumption to facilitate the killing process, with which the councils found themselves
unsuspectingly cooperating. It is the Jewish councils' experience of participating in their own destruction while acting according to
the logic of self-preservation that Diner terms the counterrational. And it is in reflecting on the Jewish experience of Nazi
counterrationality that Diner says we encounter the limits of historical understanding. Only at this limit point, according to Diner,
can we begin to "think the Nazis" via what he calls negative historical cognition. While we wouldn't want to generalize the
standpoint of the Judenräte as the essence of the Holocaust any more than we would that of the Muselmann, when read
alongside each other the arguments of Agamben and Diner strongly suggest the importance of multiplying the epistemological
standpoints from which we approach the Nazi genocide.
***A2: Colonialism***

at colonialism: US not an empire

The is mischaracterized as an empire—reciprocal economic partnerships and democratic agreements are the norm.
Ikenberry, 04. Professor of Geopolitics. G. John Ikenberry. “Illusions of Empire: Defining the New American Order” Foreign
Affairs, March/April 2004.

Is the United States an empire? If so, Ferguson's liberal empire is a more persuasive portrait than is Johnson's military empire.
But ultimately, the notion of empire is misleading -- and misses the distinctive aspects of the global political
order that has developed around U.S. power. The United States has pursued imperial policies, especially toward weak
countries in the periphery. But U.S. relations with Europe, Japan, China, and Russia cannot be described as imperial,
even when "neo" or "liberal" modifies the term. The advanced democracies operate within a "security
community" in which the use or threat of force is unthinkable. Their economies are deeply interwoven.
Together, they form a political order built on bargains, diffuse reciprocity, and an array of
intergovernmental institutions and ad hoc working relationships. This is not empire; it is a U.S.-led democratic
political order that has no name or historical antecedent.To be sure, the neoconservatives in Washington have trumpeted their own imperial vision: an era of global rule organized around the bold
unilateral exercise of military power, gradual disentanglement from the constraints of multilateralism, and an aggressive effort to spread freedom and democracy. But this vision is founded on illusions of U.S. power. It
fails to appreciate the role of cooperation and rules in the exercise and preservation of such power. Its pursuit would strip the United States of its legitimacy as the preeminent global power and severely compromise
, the
the authority that flows from such legitimacy. Ultimately, the neoconservatives are silent on the full range of global challenges and opportunities that face the United States. And as Ferguson notes

American public has no desire to run colonies or manage a global empire. Thus, there are limits on
American imperial pretensions even in a unipolar era. Ultimately, the empire debate misses the most important
international development of recent years: the long peace among great powers, which some scholars argue marks the end of
great-power war. Capitalism, democracy, and nuclear weapons all help explain this peace. But so too does the unique way in
which the United States has gone about the business of building an international order. The United States' success stems
from the creation and extension of international institutions that have limited and legitimated U.S. power.

Hegemony doesn’t equate to empire—other nations can choose to disengage from US security guarantees.

Ikenberry, 04. Professor of Geopolitics. G. John Ikenberry. “Illusions of Empire: Defining the New American Order” Foreign
Affairs, March/April 2004.
Johnson also offers little beyond passing mention about the societies presumed to be under Washington's thumb.

Domination and exploitation are, of course, not always self-evident. Military pacts and security
partnerships are clearly part of the structure of U.S. global power, and they often reinforce fragile and
corrupt governments in order to project U.S. influence. But countries can also use security ties with the
United States to their own advantage. Japan may be a subordinate security partner, but the U.S.-Japan
alliance also allows Tokyo to forgo a costly buildup of military capacity that would destabilize East Asia.
Moreover, countries do have other options: they can, and often do, escape U.S. domination simply by
asking the United States to leave. The Philippines did so, and South Korea may be next. The variety and complexity
of U.S. security ties with other states makes Johnson's simplistic view of military hegemony misleading.
at colonialism: us not an empire

Global pluralism makes empire impossible—the US has influence but not the control described by the negative.

Zelikow, 03 “Transformation of National Security” Philip Zelikow. Professor of History and Public Affairs, University of Virginia.
National Interest, Summer 2003, pg. 18-10 Lexis).

But these imperial metaphors, of whatever provenance, do not enrich our understanding; they impoverish it. They use a
metaphor of how to rule others when the problem is how to persuade and lead them. Real imperial power is sovereign
power. Sovereigns rule, and a ruler is not just the most powerful among diverse interest groups. Sovereignty means a direct
monopoly control over the organization and use of armed might. It means direct control over the administration of
justice and the definition thereof. It means control over what is bought and sold, the terms of trade and the
permission to trade, to the limit of the ruler's desires and capacities. In the modern, pluralistic world of the 21st
century, the United States does not have anything like such direct authority over other countries, nor does it
seek it. Even its informal influence in the political economy of neighboring Mexico, for instance, is far
more modest than, say, the influence the British could exert over Argentina a hundred years ago. The
purveyors of imperial metaphors suffer from a lack of imagination, and more, from a lack of appreciation for the new conditions
under which we now live. It is easier in many respects to communicate images in a cybernetic world, so that a very powerful
United States does exert a range of influences that is quite striking. But this does not negate the
proliferating pluralism of global society, nor does it suggest a will to imperial power in Washington. The
proliferation of loose empire metaphors thus distorts into banal nonsense the only precise meaning of the
term imperialism that we have. The United States is central in world politics today, not omnipotent . Nor is
the U.S. Federal government organized in such a fashion that would allow it to wield durable imperial power around the world-it
has trouble enough fashioning coherent policies within the fifty United States. Rather than exhibiting a confident will to power, we
instinctively tend, as David Brooks has put it, to "enter every conflict with the might of a muscleman and the mentality of a wimp."
We must speak of American power and of responsible ways to wield it; let us stop talking of American
empire, for there is and there will be no such thing.

The US focuses on spreading democracy- their claims of empire are outdated.

Boot, 03 (“Neither new nor nefarious: the liberal empire strikes back” Max Boot, fellow of the Council of foreign relations, Current
History, Vol. 102, Iss. 667; pg. 361 Nov. 2003. Pro Quest)

If the Europeans, with their long tradition of colonialism, have found the price of empire too high, what
chance is there that Americans, whose country was born in a revolt against empire, will replace the
colonial administrators of old?
Not much. The kind of imperial missions that the United States is likely to undertake today are very
different. The Europeans fought to subjugate "natives"; Americans will fight to bring them democracy
and the rule of law. (No one wants to put Iraq or Afghanistan permanently under the Stars and Stripes .)
European rule was justified by racial prejudices; American interventions are justified by self-defense and
human rights doctrines accepted (at least in principle) by all signatories to the Universal Declaration of Human Rights.
European expeditions were unilateral; American missions are usually blessed with international approval, whether
from the United Nations, NATO, or simply an ad hoc coalition. Even the US intervention in Iraq this year, widely
held to be "unilateral," enjoys far more international support (and hence legitimacy) than, say, the French role in Algeria in the
1950s.
at colonialism: US not an empire

Multilateralism an inevitable check on the possibility of empire.

Zelikow, 03 “Transformation of National Security” Philip Zelikow. Professor of History and Public Affairs, University of Virginia.
National Interest, Summer 2003, pg. 18-10 Lexis).

Everything that America does in the world is done multilaterally. That emphatically includes the policies
the Bush Administration considers most important, and even those that are the most "military" in
character. The global war against terrorism is being conducted through an elaborate, often hidden, network of multilateral
cooperation among scores of governments. A large number of players are interacting on intelligence, law enforcement, military
action, air transportation, shipping, financial controls and more. Ongoing military operations in Afghanistan involve several
countries, and were multilateral even at the height of American military activity, as the United States relied heavily on
relationships with Pakistan, Russia, three Central Asian governments and a variety of Afghan factions. The caricature of the
administration's unilateralism usually rests on the recitation of a by now standard list of diplomatic
actions that some other governments did not like (Kyoto, the International Criminal Court and so on). Some of these
disagreements were handled in a style and manner that seemed insensitive or simply maladroit. Unfortunately, too, the
caricature of the administration's unilateralism is willingly fed by some U.S. officials and unofficial
advisers who relish the chance to play the role of the truth teller lancing foreign obfuscation s. Sometimes
they overplay the part, sensing the license they get from working for a plain-spoken president.
at colonialism: empire good

Imperialism is good: the defeat of Nazism and the promotion of democracy are a force for good.

Boot, 03 “American Imperialism? No need to run away from Label” Max Boot, Senior fellow of the Council of foreign relations,
USA Today, May 6, 2003. http://66.102.1.104/scholar?
hl=en&lr=&q=cache:sP5soPyDtzAJ:www.attacberlin.de/fileadmin/Sommerakademie/Boot_Imperialim_fine.pdf+author:max+autho
r:boot).

Mind you, this is not meant as a condemnation. The history of American imperialism is hardly one of unadorned good doing;
there have been plenty of shameful episodes, such as the mistreatment of the Indians. But, on the whole , U.S. imperialism
has been the greatest force for good in the world during the past century. It has defeated the monstrous
evils of communism and Nazism and lesser evils such as the Taliban and Serbian ethnic cleansing. Along the way , it
has helped spread liberal institutions to countries as diverse as South Korea (news - web sites) and
Panama. Yet, while generally successful as imperialists, Americans have been loath to confirm that's what they were doing.
That's OK. Given the historical baggage that ''imperialism'' carries, there's no need for the U.S. government
to embrace the term. But it should definitely embrace the practice . That doesn't mean looting Iraq of its natural
resources; nothing could be more destructive
of our goal of building a stable government in Baghdad. It means imposing the rule of law, property rights, free
speech and other guarantees, at gunpoint if need be. This will require selecting a new ruler who is
committed to pluralism and then backing him or her to the hilt. Iran and other neighboring states won't
hesitate to impose their despotic views on Iraq; we shouldn't hesitate to impose our democratic views.
at empire: terrorism must be confronted

Pointing out flaws with imperialism is not enough—there are real threats posed by terrorism that the alternative must be able to
solve.

Gitlin, 06 - Professor of Journalism and Sociology at Columbia University - 2006(Todd, The Intellectuals And The Flag, p. 151)

During the Bush years intellectuals have had their work cut out for them exposing the arrogance of empire, piercing its
rationalizations, identifying its betrayal of patriotic traditions. But all that said, serious questions remained about what intellectuals
of the left wanted: What was to be done about fighting the jihadists and improving democracy’s chances? What roles made
sense for the United States, the United Nations, NATO, or anyone else? What was required of governments, nongovernmental
organizations, foundations, and private initiatives? Given that the Iraq War had been ill advised, what should be done next about
Iraq and Iraqis? About such questions many intellectuals of the left were understandably perplexed—and sometimes evasive.
Foreign policy wasn’t “their problem.” Their mode was critical and back-glancing, not constructive and prospective. It was useful
to raise questions about the purposes of U.S. bases abroad, for example. It was satisfying, but not especially useful, to think that
the questions answered themselves. So the intellectuals’ evasion damaged what might have been their contribution to the larger
debate that the country needed—and still needs—on its place in the world and how it protects itself. Liberal patriots would refuse
to be satisfied with knee-jerk answers but would join the hard questions as members of a society do—members who criticize in
behalf of a community of mutual aid, not marginal scoffers who have painted themselves into a corner. Liberal patriots would not
be satisfied to reply to consensus truculence with rejectionist truculence. They would not take pride in their marginality. They
would consider what they could do for our natural allies, democrats abroad. They would take it as their obligation to illuminate a
transformed world in which al Qaeda and its allies are not misinterpreted as the current rein-carnations of the eternal spirit of
anti-imperialism. They would retain curiosity and resist that hardening of the categories that is a form of self-protection against
the unprecedented.
*** Ontology ***

A2: Ontology

Preventing widespread death precedes ontological questioning


Davidson ‘89 (Arnold L., Associate Prof Philosophy – U Chicago, Critical Inquiry, Winter, p. 426)

I understand Levinas’ work to suggest


another path to the recovery of the human, one that leads through
or toward other human beings: “The dimension of the divine opens forth from the human face… Hence metaphysics is
enacted where the social relation is enacted- in our relations with men… The Other is not the incarnation of God, but precisely by his
face, in which he is disincarnate, is the manifestation of the height in which God is revealed. It is our relations with men… that give to theological concepts the sole
Levinas places ethics before ontology by beginning with our experience of the
signification they admit of.”
human face: and, in a clear reference to Heidegger’s idolatry of the village life of peasants, he associated himself with Socrates,
who preferred the city where he encountered men to the country with its trees. In his discussion of skepticism and
the problem of others, Cavell also aligns himself with this path of thought, with the recovery of the finite
human self through the acknowledgement of others: “As long as God exists, I am not alone. And couldn’t the other suffer the fate
of God?… I wish to understand how the other now bears the weight of God, shows me that I am not alone in the universe. This requires understanding the
The suppression of the other, the human,
philosophical problem of the other as the trace or scar of the departure of God [CR, p.470].”
in Heidegger’s thought accounts, I believe, for the absence, in his writing after the war, of the experience of
horror. Horror is always directed toward the human; every object of horror bears the imprint of
the human will. So Levinas can see in Heidegger’s silence about the gas chambers and death
camps “a kind of consent to the horror.” And Cavell can characterize Nazis as “those who have
lost the capacity for being horrified by what they do.” Where was Heidegger’s horror? How could he have failed to
know what he had consented to? Hannah Arendt associates Heidegger with Paul Valery’s aphorism, “Les evenements
ne sont que l’ecume des choses’ (‘Events are but the foam of things’).” I think one understands the source of her intuition. The
mass extermination of human beings, however, does not produce foam, but dust and ashes; and it
is here that questioning must stop.
A2: Ontology

It’s impossible to determine an answer to being –-- ontological questioning results in an


infinite regress and total political paralysis
Levinas and Nemo ‘85 (Emmanuel, Professor of Philosophy, and Philippe, Professor of New Philosophy, Ethics and
Infinity, p. 6-7)
Are we not in need of still more precautions? Must we not step back from this question to raise another, to
recognize the obvious circularity of asking what is the “What is . .?“ question? It seems to beg the question. Is our new
suspicion, then, that Heidegger begs the question of metaphysics when he asks “What is poetry?” or
“What is thinking?”? Yet his thought is insistently anti-metaphysical. Why, then, does he retain
the metaphysical question par excellence? Aware of just such an objection, he proposes, against the vicious circle of the
petitio principi, an alternative, productive circularity: hermeneutic questioning. To ask “What is. . .?“ does not partake of onto-theo-logy
if one acknowledges (1) that the answer can never be fixed absolutely, but calls essentially, endlessly, for additional “What is . . .?“
questions. Dialectical refinement here replaces vicious circularity. Further, beyond the openmindedness called for by dialectical refinement,
hermeneutic questioning (2) insists on avoiding subjective impositions, on avoiding reading into rather than harkening
to things. One must harken to the things themselves, ultimately to being, in a careful attunement to what is. But do the
refinement and care of the hermeneutic question — which succeed in avoiding ontotheo-logy succeed in avoiding all viciousness? Certainly they
convert a simple fallacy into a productive inquiry, they open a path for thought. But is it not the case that however much
refinement and care one brings to bear, to ask what something is leads to asking what something
else is, and so on and so forth, ad infinitum? What is disturbing in this is not so much the infinity of interpretive depth,
which has the virtue of escaping onto-theo-logy and remaining true to the way things are, to the phenomena, the coming to be and
passing away of being. Rather, the problem lies in the influence the endlessly open horizon of
such thinking exerts on the way of such thought. That is, the problem lies in what seems to be the
very virtue of hermeneutic thought, namely, the doggedness of the “What is . . .?“ question, in its
inability to escape itself, to escape being and essence.
A2: Ontology

Survival is a prerequisite to evaluating ontology – you can’t relate to the world without
being in the world
Robbins ’99 Brent Dean Robbins, doctoral student in clinical psychology @ Duquesne University. “Medard Boss.”
http://mythosandlogos.com/Boss.html

Medicine, deriving its foundations from Descartes, begins with an understanding of a split between
subject and object and between mind and body. As such, medicine approaches the human body as a thing
subject to causal, mechanistic processes like inanimate objects in nature. For Boss, however, the human being is
precisely not a thing, and thus the body cannot be understood as a thing. Rather, the body is primarily an
existential-body, the means by which we are a being-in-the-world and "body-forth" our
possibilities. It is as bodies that we exist in existential space. But this space is not the res extensa of
Descartes; it is not mere geometrical space. "Human beings," writes Boss, "are, as an open, clear realm of
perception, so essentially spatial that they dwell from the beginning with whatever is accessible
to perception, and in a way suited to the meaning they perceive." (90) That is, for human beings,
spatiality is part of the ontological structure of the human being, without which our being-in-the-
world would not be possible. This is a spatiality which is an openness to significance, to how things matter to
us. As such, spatiality is meaningful and consists of the context of significance which is the world.
As Being-there (Dasein), the human being is always with the things of the world, "actually at the place where the
thing is present." (92) In this sense, the body is not a thing, but the existential center by which things can presence to
us in our world-openness. Yet, in my experience, I am not simply here at my body; rather, it is my body which is the
openness to the "there" which is the meaningful world of perception. Things, unlike my body, gather a world of
meaning. They gather together the contextual signficance of the world as mattering to me as a human being. Thus,
too, the thing is not a representation in my head: "When we visualize something, we establish a relationship to the
thing itself, not to some mere subjective representation of it inside us." (92) In my experience, I am at the thing, I am
in-the-world as a embodied being.

Evaluate our disads on the same level as ontology – survival coincides with the opening of
ontological space
Robbins ‘99Brent Dean Robbins, doctoral student in clinical psychology @ Duquesne University. “Medard Boss.”
http://mythosandlogos.com/Boss.html

Space and time, as such, make possible and yet are equiprimordial with human bodyhood. Yet,
while natural science views the human body as some self-contained material thing, by doing so it "disregards
everything that is specifically human about human bodyhood." (100) The human body, unlike a thing, is not
limited like the material borders of inanimate objects. Rather, as a world-openness, human
embodiment is an opening onto things "there" in the world, while things are self-contained and have no
experience what-so-ever. The human body does not end at the skin, but existentially opens onto a
world of possibilities which are significant. "The borders of my bodyhood coincide with those of
my openness to the world," writes Boss. "They are in fact at any given time identical, though they
are always changing with the fluid expansion and contraction of my relationships to the world."
(103)
A2: Ontology

our ethical obligation to save lives precedes ontology – the value of life comes from human
agency, which death destroys
Robbins ‘99Brent Dean Robbins, doctoral student in clinical psychology @ Duquesne University. “Medard Boss.”
http://mythosandlogos.com/Boss.html

*THIS CARD IS GENDER-MODIFIED

"Death is an unsurpassable limit of human existence," writes Boss (119). Primarily, however, human
beings flee from death and the awareness of our mortality. But in our confrontation with death and our morality, we
discover the "relationship" which "is the basis for all feelings of reverance, fear, awe, wonder, sorrow, and deference
in the face of something greater and more powerful." (120). Boss even suggests that "the most dignified human
relationship to death" involves keeping it--as a possibility rather than an actuality--constantly in awareness without
fleeing from it. As Boss writes: "Only such a being-unto-death can guarantee the precondition that the Dasein be
able to free itself from its absorption in, its submission and surrender of itself to the things and relationships of
everyday living and to return to itself." (121) Such a recognition brings the human being back to his
responsibility for his existence. This is not simply a inward withdrawal from the world--far from
it. Rather, this responsible awareness of death as the ultimate possibility for human existence
frees the human being to be with others in a genuine way.
From this foundation--based on the existentials described above--Boss is able to articulate an understanding
of medicine and psychology which gives priority to the freedom of the human being to be itself.
By freedom, Boss does not mean a freedom to have all the possibilites, for we are finite and
limited by our factical history and death. Yet within these finite possibilities, we are free to be
who we are and to take responsibility for who we are in the world with others and alongside
things that matter. Psychotherapy comes into play in cases in which people suffer from "pathological
deficiencies of freedom," who, while constricted, still retain a degree of freedom, but a freedom which includes a
suffering from constrictedness. The therapist, in this regard, provides the client with a space to free up this
constricted existence in order to discover previously foreclosed possibilities of being in the world.
A2: Ontology

Our ethical obligation to save lives precedes ontology


Taminiaux 2003 Jacques, professor of philosophy @ Boston College. “The presence of Being and Time in Totality and Infinity”

Levinas fully agrees with the notion of a metaphysical desire as a desire for the other but he fully
disagrees with the supposed accomplishment and satisfaction of that desire in a final visibility.
Metaphysics, he says, is désir de l'invisible. The other is not at all offered to a vision of the Self. It is
desired as invisible.
Hence the meaning of the word transcendence in Levinas use of it. Since
the metaphysical desire aims to
the otherness of the other without possible satisfaction or fulfillment in an ultimate vision, the
movement of such desire is transcendence. The word designates an elevation. It is a «transascendance»(35),
Levinas says. Transascendance is a relation between myself and the absolute exteriority of the other which is such
that the Self and the Other cannot be part of a visible totality in which their relation would be symmetrical and
reversible. In other words, transascendance is a «breach of totality»(35).
Upon close inspection it appears that a confrontation with Heidegger is involved in that characterization of
metaphysics and of transcendence. The confrontation is, so to speak, condensed in Levinas' strong formula:
«Metaphysics precedes ontology». Metaphysics has precedence over ontology. By contrast, Heidegger claims
that metaphysics accomplishes itself in ontology, that is in the vision attainable by the human Dasein of what it
means to be. The Greek word for vision is theoria. Heidegger again and again insists in Being and Time on the
precedence of the bios theoretikos, contemplation as the highest way of life.
In Levinas' analysis the trouble with theoria is that it does not fit with metaphysical desire because it
does not respect the alterity of the other. To be sure it claims to let what it contemplates manifest
itself for its own sake but since its contemplation is a matter of understanding it always
renounces the marvel of exteriority by absorbing the other into the Self thanks to a third term that
the knowing subject finds in itself. So does Heidegger's ontology by finding the key to the
meaning of Being in my own temporality. The primacy of the question of Being in Heidegger's
thought leads to a self-sufficiency, to egoism. Metaphysical desire as understood by Heidegger is
a desire to be properly myself to the detriment of the Other. Ontology is an Egology.
This is what confirms Heidegger's notion of transcendence. Transcendence in Being and Time is not a movement of
elevation towards the other but a movement through which the individual Dasein, by overcoming what is not
properly its own, elevates itself to an insight into what is exclusively its own, its ownmost possibility, the possibility
of its own death. Dasein becomes authentic by confronting its own mortality. Transcendence in
Heidegger's sense is essentially a return to the Selfhood of the Self. It is a totalization.
According to Levinas there is only one way for transcendence to avoid that totalization, to be a
breach of totality: it is by being ethical, by acknowledging the primacy of the Other over the
Self.
To say that Metaphysics precedes ontology amounts to claim that Ethics precedes ontology,
whereas in Heidegger ontology precedes ethics. Levinas used to quote repeatedly Plato's famous formula:
to agathon epekeina tès ousias and to translate it into: the Good is beyond Being. It is significant that Heidegger also
used to quote repeatedly the same formula but to deprive the motto of an essentially ethical connotation by reading it
as meaning Being is beyond beings. In other words what is at stake in Plato's formula for Heidegger is merely my
elevation towards my ownmost possibility, not at all my elevation to the height of the Other.
A2: Ontology

Focus on ontology misses the boat – it ignores broader question of practice, which are key
to avoid nihilism
Backhouse 2002
Roger E., “The Economic Worldview:  Studies in the Ontology of Economics.” http://www.blackwell-synergy.com/links/doi/10.1111/1468-
0297.t01-11-00083/full/
 It might be thought that, between them, these two volumes must provide a reasonably thorough coverage of the broad range of thinking on
economic methodology from scientific realism to the idea that our view of the world is socially constructed. However, my response was that,
projects of ontology and postmodernism in
though there are papers in both volumes that are of great interest, the
themselves leave me fairly cold. The main reason for this is that the most fruitful work in
methodology over the last decade or so has arguably centred on analysing what economists do.
Thus beliefs about the economic world (economic ontology) acquire significance from the
context in which the economists who hold them are acting. There are chapters in EWV that deal with
economists' practices but to focus on ontology rather than practices is to marginalise what should be
central. Examination of practice is also relevant to the question, addressed in some chapters of PAK, of
whether postmodernism amounts to nihilism. Meaning is socially constructed but to understand
scientific knowlege it is important to take into account the constraints that are imposed by our
ability to do some things and our inability to do other things.

Ethics precedes ontology – the fundamental fact of being is vulnerability and responsibility
to the other
Merleau ’04 Chloë Taylor Merleau, doctoral candidate in philosophy @ University of Toronto, Canada, “Levinasian Ethics and Feminist
Ethics of Care.” www.sspp.us/Protected-Essays/2004-SPEP-Merleau.doc
Levinas argues against the Western philosophic tradition, and against Heidegger in particular, that ontology is not
fundamental. Rather, for Levinas, ethics, a relation to and for an other, is prior to being, grounding
human existence. Before we “are,” we are already in a relation to others, whatever the order of the
verb in this sentence. It is therefore a mistake to begin by theorizing what the being of the self is,
independent of its relations of vulnerability and responsiveness to others, because the self never
is independent of or prior to these ethical terms. Ethics is the fundamental human experience,
and is grounded in relations to others. Mainstream ethical philosophy, however, has inherited the notion of the self assumed
by traditional metaphysics, and thus has been concerned with a subject it presumes to be autonomous and free, independent of others and faced
Even the duties towards others that such ethical
with abstract questions about its own rights, duties, and freedoms.1
theories imagine have typically been duties towards other abstractly-conceived autonomous
agents who are the same as ourselves, and first and foremost is the duty to not interfere in their
rights and freedoms. Philosophy has thus not conceived of ethics as it occurs in our most
fundamental experiences, as a responsiveness to others who are vulnerable to us and to whom we
are vulnerable, and with whom we are in encounters and relations which involve difference and
inequalities in power. Philosophy has not, therefore, approached ethics in terms of the situations
in which it is perhaps most frequently and most desperately required, in relation to those who are
exposed to us in need, requiring our interference and response.

1
A2: Ontology

Our ethical obligation to the other disrupts ontology


Merleau ’04 Chloë Taylor Merleau, doctoral candidate in philosophy @ University of Toronto, Canada, “Levinasian Ethics and Feminist
Ethics of Care.”
www.sspp.us/Protected-Essays/2004-SPEP-Merleau.doc

Both feminist care theorists and Levinas have understood the receptivity towards others of ethics,
the prioritization of responsibility over freedom, as “feminine,” and both have conceived of
maternity as a paradigm (amongst others) for caring, or for being responsible for others.2 The
theorization of ethics as “feminine” occurs in Levinas’s earlier writings, while his development of maternity as trope
for the ethical relation is explored in the major work of his mature philosophy, Otherwise than Being or Beyond
Essence. In his earlier works, such as Time and the Other and Existence and Existents, Levinas develops an ethics
of alterity against the Western tradition of the Same while describing “the feminine” as having alterity as its essence
(1983, 85), and as being “the other par excellence” (1978, 85). The feminine is not different in terms of
qualities, nor different in relation to the masculine, but is difference itself, the very possibility of
ethics.3 At this stage, “the feminine” is the otherness of the Other whom one encounters in the
ethical relation, as well as being the principle of that relation. In Totality and Infinity, however, the
Other of the ethical encounter has become generic, simply human, and yet “the feminine” continues to play a crucial
role. As Catherine Chalier writes, the feminine in Totality and Infinity “stops the project of being,”
“stops this blind strength,” and thus interupts masculine ontology with ethics, disrupts military
values with her welcome, replaces transcendence with proximity and intimacy (1991, 123). In
“Judaism and the Feminine Element,” Levinas describes the masculine “outdoor” world as “hard and
cold,” alienating and ontological: “it neither clothes those who are naked nor feeds those who are hungry […]
Spirit in its masculine existence […] lives outdoors.” (1976, 33) In contrast, the feminine is indoors or domestic,
called “dwelling.” As Levinas notes, however, this “habitation is not yet the transcendence of language. The
Other who welcomes in intimacy is not the you [vous] of the face that reveals itself in a
dimension of height, but precisely the thou [tu] of familiarity.” (1979, 155) At this stage, then,
the feminine other seems no longer the Other of the ethical relation, and yet femininity remains
the principle that interupts masculinity and makes ethics possible.

2
3
A2: Spanos

Spanos' rejection of Humanism destroys any political project and/ or allies and allows the
right to take over
Perkin 1993 [J. Russell, Professor of English at Saint Mary's University, Theorizing the Culture Wars, Postmodern Culture PMC 3.3,
http://muse.jhu.edu/journals/postmodern_culture/v003/3.3r_perkin.html]
My final criticism is that Spanos. by his attempt to put all humanists into the same category and to break totally with the tradition of humanism , isolates
himself in a posture of ultra leftist purity that cuts him off from many potential political allies,
especially when, as I will note in conclusion. His practical recommendations for the practical role of an adversarial
intellectual seem similar to those of the liberal pluralists he attacks. He seems ill-informed about
what goes on in the everyday work of the academy, for instance, in the field of composition studies. Spanos laments the
"unwarranted neglect" (202) of the work of Paulo Freire, yet in reading composition and pedagogy journals over the last few years, I have noticed few thinkers who
have been so consistently cited. Spanos refers several times to the fact that the discourse of the documents comprising The Pentagon Papers was linked to the kind of
discourse that first-year composition courses produce (this was Richard Ohmann's argument): here again, however, Spanos is not up to date. For the last decade the
The academy, in
field of composition studies has been the most vigorous site of the kind of oppositional practices The End of Education recommends.
short, is more diverse. more complex, more genuinely full of difference than Spanos allows, and
it is precisely that difference that neoconservatives want to erase. By seeking to seperate out only
the pure (posthumanist) believers, Spanos seems to me to ensure his self-marginalization. For example, several
times he includes pluralists like Wayne Booth and even Gerald Graff in lists of "humanists" that include William Bennett, Roger Kimball and Dinesh D'Souza. Of
course. there is a polemical purpose to this, but it is one that is counterproductive. In fact, I would even question the validity of calling shoddy and often inaccurate
journalists like Kimball and D'Souza with the title "humanist intellectuals." Henry Louis Gates's final chapter contains some cogent criticism of the kind of position,
which Spanos has taken. Gates argues that the "hard" left's opposition to liberalism is as mistaken as its opposition to conservatism, and refers to Cornel West's
"If you don't build on liberalism, you build on air" (I 87). Building on
remarks about the field of critical legal studies,
air seems -- to me precisely what Spanos is recommending. Gates, on the other hand, criticizes "those rnassively totalizing
theories that marginalize practical political action as a jejune indulgence" (192), and endorses a coalition of liberalism and the left. Spanos misrepresents history and
fails to correctly build off others’ philosophies Perkin 1993 [J. Russell, Professor of English at Saint Mary's University, Theorizing the Culture Wars, Postmodern
the book makes huge
Culture PMC 3.3, http://muse.jhu.edu/journals/postmodern_culture/v003/3.3r_perkin.html] Another problem is that
historical assertions that have the effect of lessening difference, even while it attacks the
metaphysical principle "that identity is the condition for the possibility of difference and not the
other way around" (4; emphasis in original). This is something Spanos has in common with some followers of Derrida who turn deconstruction into a
dogma, rather than realizing that it is a strategy of reading that must take account of the particular logic of the texts being read. Spanos asserts that the classical Greeks
were characterized by "originative, differential, and errant thinking" (105), which every subsequent age, beginning with the Alexandrian Greek, through the Romans,
This not only
the Renaissance, the Enlightenment, and the Victorians, and right up to the present, misunderstood in a reifying and imperialistic appropriation.
implies a somewhat simplistic reception-history of ancient Greek culture; it also, significantly,
perpetuates a myth--the favourite American myth that Spanos in other contexts attacks in the book--of an original period of
innocence, a fall, and the possibility of redemption. There are further problems with the narrative built into The End of
Education. Humanism is always and everywhere, for Spanos, panoptic, repressive, characterized by
"the metaphysics of the centered circle," which is repeatedly attacked by reference to the same overcited passage from Derrida's
"Structure, Sign, and Play in the Discourse of the Human Sciences"--not coincidentally one of the places where Derrida allows himself to make large claims
unqualified by their derivation from reading a particular text. In order to make this assertion, Spanos must show that all apparent difference is in fact contained by the
same old metaphysical discourse. Thus, within the space of four pages, in the context of making absolute claims about Western education (or thought, or theory),
Spanos uses the following constructions: "whatever its historically specific permutations," "despite the historically specific permutations," "Apparent
historical dissimilarities," "Despite the historically specific ruptures." (12-15) Western thought, he repeats, has "always reaffirmed a nostalgic and
Spanos is a poor reader of
recuperative circuitous educational journey back to the origin" (15). This over-insistence suggests to me that
Derrida, for he is not attentive to difference at particular moments or within particular texts. He
seems to believe that one can leap bodily out of the metaphysical tradition simply by compiling
enough citations from Heidegger, whereas his rather anticlimactic final chapter shows, as Derrida recognizes more explicitly, that one
cannot escape logocentrism simply by wishing to.
A2: Spanos

Spanos ignores real violence and politics in the world


Dutton 93[Dennis, Professor of Philosophy at Cantuerbury, “Faking Your Way to Tenure” http://denisdutton.com/faking_tenure.htm] The
End of Education has a chapter entitled “The Violence of Disinterestedness.”

Now disinterestedness is not normally what I’d consider violent, at least compared, say, to a couple of skinheads
with baseball bats. But Spanos finds in its advocacy (by Arnold, Babbitt, and I.A. Richards) “a recurrent call for the
recuperation of a logocentric pedagogy in the face of historical ruptures that betrayed the
complicity of humanistic discourse with an essentially reactionary bourgeois ideology and its
discreetly repressive capitalist state apparatuses, which have dominated the vision and practices
of liberal Western industrial societies, especially in North America.” Isn’t it crazy, when you think of it? The
Western industrial societies, especially in North America, were just about the first places in the
world where the vision and practices of liberalism have been given, however imperfectly, a
chance to dominate repressive state apparatuses, rather than vice versa. Does it ever occur to
Spanos what the military police in Burma do to people they don’t like? In Iran? El Salvador?
Those cosy Marxist dictatorships in Africa? Give me the “discreet” repression of the Western
liberal societies any day. Spanos is a man stuck in the 1960s: he doesn’t notice Tiananmen
Square because he’s still obsessed with Kent State. Spanos’s theory has no historical backing and his writing style only
recreates what he rejects Bryant 97[John, Professor of English at Hofstra. “Review: Democracy, Being, and the Art of Becoming America”
College English, Vol. 59, No. 6. (Oct., 1997), pp. 705-711]
As bracing as Spanos's subversive thesis is, and despite his attempts to rectify the New Americanist approach with a finer
grounding in philosophy, its credibility is undermined by the book's wooden historicism and authoritarian style. Generally speaking, Spanos's
"thematizing" of Ahab and Ishmael amounts to reductions supported more by assertion and endless reiteration than
by textual demonstration. The result is a pronouncement, rather than an analysis, that never penetrates to
the way Melville's words work to bring readers into his ontological dilemma. Spanos's historicism is similarly lacking. He reduces
the development of American culture to a sequence of tableaux vivants with helpful intervening placards: The Puritans Hand Over the Mantle of
American Identity to Andrew Jackson. He speaks of Lewis's American Adam and Bercovitch's American Jeremiad as if they were facts, or
pieces of legislation fully endorsed by the populace, rather than cultural theses proposed by modern critics to extend, not end, debate on who we
Spanos is most effective in discussing the more immediate aspects of Vietnam, but he never makes more distant
are and why.
eras come equally to life. His repeated reference to the "Salem Witch Burnings"-they were hanged-
would be a dismissable gaffe if it did not suggest that the author is not sufficiently engaged in
historical reality to deal with it except as a set of abstractions. Spanos's use of Heidegger is necessarily

Card continues
Card Contunued
problematic. In purely metaphysical terms the philosopher's Nietzschean notion of the "will to power" is an intelligible means of comprehending
Being's temporal differentiations: very transcendental. But then, Heidegger was no Emerson; he joined the Nazi party, took his Jewish mentor
Husserl's job, sent a few colleagues racing to the border, but paradoxically conducted an affair with Hannah Arendt. Surely, Heidegger's nazism
does not invalidate the notion of errancy that so effectively explains Ishmael. However, in a study such as this, in which politics is said to derail
ontological interpretation, we naturally expect a full disclosure of an ontology's political potential. In Spanos's view, the liberal reaction to Stalin
"blinded" Americanists to the true ontological value of Moby-Dick and led us into political disaster. Interestingly enough, the optician who
crafted the lenses by which we might read Moby-Dick ontologically, with Ishmael as errant hero, was himself blind to that errancy; Heidegger
somehow missed his own point and became the authoritarian his philosophy would deny. This fact alone suggests the need for a deeper
ontological inquiry into the determinants in Melville's fiction (personality? sex? philosophy?) that cause individuals to mistake Heideggerian
fluidity and become an Ahab, or a Heidegger. To be fair, Spanos has written on Heidegger's nazism elsewhere, but his obscure and parenthetical
allusions to the issue in this book fail to take this matter to its fullest ontological extent. Spanos's style is a curious self-negation of his principal
ideas. It would be too easy to dismiss Spanos for his Heideggerian jargon and Derridean patois. I, for one, enjoy healthy licks of jargon; I love a
patois: they encourage a certain critical economy. But Spanos
uses language as a weapon to polarize readers.
He consigns past critics to tidy, benighted "post-humanist" camps and virtually ignores more recent
explorers of Melville's complex marginalizing rhetoric (including Barbara Johnson, Nina Bayrn, Lawrence Buell, Michael Rogin, Carolyn
Porter, John Samson, and myself).
This needless drawing of "boundaries" is precisely the opposite of
what Spanos (editor of bounhq 2, which seeks to break-and break again-critical boundaries) takes to be "errant" Ishmael's supreme
achievement. Spanos uses language to claim hegemony over readers even as he tries to disclose
Melville's counterhegemonic strategies. His sentences wage war against comprehension: it is
not simply that abstract subjects perform abstract acts upon abstract objects in his
sentences, but that these consmctions are nested within larger equivalent abstractions-
clause within clause--each interrupted by dashes into the contrapositive, each larded with
oxymorons and paradoxes. And when such a sentence achieves a period, Spanos begins again with "In other words. ..." But his
other words bring no relief. (Spanos's excessive use of mammoth, sometimes two-page block quotes would be offensive if not for the fact that
they provide an occasional Tahiti of literary excellence amidst the ocean of his prose.) One might think this mimicry of Heidegger's famously
dense style is a postmodern strategy to induce in readers an apt ontological crisis commensurate with Ishmael's condition, In fact, it
simply
erects a wall of language that circumscribes an academic domain alien to his ontological
project. This book is the second in a projected trilogy. Let's hope Spanos finds a more effective voice.
at spanos: cede the political

Spanos’s rejection of humanism marginalizes his theory and makes leftist coalition impossible.

Perkin, 93 – Associate Professor of English at Saint Mary's University – 1993 (J. Russell Perkin, Postmodern Culture 3.3,
“Theorizing the Culture Wars,” Project Muse).

My final criticism is that Spanos, by his attempt to put all humanists into the same category and to break totally
with the tradition of humanism, isolates himself in a posture of ultraleftist purity that cuts him off from
many potential political allies, especially when, as I will note in conclusion, his practical recommendations for the practical role of an adversarial intellectual seem similar to those of the
liberal pluralists he attacks. He seems ill-informed about what goes on in the everyday work of the academy, for instance, in the field of composition studies. Spanos laments the "unwarranted neglect" (202) of the
work of Paulo Freire, yet in reading composition and pedagogy journals over the last few years, I have noticed few thinkers who have been so consistently cited. Spanos refers several times to the fact that the
discourse of the documents comprising The Pentagon Papers was linked to the kind of discourse that first-year composition courses produce (this was Richard Ohmann's argument); here again, however, Spanos is
not up to date. For the last decade the field of composition studies has been the most vigorous site of the kind of oppositional practices The End of Education recommends. The academy, in short, is more diverse,
By seeking to separate out only the
more complex, more genuinely full of difference than Spanos allows, and it is precisely that difference that neoconservatives want to erase.

pure (posthumanist) believers, Spanos seems to me to ensure his self-marginalization. For example, several times
he includes pluralists like Wayne Booth and even Gerald Graff in lists of "humanists" that include William Bennett, Roger Kimball
and Dinesh D'Souza. Of course, there is a polemical purpose to this, but it is one that is counterproductive. In fact, I would even
question the validity of calling shoddy and often inaccurate journalists like Kimball and D'Souza with the title "humanist
intellectuals." Henry Louis Gates's final chapter contains some cogent criticism of the kind of position which Spanos has taken.
Gates argues that the "hard" left's opposition to liberalism is as mistaken as its opposition to conservatism,
and refers to Cornel West's remarks about the field of critical legal studies, "If you don't build on liberalism, you build on air"
(187). Building on air seems to me precisely what Spanos is recommending .

Spanos’s theory has no real-world implications

Lewandowski, 94 - Associate Professor and Philosophy Program Coordinator at The University of Central Missouri – 1994
(Joseph D. Lewandowsi, Philosophy and Social Criticism, “Heidegger, literary theory and social criticism,” ed. David M.
Rasmussen, P. 119)

Spanos rightly rejects the 'textuality' route in Heidegger and Criticism precisely because of its totalizing and hypostatizing
tendencies. Nevertheless, he holds on to a destructive hermeneutics as disclosure . But as I have already intimated,
disclosure alone cannot support a critical theory oriented toward emancipation. I think a critical theory
needs a less totalizing account of language, one that articulates both the emphatic linguistic capacity to
spontaneously disclose worlds - its innovative 'worlding' possibilities - and its less emphatic, but no less important,
capacity to communicate, solve problems in and criticize the world . The essential task of the social critic - and any
literary theory that wants to be critical - is to couple world disclosure with problem-solving, to mediate between the extra-ordinary
world of 'textuality' and the everyday world of 'texts'. In this alternative route, literary theory may become the kind of
emancipatory oriented critical theory it can and should be.
at spanos: no alternative

Spanos does not sufficiently connect his genealogy to specific policy recommendations—the alternative fails to influence the real
world.

Lewandowski, 94 - Associate Professor and Philosophy Program Coordinator at The University of Central Missouri – 1994
(Joseph D. Lewandowsi, Philosophy and Social Criticism, “Heidegger, literary theory and social criticism,” ed. David M.
Rasmussen, P. 115-116)

The point to be made here is that Heidegger's politics are not the only (or necessarily the largest) obstacle to coupling him with
critical theory. Hence much of Spanos's energetic defense of Heidegger against his 'humanist detractors' (particularly in his
defiant concluding chapter, 'Heidegger, Nazism, and the "Repressive Hypothesis": The American Appropriation of the Question')
is misdirected. For as McCarthy rightly points out, 'the basic issues separating critical theory from Heideggerean
ontology were not raised post hoc in reaction to Heidegger's political misdeeds but were there from the
start. Marcuse formulated them in all clarity during his time in Freiburg, when he was still inspired by the idea of a materialist
analytic of Dasein' (p. 96, emphasis added). In other words, Heidegger succumbs quite readily to an immanent critique.
Heidegger's aporias are not simply the result of his politics but father stem from the internal limits of his questioning of the 'being
that lets beings be', truth as disclosure, and destruction of the metaphysical tradition, all of which divorce reflection from social
practice and thus lack critical perspective. Spanos, however, thinks Foucault can provide an alternative materialist
grounding for an emancipatory critical theory that would obviate the objections of someone such as Marcuse. But
the turn to Foucault is no less problematic than the original turn to Heidegger. Genealogy is not critical in any
real way. Nor can it tame or augment what Spanos calls Heidegger's 'overdetermination of the ontological
site'. Foucault's analysis of power, despite its originality, is an ontology of power and not, as Spanos
thinks, a 'concrete diagnosis' (p. 138) of power mechanism. Thus it dramatizes, on a different level, the same
shortcomings of Heidegger's fundamental ontology. The 'affiliative relationship' (p. 138) that Spanos tries to develop
between Heidegger and Foucault in order to avoid the problem Marcuse faced simply cannot work. Where Heidegger
ontologizes Being, Foucault ontologizes power. The latter sees power as a strategic and intentional but subjectless mechanism
that 'endows itself' and punches out 'docile bodies', whereas the former sees Being as that neutered term and no-thing that calls
us. Foucault (like Spanos) never works out how genealogy is emancipatory, or how emancipation could be
realized collectively by actual agents in the world . The 'undefined work of freedom' the later Foucault speaks of in
'What Is Enlightenment?' remained precisely that in his work.4 The genealogy of power is as much a hypostatization as is
fundamental ontology: such hypostatizations tend to institute the impossibility of practical resistance or freedom. In short , I
don't think the Heideggerian 'dialogue' with Foucault sufficiently tames or complements Heidegger, nor
does it make his discourse (or Foucault's, for that matter) any more emancipatory or oppositional. Indeed,
Foucault's reified theory of power seems to undermine the very notion of 'Opposition', since there is no subject (but rather a
'docile' body) to do the resisting (or, in his later work, a privatized self to be self-made within a regime of truth), nor an object to
be resisted. As Said rightly points out in The World, the Text, and the Critic, 'Foucault more or less eliminates the central dialectic
of opposed forces that still underlies modern society' (p. 221, emphasis added). Foucault's theory of power is shot
through with false empirical analyses, yet Spanos seems to accept them as valid diagnoses. Spanos fails to
see, to paraphrase Said's criticisms of Foucault's theory of power , that power is neither a spider's web without the
spider, nor a smoothly functioning diagram (p. 22l).
at spanos: no truth disempowering

Spanos’s rejection of objective truth removes any way to measure the theory’s emancipatory effects.

Lewandowski, 94 - Associate Professor and Philosophy Program Coordinator at The University of Central Missouri – 1994
(Joseph D. Lewandowsi, Philosophy and Social Criticism, “Heidegger, literary theory and social criticism,” ed. David M.
Rasmussen, P. 117-118)

But radicalized or not, Spanos's trading of any possibility of 'determinate truth' for Heideggerian disclosure as
eventing of truth/untruth robs his critical theory of the necessary yardstick needed to measure
'emancipation'. Heidegger's disclosure is a cryptonormative truth; it is an event before which any critical judgment necessarily
fails. Disclosure is not a process of inquiry, but rather a revealing/concealing that befalls or overtakes us. In his eagerness to
draw out the enabling features and 'post'-humanist dimension of Heidegger's disclosure, Spanos fails to
see the inevitable and internal limits to truth as disclosure . Gadamer encounters similar problems, despite his keen
insights, when he holds on to a Heideggerian disclosure that too often undermines the power of critical reflection. And the
postmodern Italian philosopher Gianni Vattimo encounters a related problem when he attempts to take leave of modernity and
proclaim a liberating postmodernity via Heidegger's disclosure. But while a purely aesthetic theory interested in 'textuality' can
quite justifiably be grounded in truth as disclosure (as American deconstruction or Vattimo's il pensiero debole is), a truly critical
theory interested in emancipation simply cannot: some types of 'emancipation' are false and need to be rejected. Texts may very
well 'disclose' worlds in the same way that, say, the Greek temple does for Heidegger. But a genuinely critical theory
needs to be able to say what worlds are better or worse for actual agents in actual worlds - a need, I might
add, that Spanos is constantly aware of and typifies in his denunciation of American imperialism in Vietnam (and elsewhere) in
Heidegger and Criticism.
at spanos: humanism good

Humanist reforms are more effective than totalizing critique.

Good, 01 - Professor of English at the University of British Columbia – 2001 (Graham Good, Humanism betrayed, P. 7)

Liberal humanism, in my view, offers a more cogent critique of capitalist society because it generally
accepts capitalism as an economic system that is more productive and efficient than the alternatives. Yet
liberal humanism seeks to limit capitalism's social and cultural effects by preserving certain spheres -
politics, art, education - as having a limited autonomy from the imperatives of the market. This attitude of
partial acceptance and partial critique is much more realistic and effective, for example, in protesting the
commercialization of the university, or in preserving artistic standards, than the total rejection of "late-
capitalist society" that is common among academic pseudoradicals. Total opposition is more readily co-
opted by the system because it forms a mirror image. If the system is all-powerful, how can Theorists
explain the possibility or acceptability of their own opposition to it? This problem is usually evaded; but when it is
confronted, a doctrine of "necessary complicity" is often evoked. If you disbelieve in your own autonomy as an individual, you
must be liable in dark moments to suspect that you are actually working for the system. Resistance to the system is part of the
system. Total rejection flips into total acceptance and opens the way for a personal exploitation of the
academic system. Political correctness covers up careerist realpolitik.
at spanos: vietnam good

Communism was spreading in Vietnam – it had to be stopped.


Podhoretz, 82 – adviser to the US Information Agency and laureate of the Presidential Medal of Freedom – 1982 (Norman
Podhoretz, Why We Were in Vietnam, P. 11)

Indeed, for many people whose original support of American intervention in Vietnam had been based on memories of Munich,
Vietnam not only replaced it but canceled it out. To such people - the lesson of Munich had been that an expansionist
totalitarian power could not be stopped by giving in to its demands and that limited resistance at an early
stage was the only way to avoid full-scale war later on . Prime Minister Neville Chamberlain, returning to England from the conference in Munich at which Nazi
Germany's claims over Czechoslovakia had been satisfied, triumphantly declared that he was bringing with him "peace in our time." But as almost everyone would later agree, what he had actually brought with him
, if a line
was the-certainty of a world war to come-a war that Winston Churchill, the leading critic of the policy of appeasement consummated at Munich, would later call "unnecessary." According to Churchill

had been drawn against Hitler from the beginning, he would have been forced to back away, and the
sequence of events that led inexorably to the outbreak of war would have been interrupted. Obviously,
Vietnam differed in many significant ways from Central Europe in the late 1930s. But there was one great
similarity that overrode these differences in the minds of many whose understanding of such matters had been shaped by the
memory of Munich. "I'm not the village idiot," Dean Rusk, who was Secretary of State first under Kennedy and then under
Johnson, once exploded. "I know Hitler was an Austrian and Mao is a Chinese…But that is common between the two situations
is - - the phenomenon -of aggression." In-other words, in Vietnam now as in central Europe then, a totalitarian
political force - Nazism then, Communism now-was attempting to expand the area under its control . A
relatively limited degree of resistance then would have precluded the need for massive resistance
afterward. This was the lesson of Munich, and it had already been applied successfully in Western
Europe in the forties and Korea in the fifties. Surely it was applicable to Vietnam .

Vietnam was crucial for American hegemony and democracy promotion.


Podhoretz, 82 – adviser to the US Information Agency and laureate of the Presidential Medal of Freedom – 1982 (Norman
Podhoretz, Why We Were in Vietnam, P. 19-20)
Thus, on June 1, 1956, two years after delivering Schlesinger's favorite speech, Kennedy spoke before the American Friends of Vietnam on "America's Stake in Vietnam." By this time the French had been defeated,
and Vietnam had been partitioned under a set of agreements negotiated in Geneva, with a Communist regime under Ho Chi Minh established in the North and a non-Communist government under Ngo Dinh Diem set
up in the South. According to the Geneva agreements, Vietnam was to be unified under a government to be elected in 1956, but Kennedy declared that "neither the United States nor Free Vietnam [was] ever going
To Kennedy, Vietnam represented
to be a party to an election obviously stacked and subverted in advance" by the Communists of the North and their agents and allies in the South.

"the cornerstone of the Free World in Southeast Asia ," the keystone to the arch, the finger in the dike. Burma,
Thailand, India, Japan, the Philippines and obviously Laos and Cambodia . . . would be threatened if the
red tide of Communism overflowed into Vietnam. " This was the first of the four reasons Kennedy g ve for "America's
stake in Vietnam." The second was that Vietnam represented "a proving ground for democracy in Asia…the
alternative to Communist dictatorship. If this democratic experience fails, if some one million refugees
have fled the totalitarianism of the North only to find neither freedom nor security in the South, then
weakness, not strength, will characterize the meaning of democracy in the minds of still more Asians. " It
was, Kennedy said, an experiment we could not "afford to permit to fail." The third reason was that Vietnam, in addition to
representing, a test of democracy in Asia, also represented "a test of American responsibility and determination"
there. Characterizing the United States as the "godparents" of "little Vietnam" and Vietnam as "our offspring" ("We presided at its
birth, we gave assistance to its life, we have helped to shape its future"), Kennedy concluded that if Vietnam were to fall
"victim to any of the perils that threaten its existence-Communism, political anarchy, poverty and the
rest," we would be held responsible and our prestige in Asia would "sink to a new low." Finally (and most
prophetically), America's stake in Vietnam was "a very selfish one" in the sense that " American lives and American
dollars" would inevitably have to be expended if "the apparent security which has increasingly
characterized that area I under the leadership of President Diem" were to be jeopardized.

at spanos: vietnam good


Containment was necessary to prevent nuclear war with Russia

Podhoretz, 82 – adviser to the US Information Agency and laureate of the Presidential Medal of Freedom – 1982 (Norman
Podhoretz, Why We Were in Vietnam, P. 22-23)

The answer was unclear. On the one hand, the most authoritative and highly articulated public statement of the assumptions
behind containment, the famous article by the then Director of the State Department's Policy Planning Staff, George F. Kennan
(published in 1947 in Foreign Affairs, under the pseudonym "Mr. X"), could only be read to imply that in principle at least
containment was global in scope. "The main element," said Kennan, "of any United States policy toward the Soviet
Union must be that of a long-term, patient but firm and vigilant containment of Russian expansive
tendencies by the adroit and vigilant application of counter-force at a series of constantly shifting
geographical and political points, corresponding to the shifts and maneuvers of Soviet policy.” Nor did
Kennan leave any doubt as to the relation between local Communist parties and the Soviet Union: the duty of "all good
Communists" everywhere in the world, he wrote, "is the support and promotion of Soviet power, as defined in Moscow." l 2 Yet
on the other hand, three years later, Kennan's boss , Truman's Secretary of State, Dean Acheson, seemed to suggest
that the United States did not regard the independence of South Korea as a vital interest." This the Soviet
Union, the Chinese, and the North Koreans evidently all took as a signal that the forcible extension of
Communist rule to the South would not be met by the application of American counterforce. It seems
unlikely that Acheson, who as much as any one individual was the father of containment-"present," as he put it in the title of his
memoirs, 'bt the creationw-really intended to send such a signal. But whether there was a misunderstanding here or a lastminute
change of mind, the invasion of South Korea on June 25, 1950, triggered an immediate American response . Only
two days after the outbreak of the war, President Truman declared that " the attack upon Korea makes it plain beyond
all doubt that 4 Communism has passed beyond the use of subversion to conquer independent nations and
will now use armed invasion and war." l4 Not only was the U nited States now extending the principles of
containment from Europe to Asia, then; it was going even further in practice .
A2: Heidegger

Heidegger’s alternative results in Nazism to avoid the problems of the world it presents
Zizek 99[Professor of Philosophy at the University of Ljubljana. The Ticklish Subject: The Absent Centre of Political Ontology, pg 21]
The standard story about Heidegger is that he accomplished his Kehre (turn) after becoming aware of how the original project of Being and Time
leads hack to transcendental subjectivism: owing to the unreflected remainder of subjectivism (decisionism, etc.), Heidegger let
himself be seduced into his Nazi engagement; when, however, he became aware of how he had burnt his fingers’ with it,
he cleared up the remainders of subjectivism and developed the idea of the historical-epochal character of Being itself. . . One is tempted to invert
this standard story: there is a kind of ~vanishing mediator’ between Heidegger I and Heidegger II, a
position of radicalized subjectivity coinciding with its opposite that is, reduced to an empty
gesture, the impossible intersection between the ‘decisionism’ of Heidegger I and his late
‘fatalism’ (the event of Being ‘takes place’ in man, who serves as its shepherd . . .). Far from being the ‘practical consequence’ of this
radicalized subjectivity, Heidegger’s Nazi engagement was a desperate attempt to avoid it. . . . In other words,
what Heidegger later dismissed as the remainder of the subjectivist transcendental approach in Being and Time is what he should have stuck to.
Heidegger’s ultimate failure is not that he remained stuck in the horizon of transcendental
subjectivity, but that he abandoned this horizon all too quickly, before thinking out all its
inherent possibilities. Nazism was not a political expression of the ‘nihilist, demoniac potential
of modern subjectivity’ but, rather, its exact opposite: a desperate attempt to avoid this potential.
Heidegger’s Nazism can’t be separated from his philosophy Thiele 03[Leslie, Professor of Political
Science at the University of Florida. “The Ethics and Politics of Narrative” Foucault and Heidegger: Critical
Encounters] Heidegger was a Nazi and a rather unrepentant one at that. Some suggest
Heidegger’s Nazism cannot be separated from his philosophy, that indeed the former follows from the latter. The
argument, in short, is that Heidegger’s political biography pretty well tells the whole story. This position has been
rearticulated periodically since the end of the Second World War, each time creating something of an academic row. To be sure , the story
of Heidegger’s life does not well illustrate an education in sound moral and political judgment,
except perhaps as an example of a lesson left unlearned. Yet the story that Heidegger himself tells about human life, about human being in
history, can do much to cultivate moral and political judgment. I assert this despite insightful critiques of Heidegger that accuse him of ignoring
and eliding phronesis as human potentiality. My argument, then, is not that Heidegger’s work explicitly celebrates prudence, but that his
philosophical narrative facilitates its cultivation.
A2: Heidegger

Heidegger’s alternative can never yield positive change – it tips the balances toward
dogmatic authoritarianism
Thiele 03[Leslie, Professor of Political Science at the University of Florida. “The Ethics and Politics of Narrative” Foucault and Heidegger:
Critical Encounters]

The pursuit of knowledge continues unabated for the skeptic. Yet it proceeds with a suspicious eye. There are inherent limitations toand a price to pay for-the pursuit
of knowledge. Charles Scott describes Foucault's efforts in this regard: "Far from the skepticism that argues that nothing is really knowable ... genealogies embody a
sense of the historical limits that define our capacities for knowing and believing. Things are known. But they are known in ways that have considerable social and
cultural costs."" Both Heidegger and Foucault maintain[s] that there is no legitimate basis for the radical skeptic's
conviction that knowledge is impossible or unworthy of pursuit. This sort of skepticism, Heidegger
states, consists merely in an "addiction to doubt."? The skeptical nature of political philosophical
thought, in contrast, is grounded in the imperative of endless inquiry. The point for Heidegger and Foucault is to inquire not in
order to sustain doubt, but to doubt that one might better sustain inquiry. At the same time, inquiry is tempered with a sensibility of the
ethico-political costs of any "knowledge" that is gained. Doing political philosophy of this sort
might be likened to walking on a tightrope. If vertigo is experienced, a precarious balance may
be lost. Falling to one side leaves one mired in apathy, cynicism, and apoliticism. This results
when skeptical inquiry degenerates into a radical skepticism, an addictive doubt that denies the
value of (the search for) knowledge and undermines the engagements of collective life, which
invariably demand commitment (based on tentatively embraced knowledge). Falling to the other
side of the tightrope leaves one mired in dogmatic belief or blind activism. Authoritarian
ideologies come to serve as stable foundations, or a reactive iconoclasm leads to irresponsible
defiance. Apathy, cynicism, and apoliticism, on the one side, and dogmatic authoritarianism or
reactive iconoclasm, on the other, are the dangerous consequences of losing one's balance. These
states of mind and their corresponding patterns of behavior relieve the vertigo of political philosophical inquiry, but at a prohibitive cost. It has been argued that
Foucault did not so much walk the tightrope of political philosophy as straddle it, at times leaving his readers hopeless and cynical, at times egging them on to an
irresponsible monkeywrenching. For some, the Foucauldian flight from the ubiquitous powers of normalization undermines any defensible normative position.
Hopelessness accompanies lost innocence. Cynicism or nihilism become the only alternatives for those who spurn all ethical and political foundations. By refusing to
paint a picture of a better future, Foucault is said to undercut the impetus to struggle. Others focus on Foucault's development of a "tool kit" whose contents are to be
employed to deconstruct the apparatuses of modern power. Yet the danger remains that Foucault's "hyperactive" tool-kit users will be unprincipled activists, Luddites
at best, terrorists at worst. In either case, Foucault provides no overarching theoretical vision. Indeed, Foucault is upfront about his rejection of ethical and political
theories and ideals. "I think that to imagine another system is to extend our participation in the present system," Foucault stipulates. "Reject theory and all forms of
general discourse. This need for theory is still part of the system we reject."!" One might worry whether action is meant to take the place of thought. If Foucault
Heidegger obviously stumbled off it. In the 1930S, Heidegger
occasionally straddles the tightrope of political philosophy,
enclosed himself within an authoritarian system of thought grounded in ontological reifications
of a "folk" and its history. Heidegger's historicization of metaphysics led him to believe that a new philosophic epoch
was about to be inaugurated. It implicitly called for a philosophical Fuehrer who could put an end to two millennia
of ontological forgetting. The temptation for Heidegger to identify himself as this intellectual
messiah and to attach himself to an authoritarian social and political movement capable of
sustaining cultural renewal proved irresistible. Whether Heidegger ever fully recovered his balance has been the topic of much
discussion. Some argue that Heidegger's prerogative for political philosophizing was wholly
undermined by his infatuation with folk destiny, salvational gods, and political authority. 12
A2: Heidegger

The alternative will destroy ethics and only cause suffering


Thiele 03[Leslie, Professor of Political Science at the University of Florida. “The Ethics and Politics of Narrative” Foucault and Heidegger:
Critical Encounters]

The complementarity of Heidegger's and Foucault's accounts of modern demons and saving graces should not be too surprising. Foucault's
indebtedness to and fascination with Heidegger is well documented.' My intent in this chapter is neither to focus on the complementarity of these
visions, nor to outline the striking philosophical and political differences that remain in Heidegger's and Foucault's work. Rather, I attempt to
make a claim for what at first blush might appear a lost cause. Despite their originality and intellectual brilliance , Heidegger
and
Foucault are often castigated as ethico-political dead-ends. They are criticized for their
unwillingness or inability to supply the grounds for sound moral and political judgment.
Heidegger's embrace of Nazism, in particular, is frequently identified as proof positive that he has
little, if anything, to contribute to the ethico-political domain. The standard charge is that his
highly abstract form of philosophizing, empyrean ontological vantage point, and depreciation of
"das Man" undermines moral principle and political responsibility. From his philosophical heights, it is suggested,
Heidegger remained blind to human sufferings, ethical imperatives, and political practicalities.
He immunized himself against the moral sensitivity, compassion, and prudence that might have
dissuaded him from endorsing and identifying with a brutal regime. Those who embrace his
philosophy, critics warn, court similar dangers. In like fashion, it is held that Foucault dug himself into an equally deep,
though ideologically relocated, moral and political hole. Genealogical studies left Foucault convinced of the ubiquity of the disciplinary matrix.
There would be no final liberation. The sticky, normalizing webs of power were inescapable and a "hermeneutics of suspicion" quashed any hope
of gaining the ethical and political high ground.? As such, critics charge, Foucault stripped from us all reason for resistance to unjust power and
all hope of legitimating alternative ethico-political institutions. In a Foucauldian world of panoptic power that shapes wants, needs, and selves,
critics worry, one would have no justification for fighting and nothing worth fighting for.' In sum, Heidegger's and Foucault's critics suggest that
both thinkers undermine the foundations of the practical wisdom needed to ethically and politically navigate late modernity. Despite the brilliance
and originality of their thought, arguably the greatest philosopher and the greatest social and political theorist of the twentieth century remain
ungrounded ethically and divorced from political responsibility. Critics
argue that Heidegger's statements and actions
endorsing and defending Nazi authoritarianism and Foucault's radical anarchism, as displayed in
his discussions of popular justice with Maoists, demonstrate that neither thinker is capable of
supplying us with the resources for sound moral and political judgment.
A2: Heidegger

In the context of the environment it is impossible to do nothing – the environment is


dynamic and sustainable strategies for dealing with its changes must be developed
Bowman ‘01 (D.M.J.S., Northern Territory University, Australia, 2001
Journal of Biogeography, Vol 28, No. 5, Future Eating and Country Keeping:what role has environmental history in the management of
biodiversity?JSTOR)
Land managers can use the enormous complexity of landscape change and the absence of clear
goals as an excuse not to act. But this strategy ignores a basic lesson from environmental history
that it is impossible to ‘do nothing’ in dynamic systems. I suggest that the most basic goal of land
management should be to minimize the rate of species extinction, particularly in largely intact landscapes and to
maintain or restore ecosystem services, such as potable water, in degraded landscapes. In this regard a historical
perspective provides .in important context to detect marked declines in species populations and
ecosystem function. For example, the widespread decline of granivorous bird species (Franklin, 1999) and the
population crash of the endemic conifer (lslli:ris sn:rarroprca R.T. Baker & HG. Sin. (Bowman & Panton, 1993) is
clear evidence that northern Australian tropical savamsas are undergoing rapid evolutionary adjustment in response
to ecological changes initiated by European colonization, The unpotability of many inland waters and collapse of
native fish populations because of eutrophication and salinity dramatically signals the unsustainability of past and
present agriculture practices in southern Australia (State of Environment Advisory Council. 1996.) CONCLUSION
The description and explanation of environmental change is of critical importance for land
management and the conservation of biodiversiry. However, practitioners of environmental
history must accept that their studies are politically charged and that their findings are bounded
by great uncertainty. It is inevitable that conflicting interpretations of essentially the same data will arise
because authors have different value systems. Dovers (2000. p. X) sensibly advises scientists and historians that they
should enter debates about environmental history with their ‘eyes wide open and other parts of their anatomy well
covered’. Many of these tensions will he most apparent in popular works that have a tendency to gloss-over
uncertainties and over- generalize in order to make psychologically saris’ing stories. Some stories can be so
powerhil that they assume a life of their own. Rather than attempting to quash these stories, I suggest it is more
productive for ecologists to harness them to justify subsequent enquiry and to bolster land
management interventions. Although in the short term dogmatically interpreted stories can stifle
public land management debates and frustrate the incorporation of new research findings, in the
long term they will be replaced by new stories to accommodate changing knowledge and value
systems. In any case, on a practical level the rigid application of any single view across the diversity of landscapes
is doomed to failure. just as traditional ecological knowledge enables indigenous people to adapt to
particular environments I am confidetit that appropriate stories will he continually rehned to
bolster ecologically sustainable management of modern landscapes. Regardless of the
ideological struggles amongst different value systems, eventually natural selection will see the
triumph and ecological ‘fit’ stories over ecological flawed ones. Isn’t that Darwin’s law?
A2: Hedegger

Even philosophers cant agree with your alt – they cant decide on whether the ontological
requires us to believe in inherent values
ZIMMERMAN 2K2 (Michael E., Eco-Phenomenology: Back to the Earth Itself)

Philosophers have not yet agreed whether one can successfully identify and defend any property—
ontical or ontological—which would require us to accord "inherent worth " even to humans, much
less to animals and plants, not to mention the ecosystems, mountains, and rivers regarded as so worthy by
many environmentalists. Heidegger's brief accounts of the "dignity" of living beings usually focus on
their Being as physis. But for him, physis somehow means both the manifesting (Being) of beings within
the clearing, and the process whereby an organism unfolds its own structure in the life-process. In my
view, Heidegger never adequately reconciles these two aspects of physis.34 Critics charge that by
virtually equating them, Heidegger ends up in a kind of "ontological aestheticism," which celebrates the
beauty of the self-manifesting of beings at the expense of their merely "ontical" characteristics.35
Don’t hole your breath for their Alt – in the meantime do the plan
Zimmerman 2004, Pf Philosophy, Tulane, 2K4 (Michael E., Nature Revisited: New Essays in Environmental Philosophy, edited by
Bruce Foltz)

Yet, environmentalists often adopt uncritically postmodern theory’s totalizing critique of


modernity, which invites naïve celebration of supposedly eco-friendly premodern societies, the
shortcomings of which are conveniently-ignored. The same critical buzz saw that undermines
modernity’s anthropocentric institutions, moreover, calls into question such crucial
environmental concepts as wilderness, ecosystems, and even nature. Hence, environmentalists often
contend that postmodern theory affirms a subtle kind of anthropocentrism, according to which nature is merely a social construct
arising through human language, culture, and practices. A constructive postmodern theory, however, will integrate the
hermeneutics of suspicion, the critique of foundationalism, contemporary cosmology, the noble achievements of modernity, and
empirically-grounded spiritual insights in a way that contribute to more sophisticated environmentalisms, the kind that
skillfullypromote the well-being of all human and non-human life, as well as the habitats that sustain them. It would be
unwise , however, to hold one’s breath while waiting for widespread acceptance of postmodern
cosmologies and environmentalisms. At first, they will prove attractive primarily to some
members of educated elites in developed societies. This fact, however, should not be a source of
discouragement. As Nietzsche pointed out, the work of philosophers is often untimely. In the meantime, a great deal
can be accomplished on environmental fronts with the relatively anthropocentric and modernist
(free market and regulatory) approaches that appeal to the large majority of Americans and Europeans. It
may be more difficult to environmentalism to those in premodern societies or in societies that are in the process of adopting their
own versions of modern concepts, practices, and institutions. North American history during the past few
centuries shows that people are often more focused on improving their economic status than they
are on caring for the natural environment.
at heidegger: nazi

Heidegger’s philosophy is Nazism—the rejection of technology and re-connection with Being offered by National Socialism fit
with his arguments.

Wolin, 01 – Distinguished Professor of History at the City University of New York Graduate Center – 2001 (Richard Wolin,
Heidegger’s Children, P. 32)

To say that Arendt's explanation was the more successful, despite its flaws, is hardly controversial. In many respects,
Heidegger's own narrative was simply delusory, a retrospectively contrived psychological prophylaxis against his own
enthusiastic support for the regime. In Heidegger's view, everything that came to pass-the war, the
extermination camps, the German dictatorship (which he never renounced per se)-was merely a
monumental instance of the "forgetting of Being," for which the Germans bore no special responsibility.
After the war, he went so far as to insist that German fascism was unique among Western political
movements in that, for one shining moment, it had come close to mastering the vexatious "relationship
between planetary technology and modern man." In Heidegger's estimation, therein lay the "inner truth
and greatness of National Socialism." But ultimately "these people [the Nazis] were far too limited in their thinking," he
claimed. Pathetically, Heidegger was left to replay in his own mind the way things might have been had
Hitler (instead of party hacks) heeded the call of Being as relayed by Heidegger himself. Nazism might
thereby have realized its genuine historical potential. Fortunately, the world was spared the outcome of
this particular thought experiment.

Heidegger claimed that Nazism was at the heart of his philosophy and he was personally, deeply anti-Semitic.

Wolin, 01 – Distinguished Professor of History at the City University of New York Graduate Center – 2001 (Richard Wolin,
Heidegger’s Children, P. 10-11)

In May 1933, Heidegger sent a telltale telegram to Hitler expressing solidarity with recent Gleichschaltung legislation. There were
instances of political denunciation and personal betrayal. Moreover, Heidegger remained a dues-paying member of the Nazi
Party until the regime's bitter end. He continued to open his classes with the so-called "German greeting" of "Heil Hitler!" In 1936,
he confided to Lowith that his 'partisanship for National Socialism lay in the essence of his philosophy"; it
derived, he claimed, from the concept of "historicity" (which stressed the importance of authentic historical
commitment) in Being and Time.'" As the rector of Freiburg University, Heidegger was charged with enforcing the anti-
Semitic clauses of the so-called "Law for the Preservation of a Permanent Civil Service," which effectively banned Jews from
all walks of government service, including university life . Despite his later disclaimers, in his capacity as rector
Heidegger faithfully executed these laws, even though it meant banning Husserl, to whom he owed so much , from
the philosophy faculty library. In the eyes of Hannah Arendt, this action, which had affected the septuagenarian phenomenologist
so adversely, made Heidegger a "potential murderer."" At the time, Husserl complained bitterly in a letter to a former student
about Heidegger's growing anti-Semitism: "In recent years [he] has allowed his anti-Semitism to come increasingly
to the fore, even in his dealings with his groups of devoted Jewish students ," observes Husserl. "The events of
the last few weeks," he continued (referring to Heidegger's joining the Nazi Party as well as the recent university ban on Jews),
"have struck at the deepest roots of my existence."'" In 1929, Heidegger had already complained that Germany was faced with a
stark alternative: "the choice between sustaining our German intellectual life through a renewed infusion of genuine, native
teachers and educators, or abandoning it once and for all to growing Jewish influence [Verjudung]-in both the wider and narrow
sense."'

at heidegger: nazi
Heidegger’s Nazism is inexcusable – his own philosophy stressed that thought can’t be divorced from action.

Wolin, 90 - Distinguished Professor of History at the City University of New York Graduate Center - 1990 (Richard Wolin, The
Politics of Being, P. 33-34)

Although an understanding of Heidegger's political thought should in no way be reduced to the concrete political choices made
by the philosopher in the 1930s, neither is it entirely separable therefrom. And while the strategy of his apologists has been to
dissociate the philosophy from the empirical person, thereby suggesting that Heidegger's Nazism was an unessential
aberration in the hope of exempting the philosophy from political taint, this strategy will not wash for
several reasons. To begin with, Heidegger's philosophy itself would seem to rule out the artificial, traditional
philosophical separation between thought and action. In truth, much of Being and Time is concerned with
overcoming the conventional philosophical division between theoretical and practical reason; a fact that is
evident above all in the "pragmatic" point of departure of the analytic of Dasein: "Being-in- the-world"
rather than the Cartesian "thinking substance." More importantly, though, what is perhaps the central category of
Heidegger's existential ontology-the category of "authenticity''- automatically precludes such a facile
separation between philosophical outlook and concrete life-choices. As a work of fundamental ontology, Being and Time aims at delineating the
essential, existential determinants of human Being-in-the-world. Heidegger refers to these structures (e.g., "care," "fallenness," "thrownness," "Being-toward-death") as Existenzialien. The category of authenticity
demands that the ontological structures of Being and Time receive practical or ontic fulfillment; that is, the realization of these categorial determinations in actual, concrete life contexts is
essential to the coherence of the Heideggerian project. This conclusion follows of necessity from the nature of the category of authenticity itself: it would be nonsensical to speak of an "authentic Dasein" that was
Authenticity requires that ontic or practical choices and involvements-concrete
unrealized, existing in a state of mere potentiality.

decisions, engagements, and political commitments-become an essential feature of an authentic existence.

Heidegger’s Nazism was a logical consequence of his refusal of ethics.

Wolin, 90 - Distinguished Professor of History at the City University of New York Graduate Center - 1990 (Richard Wolin, The
Politics of Being, P. 65)

The consequences of this decisionistic "ethical vacuum," coupled with the prejudicial nature of
Heidegger's conservative revolutionary degradation of the modern life-world, suggests an undeniable
theoretical cogency behind Heidegger's ignominious life-choice of 1933 . In its rejection of "moral
convention-which qua convention, proves inimical to acts of heroic bravado- decisionism shows itself to be distinctly
nihilistic vis-a-vis the totality of inherited ethical paradigms.118F or this reason, the implicit political theory of Being and Time-
and in this respect, it proves a classical instance of the German conservative-authoritarian mentality of the period-remains devoid
of fundamental "liberal convictions" that might have served as an ethicopolitical bulwark against the enticement of fascism. Freed
of such bourgeois qualms, the National Socialist movement presented itself as a plausible material "filling" for
the empty vessel of authentic decision and its categorical demand for existentiell-historical content. The
summons toward an "authentic historical destiny" enunciated in Being and Time was thus provided with
an ominously appropriate response by Germany's National Revolution. The latter, in effect, was viewed
by Heidegger as 'the ontic fulfillment of the categorical demands of "historicity": it was Heidegger's own
choice of a "hero," a "destiny," and a "community."
at heidegger: nazi

Heidegger’s Dasein was easily translated into a German Dasein and an excuse for nationalism.

Wolin, 01 – Distinguished Professor of History at the City University of New York Graduate Center – 2001 (Richard Wolin,
Heidegger’s Children, P. 184-185)

What is troubling about Heidegger's standpoint is not that he judges but the basis on which he
distinguishes. His lock-step identification with the "German ideology" risks settling in advance all
questions of relative historical merit. "Capitalism," "peasant wars," "Negroes"-once the world has been
neatly divided into "historical" and "unhistorical" peoples and events, history's gray zones fade from
view. That the "Volk" that, in Heidegger's view, possessed "historicity" in the greatest abundance-the
Germans-had as of 1934 abolished political pluralism, civil liberties, and the rule of law and was in the
process of consolidating one of the most brutal dictatorships of all time, cannot help but raise additional
doubts about the "existential" grounds of Heidegger's discernment . Here, one could reverse the terms and claim that Germany of the 1930s
suffered from an excess of historicity. Conversely, the historical events and peoples that Heidegger slights could readily be incorporated into progressive historical narratives." That he fails to perceive these prospects
is attributable to his renunciation of "cosmopolitan history" and his concomitant embrace of a philosophically embellished version of German particularism or socalled Sondenveg. From an epistemological standpoint,
. The
Heidegger's difficulties derive from his decision to base ethical and political judgments on factical rather than normative terms; that is, from the Jemeinigkeit or concrete particularity of German Existenz

more one reconsiders Heidegger's philosophy of the 1930s, the more one sees that one of its guiding
leitmotifs is a refashioning of Western metaphysics in keeping with the demands of the Germanic
Dasein." He consistently rejects the "universals" that in the Western tradition occupied a position of
preeminence in favor of ethnocentric notions derived from the annals of Germanic Being-in-the-world.
The example of the airplane that brings the Fuhrer to Mussolini" is merely a paradigmatic instance of a more general trend.

Heidegger thought that labor camps could be used to attack modernity.

Wolin, 01 – Distinguished Professor of History at the City University of New York Graduate Center – 2001 (Richard Wolin,
Heidegger’s Children, P. 191)

Heidegger's concern with the importance of labor in the new Reich was a matter of philosophical as well
as political conviction. A longtime critic of the senescence and disorientation of German university life, he was of the
opinion that the labor camps would serve to reintegrate knowledge with the life of the German Volk,
whose simplicity and lack of sophistication he revered.*6A s Lijwith remarked, Heidegger "failed to notice the
destructive radicalism of the whole [Nazi] movement and the petty bourgeois character of all its 'strength-through-joy' institutions,
because he was a radical petty bourgeois himself."*' Heidegger , who hailed from the provincial lower classes, and
who, despite his manifest brilliance, was denied a university chair until the age of thirty-nine, found much
he could agree with in Nazism's dismantling of the old estates and commitment to upward social
mobility." In his view, the value of labor camps as a vehicle of ideological reeducation for politically
reticent scholars could hardly be overestimated.
at heidegger: humanism key to stop nazism

Their K has it backwards—ethical humanism is key to criticizing Nazism.

Ferry and Renaut, 90 – Professor of Political Science at the Sorbonne and Professor of Philosophy at Nantes – 1990 (Luc Ferry
and Alain Renaut, Heidegger and Modernity, trans. Franklin Philip, P. 107-108)

Whatever is true of this debate, which, it will be readily agreed, here remains open, one thing is still certain. Heidegger is not
close to Nazism because he remained a prisoner of humanism , nor because of his deliberations about authenticity
and the distinguishing property of man. For Heidegger, the distinguishing property of man is always
transcendence, and on the contrary, it was in the name of this transcendence and thus because he was still
a humanist that Heidegger could criticize the biologizing reifications of Nazi anti-Semitism. More generally,
it is very much in the name of humanism thus understood, in the name of that strictly human capacity to wrench
oneself free of natural determinations, that a criticism of the racist imaenation (in the Lacanian sense) is
possible. When, however, Heidegger makes the destiny of Being the destiny of man, when he thus returns to the antihumanist
idea of a traditional code (if only that of the history of Being), he founders in inauthenticity, and his fall makes possible the return
of the nationalistic myth and the fanatical hatred of modernity.
at heidegger: ethics too vague

Heidegger’s “call of conscience” is hopelessly vague.

Wolin, 90 - Distinguished Professor of History at the City University of New York Graduate Center - 1990 (Richard Wolin, The
Politics of Being, P. 40)

In the thought of Heidegger, it is the category of the "call of conscience" (Ruf des Gewissens) that paves the way for authentic
decision or Entschlossenheit, thereby elevating Dasein above the fallenness of the They. Yet, the discussion of the "call of
conscience" is disappointingly vague. When the question is posed as to whence the call emanates, the
specific content of the call, or how it might be recognized, we are provided with only the most
roundabout and tenuous hints. Indeed, Heidegger seems to treat the nebulousness of the call as a virtue. In part, this
evasiveness is an honest reflection of the requirements of existential analysis, which should in principle
bear no responsibility for supplying "existentiell" particulars . For were specific "ontic" directives provided, the
whole question of the "decision" at issue-the Wozu of resolve-would become superfluous. In a very real sense, it is not up to
fundamental ontology to make our choices for us. It is "we" who must decide, in accordance with what Heidegger is fond of
calling our "ownmost potentiality-for- Being." Nevertheless, these caveats should by no means exonerate existential
analysis from the charge of vacuity or insufficient concreteness.
at heidegger: unconcealment bad

The alternative of treating truth as unconcealment makes it impossible to judge true from false—this is the sort of error that
allowed Hitler to join the Nazis.

Wolin, 90 - Distinguished Professor of History at the City University of New York Graduate Center - 1990 (Richard Wolin, The
Politics of Being, P. 121-122)

Ultimately Heidegger's theory of truth succumbs to the same problem of criterionlessness that was at issue in the decisionistic
approach to human action in Being and Time. On the one hand, Heidegger seems at first to be claiming that unconcealment is
merely an ontological precondition of truth-which is, as far as it goes, certainly a plausible and valuable insight. In point of fact,
however, the nature of truth is conceptualized in terms of the dialectic of concealment and unconcealment that occurs within the
phenomenological horizon that has been opened up by a work, a world, etc. In the end, his thoroughgoing antisubjectivism,
which is radicalized in the "Turn," results in a type of ineffectual positivism: objects (beings) are no longer to be "judged" (for this
would be to subject them to subjective criteria, or, worse still, to "values"), but "disclosed" or "unveiled." Yet, once the lines
between truth and error become blurred, the distinction between authentic and inauthentic unveiling
essentially evaporates: both are victimized by error in an unspecifiable way. Heidegger could conceivably
redeem his theory of truth by an attempt, however minimal, to distinguish a true from an untrue act of unconcealment. A true
unconcealment would thus unveil a being "essentially" or as it is "in itself." But no such distinction between genuine and non-
genuine unveiling is forthcoming in his work. Instead , error (Irrnis) is paradoxically deemed a mode of
unconcealment that is valid in its own right and thus "equiprimordial" with truth . Or again, Heidegger might
have claimed that unconcealment presents a type of privileged or exemplary disclosure of beings; and judgments of truth, in turn,
could have been predicated on this exemplary mode of disclosure. But no such claim is made. Instead, all we are left with is an
unexalted, positivistic affirmation of "givenness," "beings in their immediacy," "disclosure as such." In this respect, Heidegger's
theory of Seinsgeschichte regresses behind both the Husserlian and the ancient Greek conceptions of truth. For in both cases,
truth resides not in the "givenness" of beings as such, but in a supramundane or superior mode of givenness?* As a result of his
obsession with providing a "topography" of truth-with defining the clearing or openness as a sufficient condition for the
appearance of truth as "untruth"-to the wholesale exclusion of all traditional predicative considerations, Heidegger lays himself
open to extreme judgmental incapacities. And it was this philosophically induced lack of discernment that would lead to his fatal
misapprehension of the intellectual as well as the political essence of National Socialism.
at heidegger: paralysis

Heidegger’s over determined Being so strongly that free will is impossible.

Wolin, 90 - Distinguished Professor of History at the City University of New York Graduate Center - 1990 (Richard Wolin, The
Politics of Being, P. 153)

Consequently, the major problem with Heidegger's later philosophy is that the doctrine of Being, in its oppressive
omnipotence, causes the conceptual space in which freedom can be meaningfully thought to all but
disappear. In light of this fact, Jaspers' verdict concerning Heidegger's inability to grasp the nature of human
freedom-"Heidegger doesn't know what freedom is"-becomes readily intelligible. For according to the theory of the
"destining of Being," all the worldly events we experience undergo a prior, other-wordly, metaontological
determination. Like a deus absconditus, Being "essences" or "comes to presence" in ways that are
inscrutable to the human understanding. On this point, Heidegger is emphatically clear : "The history of
Being-and not the decisions of man himself-"underlies and determines every situation et condition
humaine." But if this description of the human condition is correct, then human action is essentially
unfree, and the notion of persons as potentially autonomous actors becomes equally incoherent. For the
very possibility of a meaningful correlation between human practice and its desired ends has been
disqualified in advance: it is not we who are ultimately responsible for the outcome of our actions (for
"the advent of beings"); rather, it is the "destiny of Being .

Emphasis on releasement results in paralyzing passivity.

Wolin, 90 - Distinguished Professor of History at the City University of New York Graduate Center - 1990 (Richard Wolin, The
Politics of Being, P. 147)

As we suggested earlier, the essential thinking of the later Heidegger promotes an "eclipse of practical reason." For his
post-Kehre reformulation of the relation between Being and Dasein rebels so fervently against the
voluntarist dimension of his own earlier thinking that the very concept of "meaningful human action" is
seemingly rendered null and void. If the early Heidegger attempted to rally Dasein to "decisiveness"
(Entschlossenheit), the thought of the later Heidegger appears at times to be a summary justification of human
passivity and inaction (Gelassenheit)-so prejudicially is the balance between Sein and Mensch struck in favor of the former
term. Thus, in the later Heidegger, the campaign against practical reason develops along a two-fold front: not only is the concept
of Being grossly inflated, but the powers of human reason and will are correspondingly devalued . In the later
writings, Being assumes the character of an omnipotent primal force, a "first unmoved mover," whose
"presencing" proves to be the determinative, ultimate instance for events in the lowly world of human
affairs. In its other-worldly supremacy, this force both withdraws from the tribunal of human reason and
defies the meager capacities of human description: "A Being that not only surpasses all beings-and thus
all men-but which like an unknown God rests and 'essences' in its own truth, in that it is sometimes
present and sometimes absent, can never be explained like a being in existence; instead, it can only be
'evoked.' "
at heidegger: authoritarian

Heidegger’s philosophy rejects democracy and justifies domination of those deemed “inauthentic.”
Wolin, 90 - Distinguished Professor of History at the City University of New York Graduate Center - 1990 (Richard Wolin, The
Politics of Being, P. 46)

The political philosophical implications of this theory are as unequivocal as they are distasteful to a
democratic sensibility. On the basis of the philosophical anthropology outlined by Heidegger, the modern
conception of popular sovereignty becomes a sheer non sequitur: for those who dwell in the public sphere
of everydayness are viewed as essentially incapable of self-rule. Instead, the only viable political
philosophy that follows from this standpoint would be brazenly elitist: since the majority of citizens
remain incapable of leading meaningful lives when left to their own devices, their only hope for
"redemption" lies in the imposition of a "higher spiritual mission" from above. Indeed, this was the explicit
political conclusion drawn by Heidegger in 1933. In this way, Heidegger's political thought moves precariously in
the direction of the "Fuhrerprinzip" or "leadership principle." In essence, he reiterates, in keeping with a characteristic
antimodern bias, a strategem drawn from Platonic political philosophy: since the majority of men and women are
incapable of ruling themselves insofar as they are driven by the base part of their souls to seek after
inferior satisfactions and amusements, we in effect do them a service by ruling them from above .77T o date,
however, there has never been a satisfactory answer to the question Marx poses concerning such theories of educational
dictatorship: "Who shall educate the educator?”

The desire for “authentic” leaders justifies totalitarianism.


Wolin, 90 - Distinguished Professor of History at the City University of New York Graduate Center - 1990 (Richard Wolin, The
Politics of Being, P. 115-116)

There are many dangers lurking in the statist conception of politics advanced by Heidegger in the preceding citation. The
specifically political danger of this theory of the polis/state is that it is latently totalitarian: when the state-and the "destiny of a
historical Volk" that is its raison d'ttre-are accorded unchallenged ontological primacy as "the work for the works," the autonomy
and integrity of the other spheres of life (social, cultural, religious) disappears: they are gleichgeschaltet or immediately
subsumed within the political sphere. The Greeks could solve this potential danger via the institution of direct democracy: by
virtue of this medium, political space was opened up to its maximum extent. But in Heidegger's contemporary pan-Germanic
"repetition" of the ancient polis, the opposite is true: since his twentieth century polis/ state is integrally tied to the Fiihrerprinzip, it
becomes a Fiihrerstaat, a new form of political tyranny, in which political space shrivels up into the person of the Fuhrer and his
sycophantic entourage.6 As the remarks just cited suggest, for Heidegger, the concept of a Fiihrerstaat is unproblematical
provided there be "rulers alone, but then really rulers." That is, the rulers must be "authentic" and not imposters. And as we will
soon see, Heidegger develops a theory of world-historical "leader-creators" in order to ground his partisanship for the
Fiihrerprinzip philosophically.
at heidegger: no value to life

Heidegger’s theory reduces the value to life—he forces joyless disconnection from the real world.

Wolin, 90 - Distinguished Professor of History at the City University of New York Graduate Center - 1990 (Richard Wolin, The
Politics of Being, P. 49-50)

Heidegger's characterization of everydayness is so disproportionately negative that we are seemingly left


with no immanent prospects for realizing our authentic natures in the domain of ontic life as such . For on
the basis of his phenomenological descriptions, it would seem that the ontic sphere in general-
"worldliness" in its entirety-has been "colonized" by the They. Here, we see that Heidegger's pessimistic
philosophical anthropology and his "joyless" social ontology ultimately join forces . The result is a radical
devaluation of the life-world, that delicate substratum of everyday human sociation which existential
phenomenology claims to redeem. At this point, one might raise against Heidegger's social ontology the same charge he
levels against Husserl's theory of the pure, transcendental ego: it suffers from an impoverishment of world-relations-a fact clearly
evinced in Heidegger's self-defeating celebration of the "non-relational" character of authentic Dasein cited above. For how can
the authenticity of a Dasein that is essentially "non -relational" ever attain realization in the sphere of
ontic life?
at heidegger: no truth = nazism

Critique of the enlightenment justified Nazism.

Wolin, 90 - Distinguished Professor of History at the City University of New York Graduate Center - 1990 (Richard Wolin, The
Politics of Being, P. 152)

And thus, if upon turning to the text of a 1953 lecture we find the observation: "Thinking begins only when we have
come to know that reason, glorified for centuries, is the most stiff-necked adversary of thought " we cannot
help but conclude that in his later work, Heidegger has only sunk more deeply into the bog of Logosvergessenheit. This verdict
gives cause for dismay, for it suggests that the philosopher has drawn precisely the wrong conclusions from the
political events of 1933-1945: instead of participating in the attempt to forge, out of the ravages of
postwar Europe, a new conception of reason and truth, Heidegger himself has become an even greater
"stiff-necked" advocate of counterenlightenment. His thought seeks refuge in the recrudescence of myth: "openness
for the mystery," "the remembrance of Being," and "the mirror-play of the four-fold" (gods and mortals, heaven and earth)
becomes the mystified categorial scheme around which his later thinking revolved. The notion that analogous
counterenlightenment attitudes and doctrines might have played a key role in the spiritual preparation for
the German catastrophe is a thought that has obviously never crossed his mind .57
at heidegger: paralysis

Heidegger is unable to translate ontological insights into the real world.

Wolin, 90 - Distinguished Professor of History at the City University of New York Graduate Center - 1990 (Richard Wolin, The
Politics of Being, P. 164)

Heidegger's inability to conceptualize the sociohistorical determinants and character of modern


technology raises the oft-discussed question of the "pseudo-concreteness of his philosophy "; that is, its
apparent incapacity to fulfill its original phenomenological promise as a philosophy of "existential
concretion." The problem was already evident in the tension between the ontological and ontic levels of analysis that
dominated the existential analytic of Being and Time. For there the sphere of ontic life seemed degraded a priori as
a result of its monopolization by the "They" and its concomitant inauthentic modalities. As a result, both
the desirability and possibility of effecting the transition from the metalevel of ontology to the "factical"
realm of ontic concretion seemed problematical from the outset . Nowhere was this problem better illustrated than in
the case of the category of historicity. And thus despite Heidegger's real insight into limitations of Dilthey's historicism, the
inflexible elevation of ontology above the ontic plane virtually closes off the conceptual space wherein
real history might be thought. In truth, it can only appear as an afterthought: as the material demonstration of conclusions
already reached by the categories of existential ontology. Consequently, the "ontology of Being and Time is still
bound to the metaphysics that it rejects. The conventional tension between existentia and essentia stands
behind the difference between everyday (factical) and 'authentic historical existence.'
at heidegger: calculations good

Old flaws in calculative thought require expanding the reasoning process, not rejecting it.

Wolin, 90 - Distinguished Professor of History at the City University of New York Graduate Center - 1990 (Richard Wolin, The
Politics of Being, P. 167)

Heidegger's theory of technology ultimately collapses under the weight of its own self-imposed
conceptual limitations. And thus, the intrinsic shortcomings of his theoretical framework prevent him from
entertaining the prospect that the problem of technological domination owes more to the dearth of reason
in the modern world rather than an excess. For in modern life, the parameters of rationality have been
prematurely restricted: formal or instrumental reason has attained de facto hegemony; practical reason-
reflection on ends-has been effectively marginalized. Instead of the "overcoming" of reason
recommended by Heidegger, what is needed is an expansion of reason's boundaries, such that the
autonomous logic of instrumental rationality is subordinated to a rational reflection on ends. Similarly,
Heidegger's incessant lamentations concerning the "will to will-the theoretical prism through which he views the modern project
of human self-assertion in its entirety- only serve to confuse the problem at issue?7 That the forces of technology and industry
follow an independent logic.
at heidegger: dread of death bad

Glorification of dreading death became an excuse for violence on the battlefield.

Wolin, 01 – Distinguished Professor of History at the City University of New York Graduate Center – 2001 (Richard Wolin,
Heidegger’s Children, P. 163-164)

One of the concepts from Being and Time that Marcuse viewed with suspicion was Being-toward-death. Although its importance
has often been underplayed in the vast secondary literature on Being and Time, Being-toward-death proves a crucial way station
on the road to authenticity. Whereas everyday Dasein (the "they") systematically shuns and avoids confronting the predicament
of human finitude, authentic Dasein distinguishes itself by a willingness to confront the phenomenon of death unflinchingly. An
awareness of death's inevitability sharpens Dasein's worldly involvements and lcommitments. Since Existenz is inherently finite
(there is no salvation or eternal life), Dasein's commitment to temporality and worldliness must be radical and total. Yet, as
Marcuse notes, Heidegger's ontological characterization of death betrays a specific ontic context: the
glorification of the "front experience" in Germany following World War I. For example, in Ernst Jiinger's
provocative battle chronicles, In the Storm of Steel and War as Inner Experience, the confrontation with death in war was
elevated to ! the status of a supreme existential rite of passage . It is difficult to dissociate Heidegger's exaltation of
Being-toward-death from this postwar cultural context. Ultimately, this ethos, which emphasized the
imperatives of "sacrifice" and the importance of Nietzsche's maxim, "Have the courage to live
dangerously," found a home in the martial ethos of National Socialism. Heidegger's own political
speeches on behalf of the regime are suffused with the rhetorical bombast characteristic of this idiom .
at heidegger: permutation

Action and reflection on consequences of that action are compatible.

Padrutt, 92 – Psychiatrist and President of the Daseinsanalyse Gesellschaft – 1992 (Hanspeter Padrutt, Heidegger and the
Earth, “Heidegger and Ecology,” ed. LaDelle McWhorter, P.31)

Once in a while the conceptual interplay of theory and praxis is put against this attempt . From the philosophical point of
view the so-called practical or political dimension of the attempt is rejected, whereas from the ecological
point of view the so-called theoretical, philosophical dimension is rejected. But deeper reflection and
decisive action do not need to contradict each other. Those who shield themselves from the political
consequences might one day be confronted by the fact that no decision is still a decision that can have
consequences. And those who believe that they need not bother about thinking fail to recognize that no
philosophy is also a philosophy – e.g., a cybernetic worldview – that also has consequences.
at heidegger: being meaningless

Discussions of Being are meaningless—the concept is too abstract to be useful.

Rosen, 69 - Borden Parker Bowne Professor of Philosophy at Boston University – 1969 (Stanley Rosen, Nihilism: A
Philosophical Essay, P. 35)

Being is not a thing (the ontologists say), but the source of things. To speak of this source as the sum of all properties common
to things is to reduce the source of things to an abstraction derivative from things. It is to make things the source of Being, an
absurd reversal of the truth! Speech about things is ontic speech and, as such, diverts our attention away from
Being: seduction by ontic speech thus makes fundamental ontology impossible . Being is not an abstraction,
because abstractions are derived or constructed from concrete particulars; Being is not a construction, but the source of all
possible constructions, hence not abstract but most concrete. Finally, an abstraction is still itself a particular thing, whereas
Being, as the origin or ground of particulars, is not . Being is not a thing; rather than call it "anything at all," we
would be better advised to say that Being is nothing. Ontology is speech about no thing, and so about
nothing. But even further, since human speech is necessarily of, and in terms of, things (and so ontic), ontological speech is
also the speech of nothing: it is nothingness speaking about itself, or a gift from nothingness to man, whose own ontic speech
obscures the gift even in the act of acknowledging, receiving, or attempting t o dis-cover it.)

Discourse on being is so abstract that it renders us silent—it is nihilistic paralysis.

Rosen, 69 - Borden Parker Bowne Professor of Philosophy at Boston University – 1969 (Stanley Rosen, Nihilism: A
Philosophical Essay, P. 45-46)

I have been arguing that ontological speech, in the sense attributed to it by those who follow Heidegger’s distinction between
the ontological and ontic, is in fact silence. Ontologists of this type wish to talk about Being as distinct from
beings, and speech will simply not permit this. If this is a defect of speech, and the significance of speech
is in the deepest and final sense relative to silence, then there is no reason for what we say or for whether
we speak at all, other than the mere fact, although there is equally no reason to keep silent. The result is
absurdism or nihilism. Therefore no reason can be given which would justify our falling into such desperate straits. Every
fundamental ontological speech of the type in question is not just self-refuting but self-canceling.
A2: heidegger: link over simplified

Heidegger makes it imposisbel to distinguish between democracy and totalitarianism.

Ferry and Renaut, 90 – Professor of Political Science at the Sorbonne and Professor of Philosophy at Nantes – 1990 (Luc Ferry
and Alain Renaut, Heidegger and Modernity, trans. Franklin Philip, P. 87-88)

From this viewpoint, it is first of all clear, as we have noted, that this criticism of technology as the global concretization of
an idea of man as consciousness and will implies, like it or not, a deconstruction of democratic reamain son and hence, in
some sense, of humanism. It is also clear, however, that Heidegger's thinking, even fixed up this way, continues in
some odd way to misfire because of its one-dimensionality. Just as, on the strictly philosophical level, it
leads to lumping the various facets of modem subjectivity together in a shapeless mass and to judging that
the progression from Descartes to Kant to Nietzsche is linear and in fact inevitable; just as, on the political
level, it leads to the brutal inclusion of American liberalism in the same category with Stalinist
totalitarianism. Now this is no mere matter of taste: anyone has the right to loathe rock concerts, Disney World, and
California. Nonetheless, no one may-Hannah Arendt and Leo Strauss, who lived in the United States, did not make this mistake -
identify, in the name of a higher authority, the barbarism of the Soviet gulags with the depravities of a Western society whose
extraordinary political, social, and cultural complexity allows areas of freedom that it would wholly unwarranted to judge a priori
as mere fringes or remnants of a world in decline.
A2: Dillon: calculations good

Viewing calculative thought as equivalent to domination ensures total political paralysis.

Bronner, 04 Stephen Eric Bronner, Professor of Political Science at Rutgers University, 2004, Reclaiming the Enlightenment:
Toward a Politics of Radical Engagement, p. 3-5

“Instrumental reason” was seen as merging with what Marx termed the “commodity form” underpinning
capitalist social relations. Everything thereby became subject to the calculation of costs and benefits. Even art and
aesthetic tastes would become defined by a “culture industry”—intent only upon maximizing pro fits by seeking the lowest common denominator for its products. Instrumental rationality
was thus seen as stripping the supposedly “autonomous” individual, envisioned by the philosophes, of both the means and the will to resist manipulation by totalitarian movements. En-
lightenment now received two connotations: its historical epoch was grounded in an anthropological understanding of civilization that, from the first, projected the opposite of progress.
This gave the book its power: Horkheimer and Adorno offered not simply the critique of some prior historical moment in time, but of all human development. This made it possible to
identify enlightenment not with progress, as the philistine bourgeois might like to believe, but rather—unwittingly—with barbarism, Auschwitz, and what is still often called “the totally
administered society.” Such is the picture painted by Dialectic of Enlightenment.. But it should not be forgotten that its authors were concerned with criticizing enlightenment generally,
and the historical epoch known as the Enlightenment in particular, from the standpoint of enlightenment itself: thus the title of the work. Their masterpiece was actually “intended to
prepare the way for a positive notion of enlightenment, which will release it from entanglement in blind domination.”4 Later, in fact, Horkheimer and Adorno even talked about writing a
sequel that would have carried a title like “Rescuing the Enlightenment” (Rettung der Aufklarung).5 This reclamation project was never completed, and much time has been spent
the logic of their argument ultimately left them with little positive to
speculating about why it wasn’t. The reason, I believe, is that
say. Viewing instrumental rationality as equivalent with the rationality of domination, and this rationality
with an increasingly seamless bureaucratic order, no room existed any longer for a concrete or effective
political form of opposition: Horkheimer would thus ultimately embrace a quasi-religious “yearning for the
totally other” while Adorno became interested in a form of aesthetic resistance grounded in “negative dialectics.”
Their great work initiated a radical change in critical theory, but its metaphysical subjectivism surrendered
any systematic concern with social movements and political institutions. Neither of them ever genuinely appreciated the democratic
inheritance of the Enlightenment and thus, not only did they render critique independent of its philosophical foundations,6 but also of any practical interest it might serve. Horkheimer and
Adorno never really grasped that, in contrast to the system builder, the blinkered empiricist, or the fanatic, the philosophe always evidenced a “greater interest in the things of this world, a
greater confidence in man and his works and his reason, the growing appetite of curiosity and the growing restlessness of the unsatisfied mind—all these things form less a doctrine than a
spirit.”7 Just as Montesquieu believed it was the spirit of the laws, rather than any system of laws, that manifested the commitment to justice, the spirit of Enlightenment projected the
radical quality of that commitment and a critique of the historical limitations with which even its best thinkers are always tainted. Empiricists may deny the existence of a “spirit of the
times.” Nevertheless, historical epochs can generate an ethos, an existential stance toward reality, or what might even be termed a “project” uniting the diverse participants in a broader
intellectual trend or movement. The Enlightenment evidenced such an ethos and a peculiar stance toward reality with respect toward its transformation. Making sense of this, however, is
impossible without recognizing what became a general stylistic commitment to clarity, communicability, and what rhetoricians term “plain speech.” For their parts, however, Horkheimer
and Adorno believed that resistance against the incursions of the culture industry justified the extremely difficult, if not often opaque, writing style for which they would become famous—
or, better, infamous. Their esoteric and academic style is a far cry from that of Enlightenment intellectuals who debated first principles in public, who introduced freelance writing, who
employed satire and wit to demolish puffery and dogma, and who were preoccupied with reaching a general audience of educated readers: Lessing put the matter in the most radical form in
what became a popular saying—”Write just as you speak and it will be beautiful”—while, in a letter written to D’Alembert in April of 1766, Voltaire noted that “Twenty folio volumes will
never make a revolution: it’s the small, portable books at thirty sous that are dangerous. If the Gospel had cost 1,200 sesterces, the Christian religion would never have been established.”9

Appropriating the Enlightenment for modernity calls for reconnecting with the
vernacular. This does not imply some endorsement of anti-intellectualism. Debates in highly specialized
fields, especially those of the natural sciences, obviously demand expertise and insisting that intellectuals must “reach the masses” has always been a questionable strategy. The sub ject
.
under discussion should define the language in which it is discussed and the terms employed are valid insofar as they illuminate what cannot be said in a simpler way
Horkheimer and Adorno, however, saw the matter differently. They feared being integrated by the culture
industry, avoided political engagement, and turned freedom into the metaphysical-aesthetic preserve of
the connoisseur. They became increasingly incapable of appreciating the egalitarian impulses generated
by the Enlightenment and the ability of its advocates—Ben Franklin, Thomas Jefferson, James Madison,
Thomas Paine, and Rousseau—to argue clearly and with a political purpose.1’ Thus, whether or not their
“critical” enterprise was “dialectically” in keeping with the impulses of the past, its assumptions
prevented them from articulating anything positive for the present or the future.
Humanism Good

Humanism is critical to fighting wars and oppression – empirically proven


Radest 89 [Howard, Dean of Humanist Institute and Director of Ethical Culture Schools, “Doing Good: Humanism and the Liberal
Temptation”, Humanism Today, Vol 5. http://www.humanismtoday.org/vol5/radest.pdf]
Humanists support all
the "right" causes. We will be found defending peace and arguing for
disarmament and opposing nuclear proliferation. Our agenda will include population control and
environmental protection, fair housing and civil rights. We will attack censorship and fight for civil liberties. Separation of
church and state and religious freedom will stand high among our priorities as will "pro-choice" and public schooling. To our credit, Humanists will tend to be actively
engaged in these and other causes, although our engagement will take characteristic form. With rare exceptions as in the "freedom marches" during the 1960's in the
Humanists will be more likely to petition than to demonstrate, to lobby than to march, to
South
proclaim rather than to analyze.2 In that, we exhibit a certain confidence in the processes of
democratic change and a certain conservatism in our approach to power and the state. When pushed,
Humanists, reflecting our 18th century rationalist origins, will still exhibit confidence in schooling 3 and will reject the barricade as the way to get political
reconstruction

Humanism is inescapable – and giving up on it dooms the planet to extinction


Davis 97 [Tony Proffessor of English at Bimignham. Humanism 130]
So there will not after all be, nor indeed could that be, any definitions. The several humanisms – the civic. The
several humanisms – the civic humanism of the quattrocento Italian city-states, the Protestant humanism of sixteenth century northern Europe, the
rationalistic humanism that attended at the revolutions of enlightened modernity, and the romantic and positivistic humanisms through which the European
bourgeoisies established their hegemony over it, the revolutionary humanism that shook the world and the liberal humanism that sought to tame it, the humanism
of the Nazis and the humanism of their victims and opponents, the antihumanist humanism of Heidegger and the humanist antihumanism of Foucault and
are not reducible to one, or even to a single line or pattern. Each has its distinctive historical
Althusser –
curve, its particular discursive poetics, its own problematic scansion of the human. Each seeks, as all discourses must, to
impose its own answer to the question of ‘which is to be master’. Meanwhile, the problem of humanism remains,
for the present, an inescapable horizon within which all attempts to think about the ways in
which human being have, do, might live together in and on the world are contained. Not that the
actual humanisms described here necessarily provide a model, or even a useful history, least of all for those very numerous people, and peoples, for whom they
have been alien and oppressive. Some, at least, offer a grim warning. Certainly it should no longer be possible to formulate phrases like ‘the destiny of man’ or
‘the triumph of human reason’ without an instant consciousness of the folly and brutality they drag behind them. All humanisms, until now,
have been imperial. They speak of the human in the accents and the interests of a class, a sex, a ‘race’.
Their embrace suffocates those whom it does not ignore. The first humanists scripted the tyranny of Borgias,
Medicis and Tudors. Later humanisms dreamed of freedom and celebrated Frederick II, Bonaparte, Bismarck, Stalin. The liberators of
colonial America, like the Greek and Roman thinkers they emulated, owned slaves. At various times, not excluding the present, the circuit of
the human has excluded women, those who do not speak Greek or Latin or English, those whose complexions are not pink, children, Jews.
At the same time, though it
It is almost impossible to think of a crime that has not been committed in the name of humanity.
is clear that the master narrative of transcendental Man has outlasted its usefulness, it would
be unwise simply to abandon the ground occupied by the historical humanisms. For one
thing, some variety of humanism remains, on many occasions, the only available alternative
to bigotry and persecution. The freedom to speak and write, to organize and campaign in
defence of individual or collective interests, to protest and disobey: all these, and the prospect
of a world in which they will be secured, can only be articulated in humanist terms. It is true
that the Baconian ‘Knowledge of Causes, and Secrett Motions of Things’, harnessed to an overweening
rationality and an unbridled technological will to power, has enlarged the bounds of human empire to the
point of endangering the survival of the violated planet on which we live. But how, if not by
mobilizing collective resources of human understanding and responsibility of ‘enlightened
self-interest’ even, can that danger be turned aside?
Humanism Good

Humanism is key to preventing atrocities such as the Holocaust. By denying humanism,


they legitimize abuses of human rights
Ketels 96[Violet, associate professor of English at Temple University, The Holocaust: Remembering for the Future: ‘Havel to the
Castle!’ The Power of the Word,” The Annals of The American Academy of Political and Social Science, November, 1996]
In the Germany of the 1930s, a demonic idea was born in a demented brain; the word went forth; orders
were given, repeated, widely broadcast; and men, women, and children were herded into
death camps. Their offshore signals, cries for help, did not summon us to rescue. We had
become inured to the reality of human suffering. We could no longer hear what the words
meant or did not credit them or not enough of us joined the chorus. Shrieking victims
perished in the cold blankness of inhumane silence. We were deaf to the apocalyptic
urgency in Solzhenitsyn's declaration from the Gulag that we must check the disastrous
course of history. We were heedless of the lesson of his experience that only the unbending
strength of the human spirit, fully taking its stand on the shifting frontier of encroaching
violence and declaring "not one step further," though death may be the end of it--only this
unwavering firmness offers any genuine defense of peace for the individual, of genuine peace
for mankind at large. 2 In past human crises, writers and thinkers strained language to the breaking point to keep
alive the memory of the unimaginable, to keep the human conscience from forgetting. In the current context, however ,
intellectuals seem more devoted to abstract assaults on values than to thoughtful probing of
the moral dimensions of human experience. "Heirs of the ancient possessions of higher knowledge and
literacy skills," 3 we seem to have lost our nerve, and not only because of Holocaust history and its tragic aftermath. We feel
insecure before the empirical absolutes of hard science. We are intimidated by the "high modernist rage against mimesis and
content," 4 monstrous progeny of the union between Nietzsche and philosophical formalism, the grim proposal we have
bought into that there is no truth, no objectivity, and no disinterested knowledge. 5 Less certain about the power of
language, that "oldest flame of the [*47] humanist soul," 6 to frame a credo to live by or criteria to judge by, we are
vulnerable even to the discredited Paul de Man's indecent hint that "wars and revolutions are not empirical events . . . but
'texts' masquerading as facts." 7 Truth and reality seem more elusive than they ever were in the past;
values are pronounced to be mere fictions of ruling elites to retain power. We are
embarrassed by virtue. Words collide and crack under these new skeptical strains, dissolving into banalities the colossal
enormity of what must be expressed lest we forget. Remembering for the future has become doubly dispiriting by our having to remember
for the present, too, our having to register and confront what is wrong here and now. The reality to be fixed in memory shifts as we seek
words for it; the memory we set down is flawed by our subjectivities. It is selective, deceptive, partial, unreliable, and amoral. It plays tricks
and can be invented. It stops up its ears to shut out what it does not dare to face. 8 Lodged in our brains, such axioms, certified by science
and statistics, tempt us to concede the final irrelevance of words and memory. We have to get on with our lives. Besides, memories
reconstructed in words, even when they are documented by evidence, have not often changed the world or fended off the powerful seductions
to silence, forgetting, or denying. Especially denying, which, in the case of the Holocaust, has become an obscene industry competing in
the open market of ideas for control of our sense of the past. It is said that the Holocaust never happened. Revisionist history with a
vengeance is purveyed in words; something in words must be set against it. Yet what? How do we nerve to the task when we are
increasingly disposed to cast both words and memory in a condition of cryogenic dubiety? Not only before but also since 1945, the
criminality of governments, paraded as politics and fattening on linguistic manipulation and deliberately reimplanted memory of past real or
imagined grievance, has spread calamity across the planet. "The cancer that has eaten at the entrails of Yugoslavia since Tito's death [has]

Card Continues
Card Continued
Kosovo for its locus," but not merely as a piece of land. The country's rogue adventurers use the word "Kosovo" to reinvoke as sacred the
land where Serbs were defeated by Turks in 1389! 9 Memory of bloody massacres in 1389, sloganized and distorted in 1989, demands the
bloody revenge of new massacres and returns civilization not to its past glory but to its gory tribal wars. As Matija Beckovic, the bard of Serb
nationalism, writes, "It is as if the Serbian people waged only one battle--by widening the Kosovo charnel-house, by adding wailing upon
wailing, by counting new martyrs. to the martyrs of Kosovo. . . . Kosovo is the Serbianized [*48] history of the Flood--the Serbian New
Testament." 10 A cover of Suddeutsche Zeitung in 1994 was printed with blood donated by refugee women from Bosnia in an eerily
We stand benumbed before multiplying horrors. As Vaclav Havel
perverse afterbirth of violence revisited. 11
warned more than a decade ago,
regimes that generate them "are the avant garde of a global crisis in
civilization." The depersonalization of power in "system, ideology and apparat," pathological
suspicions about human motives and meanings, the loosening of individual responsibility, the
swiftness by which disastrous events follow one upon another "have deprived us of our
conscience, of our common sense and natural speech and thereby, of our actual humanity." 12
Nothing less than the transformation of human consciousness is likely to rescue us.
Consequentialism good

Even moral absolutists concede that catastrophic impacts come first


Haber 2002Joram Graf Haber, Distinguished Professor of Philosophy at Bergen Community College, ” Absolutism and Its
Consequentialist Critics,” p. 6.

Furthermore, not only are Anscombe, Donagan, and Geach absolut ists (in the weak sense of the term), but
so are Charles Fried and Bernard Williams. After defending what he calls absolutism (Selection VI),
Fried observes:
We can imagine extreme cases where killing an innocent person may save a whole nation. In such cases it
seems fanatical to maintain the absolute ness of the judgment, to do right even if the heavens will in fact
fall. And so the catastrophic may cause the absoluteness of right and wrong to yield "
And again:
The concept of the catastrophic is a distinct concept just because it identifies the extreme situations in which the
usual categories of judgments (including the category of right and wrong) no longer apply.14
In passages like these. Fried shows himself an advocate of "weak absolutism" by saying that catastrophic cases
produce conceptual anarchy. As Fried says later on,
I do not know . . . whether I would be willing to kill an innocent person to save the whole of humanity from
excruciating suffering and death. There are boundaries to each of these concepts themselves, and the concepts
themselves often become blurred, indeterminate, subject to judgments of prudence at those boundaries."
Thus, Fried is committed to the view that "In no situation could it be right to <|)" except when a catastrophe occurs,
but that doesn't count since in that situation the concepts of right and wrong no longer apply. Bernard Williams
suggests though does not exactly endorse a similar view when he intimates that in extreme situations (when the
consequences of not cping would be disastrous), "it cannot matter any more what happens."16

Absolutism is irrational – it commits us to producing less good than we can


Haber 2002 Joram Graf Haber, Distinguished Professor of Philosophy at Bergen Community College, ” Absolutism and Its
Consequentialist Critics,” p. 8.

With respect to its critics, consequentialists typically argue that absolutism is irrational—it commits us to the view
that we should produce less good than we are in a position to produce and prevent less evil than we are in a position
to prevent. Often, the controversy centers around hypothetical cases (such as the ones discussed by Philippa Foot in
Selection XI) where an agent is faced with performing an absolutely forbidden action or allowing a disastrous state
of affairs to obtain." In cases like "the fat man in the cave,"20'for example, the consequentialist argues (against the
absolutist) that it is simply irrational not to kill him if the alternative is to allow the others to perish. (For a different
view on a related theme, see Anscombe's "Who Is Wronged?'—Selection XII.)
Kai Nielsen (Selection XIII), for one, takes issue with the absolutist's intuition that it is never permissible to kill an
innocent person. Speaking of the "fat man in the cave," Nielsen contends that it is no less counter-intuitive to kill the
fat man than it is not to, and that this implies there is a possible world in which killing is permissible. (He also makes
the point that if this is so, then thinking of killing the fat man is hardly indicative of a corrupt mind.) Having argued,
then, that refusing to kill the fat man is at least as problematic as not killing him.
Nielsen makes two other points that are characteristic of consequentialist critics. The first is that consequentialists
typically share the same intuitions as absolutists while insisting that they can account for them in a way that
absolutists cannot. (In speaking of "the magistrate and the threatening mob,"21 for instance, Nielsen is skeptical that
consequentialism requires that the magistrate execute the innocent derelict.) The second point is that there is nothing
sacred about our moral intuitions. If, for instance, we have the intuition that it is always wrong to kill but the intuition
generates problematic counterexamples, then that is a reason to give up our intuition. Conservativism for
conservatism's sake is an irrational dogma.
Consequentialism good

Even deontology puts survival first


Fried 2002
Charles Fried, professor of law @ Harvard, “Right and Wrong as Absolute.” P. 76-77, in Absolutism and its Consequentialist Critics, edited by
Jorem Graf Haber.

Even within such boundaries we can imagine extreme cases where killing an innocent person may save a whole
nation. In such cases it seems fanatical to maintain the absoluteness of the judgment, to do right even if the heavens
will in fact fall. And so the catastrophic may cause the absoluteness of right and wrong to yield, but even then it
would be a non sequitur to argue (as consequentialists are fond of doing) that this proves that judgments of right and
wrong are always a matter of degree, depending on the relative goods to be attained and harms to be avoided. 1
believe, on the contrary, that the concept of the catastrophic is a distinct concept just because it identifies the extreme
situations in which the usual categories of judgment (including the category of right and wrong) no longer apply. At
the other end of the spectrum, there is the concept of the trivial, the de minimis where the absolute categories do not
yet apply. And the trivial also does not prove that right and wrong are really only a matter of degree. It is because
of these complexities and because the term absolute is really only suggestive of a more complex structure, that I also
refer to the norms of right and wrong not as absolute but as categorical.*) When we say that one must not grievously
harm an innocent person, that one must not lie, these are categorical prohibitions in the sense that (within limits) no
amount of good can justify them. But they are not absolute in the sense that we may never be justified in doing acts
which have these very results—the death of an innocent person, the propagation of false beliefs—as a consequence.
They are absolute in the sense that they point out certain acts we must not perform. They are not absolute in the
consequentialist's sense; they do not state that a certain state of the world is of such supreme importance that the
value of everything else must be judged by its tendency to produce that state. So here we see a complex relation
between deontological judgments on what we do and evaluative (axiological) judgments on states of the world—
with which we are also concerned. We must indeed be concerned with producing good in the world, but without
violating the absolute norms of right and wrong).
Calculability good

Calculability is a prerequisite to care for the other – without it, the other’s suffering is
inaccessible
Santilli 2003 Paul C., Siena College. “Radical Evil, Subjection, and Alain Badiou’s Ethic of the Truth” Event World Congress of The
International Society for Universal Dialogue, Pyrgos, Greece May 18-22, 2003.

From the standpoint of an ethics of subjection there is even something unnecessary or superfluous about the void of suffering in the subject
bearers of evil. For Levinas, the return to being from the ethical encounter with the face and its infinite depths is fraught with the danger the
subject will reduce the other to a "like-me," totalizing and violating the space of absolute alterity. As Chalier puts it, "Levinas conceives of the
moral subject's awakening, or the emergence of the human in being, as a response to that pre-originary subjection which is not a happenstance of
being."28 But if there really is something inaccessible about suffering itself, about the 'other' side
of what is manifestly finite, subjected, and damaged, then to a certain extent it is irrelevant to
ethics, as irrelevant as the judgment of moral progress in the subject-agent. Let me take the parent-
child relation again as an example. Suppose the child to exhibit the symptoms of an illness. Are not the
proper "ethical" questions for the parent to ask questions of measure and mathematical multiples:
How high is the fever? How long has it lasted? How far is the hospital? Can she get out of bed? Has
this happened before? These are the
questions of the doctor, the rescue squads and the police. They are questions about being,
about detail, causes
and effects. Ethically our response to the needs of must be reduced to a positivity simply because
we have access to nothing but the symptoms, which are like mine. Our primary moral responsibility
is to treat the symptoms that show up in being,
not the radically other with whom I cannot identify. Say we observe someone whose hands have been chopped off with
a machete. How would we characterize this? Would it not be slightly absurd to say, "He had his limbs severed and he suffered," as though the
cruel amputation were not horror enough. Think of the idiocy in the common platitude: "She died of cancer, but thank God, she did not suffer", as
For ethics, then, the only suffering that
though the devastating annihilation of the human by a tumor were not evil itself .
matters are the visible effects of the onslaught of the world. All other suffering is excessive and
inaccessible. Therefore, it is in being, indeed in the midst of the most elemental facts about
ourselves and other people, that we ethically encounter others by responding to their needs and
helping them as best we can.
It is precisely by identifying being and not pretending that we know any thing about suffering,
other than it is a hollow in the midst of being, that we can act responsibly. What worries me about
Levinas is that by going beyond being to what he
regards as the ethics of absolute alterity, he risks allowing the sheer, almost banal facticity of suffering to be swallowed in the infinite depths of
transcendence. Indeed, it seems to me that Levinas too often over emphasizes the importance of the emergence of the subject and the inner good
in the ethical encounter, as though the point of meeting the suffering human being was to come to an awareness of the good within oneself and
not to heal and repair. I agree with Chalier's observation that Levinas's "analyses adopt the point of view of the moral subject, not that of a person
who might be the object of its solicitude."29 Ethics has limits; there are situations like the Holocaust where to speak of a moral responsibility to
But an ethics that would be oriented to the vulnerabilities of the subjected
heal and repair seems pathetic.
(which are others, of course, but also myself) needs to address the mutilation, dismemberment,
the chronology of torture, the
numbers incarcerated, the look of the bodies, the narratives, the blood counts, the mines, knives,
machetes, and poisons. Evil really is all that. When the mind does its work, it plunges into being,
into mathematical multiples and starts counting the cells, the graveyards, and bullet wounds.
Rational practical deliberation is always about the facts that encircle the void inaccessible to
deliberation and practical reason.30
Calculability good

Calculability is key to resistance against the worst forms of violence


Campbell ’99 David, professor of international politics at the University of Newcastle. “The Deterritorialization of Responsibility:
Levinas, Derrida, and Ethics after the End of Philosophy,” published in Moral Spaces: Rethinking Ethics and World Politics, edited by
David Campbell and Michael J. Shapiro.

The finite nature of the decision may be a "madness" in the way it renders possible the impossible,
the infinite character of justice, but Derrida argues for the necessity of this madness. Most
importantly, although Derrida's argument concerning the decision has, to this point, been concerned with an
account of the procedure by which a decision is possible, it is with respect to the necessity of the decision
that Derrida begins to formulate an account of the decision that bears upon the content of the decision. In so
doing, Derrida's argument addresses more directly—more directly, I would argue, than is acknowledged by
Critchley— the concern that for politics (at least for a progressive politics) one must provide an account of the
decision to combat domination.
That undecidability resides within the decision, Derrida argues, "that justice exceeds law and
calculation, that the unpresentable exceeds the determinable cannot and should not serve as alibi
for staying out of juridico-political battles, within an institution or a state, or between institutions
or states and others."" Indeed, "incalculable justice requires us to calculate." From where does this
insistence come? What is behind, what is animating, these imperatives? It is both the character of
infinite justice as a heteronomic relationship to the other, a relationship that because of its
undecidabilily multiplies responsibility, and the fact that "left to itself, the incalculable and giving
{donatrice) idea of justice is always very close to the bad, even to the worst, for it can always be
reappropriated by the most perverse calculation."'2 The necessity of calculating the incalculable
thus responds to a duty, a duty that inhabits the instant of madness and compels the decision to
avoid "the bad," the "perverse calculation," even "the worst." This is the duty that also dwells with
deconstruction and makes it the starting point, the "at least necessary condition," for the
organization of resistance to totalitarianism in all its forms. And it is a duty that responds to
practical political concerns when we recognize that Derrida names the bad, the perverse, and the
worst as those violences "we recognize all too well without yet having thought them through, the
crimes of xenophobia, racism, anti-Semitism, religious or nationalist fanaticism."93
Util Good

THE ONLY MORAL ACTION IS TO SAVE THE MOST LIVES


 Wasserman and Strudler 2003 Philosophy and Public Affairs 31.1 

In making choices about saving people from death, what moral signif- icance should attach to the fact that one
choice involves saving more people than another? Consequentialists typically have an easy time with such questions
because they believe that the morally best choice pro- duces the best consequences and that, other things being
equal, more lives saved is a better consequence than fewer lives saved. The conse- quentialist position involves what
might be called the compensation as- sumption: the proposition that other things equal, the gain that comes from
saving a larger group of people somehow more than compensates for the loss that occurs by not saving some other,
smaller group of peo- ple. If numbers have the moral importance that consequentialists sup- pose, then it should be
at least presumptively right to sacrifice a person to save others; for example, it is unclear why one may not simply
kill an innocent person and harvest his organs if doing so is the only available way of saving the lives of people who
will die without those organs. In fact, however, the prospect of saving the lives of those people seems to provide no
reason, or an exceedingly weak one, for killing an innocent person, even if there is no other way to acquire needed
organs. One might respond in many ways to the apparent harshness of the consequentialist approach to choices
among lives. Most obviously, one might seek to qualify or constrain consequentialist reasoning by adopt- ing a
pluralist moral theory that mixes or integrates consequentialist and nonconsequentialist elements. We cannot
canvass pluralistic theo- ries in this article, but we must acknowledge that some of them are com- plex and
ingeniou~.~ Still, we suspect that they are doomed attempts to breed species that are in essence incompatible. If one
shares our doubts about the prospects for modifying or constraining consequentialism, it makes sense to look to
nonconsequentialist approaches to choices among lives-approaches that do not rely directly or indirectly on the
claim that more people saved is a better consequence. The nonconsequentialist approaches we consider treat the
failure to save the group with the greater number as a failure to respect the value or equality of the individual lives in
that group. We argue that despite their initial appeal, these approaches do not succeed, and we conclude that there
does not yet exist a cogent nonconsequentialist answer to the question of numbers. We begin with an important early
attempt by Gregory Kavka because an analysis of its weaknesses suggests the moral complexity of choices among
lives and the distinctive character of more recent efforts to understand these choices.
Util Good

Nonconsequentialist theories disregard value and equality of life.  Consequentialism is best


because it preserves the most lives.
 Wasserman and Strudler 2003 Philosophy and Public Affairs 31.1 
In making choices about saving people from death, what moral significance should attach to the fact that one choice
involves saving more people than another? Consequentialists typically have an easy time with such questions
because they believe that the morally best choice pro- duces the best consequences and that, other things being
equal, more lives saved is a better consequence than fewer lives saved. The conse- quentialist position involves what
might be called the compensation as- sumption: the proposition that other things equal, the gain that comes from
saving a larger group of people somehow more than compensates for the loss that occurs by not saving some other,
smaller group of peo- ple. If numbers have the moral importance that consequentialists sup- pose, then it should be
at least presumptively right to sacrifice a person to save others; for example, it is unclear why one may not simply
kill an innocent person and harvest his organs if doing so is the only available way of saving the lives of people who
will die without those organs. In fact, however, the prospect of saving the lives of those people seems to provide no
reason, or an exceedingly weak one, for killing an innocent person, even if there is no other way to acquire needed
organs. One might respond in many ways to the apparent harshness of the consequentialist approach to choices
among lives. Most obviously, one might seek to qualify or constrain consequentialist reasoning by adopt- ing a
pluralist moral theory that mixes or integrates consequentialist and nonconsequentialist elements. We cannot
canvass pluralistic theo- ries in this article, but we must acknowledge that some of them are com- plex and
ingeniou~.~ Still, we suspect that they are doomed attempts to breed species that are in essence incompatible. If one
shares our doubts about the prospects for modifying or constraining consequentialism, it makes sense to look to
nonconsequentialist approaches to choices among lives-approaches that do not rely directly or indirectly on the
claim that more people saved is a better consequence. The nonconsequentialist approaches we consider treat the
failure to save the group with the greater number as a failure to respect the value or equality of the individual lives in
that group. We argue that despite their initial appeal, these approaches do not succeed, and we conclude that there
does not yet exist a cogent nonconsequentialist answer to the question of numbers. We begin with an important early
attempt by Gregory Kavka because an analysis of its weaknesses suggests the moral complexity of choices among
lives and the distinctive character of more recent efforts to understand these choices.
***A2: Capitalism ***

***A2: rights K***

rights good

Rights are not perfect but do contain a radical element in their promotion of universal human dignity. They can be effectively
used to leverage the state.

Daly, Research Fellow in Philosophy, 04 (Frances, Australian National University, “The Non-citizen and the Concept of Human
Rights”, borderlands, http://www.borderlandsejournal.adelaide.edu.au/vol3no1_2004/daly_noncitizen.htm)

At its most fundamental, right is the right to something, and within the realm of natural rights or rights of the human being, it has
been principally concerned with rights against oppression and inequality in order to realize a potential for freedom. Citizen rights
have at their basis quite different values, namely, a range of political and property rights to be realized within and not against the
State. This is not to say that law associated with human rights is not, at times, itself an external form of oppression - but natural
or human right is also able to offer something quite different. The term needs to be used advisedly because of the problematic
connotations it has – but there is a tradition of natural right containing anticipatory elements of human dignity in which forms of
justice as ethically-based community survive, and it is this tradition, I would argue, which needs to be renewed. We can see this
in all struggles for human dignity in which unsatisfied demands exist for overcoming the lack of freedom of exploitation and
constraint; the inequality of degradation and humiliation; the absence of community in egoism and disunity. And so too can we
view this via the necessary reference point that a critique of right provides: by acknowledging the hypocrisy of law or the distance
between intention and realization we have an important basis for distinguishing between the problem of right and its complete
negation, such as we would see under despotic, fascistic rule. The use and abuse of right is not the same thing as a complete
absence of right, and understanding this is vital to being able to comprehend where and in what ways democratic, constitutional
States become, or are, fascistic. Natural right, or the right of the human being, occupies a space of interruption in the divide
between law and ethicality that can, on occasion, act as to reintroduce a radical pathos within right.
rights good

Rights do not create a fixed identity. When we appeal to rights in terms of universal justice they are deeply radical and able to
create open communities.

Daly, 04 (http://www.borderlandsejournal.adelaide.edu.au/vol3no1_2004/daly_noncitizen.htm,
The non-citizen and the concept of 'human rights', Frances Daly,Australian National University
2004).

Legal positivism assumes or sets out the basis for rights within a normative framework of the State that merely takes for granted
judicial postulates of the inalienability of rights, the basis of rights in property and assumptions that people are in fundamental
accord on matters of right. It is unable to imagine a realm of freedom against the State. But within rights, I would argue, we can
detect unsatisfied demands that have nothing to do with essentialist assumptions about 'man' or 'citizen'. These demands are
concerned with an understanding of human freedom in relation to values of solidarity, justice and the overcoming of alienation;
they are historical and contingent, shifting and alive, and are not about a fixed, static, generic essence of the person, or some
ahistorical or superhistorical immutable totality. What it is to be human is open and changeable, although not without
determinations, commonalities and shared properties that can emerge at various times. With the rise of individualism in the
seventeenth and eighteenth centuries, the idea of natural rights of the individual, of liberty, fraternity, and equality of the
individual – of 'inalienable' rights and normative ideals – was quite clearly conceived in terms of the citizen. What persisted of a
sense of natural justice for all, whose standard had been derived from various sources - in nature, God, a view of reason or
human nature - was undoubtedly distorted by a sense of individualism defined in terms of possession and property rights. But
this sense was not completely extinguished. It is certainly on the basis of a realm of legal positivism and its doctrines of positive
law, a realm which assumes that no element of law or right pre-exists an act of the State, that some of the basic contradictions
that Agamben highlights are likely to emerge. For it is the State that institutes types of validity for its laws on the basis of
procedure rather than any sense of morality or principles of justice. But there are other pathways to rights, other forms in which
principles of justice have been derived and enacted. And if this is the case, why must we then necessarily conclude from a
critique of legal positivism that there can be no ethical basis to rights?

Rights must be judged by their specific deployment, not abstract theory. Even if there are flaws between norms and application,
rights contain a radical element of universal dignity that can be used to leverage real change.

Daly, 04 (http://www.borderlandsejournal.adelaide.edu.au/vol3no1_2004/daly_noncitizen.htm,
The non-citizen and the concept of 'human rights', Frances Daly,Australian National University
2004).

An ahistorical disdain for legal action is merely the obverse of the process of fetishizing legality. Much theory that merely
substitutes the idea of the static essence of the person to explain the consequence of good and evil in the world with an equally
static, invariant view of authority and the State is, I would argue, ultimately eternalizing such concepts. Undoubtedly, some sort of
move beyond categories underscoring divisions within the ways people are entitled to live their lives is necessary. But much of
the power of any such critique must depend upon the manner in which the context of this life – the possible experience of acting
in the world, or 'form-of-life' - is itself understood. In the absence of any such context, what tends to emerge is a return to the
problem of rights reduced to a division of form and content, rather than the overturning of this very problematic. Only in this case,
because the content is seen to fall short of the abstraction of, for example, a "whatever singularity", the form is wholly discarded.
More importantly, by revisiting this problem via a dismissal of the context of rights, and more specifically of the possibility of
traces of the intention towards human dignity, a rich heritage of critique is sidelined. Continues... The use and abuse of right is not
the same thing as a complete absence of right, and understanding this is vital to being able to comprehend where and in what
ways democratic, constitutional States become, or are, fascistic. Natural right, or the right of the human being, occupies a space
of interruption in the divide between law and ethicality that can, on occasion, act as to reintroduce a radical pathos within right.

rights good
Rights for refugees tap into the radical core of rights—the idea that there is a universal human dignity. Past failures of rights are
not reasons to abandon the concept, they are reasons why rights must be more aggressively extended to all.

Daly, 04 (http://www.borderlandsejournal.adelaide.edu.au/vol3no1_2004/daly_noncitizen.htm,
The non-citizen and the concept of 'human rights', Frances Daly,Australian National University
2004).

Let us look then at the more specific example of the right of the refugee or right of asylum. In the 1789 Declaration of the Rights
of Man and of the Citizen there is a perceived need to set out what are described as "the natural, sacred, and inalienable rights
of man". These rights, as is well known, concern freedom, equality, the right to liberty, property, security and resistance to
oppression, the presumption of innocence, the right to opinion and religious expression and free communication. Likewise, the
1948 Universal Declaration of Human Rights restates these rights and extends an understanding of right to economic, social and
cultural rights and, perhaps most importantly from the perspective of this paper, the right to a freedom of movement and residence and the right to seek and enjoy in other countries asylum from persecution.
We have already mentioned the institutionalization of these rights in citizen rights, and that this sense of right was a creation of the nation-State. And along with this is the problematic nature of the inclusion of the
right of property as an inalienable right, which first arose as the consequence of the division of labour and has little to do with anything inherently human, and the basic difficulty that arises with a sense of innate
rights, as all rights have been acquired. We detect as well the formalism of general juridical equality with the much more normative content of the constitutional state of fundamental social division – those whose
, it would be a clear
access to education, security, work and freedom from detention can be assumed, and those whose lack of this assumption is outlined in their right to seek its guarantee. However
distortion of the struggles involved in the emergence of codified natural rights to not also mention that an essential part of a
sense of the absolute inalienability of the person was a view of individual freedom within community (the sort of idea we find
Rousseau, for instance) and the attempt to exercise limits upon the power of tyrants to curtail that freedom. That there has been
a highly variable degree of protection of these rights, or in certain cases no protection of them at all, is naturally problematic but
cannot of itself be attributed to the fact of rights themselves. The context of rights is one that is frequently unstable, and, as such,
it is important to clearly assess the place of rights within our present conditions of unfreedom. Often as a result of their denial,
human rights currently act so as to allow a questioning of the assumed authority of the State. Indeed, without a sense of rights it
would be difficult for us to understand the current absence of real freedom. If we consider the contemporary struggles of the
'Sans Papiers' in France, the several hundred thousand people whose refusal of the label 'illegal' and fight for documentation is
premised on the basis that the undermining of rights is merely a way of attacking the value of dignity for all, we can see a clear
example of the possibility that can be realized through right. The Sans Papiers are well-known for their questioning of the assumptions of immigration policies, such as the
existence of quotas, detention camps and deportations, and they argue cogently for an end to frontiers themselves. Madjiguène Cissé argues that the initiatives of those claiming their rights are basic to the survival of
communities (Cissé, 1997: 3). This is done on the basis of an appeal to rights of justice and egalitarianism. Indeed, it is not possible to understand this emancipatory struggle outside a conception of rights. 25.
Agamben views all such setting out of rights as essentially reintegrating those marginalized from citizenship into the fiction of a guaranteed community. Law only "wants to prevent and regulate" (Agamben, 2001: 1) –
– but within rights, I argue, we can also detect a potential for justice. In contrast, Agamben contends
and it is certainly the case that much law does
that legal right and the law always operate in a double apparatus of pure violence and forms of life guaranteed by a Schmittian
'state of emergency' (Agamben, 2000: 43). And although he recognizes the dire consequences of a state of emergency with the
eradication of the legal status of individuals, he views this as the force of law without law, as a mystical or fictional element, a
space devoid of law, an 'empty legal space', or 'state of exception' as Carl Schmitt refers to it, that is essential to the legal order
(Carl Schmitt, 1985: 6). What is then eliminated here is any sense of how the appeal to rights brings into question
institutionalized unfreedom and why this underlying insufficiency between the idea of right and real need is opposed by those
attempting to expand the realm of human rights. The problem with this strategy for doing away with any distinction and placing
the refugee in a position of pure potentiality is that, instead of liberating or revolutionizing the place of the refugee, it creates an
eternal present that is unable to connect the very real reality of difference with a critique of the society that victimizes the refugee
in the manner with which we are currently so familiar.
rights good

Even if the law is not perfect and culture values matter, rights still protect us from oppression.

Altman, 90 (Andrew, (Professor of Philosophy; Georgia State University) Critical Legal Studies: A Liberal Critique, page 8)

There are undoubtedly elements of the liberal tradition which exaggerate the extent to which the law alone gives contemporary
liberal societies the degree of humanity and decency they have. There are undoubtedly elements of the liberal tradition which
exaggerate the power of law to work its will against the entrenched customs and traditions of a culture. We would be wise to
keep in mind Tocqueville’s lesson about the failures of law in cultural set tings where it has tried to operate in opposition to
pervasive and deep-seated social norms. But it would be equally wrong to dismiss the protections offered by the law as
superfluous or useless. Between the area in which law is useless because it receives insufficient support from the rest of the
culture and the area in which law is superfluous because the rest of the culture provides all of the protections we can reasonably
ask for, there is a wide expanse of territory. It is within the borders of that territory that law can and does make a difference. It is
within the borders of that territory that legal rights can and do work to protect people from the evils of intolerance, prejudice, and
oppression. This is the heart of the liberal tradition in legal philosophy. It is a tradition worthy of allegiance.

Rights are the best path to liberation—if you take their alternative seriously, it would require massive coercion to create a
collective voice capable of challenging the law.

Sparer, 84 (Ed, (Law Professor, University of Pennsylvania) 36 Stan. L. Rev. 509, January).

We would do well to follow the radical approach of building upon our core human rights tradition, demonstrating the contradiction
between that tradition and our social institutions, and developing ways to fuse human rights into new cooperative institutions of
our own making. Such work requires a concern for theory which feeds social movement, but successful social movement comes
from the struggle for the realization of our basic rights, not from their disparagement. One must step outside the liberal paradigm into a realm where truth may
be experiential, where knowledge resides in world views that are themselves situated in history, where power and ideas do not exist separately. Continues... Central to the argument I have made thus far is the notion
that individual autonomy and community are not contradictions at all; rather, they shape and give meaning and richness to each other. Kennedy and other Critical legal theorists of the dominant school recognize the
latter thought. At the same time, they argue that the very interdependence of these concepts leads to the fundamental and seemingly unresolvable contradiction they embody. In an oft-quoted passage, Kennedy
states: Even when we seem to ourselves to be most alone, others are with us, incorporated in us through processes of language, cognition and feeling that are, simply as a matter of biology, collective aspects of our
individuality. Moreover, we are not always alone. We sometimes experience fusion with others, in groups of two or even two million, and it is a good rather than a bad experience. But at the same time that it forms
and protects us, the universe of others (family, friendship, bureaucracy, culture, the state) threatens us with annihilation and urges upon us forms of fusion that are quite plainly bad rather than good. A friend can
reduce me to misery with a single look. Numberless conformities, large and small abandonments of self to others, are the price of what freedom we experience in society. And the price is a high one. Through our
existence as members of collectives, we impose on others and have imposed on us hierarchical structures of power, welfare, and access to enlightenment that are illegitimate, whether based on birth into a particular
. The kicker is that the abolition of these illegitimate structures, the fashioning of an unalienated
social class or on the accident of genetic endowment
collective existence, appears to imply such a massive increase of collective control over our lives that it would defeat its purpose.
Only collective force seems capable of destroying the attitudes and institutions that collective force has itself imposed. Coercion
of the individual by the group appears to be inextricably bound up with the liberation of that same individual. If one accepts that
collective norms weigh so heavily in favor of the status quo that purely "voluntary" movement is inconceivable, then the only
alternative is the assumption of responsibility for the totalitarian domination of other people's minds -- for "forcing them to be
free."
rights good: not monolithic

Human rights are not a monolith—they can be adapted by local cultures.


Ibhawoh, 00 – Lecturer in African History and International Development Studies at the Edo State University in Nigeria – 2000
(Bonny Ibhawoh, “Between Culture and Constitution: Evaluation the Cultural Legitimacy of Human Rights in the Africa State”,
human rights quarterly 2.2, Project Muse).

This assumption tends to ignore the fact that societies are constantly in the process of change wrought by a variety of cultural,
social, and economic forces. It seems an elementary but necessary point to make that so-called traditional societies--whether in
Asia, Africa, or in Europe--were not culturally static but were eclectic, dynamic, and subject to significant alteration over time.
Traditional cultural beliefs are also neither monolithic nor unchanging. In fact they could--and were--changed in response to
different internal and external pressures. Cultural change can result from individuals being exposed to and adopting new ideas.
Individuals are actors who can influence their own fate, even if their range of choice is circumscribed by the prevalent social
structure or culture. In doing so, those who choose to adopt new ideas, though influenced by their own interest, initiate a process
of change which may influence dominant cultural traditions. Culture is thus inherently responsive to conflict between individuals
and social groups. 7 It is a network of perspectives in which different groups hold different values and world views, and in which
some groups have more power to present their versions as the true culture. The significance of this is that we proceed from the
assumption that certain cultural traditions inherently appearing in conflict with national and universal human rights standards may
in fact have the potential of being influenced through a process of change and adaptation to meet new human rights standards.

Human rights concepts are universal but their implementation varies widely.
Donnelly, 07 – Andrew Mellon Professor at the Graduate School of International Studies, University of Denver – 2007 (Jack
Donnelly, “The Relative Universality of Human Rights”, Human Rights Quarterly 29 page 281-306, Project Muse).

Human rights are (relatively) universal at the level of the concept, broad formulations such as the claims in Articles 3 and 22 of
the Universal Declaration that "everyone has the right to life, liberty and security of person" and "the right to social security." 50
Particular rights concepts, however, have multiple defensible conceptions. Any particular conception, in turn, will have many
defensible implementations. At this level—for example, the design of electoral systems to implement the right "to take part in the
government of his country, directly or through freely chosen representatives"—relativity is not merely defensible but desirable. 51
Functional and overlapping consensus universality lie primarily at the level of concepts. Most of the Universal Declaration lies at
this level as well. Although international human rights treaties often embody particular conceptions, and sometimes even
particular forms of implementation,52 they too permit a wide range of particular practices. Substantial second order variation, by
country, region, culture, or other grouping, is completely consistent with international legal and overlapping consensus
universality.

Rights enable cultural expression.


Donnelly, 07 – Andrew Mellon Professor at the Graduate School of International Studies, University of Denver – 2007 (Jack
Donnelly, “The Relative Universality of Human Rights”, Human Rights Quarterly 29 page 281-306, Project Muse).

Human rights seek to allow human beings, individually and in groups that give meaning and value to their lives, to pursue their
own visions of the good life. Such choices—so long as they are consistent with comparable rights for others and reflect a
plausible vision of human flourishing to which we can imagine a free people freely assenting—deserve our respect. In fact,
understanding human rights as a political conception of justice supported by an overlapping consensus requires us to allow
human beings, individually and collectively, considerable space to shape (relatively) universal rights to their particular purposes
—so long as they operate largely within the constraints at the level of concepts established by functional, international legal, and
overlapping consensus universality.
rights good: not western

Human rights are not inherently Western.

Donnelly, 07 – Andrew Mellon Professor at the Graduate School of International Studies, University of Denver – 2007 (Jack
Donnelly, “The Relative Universality of Human Rights”, Human Rights Quarterly 29 page 281-306, Project Muse).

The social-structural "modernity" of these ideas and practices, however, not their cultural "Westernness," deserves emphasis. 15
Human rights ideas and practices arose not from any deep Western cultural roots but from the social, economic, and political
transformations of modernity. They thus have relevance wherever those transformations have occurred, irrespective of the pre-
existing culture of the place.

Human rights not exclusive to Western countries—it si essentialist to imply that other cultures inherently oppose rights.

Donnelly, 07 – Andrew Mellon Professor at the Graduate School of International Studies, University of Denver – 2007 (Jack
Donnelly, “The Relative Universality of Human Rights”, Human Rights Quarterly 29 page 281-306, Project Muse).

It is important to remember that virtually all Western religious and philosophical doctrines through most of their history have
either rejected or ignored human rights Today, however, most adherents of most Western comprehensive doctrines endorse
human rights. And if the medieval Christian world of crusades, serfdom, and hereditary aristocracy could become today's world
of liberal and social democratic welfare states, it is hard to think of a place where a similar transformation is inconceivable.
Consider claims that "Asian values" are incompatible with internationally recognized human rights. 24. Asian values—like Western
values, African values, and most other sets of values—can be, and have been, understood as incompatible with human rights.
But they also can be and have been interpreted to support human rights, as they regularly are today in Japan, Taiwan, and
South Korea. And political developments in a growing number of Asian countries suggest that ordinary people and even
governments are increasingly viewing human rights as a contemporary political expression of their deepest ethical, cultural, and
political values and aspirations.25 No culture or comprehensive doctrine is "by nature," or in any given or fixed way, either
compatible or incompatible with human rights.
rights good: check on statism

Rights are the best model for protecting people against modern states and markets—no viable alternative has worked as well.

Donnelly, 07 – Andrew Mellon Professor at the Graduate School of International Studies, University of Denver – 2007 (Jack
Donnelly, “The Relative Universality of Human Rights”, Human Rights Quarterly 29 page 281-306, Project Muse).

The spread of modern markets and states has globalized the same threats to human dignity initially experienced in Europe.
Human rights represent the most effective response yet devised to a wide range of standard threats to human dignity that market
economies and bureaucratic states have made nearly universal across the globe. Human rights today remain the only proven
effective means to assure human dignity in societies dominated by markets and states. Although historically contingent and
relative, this functional universality fully merits the label universal—for us, today. Arguments that another state, society, or
culture has developed plausible and effective alternative mechanisms for protecting or realizing human dignity in the
contemporary world deserve serious attention. Today, however, such claims, when not advanced by repressive elites and their
supporters, usually refer to an allegedly possible world that no one yet has had the good fortune to experience. The functional
universality of human rights depends on human rights providing attractive remedies for some of the most pressing systemic
threats to human dignity. Human rights today do precisely that for a growing number of people of all cultures in all regions.
Whatever our other problems, we all must deal with market economies and bureaucratic states. Whatever our other religious,
moral, legal, and political resources, we all need equal and inalienable universal human rights to protect us from those threats.
***Capitalism***

A2: Capitalism

Total rejection of capitalism fragments resistance --– the perm solves best
Gibson-Graham ‘96 (J.K., Feminist Economists – The End of Capitalism)

One of our goals as Marxists has been to produce a knowledge of capitalism. Yet as “that which is known,” Capitalism
has become the intimate enemy. We have uncloaked the ideologically-clothed, obscure
monster, but we have installed a naked and visible monster in its place. In return for our
labors of creation, the monster has robbed us of all force. We hear – and find it easy to believe – that the
left is in disarray. Part of what produces the disarray of the left is the vision of what the left is arrayed against. When
capitalism is represented as a unified system coextensive with the nation or even the world, when it
is portrayed as crowding out all other economic forms, when it is allowed to define entire
societies, it becomes something that can only be defeated and replaced by a mass collective
movement (or by a process of systemic dissolution that such a movement might assist). The revolutionary task of
replacing capitalism now seems outmoded and unrealistic, yet we do not seem to have an
alternative conception of class transformation to take its place. The old political economic “systems” and
“structures” that call forth a vision of revolution as systemic replacement still seem to be dominant in the Marxist political
imagination. The New World Order is often represented as political fragmentation founded upon economic unification. In
this vision the economy appears as the last stronghold of unity and singularity in a world of diversity and plurality. But why
can’t the economy be fragmented too? If we theorized it as fragmented in the United States, we could being to see a huge
state sector (incorporating a variety of forms of appropriation of surplus labor), a very large sector of self-employed and
family-based producers (most noncapitalist), a huge household sector (again, quite various in terms of forms of exploitation,
with some households moving towards communal or collective appropriation and others operating in a traditional mode in
which one adult appropriates surplus labor from another). None of these things is easy to see. If capitalism takes up
the available social space, there’s no room for anything else. If capitalism cannot coexist,
there’s no possibility of anything else. If capitalism functions as a unity, it cannot be partially
or locally replaced. My intent is to help create the discursive conception under which
socialist or other noncapitalist construction becomes “realistic” present activity rather than a
ludicrous or utopian goal. To achieve this I must smash Capitalism and see it in a thousand pieces.
I must make its unity a fantasy, visible as a denial of diversity and change. Perm solves –- only using capitalism to
fight capitalism can be effective Monthly Review ‘90 (March, Vol. 41, No. 10, p. 38) No institution
is or ever has been a seamless monolith. Although the inherent mechanism of American
capitalism is as you describe it, oriented solely to profit without regard to social consequences, this
does not preclude significant portions of that very system from joining forces with the
worldwide effort for the salvation of civilization, perhaps even to the extent of furnishing the
margin of success for that very effort.
A2: Capitalism

Cap solves war


Bandow 2005 fellow @ cato 2k5 (Doug, “Spreading Capitalism Is Good for Peace” http://www.cato.org/pub_display.php?
pub_id=5193)

In a world that seems constantly aflame, one naturally asks: What causes peace? Many people, including U.S.
President George W. Bush, hope that spreading democracy will discourage war. But new research suggests that
expanding free markets is a far more important factor , leading to what Columbia University's Erik Gartzke calls a "capitalist
peace." It's a reason for even the left to support free markets. The capitalist peace theory isn't new: Montesquieu and Adam Smith believed in
it. Many of Britain's classical liberals, such as Richard Cobden, pushed free markets while opposing imperialism. But World War I
demonstrated that increased trade was not enough. The prospect of economic ruin did not prevent rampant nationalism, ethnic hatred, and
security fears from trumping the power of markets. An even greater conflict followed a generation later. Thankfully, World War II left war
essentially unthinkable among leading industrialized - and democratic - states. Support grew for the argument, going back to Immanual Kant,
that republics are less warlike than other systems. Today's corollary is that creating democracies out of dictatorships will reduce conflict.
This contention animated some support outside as well as inside the United States for the invasion of Iraq. But Gartzke argues that "the
'democratic peace' is a mirage created by the overlap between economic and political freedom." That is,
democracies typically have freer economies than do authoritarian states. Thus, while "democracy is desirable for
many reasons," he notes in a chapter in the latest volume of Economic Freedom in the World, created by the
Fraser Institute, "representative governments are unlikely to contribute directly to international peace."
Capitalism is by far the more important factor. The shift from statist mercantilism to high-tech capitalism has
transformed the economics behind war. Markets generate economic opportunities that make war less desirable.
Territorial aggrandizement no longer provides the best path to riches. Free-flowing capital markets and other
aspects of globalization simultaneously draw nations together and raise the economic price of military conflict.
Moreover, sanctions, which interfere with economic prosperity, provides a coercive step short of war to achieve
foreign policy ends. Positive economic trends are not enough to prevent war, but then, neither is democracy. It
long has been obvious that democracies are willing to fight, just usually not each other. Contends Gartzke, "liberal political
systems, in and of themselves, have no impact on whether states fight." In particular, poorer democracies perform like non-
democracies. He explains: "Democracy does not have a measurable impact, while nations with very low levels of
economic freedom are 14 times more prone to conflict than those with very high levels." Gartzke considers other
variables, including alliance memberships, nuclear deterrence, and regional differences. Although the causes of conflict vary, the relationship
between economic liberty and peace remains. His conclusion hasn't gone unchallenged. Author R.J. Rummel, an avid proponent of the
democratic peace theory, challenges Gartzke's methodology and worries that it "may well lead intelligent and policy-wise analysts and
commentators to draw the wrong conclusions about the importance of democratization." Gartzke responds in detail, noting that he
relied on the same data as most democratic peace theorists. If it is true that democratic states don't go to war,
then it also is true that "states with advanced free market economies never go to war with each other, either." The
point is not that democracy is valueless. Free political systems naturally entail free elections and are more likely to protect other forms of
liberty - civil and economic, for instance. However, democracy alone doesn't yield peace. To believe is does is dangerous:
There's no panacea for creating a conflict-free world. Capitalism stops poverty Stephens Ph.D. candidate 2k (Patrick, “The
morality of capitalism” http://www.objectivistcenter.org/cth--225-The_Morality_Capitalism.aspx) The beginning of the twenty-first
century is a great time for capitalism. Socialism has been discredited. Countries around the world are opening
their markets and removing barriers to trade. America has experienced the longest period of growth that the
world has ever seen and produced an explosion of technology that promises to reshape social structures, increase
freedom, cure disease, and extend the human life-span. It truly is a wonderful time to be alive. But despite the
prosperity that capitalism has brought to America and the West, it still suffers from an image problem. The old
Left-wing critiques are fading; Marxist arguments are rare and social experiments in rent-control, welfare
benefits, and the public ownership of capital are being abandoned. But in their place, a different critique of
capitalism is catching hold. This new critique--which, as it turns out, is not so new--does not challenge the
effectiveness of capitalism. Capitalism, it acknowledges, is better than any other system at creating wealth,
eradicating poverty, and developing technology. The new critique is aimed instead at the morality of capitalism;
it asks if wealth, mass affluence, and technology are really such good things after all.
A2: Capitalism

Non-capitalist societies destroy the environment


Dominick 98 Emeritus Professor humanities @ OSU 1998 (Raymond, “Capitalism, communism, and environmental protection:
Lessons from the German experience” http://findarticles.com/p/articles/mi_qa3854/is_199807/ai_n8795240/pg_1

At no time since its inception two hundred years ago has the ideology of free market capitalism stood more
dominant than it does today. For much of the nineteenth and twentieth centuries, communists confidently
challenged the advocates of laissez-faire, claiming that their system could produce more wealth than capitalism
and distribute it more equitably. In the process, they boasted that communism could cure a broad range of social
problems, including environmental pollution.L Following the worldwide collapse of communism,
almost all these claims proved to be false, none more so than the promise to protect the
environment. After the Iron Curtain crumbled and uncensored reporting became possible, academics and
the popular press rushed to document the massive environmental devastation in the Soviet
zone.2 The West German magazine Der Spiegel indignantly branded communist East Germany as an
"ecological outlaw of the first rank," noting, for example, that the Buna chemical works in the East dumped ten
times more mercury into its neighboring river in a day than a comparable West German plant did in a year. The
same article also reported that each of the two-cycle cars commonly operated in the East emitted
one hundred times as much carbon monoxide as a western auto equipped with a catalytic
converter. Elaborating on the air pollution problem, an article in Current History pointed out that East German
sulphur dioxide emissions per capita were the highest in the world; the burden of that particular pollutant
exceeded the corresponding figure for capitalist West Germany by a factor of twelve. Reflecting on these
and other environmental contrasts in the summer of i99o, as East and West Germany moved
toward unification, the New York Times reported that "one issue taking on urgency is how the
orderly and clean half of the country can help clean up the disheveled and polluted half....
Quick action is needed because four decades of unbridled industrial spewing and spilling in East Germany have
created an acute crisis for man and nature."3 Some commentators used the appalling evidence from the region
east of the Iron Curtain to argue that the fundamental economic principles of communism
predictably and inevitably produce environmental disaster. A Polish economist observed that "in
Marxist ideology, natural resources are free and have no intrinsic value . . . Their sole purpose
is to serve, not to constrain, humans." A West German analyst seconded this view, writing that the
"socialist labor theory of value inevitably led to serious, almost universal environmental and
health dangers." An American observer interpreted these environmental failings as the result
of the absence of capitalism: "Absent a profit motive, energy, materials, and natural resources
could be squandered without care. And they were."4 Such an implication-that capitalist economic
principles can cure the environmental crises caused by communism-dovetails perfectly with
the current zeitgeist, but it is highly questionable from an historical point of view.
A2: Zizek

Zizek justifies violence of the state through terror as a means to impose good terror
ROBINSON & TORMEY 2K5 (Andrew and Simon, “Zizek is not a radical,” THESIS ELEVEN, N80,
FEBRUARY, http://homepage.ntlworld.com/simon.tormey/articles/Zizeknotradical.pdf pg 20)

What Zizek is telling left radicals, therefore, is to abandon the notion of the state as a source of
violence and to see it as part of the solution to, rather than the problem of, reordering social life.
Zizek sees the state as a useful ally, and an instrument through which to impose the good terror.
He denounces anti-statism as idealist and hypocritical,129 and attacks the anticapitalist
movement for lacking political centralisation.130 Zizek does not offer an alternative to statist
violence; in Zizek’s world (to misquote an anarchist slogan), ‘whoever you fight for, the state
always wins’. Opponents of the war in Afghanistan and the arms trade, of police racism and
repression against demonstrators, will find no alternative in Zizek - only a new militarism, a
‘good terror’ and yet another Cheka.

Only the permutation solves. The plan creates the spring board for resistance which is a
prerequisite to solving without it the alternative would fail
ROBINSON & TORMEY 2K5 (Andrew and Simon, “Zizek is not a radical,” THESIS ELEVEN, N80,
FEBRUARY, http://homepage.ntlworld.com/simon.tormey/articles/Zizeknotradical.pdf pg 23-24)

Zizek is right to advocate a transformative stance, but wrong to posit this as a radical break constituted ex nihilo. Far
from being the disavowed supplement of capitalism, the space for thinking the not-real which is opened by
imaginaries and petty resistances is a prerequisite to building a more active resistance and ultimately, a substantial
social transformation. In practice, political revolutions emerge through the radicalisation of existing demands and
resistances – not as pure Acts occurring out of nothing. Even when they are incomprehensible from the standpoint of
‘normal’, conformist bystanders, they are a product of the development of subterranean resistances and counter
hegemonies among subaltern groups. As Jim Scott argues, when discontent among the subaltern strata generates
‘moments of madness’, insurrections and revolutions, it does so as an extension of, and in continuity with, existing
‘hidden transcripts’, dissenting imaginaries and petty resistances. As Scott’s evidence shows, resistance ‘requires an
experimental spirit and a capacity to test and exploit all the loopholes, ambiguities, silences and lapses available...
[and] setting a course for the very perimeter of what the authorities are obliged to permit or unable to prevent’. 144
Such petty resistance can pass over into more general insurrections. When prisoners at a Stalinist camp, expected to
deliberately lose a race against their guards, ‘spoiled the performance’ with a ‘pantomime of excess effort’, a ‘small
political victory had real political consequences’, producing a ‘flurry of activity’. 145 Filipino peasant uprisings often
acted out an ideology developed through a subverted version of passion plays,146 and European carnivals often
passed over into insurrection.147 Social change does not come from nothing; it requires the pre-existence of a
counter-culture involving nonconformist ideas and practices. ‘You have to know how the world isn’t in order to
change it’.148 As Gramsci puts it, before coming into existence a new society must be ‘ideally active’ in the minds of
those struggling for change.149
A2: Zizek

The Act’s form of active nihilism ultimately fails and isn’t sociall effective as it gets
repressed, allows for violence and doesn’t propose methods of solving in modes of isolation
ROBINSON & TORMEY 2K5 (Andrew and Simon, “Zizek is not a radical,” THESIS ELEVEN, N80,
FEBRUARY, http://homepage.ntlworld.com/simon.tormey/articles/Zizeknotradical.pdf pg 26)

The history of resistance gives little reason to support Zizek’s politics of the Act. The ability to Act in the manner
described by Zizek is largely absent from the subaltern strata. Mary Kay Letourneau (let us recall) did not transform
society; rather, her ‘Act’ was repressed and she was jailed. In another case discussed by Zizek, a group of Siberian
miners is said to accomplish an Act - by getting massacred.150 Since Acts are not socially effective, they cannot help
the worst-off, let alone transform society. Zizek’s assumption of the effectiveness of Acts rests on a confusion
between individual and social levels of analysis. Vaneigem eerily foresees Zizek’s ‘Act’ when he argues against
‘active nihilism’. ‘In a gloomy bar where everyone is bored to death, a drunken young man breaks his glass, then
picks up a bottle and smashes it against the wall. Nobody gets excited; the disappointed young man lets himself be
thrown out... Nobody responded to the sign which he thought was explicit. He remained alone, like the hooligan
who burns down a church or kills a policeman, at one with himself, but condemned to exile for as long as other
people remain exiled from their existence. He has not escaped from the magnetic field of isolation; he is suspended
in a zone of zero gravity’.151 The transition from this ‘wasteland of the suicide and the solitary killer’ to
revolutionary politics requires the repetition of negation in a different register, 152 connected to a positive project to
change the world and relying on the imaginaries Zizek denounces, the carnival spirit and the ability to dream. 153

Zizek’s advocacy of violence justified actions like Nazism and Soviet terror
Robinson and Tormey, 2004 (Andrew and Simon, activist and doctoral student in the School of Politics, University of
Nottingham, teaches in the schools of Politics and Critical Theory at the University of Nottingham. Thesis Eleven, University of
Nottingham, “Zizek is not radical,” http://homepage.ntlworld.com/simon.tormey/articles/Zizeknotradical.pdf, February pg 18-20)

Secondly, Zizek implies that Lenin must in some sense have ‘understood’ that the revolution would necessarily betray itself, and that all
revolutions are structurally doomed to fall short of whatever ideals and principles motivate them. He also implies that the success or failure of a
revolution has nothing to do with whether the modes of thought and action, social relations and institutions which follow are at all related to the
original revolutionary ideals and principles. What matters is that power is held by those who ‘identify with the symptom’, who call themselves
‘Proletarian’. Zizek therefore endorses the conservative claim that Lenin’s utopian moments were Machiavellian manoeuvres or at best confused
delusions, veiling his true intentions to seize power for himself or a small elite: Lenin was the ‘ultimate political strategist’.121 That Zizek
endorses the ‘Lenin’ figure despite endorsing nearly every accusation against Lenin serves to underline the degree to
which Zizek’s politics are wedded to conservative assumptions that repression, brutality and terror are ‘always with
us’. Rejecting the claim that politics could be otherwise, Zizek wishes to grasp, embrace and even revel in the
grubbiness and violence of modern politics. The moment of utopia in Russia was for Zizek realised when the Red
Guards succumbed to a destructive hedonism in moments of Bataillean excess. 122 The only difference for Zizek
between leftist ethics and the standpoint of Oliver North, the Taleban, the anti-Dreyfusards and even the Nazis is
that such ‘rightists’ legitimate their acts in reference to some higher good, whereas leftists also suspend the higher
good in a truly authentic gesture of suspension.123 The Soviet Terror is a good terror whereas the Nazi one is not,
only because the Soviet terror was allegedly more total, with everyone being potentially at risk, not only out-
groups.124 Zizek goes well beyond advocating violence as a means to an end; for Zizek, violence is part of the end
itself, the utopian excess of the Act. The closest parallel is the nihilism of Nechaev’s Catechism of a Revolution
which proclaims that ‘everything is moral that contributes to the triumph of the revolution; everything that hinders it
is immoral and criminal’.125 As Peter Marshall comments in his digest of anarchist writings and movements, the
Catechism is ‘one of the most repulsive documents in the history of terrorism’. One can only speculate what he
would have made of ‘Repeating Lenin’.126
A2: Zizek

The alternative leaves social exclusion, violence and fails at subversing capitalism
Robinson and Tormey, 2004 (Andrew and Simon, activist and doctoral student in the School of Politics, University of
Nottingham, teaches in the schools of Politics and Critical Theory at the University of Nottingham. Thesis Eleven, University of
Nottingham, “Zizek is not radical,” http://homepage.ntlworld.com/simon.tormey/articles/Zizeknotradical.pdf, pg 9)

The choice of the term ‘suspension’ is revealing, for although in Zizek’s account the surface structure of the social
system is changed during such a ‘suspension’, the deep structure of the social system as set out in Lacanian theory is
not (and cannot be) changed in the slightest. So an Act shatters capitalism, but it leaves intact many of
its most objectionable features, including social exclusion,56 violence,57 naturalisation,58
reification and myths,59 all of which are for Zizek primordial, ever-present and necessary in any
society. Further, since the Act involves submission to a Cause and a Leader, it cannot destroy the
authoritarian structure of capitalism: ‘often, one does need a leader in order to be able to “do the
impossible”... subordination to [the leader] is the highest act of freedom’. 60 So, while an Act may destroy the
specific articulations of oppression within the present system (e.g. the identification of the Real with
illegal immigrants), it necessarily produces a system which is equally oppressive.
A2: Zizek

Zizek's lack of an alternative and general ambiguity renders his anti-capitalist rhetoric
meaningless
Ernesto Laclau, Professor of Political Theory at the University of Essex, 2000, “Contingency, Hegemony,
Universality: Contemporary Dialogues on the Left”

Zizek takes a patently anti-capitalist stance, and asserts that the proponents of postmodernism 'as a rule, leave out
of sight the resignation at its heart- the acceptance of capitalism as “the only game in town”, the renunciation of any real attempt
to overcome the existing capitalist liberal regime' (SZ, p. 95). The difficulty with assertions like this is that they
mean absolutely nothing. I understand what Marx meant by overcoming the capitalist regime,
because he made it quite explicit several times. I also understand what Lenin or Trotsky meant
for the same reason. But in the work of Zizek that expression means nothing- unless he has a
secret strategic plan of which he is very careful not to inform anybody. Should we understand that
he wants to impose the dictatorship of the proletariat? Or does he want to socialize the means of production and abolish market
mechanisms? And what is his political strategy to achieve these rather peculiar aims? What is the
alternative model of society that he is postulating? Without at least the beginning of an answer
to these questions, his anti-capitalism is merely empty talk.

Zizek’s alternative is a prescription for political sterility


Ernesto Laclau, Professor of Political Theory at the University of Essex, 2000, “Contingency, Hegemony,
Universality: Contemporary Dialogues on the Left”
The imagery around the base/superstructure metaphor decisively shapes Zizek's vision of political
alternatives. Thus he distinguishes between struggles to change the system and struggles within
the system. I do not think that this distinction, posed in those terms, is a valid one. The crucial question is:
how systematic is the system? If we conceive this systematical as the result of endogenous laws
of development- as in the case of the retroactive reversal of contingency into necessity - the only alternatives are
either that those laws lead, through their operation, to the self-destruction of the system(let us
remember the debate in the Second International, on the mechanic collapse of the system) or to the system's destruction
from outside. If, on the contrary, systematicity is seen as a hegemonic construction, historical change
is conceivable as a displacement in the relations between elements- some internal and some
external to what the system had been. Questions such as the following may be asked: How is it possible to
maintain a market economy which is compatible with a high degree of social control of the
productive progress? What restructuralition of the liberal democratic institutions is necessary so
that democratic control becomes effective, and does not degenerate into regulation by an all-
powerful bureaucracy? How should democratization be conceived so that it makes possible global political effects which
are, however, compatible with the social and cultural pluralism existing in a given society? These questions are thinkable within
the Gramscian strategy of a war on position, while in Zizek's suggestion of a direct struggle for overthrowing
capitalism and abolishing liberal democracy, I can see only a prescription for political
quietism and sterility.
A2: Zizek

Universality is a joke Ernesto


Laclau, Professor of Political Theory at the University of Essex, 2000, “Contingency, Hegemony, Universality:
Contemporary Dialogues on the Left”

Zizek thinks that the degree of globality or universality of a struggle depends on its location in
the social structure: some struggles, conceived as 'class struggle'- those of the workers, especially- would
spontaneously and tendentially be more 'universal' in their effects because they take place at the
'root' of the capitalist system; while others, more 'cultural' in their aims- such as multiculturalist ones-
would be more prone to particularism and, as a result, easier to integrate into the present system
of domination. For me this is a spurious distinction. There is no struggle which has inscribed in
itself the guarantee of being the privileged locus of universalistic political effects. Workers' demands-
higher wages, shorter working hours, better conditions in the workplace, and so on- can, given the appropriate circumstances, be
as easily integrated into the system as those of any other group. Conversely, given the globalization of
capitalism, dislocations could take place which are the basis of anti-systemic movements led by
groups who are not directly part of the capitalist relations of production. So while for Zizek the
distinction between 'class struggle' and what he calls 'postmodernism' is fundamental, I tend to blur it.
A2: Zizek

Zizek’s Alt is Oppressive/Violent 1/4


Zizek’s alternative necessarily reproduces violence and oppression. It takes these things as
a given, and merely changes they ways in which violence is expressed.
Robinson and Tormey 05 (Andrew Robinson and Simon Tormey, professors of the school of Politics at
University of Nottingham, “A Ticklish Subject?Zizek and the Future of Left Radicalism,” Thesis Eleven, 8
November 2005, pg. 2)
The Act resolves all problems in a single, all-encompassing Terror which bypasses particularities and
violently stops the ‘mad dance’ of shifting identities, operating instead to ground a new political universality by opting
for the impossible, with no taboos, no a priori norms . . . respect for which would prevent us from ‘resignifying’ terror. (Butler et al., 2000: 326)
An Act is symbolic death, creatio ex nihilo, and self-grounded.17 It is the outcome of ‘an ethics grounded in
reference to the traumatic Real which resists symbolisation’, i.e. to ‘an injunction which cannot be grounded in
ontology’ ( Zizek, 1997a: 213–14), a ‘selfreferential abyss’ ( Zizek, 1997a: 223), an excessive gesture irreducible to
human considerations and necessarily arbitrary (as in Zizek, 2000: 155; 1999: 96). The suspension of ethical,
epistemological and political standards is thus not merely a necessary consequence of a Zizekian Act – it is a
defining feature. Such a suspension is necessary so a new system can be built from nothing, and anything
short of a full Act remains on enemy terrain (see also, respectively, Zizek, 2000: 155; Butler et al., 2000: 126). The choice of the term
the
‘suspension’ is revealing, for although in Zizek’s account the surface structure of the social system is changed during such a ‘suspension’,
deep structure of the social system as set out in Lacanian theory is not (and cannot be) changed,
altered or reformed. So an Act shatters capitalism, but it leaves intact many of its most objectionable
features, including social exclusion, violence, naturalization, reification and myths, all of which are for
Zizek primordial, ever-present and necessary in any society.18 Further, since the Act involves submission to
a Cause and a Leader, it cannot destroy the authoritarian structure of capitalism: ‘often, one does need
a leader in order to be able to “do the impossible”. . . subordination to [the leader] is the highest act of freedom’
( Zizek, 2001b: 246–7). So, while an Act may destroy the specific articulations of oppression within the
present system (e.g. the identification of the Real with illegal immigrants), it necessarily produces a system
which is equally oppressive. To succeed, their alternative requires an authoritarian leader capable
of engaging in horrible violence in order to bring about change.

Robinson and Tormey 05 (Andrew Robinson and Simon Tormey, professors of the school of Politics at
University of Nottingham, “A Ticklish Subject?Zizek and the Future of Left Radicalism,” Thesis Eleven, 8
November 2005, pg. 2)
Furthermore, despite Zizek’s emphasis on politics, his discussion of the Act remains resolutely
individualist – as befits its clinical origins. Zizek’s examples of Acts are nearly all isolated actions by
individuals, such as Mary Kay Letourneau’s defiance of juridical pressure to end a relationship with a youth, a
soldier in Full Metal Jacket killing his drill sergeant and himself, and the acts of Stalinist bureaucrats who
rewrote history knowing they would later be purged ( Zizek, 1997a: 21; 1999: 385–7; 2001b: 98–9). Even the Russian Revolution becomes for
Zizek a set of individual choices by Lenin, Stalin and the aforementioned bureaucrats, as opposed to the culmination of mass actions involving
This is problematic as a basis for understanding previous social
thousands of ordinary men and women.
transformations, and even more so as a recommendation for the future. The new subject Zizek envisages is
an authoritarian leader, someone capable of the ‘inherently terroristic’ action of ‘redefining the
rules of the game’ ( Zizek, 1999: 377). We would argue that this is a conservative, if not reactionary,
position. Donald Rooum’s cartoon character Wildcat surely grasps the essence of left radical ambition rather better when he states, ‘I don’t
just want freedom from the capitalists. I also want freedom from people fit to take over’ (Rooum, 1991: 24).
A2: Zizek

Zizek’s Alt is Oppressive/Violent 2/4


Zizek’s politics subordinate everything to rejecting the status quo, and result in
authoritarianism and human rights abuses
Robinson and Tormey 05 (Andrew Robinson and Simon Tormey, professors of the school of Politics at
University of Nottingham, “A Ticklish Subject?Zizek and the Future of Left Radicalism,” Thesis Eleven, 8
November 2005, pg. 2)
Yet it is still the case that Zizek mercilessly rejects the present state of the world. On the one hand, he is very aware
of problems of great significance for the left: the privatization of everything from telecommunications to genes, the
invisible exploitation of workers in sweatshops, the growing ecological crisis, and the weight of the forces lined up
to make these attacks, and the crisis they generate, seem ‘normal’.12 And yet, on the other hand, he launches
conservative-sounding attacks on liberalism and reflexivity ( Zizek, 1999: 358; Zizek, 2000: 9); the lack of a
Master ( Zizek, 1997a: 151–3, 164; 1999: 358; 2001b: 246–7); and campaigns against sexual violence
( Zizek, 1999: 285; 2000: 72, 111). He also rails against ‘permissiveness’ and ‘decadence’ and calls for a
conformist ‘normal mature subject’ prepared to submit to authority on trust and to identify
authentically with social roles ( Zizek, 1997a: 148, 193; 1999: 369, 399; 2000: 110–11, 133– 5). Though it is far
from clear that the changes he demands are unproblematically progressive, he clearly wants a comprehensive
transformation. Indeed, he dismisses others’ concerns for human rights, moderation and toleration as mere
‘humanist hysterical shirking of the act’ and announces that he doesn’t care if ‘bleeding-heart liberals’ accuse him of
‘linksfaschismus’ (Butler et al., 2000: 326; Zizek, 1999: 380). Zizek’s position thus sacrifices everything to
a core orientation. Yet the question remains, how can he reconcile such a stance with the impossibility of
imagining a radical alternative?
A2: Zizek

Zizek’s theory encourages violence as part of antagonism constitutive of humanity – it


should be rejected on the notion that it would allow for unspeakable atrocities.
Robinson and Tormey 03, (Andrew and Simon, Professor of Politics at Nottingham University and Professor
of Politics at Nottingham University, 2003 “Zizek is not a Radical,”
http://homepage.ntlworld.com/simon.tormey/articles/Zizeknotradical.pdf)
As becomes evident ‘class struggle’ is not for Zizek an empirical referent and even less a category of
Marxisant sociological analysis, but a synonym for the Lacanian Real. A progressive endorsement of ‘class
struggle’ means positing the lack of a common horizon and assuming or asserting the insolubility of political
conflict.16 It therefore involves a glorification of conflict, antagonism, terror and a militaristic
logic of carving the field into good and bad sides, as a good in itself.17 Zizek celebrates war
because it ‘undermines the complacency of our daily routine’ by introducing ‘meaningless
sacrifice and destruction’.18 He fears being trapped by a suffocating social peace or Good and so
calls on people to take a ‘militant, divisive position’ of ‘assertion of the Truth that enthuses them’.19 The content of this
Truth is a secondary issue. For Zizek, Truth has nothing to do with truth-claims and the field of ‘knowledge’. Truth is an event which ‘just
happens’, in which ‘the thing itself’ is ‘disclosed to us as what it is’.20 Truth is therefore the exaggeration which distorts any balanced system.21
A ‘truth-effect’ occurs whenever a work produces a strong emotional reaction, and it need not be
identified with empirical accuracy: lies and distortions can have a truth-effect, and factual truth can cover the
disavowal of desire and the Real.22 In this sense, therefore, Lenin and de Gaulle, St Paul and Lacan are all carriers
of the truth and therefore are progressive, ‘radical’ figures, despite the incompatibility of their doctrines. Such
individuals (and it is always individuals) violently carve the field and produce a truth-effect. That
de Gaulle and the Church are political rightists is of no importance to Zizek, since he redefines ‘right’ and ‘left’ to avoid such problems . He
also writes off the human suffering caused by carving the field as justified or even beneficial: it
has a ‘transcendental genesis’ in the subject, and its victims endure it because they obtain
jouissance from it.23 The structural occurrence of a truth-event is what matters to him - not what
kind of world results from it. This is a secondary issue - and anyway one that he thinks is impossible to discuss, since the logic of
liberal capitalism is so total that it makes alternatives unthinkable.24 One should keep the utopian possibility of alternatives open, but it should
remain empty, awaiting a content.25
A2: Zizek

Zizek’s Alt is Oppressive/Violent 3/4


Zizek’s politics require authoritarian control and violence. He endorses actions such as
Stalinist purges, and concedes that political change is impossible.
Robinson and Tormey 05 (Andrew Robinson and Simon Tormey, professors of the school of Politics at
University of Nottingham, “A Ticklish Subject?Zizek and the Future of Left Radicalism,” Thesis Eleven, 8
November 2005, pg. 2)

So the Act is a rebirth – but a rebirth as what? The parallel with Lacan’s concept of ‘traversing the fantasy’ is crucial
because, for Lacan, there is no escape from the symbolic order or the Law of the Master. We are trapped
in the existing world, complete with its dislocation, lack, alienation and antagonism, and no transcendence can
overcome the deep structure of this world, which is fixed at the level of subject-formation. The most we can hope
for is to go from incapable neurosis to mere alienated subjectivity. In Zizek’s politics, therefore, a
fundamental social transformation is impossible. After the break initiated by an Act, a system similar
to the present one is restored; the subject undergoes identification with a Cause, leading to a new ‘proper symbolic Prohibition’
revitalized by the process of rebirth ( Zizek, 1999: 154, 368), enabling one ‘effectively to realize the necessary pragmatic measures’ ( Zizek,
1997b: 72–3), which may be the same ones as today. It is on this ground that Zizek is relaxed about supporting measures that, far from
challenging or undermining the status quo, give added support to it – as, for example, in his refusal to denounce structural adjustment policies
( Zizek, 1996: 32). This is all because, in his view, it is possible to start a ‘new life’, but only by replacing one symbolic fiction with another
( Zizek, 1999: 331). As a Lacanian, Zizek is opposed to any idea of realizing utopian ‘fullness’ and thus in escaping the vicissitudes of the
Any change in the basic structure of existence, whereby one may overcome
political qua antagonism.
dislocation and disorientation, is out of the question. However, he also rejects practical solutions to
problems as a mere displacement ( Zizek, 1999: 383–4). So an Act neither solves concrete problems nor
achieves drastic improvements; it merely removes blockages to existing modes of thought and action. It
transforms the ‘constellation which generates social symptoms’ (Butler et al., 2000: 124), shifting exclusion from
one group to another, but it does not achieve either drastic or moderate concrete changes. It ‘means that we accept
the vicious circle of revolving around the object [the Real] and find jouissance in it, renouncing the myth that
jouissance is amassed somewhere else’ ( Zizek, 1988: 109–10). It also offers those who take part in it a ‘dimension
of Otherness, that moment when the absolute appears in all its fragility’, a ‘brief apparition of a future utopian
Otherness to which every authentic revolutionary stance should cling’ ( Zizek, 2000: 159–60). This
absolute, however, can only be glimpsed. The leader, Act and Cause must be betrayed so the social
order can be refounded. The leader, or ‘mediator’, ‘must erase himself [sic] from the picture’ ( Zizek, 2001b:
50), retreating to the horizon of the social to haunt history as spectre or phantasy ( Zizek, 2000: 64). Every Great
Man must be betrayed so he can assume his fame and thereby become compatible with the status quo ( Zizek, 1999:
90–1, 316); once one glimpses the sublime Universal, therefore, one must commit suicide – as Zizek
claims the Bolshevik Party did, via the Stalinist purges (1997c).
A2: Zizek

The result of the plan is to completely subordinate the individual, replicating the abuses of
concentration camps
Robinson and Tormey 05 (Andrew Robinson and Simon Tormey, professors of the school of Politics at
University of Nottingham, “A Ticklish Subject?Zizek and the Future of Left Radicalism,” Thesis Eleven, 8
November 2005, pg. 2)

The Act thus reproduces in the socio-political field the Lacanian concept of traversing the fantasy.
Traversing the fantasy involves ‘accepting’ that there is no way one can be satisfied, and therefore
a ‘full acceptance of the pain . . . as inherent to the excess of pleasure which is jouissance’, as well as a
rejection of every conception of radical difference ( Zizek, 1997a: 30–1). It means ‘an acceptance of the fact
that there is no secret treasure in me’ ( Zizek, 1997a: 10), and a transition from being the ‘nothing’ we are
today to being ‘a Nothing humbly aware of itself, a Nothing paradoxically made rich through the very awareness of
its lack’ ( Zizek, 2000: 146–7). It involves being reduced to a zero-point or ‘ultimate level’ similar to that
seen in the most broken concentration-camp inmates ( Zizek, 2001b: 76–7, 86), so the role of analysis
is ‘to throw out the baby’ in order to confront the patient with his [their] ‘dirty bathwater’ ( Zizek,
1997a: 62–3), inducing not an improvement but a transition ‘from Bad to Worse’, which is ‘inherently
“terroristic” ’ ( Zizek, 1999: 377). It is also not freedom in the usual sense, but prostration before the
call of the truth-event, ‘something violently imposed on me from the Outside through a traumatic encounter
that shatters the very foundation of my being’ ( Zizek, 1999: 377). With shades of Orwell, Zizek claims that the Act involves ‘the highest
freedom and also the utmost passivity with a reduction to a lifeless automaton who blindly performs its gestures’. In other words, in the Act
freedom equals slavery ( Zizek, 1999: 377).
A2: Zizek

Zizek’s Alt is Oppressive/Violent 4/4


Zizek’s Theory promotes violence and exclusion of the Other
Robinson 05 (Andrew, Professor of Politics at Nottingham University, 2005, “The Political Theory of
Constitutive Lack: A Critique”)

Zizek's anti-capitalism has won him friends in leftist circles, but the capitalism to which he
objects is not the capitalism of classical Marxist critique. One could, indeed, question whether Zizek is
attacking capitalism (as opposed to liberalism) at all. His "capitalism" is a stultifying world of suffocating Good
which is unbearable precisely because it lacks the dimension of violence and antagonism. It is, he says, 'boring',
'repetitive' and 'perverse' because it lacks the 'properly political' attitude of 'Us against Them'20. It therefore
eliminates the element of unconditional attachment to an unattainable Thing or Real, an element which is the core of
humanity21. It delivers what Zizek fears most: a 'pallid and anaemic, self-satisfied, tolerant peaceful
daily life'. To rectify this situation, there is a need for suffocating Good to be destroyed by
diabolical Evil22. 'Why not violence?' he rhetorically asks. 'Horrible as it may sound, I think it's a
useful antidote to all the aseptic, frustrating, politically correct pacifism'23. There must always be social
exclusion, and 'enemies of the people'24. The resulting politics involves an 'ethical duty' to accomplish an Act which
shatters the social edifice by undermining the fantasies which sustain it25. As with Mouffe, this is both a duty and
an acceptance of necessity. 'By traversing the fantasy the subject accepts the void of his nonexistence'26. On a
political level, this kind of stance leads to an acceptance of social exclusion which negates
compassion for its victims. The resultant inhumanity finds its most extreme expression in Zizek's
work, where 'today's "mad dance", the dynamic proliferation of multiple shifting identities... awaits its resolution in
a new form of Terror'27. It is also present, however, in the toned-down exclusionism of authors such as Mouffe.
Hence, democracy depends on 'the possibility of drawing a frontier between "us" and "them"', and 'always entails
relations of inclusion-exclusion'28. 'No state or political order... can exist without some form of exclusion'
experienced by its victims as coercion and violence29, and, since Mouffe assumes a state to be necessary, this means
that one must endorse exclusion and violence. (The supposed necessity of the state is derived from the supposed
need for a master-signifier or nodal point to stabilize identity and avoid psychosis, either for individuals or for
societies). What is at stake in the division between these two trends in Lacanian political theory is akin to the
distinction Vaneigem draws between "active" and "passive" nihilism30. The Laclauian trend involves an implied
ironic distance from any specific project, which maintains awareness of its contingency; overall, however, it
reinforces conformity by insisting on an institutional mediation which overcodes all the "articulations". The
Zizekian version is committed to a more violent and passionate affirmation of negativity, but one which
ultimately changes very little. The function of the Zizekian "Act" is to dissolve the self, producing a historical
event. "After the revolution", however, everything stays much the same. For all its radical pretensions,
Zizek's politics can be summed up in his attitude to neo-liberalism: 'If it works, why not try a dose of it?'31. The
phenomena which are denounced in Lacanian theory are invariably readmitted in its "small print",
and this leads to a theory which renounces both effectiveness and political radicalism.
A2: Zizek

Alternative Fails-Any type of resistance to capitalism is rejected by Zizek, the alternative


can never materialize
Robinson and Tormey 05 (Andrew Robinson and Simon Tormey, professors of the school of Politics at
University of Nottingham, “A Ticklish Subject?Zizek and the Future of Left Radicalism,” Thesis Eleven, 8
November 2005, pg. 8)

So an Act neither solves concrete problems nor achieves drastic improvements; it merely
removes blockages to existing modes of thought and action. It transforms the ‘constellation which
generates social symptoms’ (Butler et al., 2000: 124), shifting exclusion from one group to another, but it does not
achieve either drastic or moderate concrete changes. It ‘means that we accept the vicious circle of revolving around
the object [the Real] and find jouissance in it, renouncing the myth that jouissance is amassed somewhere else’
(Zizek, 1988: 109–10). It also offers those who take part in it a ‘dimension of Otherness, that moment when the
absolute appears in all its fragility’, a ‘brief apparition of a future utopian Otherness to which every authentic
revolutionary stance should cling’ (Zizek, 2000: 159–60). This absolute, however, can only be glimpsed. The leader,
Act and Cause must be betrayed so the social order can be refounded. The leader, or ‘mediator’, ‘must erase himself
[sic] from the picture’ (Zizek, 2001b: 50), retreating to the horizon of the social to haunt history as spectre or
phantasy (Zizek, 2000: 64). Every Great Man must be betrayed so he can assume his fame and thereby become
compatible with the status quo (Zizek, 1999: 90–1, 316); once one glimpses the sublime Universal, therefore, one
must commit suicide – as Zizek claims the Bolshevik Party did, via the Stalinist purges (1997c). Furthermore,
despite Zizek’s emphasis on politics, his discussion of the Act remains resolutely individualist – as befits its clinical
origins. Zizek’s examples of Acts are nearly all isolated actions by individuals, such as Mary Kay Letourneau’s
defiance of juridical pressure to end a relationship with a youth, a soldier in Full Metal Jacket killing his drill
sergeant and himself, and the acts of Stalinist bureaucrats who rewrote history knowing they would later be purged
(Zizek, 1997a: 21; 1999: 385– 7; 2001b: 98–9). Even the Russian Revolution becomes for Zizek a set of
individual choices by Lenin, Stalin and the aforementioned bureaucrats, as opposed to the
culmination of mass actions involving thousands of ordinary men and women. This is
problematic as a basis for understanding previous social transformations, and even more so as a
recommendation for the future. The new subject Zizek envisages is an authoritarian leader,
someone capable of the ‘inherently terroristic’ action of ‘redefining the rules of the game’ (Zizek,
1999: 377). We would argue that this is a conservative, if not reactionary, position. Donald
Rooum’s cartoon character Wildcat surely grasps the essence of left radical ambition rather
better when he states, ‘I don’t just want freedom from the capitalists. I also want freedom from
people fit to take over.’
A2: Zizek

The Revolution is nihilism and anti-ethical-the alternative is circular and will never solve
Robinson and Tormey 05
(Andrew Robinson and Simon Tormey, professors of the school of Politics at University of Nottingham, “A Ticklish
Subject?Zizek and the Future of Left Radicalism,” Thesis Eleven, 8 November 2005, pg. 9)

True to form, Zizek does not see mere ‘impossibility’ as a barrier to action at all. Rather, he sees a
confrontation with the impossible as a sign of the purity and authenticity of a particular action, i.e. of what he
identifies as an authentic Act. For Zizek, an authentic, radical Act necessarily comes from the repressed
Real, and involves the return of this repressed impossibility. It necessarily, therefore, surprises not only
conformist observers, but the actor; it ‘surprises/transforms the agent itself’ (Butler et al., 2000: 124). The
Act therefore opens a redemptive dimension via a ‘gesture of sublimation, of erasing the traces of one’s past .
. . and beginning again from a zero-point’ (Zizek, 2000: 127). Such an Act is for Zizek a transcendental
necessity for subjective action, ‘a quasi-transcendental unhistorical condition of possibility and . .
impossibility of historicisation’ (Zizek, 1997a: 225–6). The Act, which for Zizek is the sole criterion
of
whether one’s politics are radical, is a structural or formal category, defined (in principle)
internally and radically separated from anything which does not meet its criteria. All alternatives
that fall short of the criteria of full Acts are for Zizek necessarily complicit in capitalism, even
those which share Zizek’s hostility to liberal capitalism, and including some which fit particular
formal requirements of an Act. At best, they are hysterical ‘false acts’, providing a ‘pseudo-
radical’, pseudo-resistance which actually sustains capitalism by contributing to its ‘phantasmic
supplement’. Acts have several formal criteria which Zizek formulates differently on different
occasions. First, someone who Acts must identify with the symptom, thereby revealing a
repressed Truth and bringing the Real to the surface.
Second, they must ‘suspend’ the existing symbolic system, including its ethics, politics, and
systems of meaning and knowledge; an Act is nihilistic and extra-, even anti-, ethical (at least as
regards any conception of the good). Since Zizek denies the existence of radical social, cultural
or psycho-logical difference, he believes that everyone is equally trapped by the dominant
symbolic system, so any break with it must come from beyond meaning and positive ethics. The
commitment an Act generates must be ‘dogmatic’; it ‘cannot be refuted by any argumentation’
and is indifferent to the truth-status of the Event it refers to. An Act has its own inherent
normativity, refusing all external standards; an Act (or Decision) is circular and tautological,
based on a ‘shibboleth’, and incomprehensible except from the inside.
A2: Zizek

Alternative Fails-radical resistance fails and only continues the cycle of oppressive
alienation, hierarchies and domination
Robinson and Tormey 05 (Andrew Robinson and Simon Tormey, professors of the school of Politics at
University of Nottingham, “A Ticklish Subject?Zizek and the Future of Left Radicalism,” Thesis Eleven, 8
November 2005, pg. 1)

The work of Slavoj Zizek has become an essential reference point for debates concerning the
future of left radical thought and practice. His attacks on identity politics, multiculturalism and ‘radical
democracy’ have established him as a leading figure amongst those looking to renew the link between socialist
discourse and a transformative politics. However, we contend that despite the undeniable radicality of Zizek’s
theoretical approach, his politics offers little in the way of inspiration for the progressive left. On the
contrary, his commitment to Lacanian categories reasserts the primordial character of alienation,
hierarchy and domination, and his proposed schema for confronting the status quo, the model of
the Act, serves to reaffirm rather than contest the given. We suggest that a genuinely
transformative politics should (contra Zizek) stress the necessity for the prefiguration of
alternatives, of linking and radicalizing ‘petty’ resistances, of encouraging critical and utopian
forms of thought and activity.

Alternative Fails-Zizek’s alternative is not progressive and reinforces current political


structures through active nihilism
Robinson and Tormey 05 (Andrew Robinson and Simon Tormey, professors of the school of Politics at
University of Nottingham, “A Ticklish Subject?Zizek and the Future of Left Radicalism,” Thesis Eleven, 8
November 2005, pg. 2)

In this article we want to suggest that whilst Zizek’s


recent work is intellectually ‘radical’, this is not,
despite appearances to the contrary, a radicalism that left politics can draw sustenance or hope
from. Zizek does not offer an alternative that is genuinely progressive or transformative, but only
the negativity of what Raoul Vaneigem terms ‘active nihilism’ (1967: 178). This negativity
‘breaks’ with the present but undermines rather than generates a meaningful politics of resistance
to the system. What Zizek delivers falls short of its promise. In our view, therefore, his position
should be opposed by those genuinely concerned with advancing left radical goals and a
meaningful resistance to the neoliberal status quo.
A2: Zizek

Zizek’s alternative cannot escape the current social system – he merely shifts oppression
from one group to another.
Robinson and Tormey 03, (Andrew and Simon, Professor of Politics at Nottingham University and Professor
of Politics at Nottingham University, 2003 “Zizek is not a Radical,”
http://homepage.ntlworld.com/simon.tormey/articles/Zizeknotradical.pdf)

So the Act is a rebirth - but a rebirth as what? The parallel with Lacan’s concept of ‘traversing the fantasy’ is crucial,
because, for Lacan, there is no escape from the symbolic order or the Law of the Master. We are
trapped in the existing world, complete with its dislocation, lack, alienation and antagonism, and
no transcendence can overcome the deep structure of this world, which is fixed at the level of
subjectformation; the most we can hope for is to go from incapable neurosis to mere alienated subjectivity. In
Zizek’s politics, therefore, a fundamental social transformation is impossible. After the break
initiated by an Act, a system similar to the present one is restored; the subject undergoes identification
with a Cause,77 leading to a new ‘proper symbolic Prohibition’ revitalised by the process of rebirth,78 enabling
one ‘effectively to realize the necessary pragmatic measures’,79 which may be the same ones as
today, e.g. structural adjustment policies.80 It is possible to start a new life by replacing one
symbolic fiction with another.81 As a Lacanian, Zizek is opposed to any idea of realising utopian fullness.
Any change in the basic structure of existence, whereby one may overcome dislocation and disorientation, is out of
the question. However, he also rejects practical solutions to problems as a mere displacement. 82 So an Act
neither solves concrete problems nor achieves drastic improvements; it merely removes
blockages to existing modes of thought and action. It transforms the ‘constellation which
generates social symptoms’,83 shifting exclusion from one group to another, but it does not
achieve either drastic or moderate concrete changes. It ‘means that we accept the vicious circle of
revolving around the object [the Real] and find jouissance in it, renouncing the myth that jouissance is amassed
somewhere else’.84 It also offers those who take part in it a ‘dimension of Otherness, that moment when the
absolute appears in all its fragility’, a ‘brief apparition of a future utopian Otherness to which every authentic
revolutionary stance should cling’.85 This absolute, however, can only be glimpsed. The leader, Act and Cause must
be betrayed so the social order can be refounded. The leader, or ‘mediator’, ‘must erase himself [sic]
from the picture’,86 retreating to the horizon of the social to haunt history as spectre or phantasy.87 Every Great
Man must be betrayed so he can assume his fame and thereby become compatible with the status quo;88 once one
glimpses the sublime Universal, therefore, one must commit suicide - as Zizek claims the Bolshevik
Party did, via the Stalinist purges (‘When the Party Commits Suicide’).
A2: Zizek

Zizek’s alternative is political paralysis – progressivism built around the desire for
transformation is the only way to avoid the replication of oppression.
Robinson and Tormey 03, (Andrew and Simon, Professor of Politics at Nottingham University and Professor
of Politics at Nottingham University, 2003 “Zizek is not a Radical,”
http://homepage.ntlworld.com/simon.tormey/articles/Zizeknotradical.pdf)

Zizek’s politics are not merely impossible, but potentially despotic, and also (between support
for a Master, acceptance of pain and alienation, militarism and the restoration of order)
tendentially conservative. They serve only to discredit the left and further alienate those it seeks
to mobilise. Instead, a transformative politics should be a process of transformation, an alinear,
rhizomatic, multiform plurality of resistances, initiatives, and, indeed, acts, which are sometimes spectacular
and carnivalesque, sometimes prefigurative, sometimes subterranean, sometimes rooted in institutional change
and reform, sometimes directly revolutionary. Zizek’s model of the pledged group, bound together by
the One who Acts, is entirely irrelevant to the contemporary world and would be a step
backwards from the decentred character of current leftradical politics. Nor need this decentring be seen
as a weakness as Zizek insists. It can be a strength, protecting radical politics from self-appointed elites,
transformism, infiltration, defeat through the ‘neutralisation’ of leaders, and the threat of a repeat of the Stalinist
betrayal. In contrast with Zizek’s stress on subordination, exclusivity, hierarchy and violence, the tendency of anti-
capitalists and others to adopt anti-authoritarian, heterogeneous, inclusive and multiform types of
activity offer a better chance of effectively overcoming the homogenising logic of capitalism and
of winning support among wider circles of those dissatisfied with it. Similarly, the emphasis on
direct action - which can include ludic, carnivalesque and non-violent actions as well as more overtly
confrontational ones - generates the possibility of empowerment through involvement in and support
for the myriad causes which make up the anti-capitalist resistance. This resistance stands in stark
contrast to the desert of ‘heroic’ isolation advocated by Zizek, which, as Laclau puts it, is ‘a
prescription for political quietism and sterility’.154 Zizek is right that we should aim to overcome the
‘impossibilities’ of capitalism, but this overcoming should involve the active prefiguration and
construction in actuality of alternative social forms, not a simple (and actually impossible) break
with everything which exists of the kind imagined by Zizek. It is important that radicals invoke
‘utopias’, but in an active way, in the forms of organisation, ‘disorganisation’, and activity we
adopt, in the spaces we create for resistance, and in the prefiguration of alternative economic,
political and social forms. Utopian imaginaries express what is at stake in left radicalism: that what
exists does not exist of necessity, and that the contingency of social institutions and practices makes
possible the overthrow of existing institutions and the construction or creation of different
practices, social relations, and conceptions of the world. The most Zizek allows to radicals is the ability
to ‘glimpse’ utopia while enacting the reconstruction of oppression. Radicals should go further,
and bring this imagined ‘other place’ into actual existence. Through enacting utopia, we have the ability
to bring the ‘no-where’ into the ‘now-here’
A2: Zizek

Their alternative as presented in this debate round is not the radical act – it fails to meet
Zizek’s own standards of what constitutes the Act. Zizek calls this a “false act,” one which
merely serves to continue capitalism.
Robinson and Tormey 03, (Andrew and Simon, Professor of Politics at Nottingham University and Professor
of Politics at Nottingham University, 2003 “Zizek is not a Radical,”
http://homepage.ntlworld.com/simon.tormey/articles/Zizeknotradical.pdf)

Caught in the Act The answer is that Zizek does not see impossibility as a barrier to action. Rather, he sees it as a sign of the purity and
authenticity of a particular action, i.e. of what he identifies as an authentic Act. For Zizek, an authentic, radical Act
necessarily comes from the repressed Real, and involves the return of this repressed impossibility. It necessarily,
therefore, surprises not only conformist observers, but the actor; it ‘surprises/transforms the agent
itself’.37 The Act therefore opens a redemptive dimension via a ‘gesture of sublimation, of
erasing the traces of one’s past … and beginning again from a zeropoint’. 38 Such an Act is for Zizek a
transcendental necessity for subjective action, ‘a quasi-transcendental unhistorical condition of possibility and … impossibility of historicisation’.
The Act, which for Zizek is the sole criterion of whether one’s politics are radical, is a structural or formal category, defined (in principle)
All alternatives - even those which share
internally and radically separated from anything which does not meet its criteria .
Zizek’s hostility to liberal capitalism, and including some which fit particular formal
requirements of an Act - which fall short of the criteria of full Acts are for Zizek necessarily
complicit in capitalism. At best, they are hysterical ‘false acts’, providing a pseudo-radical
pseudo-resistance which actually sustains capitalism by contributing to its phantasmic
supplement.40 Acts have several formal criteria which Zizek formulates differently on different occasions. Firstly,
someone who Acts must identify with the symptom, thereby revealing a repressed Truth and bringing the Real to
the surface. Secondly, they must ‘suspend’ the existing symbolic system, including its ethics, politics,
and systems of meaning and knowledge;41 an Act is nihilistic and extra-, even anti-, ethical (at least as regards any
conception of the good). Since Zizek denies the existence of radical social, cultural or psychological difference, he believes that everyone is
equally trapped by the dominant symbolic system, so any
break with it must come from beyond meaning and
positive ethics. The commitment an Act generates must be ‘dogmatic’; it ‘cannot be refuted by any
argumentation’ and is indifferent to the truth-status of the Event it refers to.42 An Act has its
own inherent normativity, refusing all external standards;43 an Act (or Decision) is circular and
tautological,44 based on a shibboleth,45 and incomprehensible except from the inside.46 It is a response to
an ethical injunction beyond ordinary ethical norms, so that ‘although what I am about to do will have catastrophic
consequences for my well-being and for the well-being of my nearest and dearest, none the less I simply have to do
it, because of the inexorable ethical injunction’.47 The Act resolves all problems in a single, all-encompassing
Terror which bypasses particularities and violently stops the ‘mad dance’ of shifting identities, operating instead ‘ to
ground a new political universality by opting for the impossible, with no taboos, no a priori
norms... respect for which would prevent us from ‘resignifying’ terror, the ruthless exercise of power, the spirit of
sacrifice’.48 An Act is symbolic death,49 creatio ex nihilo and self-grounded.50 It is the outcome of ‘an ethics
grounded in reference to the traumatic Real which resists symbolisation’, i.e. to ‘an injunction which cannot
be grounded in ontology’,51 a ‘selfreferential abyss’,52 an excessive gesture irreducible to human
considerations and necessarily arbitrary.53 The suspension of ethical, epistemological and political
standards is not a necessary consequence of a Zizekian Act - it is a defining feature. It is necessary so a
new system can be built from nothing,54 and anything short of a full Act remains on enemy
terrain.55
A2: Zizek

Zizek’s alternative is genocide – it requires acceptance of human nature as violent and


extermination of those who get in the way of the revolution.
Robinson and Tormey 03, (Andrew and Simon, Professor of Politics at Nottingham University and Professor
of Politics at Nottingham University, 2003 “Zizek is not a Radical,”
http://homepage.ntlworld.com/simon.tormey/articles/Zizeknotradical.pdf)

Secondly, Zizek implies that Lenin must in some sense have ‘understood’ that the revolution would necessarily
betray itself, and that all revolutions are structurally doomed to fall short of whatever ideals and principles motivate
them. He also implies that the success or failure of a revolution has nothing to do with whether the modes of thought
and action, social relations and institutions which follow are at all related to the original revolutionary ideals and
principles. What matters is that power is held by those who ‘identify with the symptom’, who call themselves
‘Proletarian’. Zizek therefore endorses the conservative claim that Lenin’s utopian moments were
Machiavellian manoeuvres or at best confused delusions, veiling his true intentions to seize
power for himself or a small elite: Lenin was the ‘ultimate political strategist’.121 That Zizek endorses the
‘Lenin’ figure despite endorsing nearly every accusation against Lenin serves to underline the degree to which
Zizek’s politics are wedded to conservative assumptions that repression, brutality and terror are
‘always with us’. Rejecting the claim that politics could be otherwise, Zizek wishes to grasp,
embrace and even revel in the grubbiness and violence of modern politics. The moment of utopia
in Russia was for Zizek realised when the Red Guards succumbed to a destructive hedonism in
moments of Bataillean excess.122 The only difference for Zizek between leftist ethics and the standpoint of Oliver
North, the Taleban, the anti-Dreyfusards and even the Nazis is that such ‘rightists’ legitimate their acts in reference
to some higher good, whereas leftists also suspend the higher good in a truly authentic gesture of suspension.123
The Soviet Terror is a good terror whereas the Nazi one is not, only because the Soviet terror
was allegedly more total, with everyone being potentially at risk, not only out-groups.124 Zizek
goes well beyond advocating violence as a means to an end; for Zizek, violence is part of the end
itself, the utopian excess of the Act. The closest parallel is the nihilism of Nechaev’s Catechism of a
Revolution which proclaims that ‘everything is moral that contributes to the triumph of the
revolution; everything that hinders it is immoral and criminal’.125 As Peter Marshall comments
in his digest of anarchist writings and movements, the Catechism is ‘one of the most repulsive
documents in the history of terrorism’. One can only speculate what he would have made of
‘Repeating Lenin’.126
A2: Zizek

Perm Solves-Challenging leftist theories is the only way to create true political
progressivism
Robinson 05 (Andrew, Professor of Politics at Nottingham University, 2005, “The Political Theory of
Constitutive Lack: A Critique”)

Amongst a plethora of radical theoretical perspectives, a new paradigm is slowly becoming


hegemonic. Inspired by the work of Jacques Lacan, theorists are increasingly turning to the
concept of "constitutive lack" to find a way out of the impasses of classical Marxist, speculative
and analytical approaches to political theory. Beneath the debates between rivals such as Ernesto
Laclau and Slavoj Zizek, there is a unity of purpose about the parameters of political theory.
Across the work of authors such as Zizek, Laclau, Chantal Mouffe, Yannis Stavrakakis, David
Howarth, Renata Salecl, Jason Glynos, Aletta Norval and Saul Newman, there is a central set of
motifs and claims which mark out a distinct tradition within contemporary political thought. The
idea of "constitutive lack", constructed as an ontological claim, operates also in these theories as
a normative concept, and it is used to found normative claims. The title of Alenka Zupančič's
most famous book Ethics of the Real summarises the outlook of all these authors1. The challenge
posed by this influential perspective is too important to ignore. Its paradigmatic structure - the
shared, often unconscious and unreflexive, assumptions which unite its various proponents in a
single way of thinking and arguing - is becoming the dominant trend in (ostensibly) radical
theory. It is accounting for a growing number of submitted and published articles and is gaining
a growing support among researchers and graduates. It has almost invisibly gained a foothold in
theoretical literature significant enough to raise its influence to a level second only, perhaps, to
the analytical/Rawlsian tradition. This is at least partly due to its radical pretensions. It is,
however, crucial to challenge it, because its political effects are to paralyse "radical" theory. It
provides a very weak basis for any kind of politics, and certainly no basis for a radical or
transformative agenda. It is, in short, a surrogate radicalism, a theoretical placebo which does not
live up to the promises it makes. This article examines this paradigm through a critique of its
founding concept. In contrast to the claims of authors such as Laclau to have escaped the
"essentialism" of classical political theory, I shall demonstrate that the idea of "constitutive lack"
involves the reintroduction of myth and essentialism into political theory. I shall demonstrate that
Lacanian political theory cannot meet its claims to be "radical" and "anti-essentialist", and its
central arguments are analytically flawed. First of all, however, I shall outline the parameters of
this new theoretical paradigm2.
A2: Zizek

Alt Fails and Impact Turn-Zizek does not provide any social transformation theory and
endorses an endless cycle of war and violence
Robinson and Tormey 05 (Andrew Robinson and Simon Tormey, professors of the school of Politics at
University of Nottingham, “A Ticklish Subject?Zizek and the Future of Left Radicalism,” Thesis Eleven, 8
November 2005, pg. 2-3)

Against this alleged pseudo-radicalism, Zizek revives traditional leftist concepts such as ‘class
struggle’. He ignores, however, the ‘orthodox’ left meaning of such terms, rearticulating them in
a sophisticated Hegelian and Lacanian vocabulary. Yet problems remain: Zizek’s version of
‘class struggle’ does not map on to traditional conceptions of an empirical working class, and
Zizek’s ‘proletariat’ is avowedly ‘mythical’. He also rejects newer forms of struggle such as the
anti-capitalist movement and the 1968 uprisings, thereby reproducing a problem common in
radical theory: his theory has no link to radical politics in an immediate sense. Nevertheless, he
has a theory of how such a politics should look, which he uses to judge existing political
radicalisms. So how does Zizek see radical politics emerging? Zizek does not offer much by way
of a positive social agenda. He does not have anything approximating to a ‘programme’, nor a
model of the kind of society he seeks, nor a theory of the construction of alternatives in the
present. Indeed, the more one looks at the matter, the more difficult it becomes to pin Zizek
down to any ‘line’ or ‘position’. He seems at first sight to regard social transformation not as
something ‘possible’ to be theorized and advanced, but as a fundamental ‘impossibility’ because
the influence of the dominant symbolic system is so great that it makes alternatives unthinkable.
A fundamental transformation, however, is clearly the only answer to the otherwise compelling
vision of contemporary crisis Zizek offers. Can he escape this contradiction? His attempt to do so
revolves around a reclassification of ‘impossibility’ as an active element in generating action.
Asserting or pursuing the impossible becomes in Zizek’s account not only possible but desirable.
So how then can the left advance its ‘impossible’ politics? How is a now ‘impossible’ model of
class struggle to be transformed into a politics relevant to the present period? As becomes
evident, ‘class struggle’ is not for Zizek an empirical referent and even less a category of
Marxisant sociological analysis, but a synonym for the Lacanian Real. A progressive
endorsement of ‘class struggle’ means positing the lack of a common horizon and assuming or
asserting the insolubility of political conflict. It therefore involves a glorification of conflict,
antagonism, terror and a militaristic logic of carving the field into good and bad sides, as a good
in itself (see, for example, the discussion in Zizek, 2000: 57, 126). Zizek celebrates war because
it ‘undermines the complacency of our daily routine’ by introducing ‘meaningless sacrifice and
destruction’ (Zizek, 1999: 105). He fears being trapped by a suffocating social peace or Good
and so calls on people to take a ‘militant, divisive position’ of ‘assertion of the Truth that
enthuses them’ (Zizek, 2001b: 237–8).
at lacan/zizek: “the act” fails

Zizek’s Act fails to accomplish fundamental change—it is merely therapeutic for individuals.

Robinson 04 (Andrew, PhD, political theory, University of Nottingham, “Introduction: The Basic Zizekian Model,” Theory Blog,
http://andyrobinsontheoryblog.blogspot.com/2004/11/zizek-notes-and-work-in-progress_15.html)

Why does Zizek support the Act? Although he connects the Act to 'radicalism', he does not state anywhere that the Act
accomplishes any fundamental change in the deep structure of existence; at best, it can temporarily suspend (for instance)
exclusion. This is not an attempt to achieve a better world (still less a perfect one!) but a purely structural attempt to restore
something which Zizek thinks is missing. In this sense, even in its 'radicalism', the Act is conservative. Zizek is concerned that
the matrix of sublimation - the possibility of producing 'sublime' objects which seem to encapsulate the absolute - is under threat
(FA 26; elsewhere, Zizek attacks postmodernists and other 'new sophists' for this). The Act in whatever form reproduces the
possibility of sublimity; in this sense, it reproduces old certainties in new forms, undermining all the gains made by theories of
historicity and contingency. The purpose of the Act, which Zizek has transplanted from psychoanalytic practice (directed at
individual psyches) to socio-political practice (directed at entire social systems) without considering whether this is possible or
appropriate, is primarily therapeutic. The role of the Act is to solve the antinomy of the present by asserting a Real against the
combined Imaginary and Real of simulacra, thereby reintroducing the impossibility that shatters the Imaginary, enabling us to
traverse the fantasy (TS 374; the fantasy is the extimate kernel of libidinal investment which Zizek sees lurking almost
everywhere). Zizek seems to be restoring to psychoanalysis a naive conception of psychological health: via the ex nihilo act, one
can escape the logic of the symptom (DSST 178).

Zizek’s Act is radically nihilistic and accomplishes nothing political.

Robinson 04 (Andrew, PhD, political theory, University of Nottingham, “Introduction: The Basic Zizekian Model,” Theory Blog,
http://andyrobinsontheoryblog.blogspot.com/2004/11/zizek-notes-and-work-in-progress_15.html)

It is important to realise that the Act is not revolutionary in the sense of creating something new on the basis of an ideal, or an
imaginary, or the restoration of an authentic pre-alienated state, or any other process which would allow one to create something
on the basis of a project and praxis. The Act is radically nihilistic (see below). For Zizek, the subject can change nothing - all it
can do is add itself to reality by an act of claiming responsibility for the given (SOI 221). Zizek is a little inconsistent on the relationship between the Act and the
existing system, but on the whole, he seems to see Acts as occurring for the system, against imaginaries and especially the extimate kernel of fantasy. Christianity did not so much suspend the law, says Zizek, as
suspend its obscene supplement (FA 130) (i.e. extimate kernel). Zizek thinks fantasy is fundamentally inconsistent, so it is an "ethical duty" to put this on display, in order to disrupt fantasy (PF 74; see
CONSERVATISM on Zizek's tendency to conflate 'displaying' with 'doing', so that the boundary between being a sexist or a fascist and displaying sexism or fascism to disrupt it is unclear). Zizek is inconsistent,
however, since there are also occasions when he seems to want to encourage fantasies (TS 51). Crucially, the Act is also a form of decisiveness. Zizek wants to pin down vacillating signifiers without using a Master-
Signifier or quilting-point, he says on one occasion (FA 139-40). Elsewhere (eg. on Chavez and Lenin), he seems to rather like the Master or "One" whose Act 'quilts' the field. Either way, the Act seems to give a
certain focus to discourse, acting as a centre. As his discussions of the vanishing mediator show, he sees the Act establishing a new set of symbolic and imaginary discourses which restore the role of the master-
signifier, by directly adopting the position of the extimate kernel. Zizek also sees the Act as a resolution of a dilemma. According to Zizek, Good assumes (and therefore produces) Evil, and the Act escapes the
resulting dilemma by breaking with Good (TS 382; this is also what distinguishes the Act as diabolical Evil from everyday evil - crime, the Holocaust and so on). For Zizek, denial of the possibility of the Act is the root
). What seems completely missing here is any case for the Act that in any way justifies ethically the terrible nature of the
of evil (TS 376
Act, both for its perpetrator and for others; one can only really accept Zizek's Act if one places at the core of one's belief-system
the importance of resolving dilemmas in some supposed deep structure of existence, so what matters is not human or social
consequences or any specific beliefs, but merely the adoption of a structural position which solves contradictions in and thereby
overcomes the problems of a structure. Despite Zizek's repeated use of the term "ethics", therefore, this is in many ways not an
ethical system at all, but a kind of model of structural problem-solving - a "therapy" for society, passed off as ethics.
at lacan/zizek: “the act” fails

Zizek’s alternative “act” conceptualizes culture as having so much power over individuals that liberation is impossible.
Robinson 04 (Andrew, PhD, political theory, University of Nottingham, “Introduction: The Basic Zizekian Model,” Theory Blog,
http://andyrobinsontheoryblog.blogspot.com/2004/11/zizek-notes-and-work-in-progress_15.html)

The Act is a fundamentally negative occurrence in which one strips oneself of all human dignity and 'recognises' that one is
nothing but excrement, that there is no 'little treasure' inside and that the subject is nothing but a void. (It is therefore utterly
incompatible with approaches which involve action - eg. praxis - as a humanising phenomenon). "By traversing the fantasy, the
subject accepts the void of his nonexistence" (TS 281). Traversing the fantasy leads to subjective destitution: abandoning the
notion of something 'in me more than myself' and recognising that the big Other is nothing but a semblance. This involves a
change in one's worldview: the "analyst's desire" makes possible a community minus its phantasmic support, without any need for a 'subject supposed to...' (know, enjoy or believe) (TS 296). (In this
passage Zizek portrays the Act as leading to a fundamental shift in character-structure, although this is not a claim he repeats consistently). An Act is defined by the characteristic that it "surprises/transforms the
agent itself" (CHU 124; a choice in the usual sense cannot therefore be an Act). It involves subjective destitution , a (supposedly) liberating moment, "the anti-ideological gesture par excellence
by means of which I renounce the treasure within myself and fully admit my dependence on the externality of symbolic apparatuses - that is to say, fully assume the fact that my very self-experience of a subject who
was already there prior to the external process of interpellation is a retrospective misrecognition brought about by the process of interpellation" (CHU 134; NB how this means endorsing control by the system, not
). The Act therefore involves an utter prostration before symbolic apparatuses: NOT the liberation of the
opposing it; cf. MATERIALISM
human from the system, but the total victory of the system over humans (cf. Zizek's support for Big Brother-type surveillance; see
MARX).

The Act is indeterminate nonsense—there is no way to know if a political intervention meets the criteria.

Robinson 04 (Andrew, PhD, political theory, University of Nottingham, “Introduction: The Basic Zizekian Model,” Theory Blog,
http://andyrobinsontheoryblog.blogspot.com/2004/11/zizek-notes-and-work-in-progress_15.html)

Not surprisingly given that he sees the Act as shattering meaning, Zizek wants a commitment which is "dogmatic", "cannot be
refuted by any 'argumentation' " and "does not ask for good reasons", and which is "indifferent" to the truth-status of the Event it
refers to (TS ****; find reference). A Decision (Act) is circular, a shibboleth, and a creative act which nevertheless reveals a
constitutive void which is invisible (TS 138; NB the slippage between epistemology and ontology here: how do we know the Act
is revealing rather than creating the void?). Law is legitimated by transference: it is only convincing to those who already believe
(SOI 38). The Act subverts a given field as such and achieves the apparently 'impossible' by retroactively creating the conditions
of its possibility by changing its conditions (CHU 121). It has its own inherent normativity, lacking any simple external standards
(TS 388) As well as being problematic in itself, this kind of open advocation of irrationalism and dogmatism would seem to rule
out the possibility of empirically or rationally assessing the validity of a particular Act: by definition an Act is not open to such
assessment, so one cannot judge between a false (eg. Nazi) and a true Act, since this would involve precisely such a rational
and empirical process of assessment ("good reasons" and truth-status). This raises problems for Zizek's attempts to distance
himself from Nazism (see below, on false acts). Also, Zizek is being inconsistent in trying to defend such an attack on
communication by communicative means (can one make a rational case against rationality?).
at lacan/zizek: “the act” fails

The Zizekian “Act” requires an abandonment of ethics and accepting an obliteration of the self.

Robinson 04 (Andrew, PhD, political theory, University of Nottingham, “Introduction: The Basic Zizekian Model,” Theory Blog,
http://andyrobinsontheoryblog.blogspot.com/2004/11/zizek-notes-and-work-in-progress_15.html)

Zizek's theory of the Act presupposes a belief that we are all basically worthless. "The ultimate level of the ethical experience" is
found in the utterly broken victim of the Nazi or Stalinist camps (DSST 86), which means one "will be surprised to learn how even
the darkest Stalinism harbours a redemptive dimension" (DSST 88). Humanity per se is reducible to the most broken concentration camp inmates (i.e. the ones who have
gone beyond trying o reconstruct meaning through petty resistances; referred to in the camps as "Muslims" or "Musselmen" because of their resemblance to famine victims); these people were not dehumanised by
the Nazis, but rather, express an inhuman kernel of humanity (DSST 76-7). This kind of person is the " 'zero-level' of humanity" which makes human symbolic engagement possible by wiping the slate of animal
instincts (DSST 77; NB the strong binary operative here, which is totally flawed: dogs show similar modes of action when exposed to similar situations, such as Seligman's dogs in the 'learned helplessness'
experiments). Zizek thinks we all have had to go through this experience (DSST 77-8). This experience also negates the concept of authenticity (though not enough to stop Zizek using it elsewhere): one can't say
such victims are involved in an authentic existential project, but it would be cynical to say they are living an inauthentic existence since it is others, not themselves, who degrade them (DSST 78-9; I don't actually see
why an external basis for subordination would affect the concept of authenticity in the slightest; perhaps it would affect the strongest versions which assume pure freedom, but it would not undermine, for instance, the
later Sartre, since in this case the authenticity of the project has been defeated by the practico-inert, leading to a state of existence he terms "exis": a degraded existence without project). I think a Deleuzian analysis
would be more appropriate here: the dehumanisation of these victims results from the (temporary) total victory of the Oedipal/authoritarian cage: flows and breaks are cut off or utterly contained within an order of
power/knowledge, with the political conclusion being that freedom exists in a struggle with domination and that the struggle for freedom is necessary to prevent us being reduced to this level. But this would be partly a
; Zizek cannot in all seriousness criticise the inhumanity
causal account, whereas Zizek seems to want a pure ethics. Where Zizek's account leads politically is far more sinister
of the concentration camps if they simply reveal our essence, and it is hard to see how one could oppose the Nazis if they did not
dehumanise their victims or treat them inhumanely. Indeed, such an excremental reduction is something Zizek elsewhere
praises, and his attempts to distance himself from Nazism have nothing to do with the inhumanity of the camps; rather, they
revolve around nit-picking over whether the Nazis really traversed the fantasy or stopped short at a false act (see below). The
Act is a submission: revolutionaries should become "followers" of the truth-event and its call (TS 227; this reproduces with a
reversed sign Vaneigem's concept of the Cause as a form of alienation. cf. Donald Rooum's cartoon Wildcat: "I don't just want
freedom from the capitalists, I also want freedom from people fit to take over"). Love is "nothing but" an act of self-erasure which
breaks the chain of justice (DSST 49-50). Zizek demands submission to radically exterior, meaningless injunctions, "experienced
as a radically traumatic intrusion", which "a renewed Left should aim at fully endorsing"; "something violently imposed on me
from the Outside through a traumatic encounter that shatters the very foundations of my being" (TS 212). It also involves the
negation of dignity: Zizek refers to "heroically renouncing the last vestiges of narcissistic dignity and accomplishing the act for
which one is grotesquely inadequate" (TS 352). The heroism of the act is to openly endorse a transition "from Bad to Worse",
and for this reason, a true act, which redefines the 'rules of the game', is "inherently 'terroristic' " (TS 377). Thus, instead of the
"liberal trap" of respecting some rights and rejecting obligatory Party lines, one should seek the "good terror", i.e. choosing what
one has to do (TS 378). Any qualms are dismissed by Zizek as "humanist hysterical shirking the act" (TS 380; NB this misuse of clinical categories in socio-ideological analysis quickly leads Zizek into
problems: the Lacanian categories obsessional/hysterical/psychotic/perverse are strictly incompatible, whereas it is quite clear that a theorist who 'hysterically' rejects terror may easily also 'psychotically' believe in
literality and 'perversely' believe in decoded flows). The Act involves accepting utter self-obliteration, and rejecting all compassion (TS 378).
at lacan/zizek: “the act” fails

Zizek’s “act” erases all compassion for others.

Robinson 04 (Andrew, PhD, political theory, University of Nottingham, “Introduction: The Basic Zizekian Model,” Theory Blog,
http://andyrobinsontheoryblog.blogspot.com/2004/11/zizek-notes-and-work-in-progress_15.html)

Assuming an Act means rejecting all concern for others and making oneself, to all intents and purposes, a rock. In the Act, one
"assumes... the full burden of freedom impervious to any call of the Other" (DSST 175). Whereas in Derrida and other
postmodernists, argues Zizek, ethics is a response to the call of the Other, either abyssal or actual, in Zizek's Lacan the ethical
act proper suspends both of these along with the rest of the 'big Other' (DSST 161). Zizek loathes 'soft-heartedness' because it
"blurs the subject's pure ethical stance". In this passage, he is referring to Stalinist views; but his criticism of them is not of this
loathing; rather, he thinks "that they were not 'pure' enough" because they got caught in an emotional sense of duty (DSST 111).
This according to Zizek is the difference between Lenin and Stalin: Zizek's Lenin did not become emotionally attached to his Act
(DSST 113). Zizek's ethical anti-humanism goes so far that he advocates hating the beloved out of love (FA 126), because what
one should love is not their human person. Zizek also endorses Kant's attempt to purge ethics of historical contents, including
compassion and concern for others (PF 232-3).

Zizek’s alternative fails to transform the existing order—it is a shot in the dark.

Robinson 04 (Andrew, PhD, political theory, University of Nottingham, “Introduction: The Basic Zizekian Model,” Theory Blog,
http://andyrobinsontheoryblog.blogspot.com/2004/11/zizek-notes-and-work-in-progress_15.html)

How one locates the Act in relation to revolution depends just how fundamentally the change involved in a revolution is
conceived. The Act according to Zizek disrupts/overthrows the existing order of Imaginary and Symbolic alignments (though this
does not of course make it revolutionary in practice); however, his account seems to involve the restoration of the basic structure
of the social system subsequently, so there is no possibility of meaningful change in terms of overcoming social oppression and
exclusion or the irrationalities of ideology. (This also leaves the question of why an Act would lead to anything better; indeed,
Zizek denies that it would. So why opt for an Act?). In a sense, the Act is conservative. Traversing the fantasy involves the act of 'accepting' there is no way one can ever be satisfied:
a direct relation to the objet petit a (i.e. desired object) minus the screen of fantasy, involving "a full acceptance of the pain... as inherent to the excess of pleasure which is jouissance" (PF 30). This means accepting
"radical ontological closure" - i.e. 'accepting' that there is no radical difference - and also that "we renounce every opening, every belief in the messianic Otherness", including, for instance, Derridean and Levinasian
concepts of being 'out-of-joint' (PF 31), especially the idea of jouissance being amassed elsewhere. This leads one into the realm of drive; one becomes "eternal-'undead' " (PF 31). (Zizek is here replacing an
irrational belief that jouissance is amassed elsewhere with an irrational belief that it isn't; the existence or non-existence of difference and Otherness is an empirical question, and Zizek's refusal to accept that radical
). Crucially, the Act does not involve overcoming Law and the system. It
Otherness could exist renders his theory potentially extremely normalist and ethnocentric
involves suspending them, so they can be resurrected or resuscitated on a new basis. Although the Act is a 'shot in the dark'
(preventing voluntary reconstruction/transformation of society), nevertheless it always involves a necessary betrayal (see TS)
which reproduces the Oedipal/authoritarian structure of the world; the vanishing mediator always vanishes so as to restore the system. It is interesting to note Zizek's
insistence on using the word "suspension" (St Paul's suspension of the law, the leftist suspension of the ethics, and so on). The suspension of the Law, as shown in Zizek's quote from St Paul (TS 150-1), is clearly in
fact something more: it is in a sense psychotic, breaking with both Law and desire. But it is a suspension because it resurrects Law in the more total form of the Cause. It is interesting that Zizek chooses the word
"suspension". If Zizek has in mind a destruction or fundamental transformation of the Law or ethics, there are so many better terms he could have chosen: abolition, destruction, smashing, overcoming, transcending,
sublating, surpassing and so on. That he (more-or-less consistently) uses the term "suspension" is therefore probably significant. This term implies a temporary absence of the phenomenon in question, as opposed
: what is suspended (Law, ethics, etc.) nevertheless returns in the same basic
to its permanent destruction, replacement, or even transformation. In other words
form as before (which presumably means its structural nature is basically the same).
at lacan/zizek: “the act” fails

Zizek’s Act relies on extreme individualism—it results in no social change.

Robinson 04 (Andrew, PhD, political theory, University of Nottingham, “Introduction: The Basic Zizekian Model,” Theory Blog,
http://andyrobinsontheoryblog.blogspot.com/2004/11/zizek-notes-and-work-in-progress_15.html)

The category of the Act involves extreme methodological individualism. The assumption that an individual Act can alter society
as a whole, whatever its earth-shattering psychological consequences for a particular individual, is deeply flawed. This problem
is related to Zizek's inappropriate expansion of what are at root clinical/therapeutic concepts into socio-political analysis.
Individual Acts do not have direct social effects. The Mary Kay Letourneau case, for instance, has not substantially changed
popular perceptions of non-abusive relations between legal- and illegal-age people; it certainly has not shattered the social
structure. Rather, Letourneau has been anathematised and victimised by the state. On a social level, the Act is impotent and
politically irrelevant; it has no transformative role and makes sense only in a closed analytical system. Even when Acts of Zizek's
type do have social effects, there is no reason to believe that these effects shatter or reformulate entire social structures . Zizek's
account here rests on psychologising social structures, imagining that these structures rest on the same basis as a Lacanian account of the psyche. Actually, a single act on the superficial level is unlikely to alter the
social structure any more than a tiny amount. For instance: suppose Letourneau's Act worked; suppose the law was changed to make love a defence for consensual sex across the age-of-consent boundary. Would
this have any deep-rooted social effects? Surely not. Such changes have not, for instance, taken us very far towards gay liberation; the situation is better than it was, but the social position of gay men has not been
. Acts are impotent against deep prejudices.
reshaped dramatically
Since Acts do not have meaningful social effects, they cannot really help the worst-off group (social symptom). If the "cathartic
moment" of a break with the dominant ideology only occurs in a single individual, the social system would not be harmed. To be
effective, it would have to produce a new conception of the world which is expansive and convinces wide strata of the population.
Zizek is missing the significance of revolutions such as in Russia when he sees them as pure Acts by leaders; this is an
intentionalist delusion. As Gramsci rightly puts it, each revolution involves an "intense critical labour" whereby a new conception
of the world is formulated, spread and used to create a collective will. The collective will does not simply spring miraculously from
a leader's whim.

The “act” is so radically negative that it is unable to formulate new politics.

Robinson 04 (Andrew, PhD, political theory, University of Nottingham, “Introduction: The Basic Zizekian Model,” Theory Blog,
http://andyrobinsontheoryblog.blogspot.com/2004/11/zizek-notes-and-work-in-progress_15.html)

Because of his extreme methodological individualism, Zizek ends up with a highly intentionalist, leader-fixated model of politics
which is authoritarian and also exaggerates the role of leaders both in practice and potentially. Stalinism, for instance, was not a result of an Act by
Stalin and Lenin; it was a social-structural phenomenon involving the actions of many individuals, with a "history of everyday life" and structural dynamics such as intrabureaucratic competition, resulting from the
mode (or modes) of thought and action it involved. The extension of clinical categories into society requires the reduction of concepts which are usually diverse to singularity: one unconscious, symptom, fundamental
Zizek's politics are "a prescription for political
fantasy, etc. for entire societies or even the whole of humanity. This is in contradiction with psychoanalytic practice and also is implausible.
quietism and sterility" (Laclau, CHU 293). I disagree with Laclau's reasons for claiming this, but the conclusion is valid: the Act
has little practical political relevance, and Zizek's sectarianism (see RESISTANCE) leaves him aloof from actual political
struggles. Zizek seems to have no real sense of what is important in politics. For Zizek, the main issue is reviving the category of the Act, to fill a supposed structural void. But there are many concrete issues
which are many times more important: closing down the WTO, fighting back against the wave of police repression, stopping the wholesale commodification of society, stopping environmental destruction, stopping
Bush's racist war, smashing capitalism, etc. 'Restoring the properly ethical dimension of the act' only matters to someone who is so trapped in his own theory that he thinks the whole world revolves around it. (What
. Zizek
did Wittgenstein say about philosophy and masturbation?). Zizek should let the fly out the jar! The abstract and essentialist pursuit of the "act proper" is a distraction from contingent political struggles
lacks, and is presumably unable on principle to formulate, a positive conception of what should replace the present system. His
suggestions are either vague and naive (socialising cyberspace, for instance), reproduce capitalism (the necessity of betrayal),
or set up something worse (terror). Zizek's endorsement of "absolute negativity" is a barrier to his developing actual alternatives.
at lacan/zizek: “the act” fails

The alternative fails: Lacan under-develops the connection between individual psyches and universal understandings. They will
not be able to explain how one person thinking about the mysterious Act will change society.

Robinson (PhD Political Theory, University of Nottingham) 05 (Theory and Event, Andrew, 8:1, The Political Theory of
Constitutive Lack: A Critique).

Lacanian analysis consists mainly of an exercise in projection.  As a result, Lacanian "explanations" often look more
propagandistic or pedagogical than explanatory.  A particular case is dealt with only in order to, and to the extent that it can,
confirm the already-formulated structural theory .  Judith Butler criticizes Žižek's method on the grounds that 'theory is applied to its examples', as if
'already true, prior to its exemplification'.  'The theory is articulated on its self-sufficiency, and then shifts register only for the pedagogical purpose of illustrating
an already accomplished truth'.  It is therefore 'a theoretical fetish that disavows the conditions of its own emergence'52.  She alleges that Lacanian
psychoanalysis 'becomes a theological project' and also 'a way to avoid the rather messy psychic and social entanglement' involved in studying specific
cases53.  Similarly, Dominick LaCapra objects to the idea of constitutive lack because specific 'losses cannot be adequately addressed when they are enveloped
in an overly generalised discourse of absence... Conversely, absence at a "foundational" level cannot simply be derived from particular historical losses'54. 
Attacking 'the long story of conflating absence with loss that becomes constitutive instead of historical'55, he accuses several theorists of eliding the difference
between absence and loss, with 'confusing and dubious results', including a 'tendency to avoid addressing historical problems, including losses, in sufficiently
specific terms', and a tendency to 'enshroud, perhaps even to etherealise, them in a generalised discourse of absence'56.  Daniel Bensaïd draws out the political
consequences of the projection of absolutes into politics.  'The fetishism of the absolute event involves... a suppression of historical intelligibility, necessary to its
depoliticization'.  The space from which politics is evacuated 'becomes... a suitable place for abstractions, delusions and hypostases'. 
Instead of actual social forces, there are 'shadows and spectres'. The operation of the logic of projection is predictable.  According to Lacanians, there is a basic
structure (sometimes called a 'ground' or 'matrix') from which all social phenomena arise, and this structure, which remains unchanged in all eventualities, is the
reference-point from which particular cases are viewed.  The "fit" between theory and evidence is constructed monologically by the reduction of the latter to the
former, or by selectivity in inclusion and reading of examples .  At its simplest, the Lacanian myth functions by a short-circuit between a
particular instance and statements containing words such as "all", "always", "never", "necessity" and so on.  A contingent
example or a generic reference to "experience" is used, misleadingly, to found a claim with supposed universal validity.  For
instance, Stavrakakis uses the fact that existing belief-systems are based on exclusions as a basis to claim that all belief-
systems are necessarily based on exclusions58, and claims that particular traumas express an 'ultimate impossibility'59. 
Similarly, Laclau and Mouffe use the fact that a particular antagonism can disrupt a particular fixed identity to claim that the social
as such is penetrated and constituted by antagonism as such60.  Phenomena are often analysed as outgrowths of something
exterior to the situation in question.  For instance, Žižek's concept of the "social symptom" depends on a reduction of the acts of
one particular series of people (the "socially excluded", "fundamentalists", Serbian paramilitaries, etc.) to a psychological function
in the psyche of a different group (westerners).  The "real" is a supposedly self-identical principle which is used to reduce any
and all qualitative differences between situations to a relation of formal equivalence.  This shows how mythical characteristics
can be projected from the outside, although it also raises different problems: the under-conceptualization of the relationship
between individual psyches and collective phenomena in Lacanian theory, and a related tendency for psychological concepts to
acquire an ersatz agency similar to that of a Marxian fetish.  "The Real" or "antagonism" occurs in phrases which have it doing or
causing something. As Barthes shows, myth offers the psychological benefits of empiricism without the epistemological costs. 
at lacan/zizek: “the act” fails

Zizek’s alternative is authoritarian—his concept of human nature requires political domination.

Robinson 04 (Andrew, PhD, political theory, University of Nottingham, “Introduction: The Basic Zizekian Model,” Theory Blog,
http://andyrobinsontheoryblog.blogspot.com/2004/11/zizek-notes-and-work-in-progress_15.html)

The Act also seems to be authoritarian in the sense that it involves an unfounded imposition of will which reshapes the symbolic
edifice. Perhaps even worse is Zizek's conception of human nature. Zizek thinks people are basically too chaotic to live without
rulers, repeating the claims of the likes of Hobbes. He sees 'unruliness' and going to the end beyond every human measure as a
primordial drive and part of human nature - a drive ethics tries to contain - a drive involving "clinging to wild egotistical freedom
unbound by any constraints" which "has to be broken and 'gentrified' by the pressure of education" (PF 236-7). Humanity is as
such unnaturally prone to excess, and has to be gentrified through institutions (PF 135). There is a basic drive to dis-attach from
the world which fantasy is a protection against (TS 289). The role of paternal Law is to expose people to the harsh demands of
social reality, demands which lead to entry into desire (FA 76; Zizek is presumably some kind of expectationist). He even seems
to endorse Kant's view that people need a Master and (hierarchic) discipline to tame their 'unruly' insistence on their own will and
force them to submit to being placed in subjection to "the laws of mankind and brought to feel their constraint" (TS 36 - clearly a
substitutionist term). So Zizek endorses Kant's work on education, where he claims the role of schools is not for children to learn
but to accustom them "to sitting still and doing exactly what they are told", to "counteract man's natural unruliness" (TS 36)!
(Zizek also conflates social control with the unrelated issue of "venturing wildly and rashly into danger" in this discussion of Kant).
Once accustomed to freedom, one will do anything for it, so this urge must be "smoothed down" (TS 36). Zizek calls this text of
Kant's a "marvellous text" (TS 36). He also makes the (apparently contradictory with all the above, but equally conservative)
claim that "a human being is... in need of firm roots" and that this basic need is the root of the symbolic order (CHU 250). On the
whole Zizek seems to be endorsing a conservative or even reactionary view of human nature; though this is not entirely clear.
at lacan/zizek “the act” fails

Zizek’s political stance is violent and feeds into power. The notion of the “Act” has so few limits that there is nothing to prevent
elites from deploying it to violently maintain power.

Robinson (PhD Political Theory, University of Nottingham) 05 (Theory and Event, Andrew, 8:1, The Political Theory of
Constitutive Lack: A Critique).
    
On a political level, this kind of stance leads to an acceptance of social exclusion which negates compassion for its victims.  The
resultant inhumanity finds its most extreme expression in Žižek's work, where 'today's "mad dance", the dynamic proliferation of
multiple shifting identities... awaits its resolution in a new form of Terror'.  It is also present, however, in the toned-down
exclusionism of authors such as Mouffe.  Hence, democracy depends on 'the possibility of drawing a frontier between "us" and
"them"', and 'always entails relations of inclusion-exclusion'28.  'No state or political order... can exist without some form of
exclusion' experienced by its victims as coercion and violence29, and, since Mouffe assumes a state to be necessary, this
means that one must endorse exclusion and violence.  (The supposed necessity of the state is derived from the supposed need
for a master-signifier or nodal point to stabilize identity and avoid psychosis, either for individuals or for societies).  What is at
stake in the division between these two trends in Lacanian political theory is akin to the distinction Vaneigem draws between
"active" and "passive" nihilism30.  The Laclauian trend involves an implied ironic distance from any specific project, which
maintains awareness of its contingency; overall, however, it reinforces conformity by insisting on an institutional mediation which
overcodes all the "articulations".  The Žižekian version is committed to a more violent and passionate affirmation of negativity,
but one which ultimately changes very little.  The function of the Žižekian "Act" is to dissolve the self, producing a historical
event.  "After the revolution", however, everything stays much the same.  For all its radical pretensions, Žižek's politics can be
summed up in his attitude to neo-liberalism: 'If it works, why not try a dose of it?'31.  The phenomena which are denounced in
Lacanian theory are invariably readmitted in its "small print", and this leads to a theory which renounces both effectiveness and
political radicalism. It is in this pragmatism that the ambiguity of Lacanian political theory resides, for, while on a theoretical level
it is based on an almost sectarian "radicalism", denouncing everything that exists for its complicity in illusions and guilt for the
present, its "alternative" is little different from what it condemns (the assumption apparently being that the "symbolic" change in
the psychological coordinates of attachments in reality is directly effective, a claim assumed – wrongly – to follow from the claim
that social reality is constructed discursively).  Just like in the process of psychoanalytic cure, nothing actually changes on the
level of specific characteristics.  The only change is in how one relates to the characteristics , a process Žižek terms 'dotting the "i's"' in reality,
recognizing and thereby installing necessity32.  All that changes, in other words, is the interpretation: as long as they are reconceived as expressions of constitutive lack, the old politics are acceptable.   Thus, Žižek
claims that de Gaulle's "Act" succeeded by allowing him 'effectively to realize the necessary pragmatic measures' which others pursued unsuccessfully33.  More recent examples of Žižek's pragmatism include that
his alternative to the U.S. war in Afghanistan is only that 'the punishment of those responsible' should be done in a spirit of 'sad duty', not 'exhilarating retaliation'34, and his "solution" to the Palestine-Israel crisis,
which is NATO control of the occupied territories35.  If this is the case for Žižek, the ultra-"radical" "Marxist-Leninist" Lacanian, it is so much the more so for his more moderate adversaries.  Jason Glynos, for
instance, offers an uncompromizing critique of the construction of guilt and innocence in anti-"crime" rhetoric, demanding that demonization of deviants be abandoned, only to insist as an afterthought that, 'of course,
.  Lacanian theory tends, therefore, to produce an "anything goes" attitude to state action:
this... does not mean that their offences should go unpunished'36
because everything else is contingent, nothing is to limit the practical consideration of tactics by dominant elites.  
at lacan/zizek: revolution fails

Zizek offers no clear alternative—capitalism is inevitable.

Robinson 04 (Andrew, PhD, political theory, University of Nottingham, “Introduction: The Basic Zizekian Model,” Theory Blog,
http://andyrobinsontheoryblog.blogspot.com/2004/11/zizek-notes-and-work-in-progress_15.html)

It is by no means clear that Zizek thinks alternatives to capitalism are possible, or that he wants them. He seems to want to
destroy capitalism, on his definition of it (see CAPITALISM, CONSERVATISM), which sets up a rather conservative target
(liberalism, permissiveness, decadence, 'flabbiness', etc.). It is less clear that he wants to destroy it by any other criterion: he
endorses work ethics and authoritarianism, and he has posited so much of the deep structure of society as unchangeable as to
render the space for change highly limited. Laclau attacks Zizek on this subject. Despite "r-r-revolutionary zeal", Zizek is no more
proposing a thoroughly different economic and political regime than Laclau. Zizek lets us know nothing about his alternative,
Laclau says (actually, this is not strictly true, though he does tell us very little); he only tells us that it isn't liberal democracy or
capitalism. Laclau is concerned it could mean Stalinism, despite Zizek's earlier resistance against this (NB Zizek dislikes late,
post-Stalin Stalinism with a human face, but distinguishes this from the earlier Stalinism - what he resisted was the former);
Laclau suspects Zizek simply doesn't know what his alternative is (CHU 289). How does Zizek respond to this? He uses it to
pathologise Laclau, claiming he cannot imagine an alternative and so thinks there isn't one (which Laclau actually never states).
at lacan/zizek: revolution fails

Zizek’s alternative fails—he claims that capitalism must be over thrown but has no means of accomplishing this goal.

Boynton, 98 (Director of NYU's Graduate Magazine Journalism Program, Robert, "Enjoy Your Žižek!" Lingua Franca, October,
http://www.robertboynton.com/articleDisplay.php?article_id=43)

"Authentic politics is the art of the impossible," he writes. "It changes the very parameters of what is considered "possible' in the
existing constellation." This is a noble vision, but when Zizek turns to history, he finds only fleeting examples of genuine politics
in action: in ancient Athens; in the proclamations of the Third Estate during the French Revolution; in the Polish Solidarity movement; and in the last, heady days of the East German Republic before the Wall
came down and the crowds stopped chanting "Wir sind das Volk" ("We are the people!") and began chanting "Wir sind ein Volk" ("We are a/one people!"). The shift from definite to indefinite article, writes Zizek,
marked "the closure of the momentary authentic political opening, the reappropriation of the democratic impetus by the thrust towards reunification of Germany, which meant rejoining Western Germany's liberal-
capitalist police/political order." In articulating his political credo, Zizek attempts to synthesize three unlikely–perhaps incompatible–sources: Lacan's notion of the subject as a "pure void" that is "radically out of joint"
with the world, Marx's political economy, and St. Paul's conviction that universal truth is the only force capable of recognizing the needs of the particular. Zizek is fond of calling himself a "Pauline materialist," and he
admires St. Paul's muscular vision. He believes that the post-political deadlock can be broken only by a gesture that undermines "capitalist globalization from the standpoint of universal truth in the same way that
. AS
Pauline Christianity did to the Roman global empire." He adds: "My dream is to combine an extremely dark, pessimistic belief that life is basically horrible and contingent, with a revolutionary social attitude
PHILOSOPHY, Zizek's argument is breathtaking, but as social prescription, "dream" may be an apt word. The only way to
combat the dominance of global capitalism, he argues, is through a "direct socialization of the productive process"–an agenda
that is unlikely to play well in Slovenia, which is now enjoying many of the fruits of Western consumer capitalism. When pressed
to specify what controlling the productive process might look like, Zizek admits he doesn't know, although he feels certain that an
alternative to capitalism will emerge and that the public debate must be opened up to include subjects like control over genetic
engineering. Like many who call for a return to the primacy of economics, Zizek has only the most tenuous grasp of the subject.

Zizek’s alternative is political nihilism – he supplies no method for over throwing capitalism.

Laclau 04 Ernesto Laclau, Professor of Political Theory at the University of Essex and Visiting Professor of Comparative
Literature at SUNY-Buffalo, 2004, Umbr(a): War, p. 33-34

Here we reach the crux of the difficulties to be found in Zizek. On the one hand, he is committed to a theory of
the full revolutionary act that would operate in its own name, without being invested in any object outside itself.
On the other hand, the capitalist system, as the dominating, underlying mechanism, is the reality with which
the emancipatory act has to break. The conclusion from both premises is that there is no valid emancipatory
struggle except one that is fully and directly anti-capitalist. In his words: “I believe in the central structuring role
of the anti-capitalist struggle.” The problem, however, is this: he gives no indication of what an anti-capitalist
struggle might be. Zizek quickly dismisses multicultural, anti-sexist, and anti-racist struggles as not being
directly anti-capitalist. Nor does he sanction the traditional aims of the Left, linked more directly to the
economy: the demands for higher wages, for industrial democracy, for control of the labor process, for a
progressive distribution of income, are not proposed as anti-capitalist either. Does he imagine that the Luddites’
proposal to destroy all the machines would bring an end to capitalism? Not a single line in Zizek’s work gives
an example of what he considers an anti-capitalist struggle. One is left wondering whether he is
anticipating an invasion of beings from another planet, or as he once suggested, some kind of ecological
catastrophe that would not transform the world but cause it to fall apart. So where has the whole argument
gone wrong? In its very premises. Since Zizek refuses to apply the hegemonic logic to strategico-political
thought, he is stranded in a blind alley. He has to dismiss all “partial” struggles as internal to the “system”
(whatever that means), and the “Thing” being unachievable, he is left without any concrete historical actor
for his anti-capitalist struggle. Conclusion: Zizek cannot provide any theory of the emancipatory subject.
At the same time, since his systemic totality, being a ground, is regulated exclusively by its own internal
laws, the only option is to wait for these laws to produce the totality of its effects. Ergo: political nihilism.
at lacan/zizek: does not apply to aff

It does not make sense to have a fixed understanding of the political that gets applied to all situations. They will have persuasive
descriptions of Lacanian theory but nothing that applies it to our aff.

Robinson (PhD Political Theory, University of Nottingham) 05 (Theory and Event, Andrew, 8:1, The Political Theory of
Constitutive Lack: A Critique).

One of the functions of myth is to cut out what Trevor Pateman terms the "middle level" of analytical concepts, establishing a
short-circuit between high-level generalizations and ultra-specific (pseudo-) concrete instances.  In Barthes's classic case of an
image of a black soldier saluting the French flag, this individual action is implicitly connected to highly abstract concepts such as
nationalism, without the mediation of the particularities of his situation.  (These particularities, if revealed, could undermine the
myth.  Perhaps he enlisted for financial reasons, or due to threats of violence).  Thus, while myths provide an analysis of sorts,
their basic operation is anti-analytical: the analytical schema is fixed in advance, and the relationship between this schema and
the instances it organizes is hierarchically ordered to the exclusive advantage of the former.   This is precisely what happens in
Lacanian analyses of specific political and cultural phenomena.  Žižek specifically advocates 'sweeping generalizations' and
short-cuts between specific instances and high-level abstractions, evading the "middle level".  'The correct dialectical
procedure... can be best described as a direct jump from the singular to the universal, bypassing the mid-level of particularity'. 
He wants a 'direct jump from the singular to the universal', without reference to particular contexts. 

Prefer our specific solvency evidence over their generic theory. Saying that something that cannot be defined will come back to
haunt us is the ultimate hidden DA. We will defend against any specific case turn but cannot defend against a turn that they say
is beyond our grasp.

Robinson (PhD Political Theory, University of Nottingham) 05 (Theory and Event, Andrew, 8:1, The Political Theory of
Constitutive Lack: A Critique).
    
The technical term operates in much the same way as in positivistic theories, where the use of a noun turns a set of observed
"facts" into a "law".  Lack (in the sense of the verb "to lack") is explained by means of a nominalized lack (for instance, the failure
of society by the fact of antagonism), and the various versions of nominalized lack are arranged in sentences involving the verb
"to be".  It is not simply a relation of dislocation but a theoretical entity in its own right.   For instance, '"class struggle" is that on
account of which every direct reference to universality... is... "biased", dislocated with regard to its literal meaning.   "Class
struggle" is the Marxist name for this basic "operator of dislocation"'90.  One might compare this formula to the statement, "I don't
know what causes dislocation".  Žižek also refers to history 'as a series of ultimately failed attempts to deal with the same
"unhistorical", traumatic kernel’.  Dallmayr similarly writes of Laclau and Mouffe's concept of antagonism that 'negativity
designates not simply a lack but a "nihilating" potency', 'a nihilating ferment with real effects'92, and Newman writes of a 'creative
and constitutive absence'. Butler notes that 'the "real" that is a "rock" or a "kernel" or sometimes a "substance" is also, and
sometimes within the same sentence, "a loss", a "negativity"'94.  Constitutive lack is a positivity - an "operator of dislocation", a
"nihilating" element  - in the Lacanian vocabulary.  It is this process of mythical construction which allows lack to be defined
precisely, and which therefore meets (for instance) Newman's criterion that it be less 'radically underdefined' than Derrida's
concept of lack95.  One can only avoid an "I-don't-know" being underdefined if one misrepresents it mythically.
at lacan/zizek: non-falsifiable

Lacan’s explanation of the Real requires a leap of faith similar to religion. You are asked to believe in it because it ‘resists
symbolization’ or it is beyond our understanding. Yet, it is credited with determining all social conflicts.

Robinson (PhD Political Theory, University of Nottingham) 05 (Theory and Event, Andrew, 8:1, The Political Theory of
Constitutive Lack: A Critique).

This passage could almost have been written with the "Lacanian Real" in mind.  The characteristic of the Real is precisely that
one can invoke it without defining it (since it is "beyond symbolization"), and that the accidental failure of language, or indeed a
contingent failure in social praxis, is identified with an ontological resistance to symbolization projected into Being itself.  For
instance, Žižek's classification of the Nation as a Thing rests on the claim that 'the only way we can determine it is by... empty
tautology', and that it is a 'semantic void'63.  Similarly, he claims that 'the tautological gesture of the Master-Signifier', an empty
performative which retroactively turns presuppositions into conclusions, is necessary, and also that tautology is the only way
historical change can occur64.  He even declares constitutive lack (in this case, termed the "death drive") to be a tautology.
Lacanian references to "the Real" or "antagonism" as the cause of a contingent failure are reminiscent of Robert Teflon's
definition of God: 'an explanation which means "I have no explanation"'.  An "ethics of the Real" is a minor ethical salvation which
says very little in positive terms, but which can pose in macho terms as a "hard" acceptance of terrifying realities.  It authorizes
truth-claims - in Laclau's language, a 'reality' which is 'before our eyes67', or in Newman's, a 'harsh reality' hidden beneath a
protective veil68 - without the attendant risks.  Some Lacanian theorists also show indications of a commitment based on the
particular kind of "euphoric" enjoyment Barthes associates with myths.  Laclau in particular emphasizes his belief in the
'exhilarating' significance of the present69, hinting that he is committed to euphoric investments generated through the repetition
of the same.
at lacan/zizek: non-falsifiable

Their vision of politics is a non-falsifiable myth: The root of the Lacanian subject is structured around the “lack.” The problem is
that there is nothing to support this idea of a missing reality. The other team may be able to explain Lacan’s ideas but not why
they are true.

Robinson (PhD Political Theory, University of Nottingham) 05 (Theory and Event, Andrew, 8:1, The Political Theory of
Constitutive Lack: A Critique).

More precisely, I would maintain that "constitutive lack" is an instance of a Barthesian myth.  It is, after all, the function of myth to
do exactly what this concept does: to assert the empty facticity of a particular ideological schema while rejecting any need to
argue for its assumptions.  'Myth does not deny things; on the contrary, its function is to talk about them; simply, it purifies them,
it makes them innocent, it gives them a natural and eternal justification, it is a clarity which is not that of an explanation but that of
a statement of fact'.  This is precisely the status of "constitutive lack": a supposed fact which is supposed to operate above and
beyond explanation, on an ontological level instantly accessible to those with the courage to accept it.  Myths operate to
construct euphoric enjoyment for those who use them, but their operation is in conflict with the social context with which they
interact.  This is because their operation is connotative: they are "received" rather than "read" and open only to a "readerly" and
not a "writerly" interpretation.  A myth is a second-order signification attached to an already-constructed denotative sign, and the
ideological message projected into this sign is constructed outside the context of the signified.  A myth is therefore, in Alfred
Korzybski's sense, intensional: its meaning derives from a prior linguistic schema, not from interaction with the world in its
complexity.  Furthermore, myths have a repressive social function, carrying in Barthes's words an 'order not to think'.  They are
necessarily projected onto or imposed on actual people and events, under the cover of this order.  The "triumph of literature" in
the Dominici trial consists precisely in this projection of an externally-constructed mythical schema as a way of avoiding
engagement with something one does not understand. Lacanian theory, like Barthesian myths, involves a prior idea of a
structural matrix which is not open to change in the light of the instances to which it is applied.   Žižek's writes of a 'pre-ontological
dimension which precedes and eludes the construction of reality'42, while Laclau suggests there is a formal structure of any
chain of equivalences which necessitates the logic of hegemony43.  Specific analyses are referred back to this underlying
structure as its necessary expressions, without apparently being able to alter it; for instance, 'those who triggered the process of
democratization in eastern Europe... are not those who today enjoy its fruits, not because of a simple usurpation... but because
of a deeper structural logic'44.  In most instances, the mythical operation of the idea of "constitutive lack" is implicit, revealed only
by a rhetoric of denunciation. For instance, Mouffe accuses liberalism of an 'incapacity... to grasp... the irreducible character of
antagonism'45, while Žižek claims that a 'dimension' is 'lost' in Butler's work because of her failure to conceive of "trouble" as
constitutive of "gender"46.   This language of "denial" which is invoked to silence critics is a clear example of Barthes's "order not
to think": one is not to think about the idea of "constitutive lack", one is simply to "accept" it, under pain of invalidation.   If
someone else disagrees, s/he can simply be told that there is something crucial missing from her/his theory.  Indeed, critics are
as likely to be accused of being "dangerous" as to be accused of being wrong. 
at lacan/zizek: conservative politics

Turn: Lacanian criticism is analytically radical but breaks down into very conservative politics.

Robinson (PhD Political Theory, University of Nottingham) 05 (Theory and Event, Andrew, 8:1, The Political Theory of
Constitutive Lack: A Critique).

There is more than an accidental relationship between the mythical operation of the concept of "constitutive lack" and Lacanians'
conservative and pragmatist politics.  Myth is a way of reducing thought to the present: the isolated signs which are included in
the mythical gesture are thereby attached to extra-historical abstractions.  On an analytical level, Lacanian theory can be very
"radical", unscrupulously exposing the underlying relations and assumptions concealed beneath officially-sanctioned discourse. 
This radicalism, however, never translates into political conclusions: as shown above, a radical rejection of anti-"crime" rhetoric
turns into an endorsement of punishment, and a radical critique of neo-liberalism turns into a pragmatist endorsement of
structural adjustment.  It is as if there is a magical barrier between theory and politics which insulates the latter from the former. 
One should recall a remark once made by Wilhelm Reich: 'You plead for happiness in life, but security means more to you'133. 
Lacanians have a "radical" theory oriented towards happiness, but politically, their primary concern is security.  As long as they
are engaged in politically ineffectual critique, Lacanians will denounce and criticize the social system, but once it comes to
practical problems, the "order not to think" becomes operative. This "magic" barrier is the alibi function of myth.   The short-circuit
between specific instances and high-level abstractions is politically consequential. 

The deep negativity toward politics makes Lacanian analysis collapse into reactionary politics.

Robinson (PhD Political Theory, University of Nottingham) 05 (Theory and Event, Andrew, 8:1, The Political Theory of
Constitutive Lack: A Critique).

The political function of Lacanian theory is to preclude critique by encoding the present as myth. There is a danger of a stultifying
conservatism arising from within Lacanian political theory, echoing the 'terrifying conservatism' Deleuze suggests is active in any
reduction of history to negativity136.  The addition of an "always" to contemporary evils amounts to a "pessimism of the will", or a
"repressive reduction of thought to the present".  Stavrakakis, for instance, claims that attempts to find causes and thereby to
solve problems are always fantasmatic137, while Žižek states that an object which is perceived as blocking something does
nothing but materialize the already-operative constitutive lack138.  While this does not strictly entail the necessity of a
conservative attitude to the possibility of any specific reform, it creates a danger of discursive slippage and hostility to
"utopianism" which could have conservative consequences.  Even if Lacanians believe in surplus/contingent as well as
constitutive lack, there are no standards for distinguishing the two.  If one cannot tell which social blockages result from
constitutive lack and which are contingent, how can one know they are not all of the latter type?  And even if constitutive lack
exists, Lacanian theory runs a risk of "misdiagnoses" which have a neophobe or even reactionary effect.  To take an imagined
example, a Lacanian living in France in 1788 would probably conclude that democracy is a utopian fantasmatic ideal and would
settle for a pragmatic reinterpretation of the ancién regime.  Laclau and Mouffe's hostility to workers' councils and Žižek's
insistence on the need for a state and a Party139 exemplify this neophobe tendency.  The pervasive negativity and cynicism of
Lacanian theory offers little basis for constructive activity.   Instead of radical transformation, one is left with a pragmatics of
"containment" which involves a conservative de-problematization of the worst aspects of the status quo.   The inactivity it
counsels would make its claims a self-fulfilling prophecy by acting as a barrier to transformative activity.
at lacan/zizek: conservative politics

The alternative encourages oppressive social relations—the idea we are driven by joussiance is essentially a justification for
sadomasochism—including the attacking oneself or accepting totalitarianism as an authentic political act.

Robinson (PhD Political Theory, University of Nottingham) 05 (Theory and Event, Andrew, 8:1, The Political Theory of
Constitutive Lack: A Critique).
    
The "death instinct" is connected to an idea of primordial masochism which, in the form of "aphanisis" or "subjective destitution",
recurs throughout Lacanian political theory.  Žižek in particular advocates masochism, in the guise of "shooting at" or "beating"
oneself, as a radical gesture which reveals the essence of the self and breaks the constraints of an oppressive reality, although
the masochistic gesture is present in all Lacanian theorists.  The death instinct is typified by Žižek as a pathological (in the
Kantian sense), contingent attitude which finds satisfaction in the process of self-blockage109.  It is identical with the Lacanian
concept of jouissance or enjoyment.  For him, 'enjoyment (jouissance) is not to be equated with pleasure: enjoyment is precisely
"pleasure in unpleasure"; it designates the paradoxical satisfaction procured by a painful encounter with a Thing that perturbs the
equilibrium of the pleasure principle.  In other words, enjoyment is located "beyond the pleasure principle"'110.  It is also the core
of the self, since enjoyment is 'the only "substance" acknowledged by psychoanalysis', and 'the subject fully "exists" only through
enjoyment'111.  Primordial masochism is therefore central to the Lacanian concept of the Real, which depends on there being a
universal moment at which active desire - sometimes given the slightly misleading name of the "pleasure principle" - is
suspended, not for a greater or delayed pleasure, but out of a direct desire for unpleasure (i.e. a primary reactive desire). 
Furthermore, this reactive desire is supposed to be ontologically prior to active desire.  Dominick LaCapra offers a similar but
distinct critique to my own, claiming that Lacanian and similar theories induce a post-traumatic compulsion repetition or an
'endless, quasi-transcendental grieving that may be indistinguishable from interminable melancholy'. Reich has already provided
a rebuttal of "primordial masochism", which, paradoxically given Žižek's claims to radicalism, was denounced by orthodox
Freudians as communist propaganda.  In Reich's view, masochism operates as a relief at a lesser pain which operates as
armouring against anxiety about an underlying trauma113.  Regardless of what one thinks of Reich's specific account of the
origins of masochism, what is crucial is his critique of the idea of a death drive.  'Such hypotheses as are criticised here are often
only a sign of therapeutic failure.  For if one explains masochism by a death instinct, one confirms to the patient his [sic] alleged
will to suffer'.  Thus, Lacanian metaphysics conceal Lacanians' encouragement of a variety of neurosis complicit with oppressive
social realities.  Politically, the thesis of primordial masochism provides a mystifying cover for the social forces which cause and
benefit from the contingent emergence of masochistic attachments (i.e. sadistic power apparatuses).  One could compare this
remark to Butler's claim that Žižek 'defends the trauma of the real... over and against a different kind of threat'115
at lacan/zizek: conservative politics

Zizek’s alternative is pessimistic and authoritarian – his theory precludes a democratic politics.

Breger, 01 Assistant Professor of Germanic Studies at Indiana, 2001 (Claudia, Diacritics 31.1 (2001) 73-90, "The Leader's Two
Bodies: Slavoj Zizek's Postmodern Political Theology," project muse)

More than ten years later—after a decade of authoritarian rule, war, and genocide in former Yugoslavia—recent revolutionary
events in Serbia once more allow one to hope for a thorough democratization of the region. In a newspaper article evaluating the
uprising, however, Zizek warned that these hopes might be premature: while Milosevic could find his new role as "a Serbian
Jesus Christ," taking upon him all the "sins" committed by his people, Kostunica and his "democratic" nationalism might
represent "nothing but Milosevic in the 'normal' version, without the excess" [Zizek, "Gewalt"]. Zizek was not alone in warning
that the new government in Yugoslavia might not bring an end to Serbian nationalist politics. The pessimistic scenario Zizek
evoked on this occasion, however, was not simply the result of his evaluation of the current political constellation in Serbia.
Rather, the fantasy of the necessary return of the leader is connected to his political theory—a theory that does not allow for
more optimistic scenarios of democratization and the diminution of nationalism in society. My reading of Zizek's work thus argues
for a reevaluation of his theory in terms of its implicit authoritarian politics. The need for such a reevaluation is also suggested by
Laclau toward the end of his recent exchange with Judith Butler and Zizek when he admits that "the more our discussions
progressed, the more I realized that my sympathy for Zizek's politics was largely the result of a mirage" [Laclau, "Constructing
Universality" 292]. Laclau now criticizes Zizek's radical Marxist rhetoric by suggesting that he "wants to do away with liberal
democratic regimes" without specifying a political alternative [289], and describes Zizek's discourse as "schizophrenically split
between a highly sophisticated Lacanian analysis and an insufficiently deconstructed traditional Marxism" [205]. On [End Page
73] the other hand, he also problematizes Zizek's "psychoanalytic discourse" as "not truly political" [289]. My argument primarily
starts from this latter point: the antidemocratic—and, as I will argue, both antifeminist and anti-Semitic—moment of Zizek's theory
is to be located not only in the way he performs Marxism, but also in the way he performs Lacanian psychoanalysis. While, in
other words, Zizek's skepticism vis-à-vis democracy is obviously informed by, and inseparable from, Marxist critiques of "liberal,"
"representative" democracy, his failure to elaborate alternative visions of political change towards egalitarian and/or plural
scenarios of society cannot be explained solely by his Marxist perspective..
at lacan/zizek: essentialism turn

The alternative links to the critique: the Lacanian notion of a “constitutive element” that is at the root of all political fantasy is just
as essentialist as they claim the affirmative to be. In other words, Lacan repeats the error of trying to create what he critiques.

Robinson (PhD Political Theory, University of Nottingham) 05 (Theory and Event, Andrew, 8:1, The Political Theory of
Constitutive Lack: A Critique).

Lacanians assume that the idea of a founding negativity is not essentialist, whereas any idea of an autonomous positive or
affirmative force, even if constructed as active, undefinable, changing and/or incomplete, is essentialist. The reason Lacanians
can claim to be "anti-essentialist" is that there is a radical rupture between the form and content of Lacanian theory.   The
"acceptance of contingency" constructed around the idea of "constitutive lack" is a closing, not an opening, gesture, and is itself
"essentialist" and non-contingent.  Many Lacanian claims are not at all contingent, but are posited as ahistorical absolutes.  To
take an instance from Mouffe's work, 'power and antagonism' are supposed to have an 'ineradicable character' so that 'any social
objectivity is constituted through acts of power' and will show traces of exclusions.  One could hardly find a clearer example
anywhere of a claim about a fixed basic structure of Being.  One could also note again the frequency of words such as "all" and
"always" in the Lacanian vocabulary.  Ludwig Wittgenstein argues that 'if someone wished to say: "There is something common
to all these constructions - namely the disjunction of all their common properties" - I should reply: Now you are only playing with
words'77.  Lacanian theory seems, indeed, to be treating disjunction as a basis for similarity, thus simply "playing with
words"."contingency" embraced in Lacanian theory is not an openness which exceeds specifiable positivities, but a positivity
posing as negativity.  The relationship between contingency and "constitutive lack" is like the relationship between Germans and
"Germanness", or tables and "tableness", in the work of Barthes.  One could speak, therefore, of a "lack-ness" or a "contingency-
ness" or an "antagonism-ness" in Lacanian political theory, and of this theory as a claim to fullness with this reified "lack-ness" as
one of the positive elements within the fullness.  One sometimes finds direct instances of such mythical vocabulary, as for
instance when Stavrakakis demands acknowledgement of 'event-ness and negativity'78.  Indeed, it is an especially closed
variety of fullness, with core ideas posited as unquestionable dogmas and the entire structure virtually immune to falsification.
at lacan/zizek: essentialism turn

Their alternative links to the critique. Their claim is that the aff is a quest for a new-master signifier that will fail because of the
inevitable re-emergence of the Real. However, that ontological statement about humanity is itself, a new master-signifier.

Robinson (PhD Political Theory, University of Nottingham) 05 (Theory and Event, Andrew, 8:1, The Political Theory of
Constitutive Lack: A Critique).

The gap between the two kinds of contingency is also suggested by the Lacanian insistence on the "need" for a master-signifier
(or "nodal point"), i.e. a particular signifier which fills the position of universality, a 'symbolic injunction which relies only on its own
act of enunciation'116.  It is through such a gesture that one establishes a logic of sameness, and such a logic seems to be
desired by Lacanians.  Butler remarks that Žižek's text is a 'project of mastery' and a discourse of the law in which 'the
"contingency" of language is mastered in and by a textual practice which speaks as the law'. He demands a '"New Harmony",
sustained by a newly emerged Master-Signifier'.  This insistence on a master-signifier is an anti-contingent gesture, especially in its rejection of the multiordinality of language.  It is,
after all, this multiordinality (the possibility of making a statement about any other statement) which renders language an open rather than a closed system.  The "need" for a master-signifier seems to be a "need" to
restore an illusion of closure, the "need" for metacommunication to operate in a repressive rather than an open way.  This "need" arises because the mythical concept of "constitutive lack" is located in an entire
mythical narrative in which it relates to other abstractions.  In the work of Laclau and Mouffe, this expresses itself in the demand for a "hegemonic" agent who contingently expresses the idea of social order "as
such". One should recall that such an order is impossible, since antagonism is constitutive of social relations, and that the
hegemonic gesture therefore requires an exclusion.  Thus, the establishment of a hegemonic master-signifier is merely a useful
illusion.  The alternative to demanding a master-signifier - an illusion of order where there is none - would be to reject the pursuit
of the ordering function itself, and to embrace a "rhizomatic" politics which goes beyond this pursuit.  In Laclau and Mouffe's
work, however, the "need" for a social order, and a state to embody it, is never questioned, and, even in Zizek's texts, the "Act"
which smashes the social order is to be followed by a necessary restoration of order118.  This necessity is derived ontologically:
people are, says Žižek, 'in need of firm roots'119.  The tautological gesture of establishing a master-signifier by restrospectively positing conditions of an object as its components,
thereby 'blocking any further inquiry into the social meaning' of what it quilts (i.e. repressive metacommunication), is a structural necessity120.  This is because 'discourse itself is in its fundamental structure
"authoritarian"'.  The role of the analyst is not to challenge the place of the master, but to occupy it in such a way as to expose its underlying contingency121.  The master-signifier, also termed the One, demonstrates
the centrality of a logic of place in Lacanian theory.  Lacanians assume that constitutive lack necessitates the construction of a positive space which a particular agent can fill (albeit contingently), which embodies the
emptiness/negativity as such.  Therefore, the commitment to master-signifiers and the state involves a continuation of an essentialist image of positivity, with "lack" operating structurally as the master-signifier of
). The idea of "constitutive lack" is supposed to entail a rejection of neutral
Lacanian theory itself (not as a subversion of positivity, but as a particular positive element
and universal standpoints, and it is this rejection which constructs it as an "anti-essentialist" position.   In practice, however,
Lacanians restore the idea of a universal framework through the backdoor.   Beneath the idea that "there is no neutral
universality" lurks a claim to know precisely such a "neutral universality" and to claim a privileged position on this basis.   A
consistent belief in contingency and "anti-essentialism" entails scepticism about the idea of constitutive lack.  After all, how does
one know that the appearance that 'experience' shows lack to be constitutive reflects an underlying universality, as opposed to
the contingent or even simulated effects of a particular discourse or episteme?  Alongside its opponents, shouldn't Lacanian
theory also be haunted by its own fallibility and incompletion?  There is a paradox in the idea of radical choice, for it is unclear
whether Lacanians believe this should be applied reflexively.  Is the choice of Lacanian theory itself an ungrounded Decision?  If
so, the theory loses the universalist status it implicitly claims.   If not, it would seem to be the kind of structural theory it attacks .  A
complete structural theory would seem to assume an extra-contingent standpoint, even if the structure includes a reference to
constitutive lack.  Such a theory would seem to be a radical negation of the incompletion of "I don't know".
at lacan/zizek: 9/11 arguments bad

Zizek’s arguments about 9/11 fail to recognize that the shock of the attack was not in its simulated nature but in its impact on real
people. We need to focus on the actual effects of violence and how to redress it.

Crosswhite, 01 (Associate Professor of English at University of Oregon, 2001 Jim, “A Response to Slavoj Zizek's "Welcome to
the Desert of the Real!",” September 25, http://www.uoregon.edu/~jcross/response_to_zizek.htm)

But to say that what happened on September 11 is like the scene in the Matrix where Morpheus introduces the Keanu Reeves
character to the "desert of the real" is to say something that belongs on a Fox Network talk show. For what Americans is it true that the events of
September 11 broke into an "insulated artificial universe" that generated an image of a diabolical outsider? Let's not consider the 5,000 incinerated and dismembered men and women and children who suffered from
disease and injury like all people, who cleaned toilets and coughed up phlegm and changed diapers and actually occupied with what was once their real bodies those towers which, for Zizek, stand for virtual
capitalism. They can't be the ones whose delusions generated the fantasy of a diabolical outsider. None of them, none of their surviving children, none of their fellow citizens fantasized Bin Laden's ruling that it is an
individual duty for every Muslim who can do it in any country in which it is possible to do it, to kill the Americans, military and civilians. So for whom has the fantastic "outside" broken in and smashed, with "shattering
impact," an immaterial world of delusion? For whom does Osama Bin Laden appear as a character from a James Bond film? For whom did the events of September 11 arrive with the painful awareness that we were
living in an artificial insulated reality? For whom do the people and events in this massacre of innocents appear solely in the shapes of film and television? Perhaps, perhaps the Americans living in an insulated,
artificial reality are the characters in American television shows and in increasingly intertextual American films. Perhaps these are the Americans Zizek is listening to, watching, imagining. But here is the true
"shattering impact:" that 5,000 innocent people who lived real lives in real, vulnerable human bodies, who bore real children, suffered real disease and injury and
pain, bled real blood; 5,000 real people who helped to sustain a cosmopolitan city of millions and millions of other real people of different ethnic groups and religions and languages, real citizens who had achieved a
great measure of peace and hope, who had been slowly and successfully bringing down the New York City crime rate; that 5,000 of these people would have their real bodies and lives erased in a matter of minutes,
This is the shock. This is the disbelief. Not the
and that only body parts, the vapors of the incinerated, and the grieving and the sorrowful and the orphans would remain.
shattering of an illusion but the shattering of those real people and their real bodies. Not the shattering of a virtual reality, but the erasing of what was real. This
is why the people of New York wept in the streets, why the tears and grief will continue. And this is why, in their grief, the survivors will struggle to preserve a memory of what was real, and to keep this memory of
what was real from evanescing into someone else's symbol, or fantasy, or tool. Were the real lives they led less real for any happiness or peace they achieved? Are the unfathomable sufferings of Rwanda and what
happened in Sarajevo to be the measure of what is most real? And yet in Zizek's writing, what happened on September 11 is not real but symbolic, as it seems to have been for the murderers, too: "the actual effect
of these bombings is much more symbolic than real." We are just "getting a taste of" what goes on around the world "on a daily basis." OK, perhaps we are insulated and ignorant. But where are 5,000 innocents
being incinerated by murderers on a daily basis? If Zizek is saying that Americans should be more knowledgeable about the lives and sufferings of other peoples whose lives and sufferings are entangled with
America's own history, then who would disagree? If Zizek is saying that American power and its direct involvement in international affairs create a special responsibility for our educational systems and our media to
provide us with a knowledge of global matters that we have not yet achieved, then who would disagree? If he is saying that Americans should comprehend more deeply how people in other parts of the world
comprehend us, once more, who would disagree? If he is saying that real understanding of geographically distant others is endangered and distorted by the fantasies of film and television, are there educated
Americans who have not heard this? Is the struggle to educate a democratic citizenship adequate to our time and the realities of globalization unique to the United States? That would be hard to believe. However, it
must be conceded by all that the U.S. faces one special difficulty and so a special but obligatory struggle here. Many of its citizens will never have a first hand experience of Europe or the Middle East or Africa or Asia
or even South America. I can drive or fly 3,000 miles and never leave my country. At best, I can get to Mexico or Canada. This would take someone living in France through all of Europe and into central Asia, or into
Zizek
the center of Africa. The problems of truly comprehending these others whose languages are rarely spoken anywhere near you and into whose actual presence you will never come are not trivial. But
seems to be saying something more than all of this. He seems to know more than most of us know. He knows that "the ultimate truth of the capitalist
utilitarian de-spiritualized universe is the de-materialization of the 'real life' itself, its reversal into a spectral show." This is difficult to comprehend. Is this the "ultimate truth" about a real nation, about real people, about
a real, existing economic system, about an ethical theory, about a fantasy of real people, or about movies or television or what? The problem may be that many of us cannot imagine that "capitalism" (is it one thing?),
it is difficult to understand what he is asking at the end: "Or will America finally risk
which is after all something historical, has an "ultimate truth." And
stepping through the fantasmatic screen separating it from the Outside World, accepting its arrival into the Real world, making
the long-overdue move from 'A thing like this should not happen HERE!' to 'A thing like this should not happen ANYWHERE!'." Of
course, to abandon the "here" for the "anywhere" would be foolish. We are in real bodies in real places with real limitations and
with real work to do. It is not simply a "fantasmatic screen" that deeply attaches people in a unique way to the sufferings of their neighbors and their fellow citizens. But the
demand that Zizek makes is neither unfamiliar nor inappropriate. It is more than worth pursuing. What can we do to work to see that what the people of New York City suffered on September 11 does not happen
anywhere, neither in the U.S. nor anywhere else? The reactions of the American government now threaten regions all over the world and seriously threaten liberty and privacy and tolerance in the United States. The
Certainly we must struggle to sustain serious
American past carries humanitarian successes and catastrophic failures and genocide. Perhaps fantastic critique has a role to play.
social criticism through threatening times, but unless we are simply displaying critical virtuosity, we must achieve a kind of
criticism that is reasonably concrete, less pretending to ultimate truths of history, more capable of acknowledging the real
suffering of real people, criticism that is not too proud to descend to the practicable. What do we seek now? First, to avert a catastrophe. We must undo the
terrorist networks and prevent American anger and power from leading us into the catastrophic roles that seem to have been scripted for us. Five thousand innocents are murdered in New York City. That is more
Averting an escalation of global violence is the immediate and
than enough. Every dead innocent fuels more anger, either from the powerless or from the powerful.
pressing task. Undoing and weakening the terrorist networks, withdrawing support from them, arresting the guilty—everyone who
is not already a monster must be persuaded to join in this.
at lacan/zizek: no connection alt and ballot

There is no connection between the ballot and the alterative—Zizek argues that the alternative cannot be consciously brought
about—it can only be recognized in hindsight.

Tell, 04 Communication Arts and Sciences at The Pennsylvania State University, 2004
(David, “On Belief (Review),” Philosophy and Rhetoric 37.1 (2004) 96-99, Project MUSE)

Most scholars of rhetoric, however, will not be satisfied with Zizek's belief. For although this belief provides the necessary
subjective conditions for public intervention, it is difficult to imagine it being publicly deployed. This belief is, after all, radically
privatized; it is the internal repetition of a "primordial decision," or an "unconscious atemporal deed" (147). One must wonder
about the public possibilities of such a private (and subconscious) experience. Moreover, most rhetoricians may well be troubled
by Zizek's claim that all "acts proper"—acts of actual freedom—occur outside the symbolic order. Insofar as rhetoric can be
considered symbolic action, then, its action can never provide for innovative intervention into the public sphere. Zizek admits as
much in an endnote: "true acts of freedom are choices/decisions which we make while unaware of it—we never decide (in the
present tense); all of a sudden, we just take note of how we have already decided " (156n46). It is precisely here that the
rhetorician will not be satisfied: if Rorty marginalized the rhetorical purchase of [End Page 98] belief by banishing it to the private
sphere, Zizek does so by marginalizing rhetoric itself.
A2: Empire (Hardt+Negri)

Hardt and Negri’s abstract theory of revolution destroys any possibility for political change
Quinby ‘04 (Lee, Chair Distinguished Teaching in Humanities – Hobart and William Smith Colleges, Empire’s New Clothes, p. 233)
Demonstrating Empire’s millennial drift is a complicated undertaking, in no small part because of Hardt’s and
Negri’s tendency to say one thing and yet do another. For example, even though they explicitly claim a nonprophetic
stance by stating that they can see “only shadows of the figures that will animate our future” (205), much of what
actually animates the book is its prophetic vision of the nature and role of the militant, the poor, the nomad, the new
barbarian, and the multitude. In place of specific and concrete analysis—a hallmark of a genealogical approach—
they stamp their theory with messianic categories that diminish rather than expand our understanding of productive
and reproductive life. This contradiction is particularly noteworthy because Empire’s millennialism is what makes it
compelling. Millennial rhetoric stirs the imagination toward exhilarating poles of fear and hope, promising a culmi -
nating and righteous telos to those who adhere to its tenets of belief. It is hard not to be drawn in. A second
interrelated contradiction arises from the fact that Hardt and Negri specifically reject transcendence, making
numerous explicit claims for the immanence of their materialist approach , often drawing on Foucault to help make
their case. In their opening pages, for example, they “rule out” the “idea that order is dictated by a single power and
a single center of rationality transcendent to global forces” (3). Nevertheless, their recurrent appeals to certain
categories of thought cast their theoretical framework back into transcen dental molds integral to millennialism,
which is both totalizing and abstractionist in its history and basic formulation. Hardt and Negri reject political
action and representation, dooming their revolution Passavant ‘04 (Paul, Professor of Political Science –
Hobart and William Smith Colleges, Empire’s New Clothes, p. 115-6) What if there was a revolution? Inevitably,
there would be disagreements over whether there should be rights in addition to or other than those Hardt and Negri
claimed for the multitude. In practice, there would be struggles in which the rights claims Hardt and Negri have put
forward might be considered antagonistic to other rights that people believe are essential to justice. But even in the
unlikely event of total agreement on a total list of multitudinous rights, disagreements would arise whenever rights
claims were concretized in specific circumstances. Outcomes of these interpretive battles over what the rights of
the multitude oblige in given circumstances would position those interpreta tions that do not carry the day as not-
multitudinous, if not antimultitudinous. Where there is politics, there is representation, as “we” need to figure out
who “we” are and what our commitments mean in particular circumstances, circumstances that will be different
from those under which we committed ourselves previously. And where there is representation, there is difference
and exclusion, as Hardt and Negri know and fear. Hardt and Negri avoid the problematic of representation in order
to save the inclusiveness of the multitude, but they cannot do this without sacrificing the political significance of
the multitude. For the multitude to take on political significance, it will become rent by difference and repression.
But sacrificing the chaos of anything goes leads to the possibility that something (good?) can happen. There will be
exclusions and repressions involved in any one move toward justice. Rather than complacency or a false
universality, we must make what remains after an (necessarily) incomplete move toward justice the fuel for
subsequent moves toward justice, since total justice will elude our time.
A2: Empire (Hardt+Negri)

Turn- the Multitude would devastate the left, they favor laborer over environmentalist
ignoring and creating and fostering dogmatic dissent
Tampino, 2005 (Nicholas, Assistant Professor of Government at Hamilton College
http://muse.jhu.edu/journals/theory_and_event/v008/8.2tampio.html, “Can the Multitude Save the Left?”

The concept of the multitude, however, lays out a questionable and dangerous project for the Left. First,
the concept assumes that every significant disagreement within the Left can, in their words, melt
away. In Seattle, rank-and-file unionists joined environmentalists marching with green sea turtle puppets. This
event, for Hardt and Negri, signals an epochal shift in the relationship between these two groups. The magic of
Seattle was to show that these many grievances were not just a random, haphazard collection, a cacophony of
different voices, but a chorus that spoke in common against the global system.7 Enduring harmony between
unionists and environmentalists in the Pacific Northwest would, indeed, be magical. Since the
passage of the Endangered Species Act of 1973, these two groups have battled recurrently over the management of
natural resources in Oregon and Washington. The most famous confrontation, perhaps, occurred when loggers
burned spotted owls in effigy to protest environmental regulations protecting the animals' habitat. There are also
ongoing debates over how to regulate the salmon and pollock industries. Though some unionists undoubtedly favor
sustainable fishing and logging, there may be another explanation for why unionists marched alongside
environmentalists in Seattle. The American fishing industry, according to Elizabeth R. DeSombre and J. Samuel
Barkin, had an economic incentive to dispute the WTO decision vetoing the United States' prohibition of shrimp
imported from countries that do not use "turtle excluder devices" (TEDs) on their nets. 8 The U.S. law banning
shrimp from countries such as Mexico acted, in effect, as a tariff. Spokesmen for the fishing industry, in fact,
focused on the economic impact of importing cheap shrimp, not on the welfare of sea turtles. This does not mean
that unionists and environmentalists cannot collaborate on legislation or policy. It does suggest,
however, that conflict between environmentalists and unionists - over vision and strategy - may
persist after the events of 1999 . Hardt and Negri do not seem to value deep disagreement (rather than
deep diversity) within the Left. In the dispute between labor and environmentalism, Hardt and Negri seem
squarely on the side of labor. In Empire, they criticize as "primordial" and "romantic" environmentalist claims
about the sanctity of nature and biodiversity.9 But Hardt and Negri never discuss environmental politics
at length in Empire or Multitude.10 The second problem with the concept of the multitude, thus,
is the dogmatism it fosters in those facing dissent — in this case, about humanity's relationship to the
environment. In an interview, Negri says the following about critics of or obstacles to the multitude: " Any attempt
to stand in the way of this unification and the consequent recognition of common objectives is
reactionary, or, rather, expresses sectarian and inimical operations."11 What happens, though, when
unionists and environmentalists stand on opposite sides of the barricades? Is it productive, then, to use the language
of "sectarian and inimical operations"? It is odd that Hardt and Negri, who end Multitude with an appeal to James
Madison, do not see the intimate connection between liberty and faction. The concept of the multitude, in
short, seems more likely to harm the Left than to help it. The Left ought to engage in the
challenging, provisional, but necessary work of building coalitions rather than wait for a secular
Messiah.
A2: Empire (Hardt+Negri)

Alt fails
Angus, 04 (Ian Empire,Borders,Place: A Critique of Hardt and Negri’s Concept of Empire, Theory and Event, 2004,
http://muse.jhu.edu/journals/theory_and_event/v007/7.3angus.html)

Hardt and Negri think that we have arrived at this historical turning point, that classes (along with nations or
peoples) are no longer the subjects of history. Instead the individual has become such (or is in the process of
becoming such). This turning point gives rise to the formation of what they call the “multitude,” defined in terms of
the “totality of productive and creative subjectivities.” Why and how would this turning point occur? Hardt and
Negri’s texts are quite vague on these questions. They talk about the transition to “cognitive capitalism” or the
emergence of “immaterial production,” the new “networked” society or “deterritorialization.” They make reference
to Foucault’s propositions concerning the transition from the disciplinary society to the society of control.
Everything that has been said over the past thirty years, whether good or bad, depending on one’s viewpoint,
whether indisputable because platitudinous or strongly debatable, is thrown pell-mell into a great pot in preparation
for the future. A compendium of current fashions does not easily lead to conviction. The similarity to the theses
formulated by Manuel Castells concerning the “networked society” and to the ideas popularized by Jeremy Rifkin,
Robert B. Reich, and other American popularizers is such that one is entitled to pose the question: what is new and
important in all this hodgepodge of ideas? I will propose then another hypothesis to account for the invention of the
“multitude” in question. Our moment is one of defeat for the powerful social and political movements that shaped
the twentieth century (workers’, socialist, and national liberation movements). The loss of perspective that any
defeat involves leads to ephemeral unrest and the profusion of para-theoretical propositions that both legitimate that
unrest and give rise to the belief that it constitutes an “effective” means for “transforming the world” (even without
wanting to), in the good sense of the term moreover. One can only gradually solidify new formulations that are both
coherent and effective by distancing oneself from the past, rather than proposing a “remake” of it, and by effectively
integrating new realities produced by social evolution in all its dimensions. Such contributions, both debatable and
diverse, certainly exist. I do not include Hardt and Negri’s discourse among them. The propositions that Hardt and
Negri draw from their discourse on the “multitude” bear witness, even in their very formulation, to the impasse in
which they are trapped. The first of these propositions concerns democracy that, for the first time in history, is
supposedly on the verge of becoming a real possibility on the global scale. Moreover, the multitude is defined as the
“constitutive” force of democracy. This is a wonderfully naïve proposition. Are we moving in this direction?
Beyond a few superficial appearances (some elections here or there), which obviously satisfy the liberal powers
(particularly Washington), democracy—both necessary and possible—is in crisis. It is threatened with losing its
legitimacy to the advantage of religious or ethnic fundamentalisms (I do not consider the ethnocratic regimes of the
former Yugoslavia as democratic progress!). Do elections that overturn the power of one criminal gang (for
example, one in the service of the Russian autocracy) to replace it with another one (financed by the CIA!)
constitute progress for democracy or a manipulated farce? Is not the unfolding of the imperialist project for control
of the planet at the origin of the frontal attacks that are reducing basic democratic rights in the United States? Is not
the liberal consensus in Europe, around which the major political forces of right and left have united, in the process
of delegitimizing electoral procedures? Hardt and Negri are silent on all these questions.
A2: Empire (Hardt+Negri)

The Empire’s pursuit of total control creates contradictions and a paradox of power – the
unification within Empire is key to resistance.
Wolfe, 2001 (Alan, director of the Boisi Center for Religion and American Public Life at Boston College, “The Snake”, The New Republic
Online, October 4th, http://www.powells.com/review/2001_10_04)

But Hardt and Negri will have none of this talk of human nature, or use value, or labor power. Capital will exploit wherever and
whatever it can. With bio-power in command, our bodies are no longer irreducibly ours. Our bodies have instead turned against
themselves; they are the very instruments by which we are controlled by forces external to us. We therefore have to "recognize
our posthuman bodies and minds" and see ourselves "for the simians and cyborgs we are" before we can begin to unleash
whatever creative powers we may have left over. But all is not lost for us simians and cyborgs. Unlike the writers of the
Frankfurt School, who also emphasized the authoritarian character of contemporary capitalism, writers such as Gilles Deleuze
and FÈlix Guattari, who are the true intellectual heroes of Empire, recognize that efforts at total control create
contradictions of their own. Here, in prose that insults language, is how Hardt and Negri summarize what they have
understood: "The analysis of real subsumption, when this is understood as investing not only the economic or only the cultural
dimension of society but rather the social bios itself, and when it is attentive to the modalities of disciplinarity and/or control,
disrupts the linear and totalitarian figure of capitalist development." What this means is that under Empire there emerges
a "paradox of power" in which all elements of social life are unified, but the very act of
unification "reveals a new context, a new milieu of maximum plurality and uncontainable
singularization—a milieu of the event." Even when Empire seems to rule everywhere and over
everything, there are opportunities for resistance, if only those opportunities can be grasped and
seen.]

The negative’s alternative is wrong – resistance can only succeed by working within the
conditions of the Empire.
Foster, 2001 (John Bellamy, editor of Monthly Review and author of Marx’s Ecology: Materialism and Nature and The Vulnerable Planet,
“Imperialism and ‘Empire’” Monthly Review, http://www.monthlyreview.org/1201jbf.htm)

Empire, the name they give to this new world order, is a product of the struggle over sovereignty and constitutionalism at the
global level in an age in which a new global Jeffersonianism—the expansion of the U.S. constitutional form into the global realm
—has become possible. Local struggles against Empire are opposed by these authors, who believe that
the struggle now is simply over the form globalization will take—and the extent to which Empire
will live up to its promise of bringing to fruition “the global expansion of the internal U.S.
constitutional project” Their argument supports the efforts of the “multitude against Empire”—
that is, the struggle of the multitude to become an autonomous political subject—yet this can
only take place, they argue, within “the ontological conditions that Empire presents”
at hardt/negri: alternative  terrorism

Hardt and Negri’s alternative is an endorsement of terrorism – the multitude’s revolt against capitalism is empirically violent.

Balakrishnan, 2000 (Political Science Professor at University of Chicago and member of the editorial board of New
Left Review , Gopal, “Hardt and Negri’s Empire”, New Left Review, September-October, http://newleftreview.org/A2275)

In addition to having a career as an influential political philosopher, with widely–translated books on Spinoza and Marx to his
credit, Negri is a convicted terrorist. In 1979, the Italian government arrested Negri, at the time a political science professor at the
University of Padua, and accused him of being the secret brains behind the Red Brigades, the Italian version of the Weathermen
in the U.S. or the Baader–Meinhoff Gang in West Germany—left–wing groups that during the 1970s sought to overthrow
capitalism through campaigns of terrorist violence. Italian authorities believed that Negri himself planned the infamous 1979
kidnapping and murder of Aldo Moro, the leader of Italy’s Christian Democratic Party. Just before Aldo’s execution, his distraught
wife got a taunting phone call, telling her that her husband was about to die. The voice was allegedly Negri’s. Unable to build a
strong enough case to try the philosopher for murder, Italian authorities convicted him on lesser charges of “armed insurrection
against the state.” Negri’s theoretical work was in keeping with his terrorist activities. He had become the leading voice of Italy’s
ultra–Left by advancing an inventive reinterpretation of Marx’s Grundrisse that located the agent of social revolution not among
the industrial proletariat, largely co–opted as it was by capitalist wealth and bourgeois democratic freedoms, but among those
marginalized from economic and political life: the criminal, the part–time worker, the unemployed. These dispossessed souls,
Negri felt, would be far quicker to unleash the riotous confrontations with the state that he saw as necessary to destroying
capitalism

Hardt and Negri’s alternative justifies terrorist attacks.

Wolfe, 01 (Alan, director of the Boisi Center for Religion and American Public Life at Boston College, “The Snake”, The New
Republic Online, October 4th, http://www.powells.com/review/2001_10_04)

We cannot know, of course, whether Hardt and Negri, in the light of the recent atrocities at the World Trade Center and the
Pentagon, will want to change their minds about the progressive potential of Islamic fundamentalism. But their book gives no
grounds on which such attacks can be condemned. For if being against the West is the sine qua non of good and effective
protest, well, no one could accuse the murderers in New York and Washington of not being against Western hegemony. And if it
is true, as Hardt and Negri blithely claim, that efforts to find legitimate reasons for intervening in world affairs are only a
smokescreen for the exercise of hegemonic power, then the way is cleared for each and every illegitimate act of global
intervention, since in the postmodern world of this book no justifiable distinctions between good and evil acts can ever be made.
at hardt/negri: alternative  terrorism

Hardt and Negri are so eager to oppose capitalism that their alternative embraces terrorism and misogyny
Wolfe, 01 (Alan, director of the Boisi Center for Religion and American Public Life at Boston College, “The Snake”, The New
Republic Online, October 4th, http://www.powells.com/review/2001_10_04)

The authors of Empire see no reason to exclude explicit reactionaries, including religious fundamentalists, from the catalogue of
post-Fordist movements that they admire. Fundamentalists, they write, are often portrayed as anti-modernist, but this is Western
propaganda. "It is more accurate and more useful...to understand the various fundamentalism [sic] not as the re-creation of a
pre-modern world, but rather as a powerful refusal of the contemporary historical passage in course." Neglecting to mention the
Taliban's treatment of women, Hardt and Negri go out of their way to reassure readers of the genuinely subversive nature of the
Islamic version of fundamentalism. These movements are motivated not by nostalgic attempts to reconstruct the past, but by
"original thought." They are anti-Western, which means that they are anti-capitalist. Properly understood, they are postmodern
rather than premodern, since they engage in a refusal of Western hegemony, with the proviso that fundamentalism speaks to the
losers in the globalization project and postmodernism to the winners. Hardt and Negri even leave the impression that, if they had
to choose between the postmodernists in Western universities and the fundamentalists in Iran, they would prefer the latter: "The
losers in the process of globalization might indeed be the ones who give us the strongest indication of the transformation in
process."

Hardt and Negri support terrorists over democracy—it is political lunacy.


Balakrishnan, 2000 (Political Science Professor at University of Chicago and member of the editorial board of New
Left Review, Gopal, “Hardt and Negri’s Empire”, New Left Review, September-October, http://newleftreview.org/A2275)

Apolitical abstraction and wild–eyed utopianism, a terroristic approach to political argument, hatred for flesh and blood human
beings, nihilism: Empire is a poisonous brew of bad ideas. It belongs with Mein Kampf in the library of political madness. Do
Empire’s many fans really believe their own praise? Does Time really think it’s “smart” to call for the eradication of private
property, celebrate revolutionary violence, whitewash totalitarianism, and pour contempt on the genuine achievements of liberal
democracies and capitalist economics? Would Frederic Jameson like to give up his big salary at Duke? To ask such questions is
to answer them. The far left’s pleasure is in the adolescent thrill of perpetual rebellion. Too many who should know better refuse
to grow up. The ghost of Marx haunts us still. For all its infantilism, the kind of hatred Hardt and Negri express for our flawed but
decent democratic capitalist institutions—the best political and economic arrangements man has yet devised and the outcome of
centuries of difficult trial and error—is dangerous, especially since it’s so common in the university and media. It seems to
support Islamist revolutionary hopes, the increasingly violent anti–globalization movement, and kindred political lunacies.

Empire anarchist rhetoric which praises terrorism and totalitarianism.


Wolfe, 01 (Alan, director of the Boisi Center for Religion and American Public Life at Boston College,
“The Snake”, The New Republic Online, October 4th, http://www.powells.com/review/2001_10_04)

The anarchist flavor of Empire is conveyed most strikingly by its romanticization of violence. Although by now everyone knows
that there are terrorists in this world, there are no terrorists in Hardt and Negri's book. There are only people who are called
terrorists, "a crude conception and terminological reduction that is rooted in a police mentality." Terms such as "ethnic terrorists"
and "drug mafias" appear within quotation marks, as if no serious revolutionary could believe that there were such things.
"Totalitarianism" is another pure construct, simply an invention of cold war ideology, that has been used to "denounce the
destruction of the democratic sphere...." Certainly the term has little to do with actual life in the Soviet Union, which Hardt and
Negri describe as "a society criss-crossed by extremely strong instances of creativity and freedom."
at hardt/negri: alternative justifies holocaust

Hardt and Negri argue that the state as always bad and resistance movements are always good – this ignores a critical
distinction between democracy and totalitarianism that downplays the Holocaust.

Wolfe, 01 (Alan, director of the Boisi Center for Religion and American Public Life at Boston College, “The Snake”, The New
Republic Online, October 4th, http://www.powells.com/review/2001_10_04)

Negri, when not in prison, has been a political philosopher, and he is the author of numerous books, manifestos, and theses on
subjects ranging from Spinoza's metaphysics to the nature of insurgency under contemporary capitalism. In nearly all this work,
as in Empire, he invariably associates violence with states in the exercise of their power, never with opposition groups and their
tactics. For the latter, any action, no matter how insurrectionary, is justified. For the former, any action, no matter how peaceful,
is terrorism in disguise. From this warped perspective, all states are equally bad and all movements of opposition are equally
good. Only the working of such a myopia can help the reader to understand why the authors of Empire are incapable of
mustering any rigorous historical or moral consciousness of Nazism and its policy of Jewish extermination. In their view Nazism
is capitalism, and that is the end of the story. Nazi Germany, Hardt and Negri write, far from a unique excursion into human evil,
"is the ideal type of the transformation of modern sovereignty into national sovereignty and of its articulation into capitalist
form...." Since Nazism is merely normal capitalism — this point of view was once associated with the Frankfurt School, and it
survives almost nowhere outside the pages of this book — there is no reason to single out the Nazis or their sympathizers for
crimes against humanity. Astonishingly, Hardt and Negri are worse than neutral in their discussion of the Nazi period: they
actually heap praise on the ordinary Germans who supported the regime. The obedience of these citizens is called "exemplary"
in this book. The authors also celebrate "their military and civil valor in the service of the nation," before moving on to identify the
victims whom they valorously helped to send to Buchenwald as "communists, homosexuals, Gypsies, and others," the latter,
presumably, being the Jews (whom Hardt and Negri reserve for Auschwitz). I am not making this up. Lest anyone consider these
apologetics for Nazism a misreading of my own — how can good leftists, after all, engage in a downplaying of the Holocaust? —
Hardt and Negri twice acknowledge that they are completely fed up with the whole question of totalitarianism.
at hardt/negri: globalization good

Globalization increases world prosperity and freedom – Hardt and Negri provide no evidence to the contrary.
Balakrishnan, 2000 (Political Science Professor at University of Chicago and member of the editorial board of New Left
Review, Gopal, “Hardt and Negri’s Empire”, New Left Review, September-October, http://newleftreview.org/A2275)

Inseparable from the failure to think politically, Hardt and Negri, like the rioters endlessly disrupting World Trade Organization
meetings, offer no evidence to support their basic charge that economic globalization is causing wide–scale planetary misery.
Predictably, this past summer, as the G–8 meeting got underway in Genoa, Italy, the New York Times chose these two “joyful”
Communists to write a lengthy op–ed extolling the virtues of anti–globalization rioters. The truth about globalization is exactly the
reverse of what Hardt and Negri assert. Globalization is dramatically increasing world prosperity and freedom. As the
Economist’s John Micklethwait and Adrian Wooldridge point out, in the half century since the foundation of the General
Agreement on Tariffs and Trade (GATT), the world economy has expanded six–fold, in part because trade has increased 1,600
percent; nations open to trade grow nearly twice as fast as those that aren’t; and World Bank data show that during the past
decade of accelerated economic globalization, approximately 800 million people escaped poverty.]

Globalization allows for justice, solidarity, and democracy.


Gills –02 [Barry K, Chair of the World Historical Systems theory group of the International Studies Association and a faculty
affiliate of the Globalization Research Center of the University of Hawaii, “Democratizing Globalization and Globalizing
Democracy, May, Annals of the American Academy of Political and Social Science, May]

If there is global capitalism, then the system gives rise to and in fact requires fundamental counterparts, including global justice,
global solidarity, global democracy, and global citizenship, the last of these perhaps being especially significant. We need a
credible political theory of global democracy based on the new concept of global citizenship rather than merely a pragmatic
problem-solving approach. If democracy is a process of building countervailing powers, then the democratic theory we have at
present, which is based on countries and their domestic political order, must be transposed to the global level. To do so, we must
also elevate or transpose the classic enlightenment democratic ideals of equality, justice, solidarity (fraternity), and liberty to the
global level.

Globalization is inevitable and key to spreading democracy and community empowerment.


Gills –02 [Barry K, Chair of the World Historical Systems theory group of the International Studies Association and a faculty
affiliate of the Globalization Research Center of the University of Hawaii, “Democratizing Globalization and Globalizing
Democracy, May, Annals of the American Academy of Political and Social Science, May]

Thus, there is likewise a historical dialectic between globalization and democratization, a process that is unavoidable. I firmly
believe, on both historical and moral grounds, that this historical dialectic leads strongly, even inexorably, toward the practices
and theory of global democracy, that is, to the globalization of democracy and the democratization of globalization. Insofar as
neoliberal economic globalization has succeeded, it creates the conditions for further critical social responses that lead to
renewed struggles for democratic freedoms and participation by the ordinary people affected by these changes. In these
processes of renewed democratic struggles, we may expect to see continued efforts at self-government by many peoples and
also expanded representation. Globalization allows the transcending of old established and fixed territorial units and borders of
political representation, thus allowing a more territorially diffuse pattern of political community to emerge, and to do so globally.
This process deepens democracy by extending it to the global arena but moreover by also devolving power to self-constituting
communities seeking self-government and representation in the political order, whether this be on a local, national, regional, or
global level.
at hardt/negri: capitalism good

Capitalism is not perfect but is better than Hardt and Negri’s alternative—which would devolve into totalitarianism.
Balakrishnan, 2000 (Political Science Professor at University of Chicago and member of the editorial board of New Left
Review, Gopal, “Hardt and Negri’s Empire”, New Left Review, September-October, http://newleftreview.org/A2275)

Needless to say, economic globalization isn’t without its downside. As I’ve argued in these pages (see “Capitalism and the
Suicide of Culture,” February 2000), it can—there’s no necessity at work—amplify and disseminate some of the less attractive
aspects of today’s libertine culture. But on balance, as neoconservative sociologist Peter L. Berger has suggested, the empirical
evidence proves it far preferable to any alternative economic order we know of. It has profoundly diminished human suffering. If
Hardt and Negri’s depiction of global capitalism is mendacious, their hazy alternative to it—absolute democracy, open borders,
equal compensation—is apolitical utopian nonsense. How would such schemes actually work? Hardt and Negri never say. Do
they truly think that “annulling” private property and eliminating nations, if it were somehow possible, would be liberating?
Wouldn’t it lead to a totalitarian increase in political power, as in the old Soviet Union? But then Hardt and Negri seem to look
back fondly on Lenin and Stalin’s dark regime. “Cold war ideology called that society totalitarian,” they complain, “but in fact it
was a society criss–crossed by extremely strong instances of creativity and freedom, just as strong as the rhythms of economic
development and cultural modernization.” To which one can only respond: Have they never read a page of Solzhenitsyn?
Moreover, as filled with admiration as Hardt and Negri are toward the Soviet Union, they are contemptuous toward the decencies
and the humbleoften not so humble—freedoms of democratic capitalist societies.
at hardt/negri: at historical argument

Hardt and Negri’s historical examples are highly selective and poorly developed.
Petras, 01 (James, Professor of Sociology at Binghamton University, “Empire With Imperialism”, Rebellion: Petras Essays in
English, October 29, http://www.rebelion.org/petras/english/negri010102.htm)

The authors argue early on that the intellectual origins the U.S. revolution can be traced to Spinoza and Machiavelli. Rousseau
and Locke are given short shrift, despite their greater immediate relevance. Extended and tendencious discussions of
sovereignty are interspersed with reductionist assertions which collapse or omit numerous variations. For example, in their
discussion of totalitarianism and the nation-state they argue "If Nazi Germany is ideal type of the transformation of modern
sovereignty into national sovereignty and of the articulation in its capitalist form, Stalinist Russia is the ideal type of the
transmission of popular interest and the cruel logics that follow from it into a project of national modernization, mobilizing for its
own purposes the productive forces that yearn for liberation from capitalism" (p.110). I have quoted extensively in order to
illustrate the confused, illogical, unhistorical nature of the author's broad and vacuous generalizations. What empirical or
historical basis is there for claiming Nazi Germany is the "ideal type"? National sovereignty pre-existed the Nazis and continues
after its demise in non-totalitarian settings. If Stalin's Russian embodied "popular interest" why should anyone seek to be
liberated from it? "Cruel logic" of "popular interests" is stuff from the ancien regime - hardly the basis for orienting the "multitudes"
which the writers describe to be the new agencies for democratizing the world. The authors engage in what George Saboul once
referred to as the "vacuum cleaner" approach to history: a little of ancient history, a smattering of exegesis of elementary political
theory, a plus and minus evaluation of post-modernism, a celebration of U.S. constitutionalism, a brief synopsis of colonialism
and post-colonialism. These discursive forays provide an intellectual gloss for the core argument dealing with the contemporary
world: the disappearance of imperialism; the obsolescence of imperial states, nation states (and boundaries) and the
ascendancy of an ill-defined Empire, globalization, and supra-national governing bodies, apparently resembling the United
Nations.
at hardt/negri: no qualified data

Hardt and Negri’s have no supporting data.


Wolfe, 01 (Alan, director of the Boisi Center for Religion and American Public Life at Boston College, “The Snake”, The New
Republic Online, October 4th, http://www.powells.com/review/2001_10_04)

Most of Empire is an exercise in nominalism, in the attempt to name, rather than to describe, to analyze, or even to condemn, the
new order that its authors see emerging. Although it is presumably devoted to outlining the contours of a new mode of
production, the book contains no data, offers no effort to demonstrate who owns what or holds power over whom, and provides
no indicators of any of the deplorable conditions that it discusses. As if once again to distinguish itself from Marx, Empire, like the
left Hegelians whom Marx once attacked, moves entirely at the level of ideas. Unlike the left Hegelians, however, Hardt and
Negri handle ideas incompetently.
at hardt/negri: multitude fails

The multitude will never be unified—“workers” stand for different objectives and never coordinate.
Wolfe, 01 (Alan, director of the Boisi Center for Religion and American Public Life at Boston College, “The Snake”, The New
Republic Online, October 4th, http://www.powells.com/review/2001_10_04)

Never saying so explicitly, the authors of this book, in identifying their hopes with such disparate movements of protest whatever
their targets or their political coloration, are throwing over the most central proposition of Marxism: class consciousness. Workers
no longer need to be aware of themselves as workers in order to bring down capitalism. They need not develop a revolutionary
strategy, for under contemporary conditions "it may no longer be useful to insist on the old distinction between strategy and
tactics." They do not even need to be workers. All that is required is that they set themselves up against power, whatever and
wherever power happens to be. Never mind that movements that do so can stand for wildly different objectives — an open
society here, a closed society there; or that they are also, as Hardt and Negri point out, often unable or unwilling to communicate
with each other. Indeed, as Hardt and Negri do not point out, they might, if they had the chance, prefer to kill one another.

Hardt and Negri admit they have no idea how the multitude will rise up and over throw Empire.
Wolfe, 01 (Alan, director of the Boisi Center for Religion and American Public Life at Boston College, “The Snake”, The New
Republic Online, October 4th, http://www.powells.com/review/2001_10_04)

And redemption will come from the multitude, who despite their oppression under empire — or Empire — remain pure in heart. In
them, one can see the emergence of the new city that will put us at one with the world. Unlike Augustine's, of course, their city
cannot be the divine one, since "the multitude today...resides on the imperial surfaces where there is no God the Father and no
transcendence." Instead, they will create "the earthly city of the multitude," which the authors esoterically define as "the absolute
constitution of labor and cooperation." About the practical question of how this can be done, Hardt and Negri have nothing
significant to say. "The only response that we can give to these questions is that the action of the multitude becomes political
primarily when it begins to confront directly and with an adequate consciousness the central repressive operations of Empire."
This, too, is a Christian conception of revolution. We cannot know how we will be saved; we must recognize that if only we have
faith, a way will be found.
at hardt/negri: nation-state strong

The nation state is not dying away—their economic analysis is superficial.


Petras, 01 (James, Professor of Sociology at Binghamton University, “Empire With Imperialism”, Rebelion: Petras Essays in
English, October 29, http://www.rebelion.org/petras/english/negri010102.htm)

Assumption 2: The old nation-state governments have been superseded by a new world government, made up of the heads of
the IFI, the WTO, and the heads of the MNCs (p. 326). This is an argument that is based on a superficial discussion of
epiphenomena, rather than a deeper analytical view of the structure of power. While it is true that the IFIs make many important
decisions in a great many geographical locations affecting significant economic and social sectors, these decisions and the
decision-makers are closely linked to the imperial states and the MNCs which influence them. All top IFI officials are appointed
by their national/imperial governments. All their crucial policy guide lines that dictate their loans and conditions for lending are set
by the finance, treasury and economy ministers of the imperial states. The vast majority of funds for the IFIs come from the
imperial states. Representation on the executive board of the IFI is based on the proportion of funding by the imperial states. The
IMF and the WB have always been led by individuals from the U.S. or E.U . Hardt and Negri's vision of IFI power is based on a discussion of derived power not its
imperial states source. In this sense, international power is based in the imperial states not on supra-national entities. The latter concept grossly overestimates the autonomy of the IFIs and underestimates their
subordination to the imperial states. The real significance of the IFIs is how they magnify, extend and deepen the power of the imperial states and how they become terrain for competition between rival imperial
states. Far from superseding the old states, the IFIs have strengthened their positions. Assumption 3: One of the common arguments of globalist theorists like Hardt and Negri is that an information revolution has
. The claims that
taken place that has eliminated state borders, transformed capitalism and created a new epoch (p.145) by providing a new impetus to the development of the productive forces
information technologies have revolutionized economies and thus created a new global economy in which nation states and
national economies have become superfluous is extremely dubious. A comparison of productivity growth in the U.S. over the
past half century fails to support the globalist argument. Between 1953-72, before the so-called information revolution in the U.S. productivity grew an average 2.5%; with the
introduction of computers, productivity growth between 1973-95 was less than half. Even in the so-called boom period of 1995-99, productivity growth was 2.5% about the same as the pre-computer period. Japan
which makes the most extensive use of computers and robots has witnessed a decade of stagnation and crises. During the year 2000-01, the information sector went into a deep crises, tens of thousands were fired,
hundreds of firms went bankrupt, stocks dropped in value some 80%. The speculative bubble, that defined the so-called information economy, burst. Moreover, the major source of growth of productivity claimed by
the globalists was in the computerization of the area of computer manufacture. Studies have shown that computer use in offices is directed more toward personal use than to exchanging ideas. Estimates run up to
60% of computer time is spent in activity unrelated to the enterprise. Computer manufacturers account for 1.2% of the U.S. economy and less than 5% of capital stock. Moreover, the U.S. population census provides
another explanation for the higher productivity figures - the 5 million illegal immigrants who have flooded the U.S. labor market in the 1990s. Since productivity is measured by the output per estimated worker, the 5
With the decline of the information economy and its
million uncounted workers inflate the productivity data. If the 5 million are included the productivity figures would deflate.
stock valuations it becomes clear that the "information revolution" is not the transcendent force defining the economies of the
major imperial states, let alone defining a new world order. The fact that most people have computers and browse, that some
firms have better control over their inventories does not mean that power has shifted beyond the nation-state. The publicists'
claims about the "information revolution" ring hollow, as the investors in the world stock markets move funds toward the real
economy and away from the high tech firms which show no profits and increasing losses.
at hardt/negri: nation-state strong

Hardt and Negri’s examples of globalization all ignore the crucial role of the nation-state in creating those trends—the nation is
not dying away.

Post, 02 (Charlie, member of Solidarity’s National Committee, “Review: Empire and Revolution”, International Viewpoint
Magazine, http://www.internationalviewpoint.org/article.php3?id_article=435)

The result of the internationalisation of lean production over the past two decades has not been a ’smooth’ or ’decentred global
network’ or ’empire’ that Hardt and Negri claim. Quite the opposite, the centres of accumulation and social power remain in the
centres of advanced capitalism in Western Europe, the US and Japan. Global uneven and combined development - the growing
gap in incomes, production and the like - between this global ’north’ and the global ’south’ has only grown wider. Some regions of
the former ’third world’ have become centres of labour-intensive assembly and parts production (the ’Newly Industrialized
Countries’ of Mexico, Brazil, South Africa, South Korea, Taiwan), becoming extensions of capitalist accumulation cantered in the
’north.’ However, vast expanses of the globe (sub-Saharan Africa) remain at best sites of raw material extraction, or at worst
huge labour reserves, marked by extreme poverty and capitalist-created famine and natural disasters. Hardt and Negri’s claims
that the nation-state and inter-imperialist rivalry have declined in importance with the rise of ’empire’ and various institutions of
’global governance’ (World Bank, IMF, WTO, G7, EU, NATO, etc) lack theoretical and even empirical plausibility. The ’declining
effectiveness’ of the nation-state can be traced clearly through the evolution of a whole series of global juridico-economic bodies,
such as GATT, the World Trade Organization, the World Bank, and the IMF. The globalisation of production and circulation,
supported by this supranational juridical scaffolding, supersedes the effectiveness of national juridical structures (p 337). Clearly,
this ’supranational juridical scaffolding’ has been crucial in changing the political environment for capitalist accumulation over the
past two decades. Clearly, ’neo-liberalism’ - the dismantling of the rules that restrict corporations at home and abroad - would be
impossible without these ’global juridico-economic bodies.’ However, the growing importance of these trans-national
organizations does not mean that, in the words of Hardt and Negri ’state functions and constitutional elements have effectively
been displaced to other levels and domains’ (p. 307). On the contrary, the ability of these global political bodies to operate
effectively requires, in many ways, the strengthening of the national-capitalist state. [Kim Moody presents a compelling
alternative analysis. The trans-national corporations (TNCs) have neither the desire nor ability to create a world state. They have
opted instead for a system of multilateral agreements and institutions that they hope will provide coherence and order the world
market. Through their ’home’ governments, the TNCs have attempted to negotiate forms of regulation through the GATT, the
new WTO, and the various regional and multilateral trade agreements. They have also transformed some of the old Bretton
Woods institutions, notably the World Bank and IMF. [9] To ensure the unhindered operations of the trans-nationals and protect
private business property, these global political institutions require national capitalist states capable of denationalising industries,
abolishing social welfare programs and labour regulations, generally deregulating their capital, labour and commodities markets,
and containing challenges from below. Put simply, rather than representing a simple shift of political powers ’upward’ from the
nation-state to the ’global juridico-economic bodies’, the development of the WTO, EU, and the like actually enhance the role of
the nation-state.
at hardt/negri: at biopower Impact

Powerless people depend upon biopolitics to keep them alive—for example, those suffering from HIV/AIDS would be squeezed
out of Hardt and Negri’s society because they would be non-productive workers.

Bull, 01 (Malcolm, head of art history and theory at Oxford University, “You Can’t Build a New Society with a Stanley Knife”,
London Review of Books, Vol. 23, No. 19, http://www.lrb.co.uk/v23/n19/bull01_.html)

It would, I think, be difficult for Hardt and Negri to turn their argument around in this way. Although they recognise the function of
society in the production of individual subjectivities they barely acknowledge its role in the production of power. Using Foucault's
model of biopower, they argue that power constitutes society, not the other way round: 'Power, as it produces, organises; as it
organises, it speaks and expresses itself as authority.' In reply to Machiavelli's observation that the project of constructing a new
society needs arms and money, they cite Spinoza and ask: 'Don't we already possess them? Don't the necessary weapons
reside precisely within the creative and prophetic power of the multitude?' No one is powerless; even the old, the sick and the
unemployed are engaged in the 'immaterial labour' that produces 'total social capital'. Sounding a bit like Ali G, they conclude:
'The poor itself is power. There is World Poverty, but there is above all World Possibility, and only the poor is capable of this.' It is
difficult to see how this analysis comprehends the reality of powerlessness. You may be able to threaten the world with a Stanley
knife, but you cannot build a new society with one. Insofar as the problems of the powerless have been addressed in recent
years it is often through a dynamic that works in the opposite direction to the one Hardt and Negri suggest. Their response to
globalisation is to maintain that since we have not contracted into global society, we still have all the power we need to change it.
The alternative is to argue that a geographically boundless society must also be a totally inclusive society. The latter is an
extension of what used to be called the politics of recognition. Globalisation may have replaced multiculturalism as the focus of
contemporary political debate, but there is an underlying continuity: the concern of anti-globalisation protesters with remote
regions of the world, with the lives of people unlike themselves, and with species of animals and plants that most have seen only
on TV is predicated on an unparalleled imaginative identification with the Other. This totalisation of the politics of recognition from
the local to the global is what has given momentum to campaigns such as the one for African Aids victims; here, it is a question
of sympathy rather than sovereignty, of justice rather than power. In many cases, unless the powerful recognised some kinship
with them, the powerless would just die. Capitalism has no need for the 'immaterial labour' of millions now living. For powerless
human beings, as for other species, autonomy leads to extinction.
at hardt/negri: alt fails

The alternative fails: there are far too many fragmented cultures to form an effective multitude and social organization is needed
to combat oppression.

Angus, 04 (“Empire, Borders, Place: A Critique of Hardt and Negri’s Concept of Empire.” Theory & Event 7:3 Ian Angus, Project
Muse, 2004)

The “aspirations” of the multitude established as the constitutive force of the future are reduced to very
little: freedom, particularly to emigrate, and the right to a socially guaranteed income. In the undoubted care
not to venture outside what is permitted by American liberalism, the project deliberately ignores everything that could be qualified
as the heritage of the workers’ and socialist movement, in particular the equality rejected by the political culture of the United
States. It is difficult to believe in the transformative power of an emerging global (and European) citizenship while the policies
implemented fundamentally deprive citizenship of its effectiveness. The construction of a real alternative to the
contemporary system of globalized liberal capitalism involves other requirements, in particular the
recognition of the gigantic variety of needs and aspirations of the popular classes throughout the world. In
fact, Hardt and Negri experience much difficulty in imagining the societies of the periphery (85 percent of
the human population). The debates concerning the tactics and strategy of building a democratic and
progressive alternative that would be effective in the concrete and specific conditions of the different
countries and regions of the world never appear to have interested them. Would the “democracy” promoted by the
intervention of the United States permit going beyond an electoral farce like the one in the Ukraine, for example? Can one
reduce the rights of the “poor” who people the planet to the right to “emigrate” to the opulent West? A socially guaranteed income
may be a justifiable demand. But can one have the naiveté to believe that its adoption would abolish the capitalist relation, which
allows capital to employ labor (and, consequently, to exploit and oppress it), to the advantage of the worker who would from that
point on be in a position to use capital freely and so be able to affirm the potential of his or her creativity? The reduction of
the subject of history to the “individual” and the uniting of such individuals into a “multitude” dispose of
the true questions concerning the reconstruction of subjects of history equal to the challenges of our era.
One could point to many other important contributions to oppose to the silence of Hardt and Negri on this subject. Undoubtedly,
historic socialisms and communisms had a tendency to reduce the major subject of modern history to the “working class.”
Moreover, this is a reproach that could be leveled at the Negri of workerism. In counterpoint, I have
proposed an analysis of the subject of history as formed from particular social blocs capable, in
successive phases of popular struggle, of effectively transforming the social relations of force to the
advantage of the dominated classes and peoples. At the present time, to take up the challenge implies that one
is moving forward in the formation of democratic, popular, and national hegemonic blocs capable of
overcoming the powers exercised by both the hegemonic imperialist blocs and the hegemonic comprador
blocs. The formation of such blocs takes place in concrete conditions that are very different from one country to another so that
no general model (whether in the style of the “multitude” or some other) makes sense. In this perspective, the combination of
democratic advances and social progress will be part of the long transition to world socialism , just as the affirmation of the
autonomy of peoples, nations, and states will make it possible to substitute a negotiated globalization for
the unilateral globalization imposed by dominant capital (which Empire praises!) and thus gradually
deconstruct the current imperialist system.
at hardt/negri: no multitude

The dramatic centralization of state power that has come after 9/11 disproves Hardt and Negri’s theory that the economic base is
more important than the state

Steinmetz, (Sociology Professor, Michigan) 03 (George, Public Culture 15.2 (2003) 323-345,
The State of Emergency and the Revival of American Imperialism: Toward an Authoritarian Post-Fordism).

The manifest regulatory changes over the past year have focused primarily on the structure and role of the (U.S.) state. The
current "state of emergency," the threat of terrorism, is constructed as a specifically political crisis, a shaken sense of political
sovereignty. This massive campaign to recentralize power began, somewhat ironically, just at the moment when globalization
theorists (including Hardt and Negri) were reaching a consensus that the state was being overshadowed by transnational,
regional, and local organizations. The refocusing of political power on the level of the American national state has been most
evident in the area of U.S. geopolitical strategy (unilateralism and preemptive military strikes), but much of the new regulatory activity has focused on the state apparatus itself and the
"domestic" level of politics, with the creation of a huge new government agency (the Department of Homeland Security), transformations of the legal system (e.g., secret trials and arrests, indefinite detentions), and
intensified domestic surveillance: first with the 2001 USA Patriot Act, which dramatically relaxed restrictions on search and seizure; then with the Total Information Awareness Program, which collects and analyzes
vast amounts of data on private communications and commercial transactions; and most recently with the proposed Domestic Security Enhancement Act of 2003.

Hardt and Negri are wrong, the state is still the central locus of power

Steinmetz, (Sociology Professor, Michigan) 03 (George, Public Culture 15.2 (2003) 323-345,
The State of Emergency and the Revival of American Imperialism: Toward an Authoritarian Post-Fordism).

The argument for the obsolescence of the strong state becomes less convincing when the U.S. government is busy spying on
and arresting its subjects, denying them public trials and lawyers, and threatening to conduct an ever-expanding global war
against terrorism defined in ever more open-ended terms. If the essence of politics is defining the enemy, as Carl Schmitt
argued, the state apparatuses of the United States seem to have dominated this field domestically since September 11, with the
news media playing the role of a supine, supporting chorus. Even questioning the state's decisions has been qualified as
treasonous

Hardt and Negri mis-describe the era. Information and cross border flows of people have both become MORE difficult, not less
so.

Steinmetz, (Sociology Professor, Michigan) 03 (George, Public Culture 15.2 (2003) 323-345,
The State of Emergency and the Revival of American Imperialism: Toward an Authoritarian Post-Fordism).

Within Empire, according to Hardt and Negri, "increasingly less will passports or legal documents be able to regulate our
movements across borders" (397); John Ashcroft has insisted, by contrast, that America's national borders will never again be
open and fluid. The government's secret drumhead trials are the opposite of the transparently open model envisioned by the
authors of Empire. The attorney general's seeming disdain for civil liberties (except the Second Amendment), the chilling effect of
current policies on immigration, international students and scholars, and Muslims in the United States—all of this seems
diametrically opposed to the deterritorializing tendencies of Empire. Developments that seem to run directly counter to the notion
of a "democratic network structure" of information (Hardt and Negri 2000: 299) include the provisions in the USA Patriot Act that
allow the FBI to search records of books and Internet sites used; the Total Information Awareness program; and the brief but
significant career of the "Office of Strategic Influence," whose purpose was to distribute both true and false news items (see Dao
and Schmitt 2002). 19 Clearly we are moving into a new political space.
at hardt/negri: no empire

Hardt and Negri are wrong about recent history so their alternative is doomed to failure: the Empire that they describe has
dissipated.

Steinmetz, (Sociology Professor, Michigan) 03 (George, Public Culture 15.2 (2003) 323-345,
The State of Emergency and the Revival of American Imperialism: Toward an Authoritarian Post-Fordism).

I will argue here, however, that Empire should be understood as a historical reflection on the post-Fordist formation that
crystallized in the 1990s and that is now coming to an end rather than a consideration of the present and the future. Hardt and
Negri's arresting portrait of Empire is undercut by an explanatory framework that suffers from many of the epistemological
shortcomings of traditional Marxism. The authors' residual reductionism and their adherence to a Marxian end-of-history story
line prevent them from grasping the transitory nature of the relatively decentered political formation that they call "Empire." In
addition to the political shock of September 11 and the ensuing war on terrorism, the 1990s also differed in other ways from the
current period. As Slavoj Zizek points out, this was a period in which there was no dominant "schematization" of the "figure of the
Enemy" around a single "central image." Capitalist profit rates were so satisfying as to make this lack unproblematic. Nor was
there anything like September 11 during this decade: no direct and radical attack on the American node in the network of global
capitalism. The ideological, political, and economic conditions for the decentered, multivalent system of Empire described by
Hardt and Negri thus seem to have disappeared in the past year and a half.
at hardt/negri: hurts movements

Empire theory is theoretically indefensible and disabling to resistance movements because they could misdirect their efforts.
Instead of a moment of transformation, we are faced with a dramatic consolidation of state power and capitalist hegemony.

Steinmetz, (Sociology Professor, Michigan) 03 (George, Public Culture 15.2 (2003) 323-345,
The State of Emergency and the Revival of American Imperialism: Toward an Authoritarian Post-Fordism).

Contra such theorists as Hardt and Negri, there is little support for arguments that capitalist history has entered its final phase,
that with the coming of Empire the multitudes have reached a stage in which "pushing through to come out the other side"
becomes a realistic possibility. These authors link the rise of Fordism to the "great economic crisis of 1929" (Hardt and Negri
2000: 241) and acknowledge the role of the economic crisis of the 1970s in creating the conditions for the transition to post-
Fordism. Yet they do not entertain the possibility that Empire itself could enter into a political crisis, like the one we are currently
witnessing, and give rise to a new imperialism. Nor do they consider the possibility that a more systemic economic crisis might
give rise to a mode of regulation that is neither imperial nor imperialist, but protectionist and neocolonial. Each period of core
hegemony has nurtured the illusion among enthusiasts of capitalism that it has reached its apotheosis and the parallel fantasy
among leftists that capitalism is on its last legs. Hugo Grotius ([1625] 1901), writing during the golden age of Dutch hegemony,
believed that his own world was the ultimate one. (Not surprisingly, Grotius's name is often heard in current discussions of U.S.
foreign policy.) The events leading up to the 1848 revolutions in Europe, during the era of British hegemony, famously led Marx
and Engels in the Communist Manifesto to predict "an immediately following proletarian revolution." To take a more recent
example, Ernest Mandel (1975: 125) believed that late capitalism—the title of his book, published in 1972 at the beginning of the
death throes of Fordism but written at the end of the first era of postwar American hegemony—had entered a terminal period of
"overall social crisis." The final sentence of Late Capitalism announced that "the final abolition of capitalism. . . is now
approaching." Insisting that there is something ultimate about Empire is not only theoretically indefensible but could actually be
disabling for movements of resistance, for such arguments may desensitize readers to the possibility of further mutations of
capitalism and modes of social regulation. Without pushing for a cyclical view of history, which Hardt and Negri rightly reject, one
need not fall back on its inverse, a teleological or truncated narrative.
***A2: Language***

at langauge ks: cede the politcal

A focus on discourse substitutes philosophical musing for material politics.

Taft-Kaufman, 95 - Professor, Department of Speech Communication And Dramatic Arts, Central Michigan University – 1995
(Jill, “Other ways: Postmodernism and performance praxis,”  The Southern Communication Journal, Vol.60, Iss. 3;  pg. 222)

In its elevation of language to the primary analysis of social life and its relegation of the de-centered subject to a set of language
positions, postmodernism ignores the way real people make their way in the world. While the notion of decentering does much to
remedy the idea of an essential, unchanging self, it also presents problems. According to Clarke (1991): Having established the
material quality of ideology, everything else we had hitherto thought of as material has disappeared. There is nothing outside of
ideology (or discourse). Where Althusser was concerned with ideology as the imaginary relations of subjects to the real relations
of their existence, the connective quality of this view of ideology has been dissolved because it lays claim to an outside, a real,
an extra-discursive for which there exists no epistemological warrant without lapsing back into the bad old ways of empiricism or
metaphysics. (pp. 25-26) Clarke explains how the same disconnection between the discursive and the extra-discursive has been
performed in semiological analysis: Where it used to contain a relation between the signifier (the representation) and the signified
(the referent), antiempiricism has taken the formal arbitrariness of the connection between the signifier and signified and
replaced it with the abolition of the signified (there can be no real objects out there, because there is no out there for real objects
to be). (p. 26) To the postmodernist, then, real objects have vanished. So, too, have real people. Smith (1988) suggests that
postmodernism has canonized doubt about the availability of the referent to the point that "the real often disappears from
consideration" (p. 159). Real individuals become abstractions. Subject positions rather than subjects are the focus. The
emphasis on subject positions or construction of the discursive self engenders an accompanying critical sense of irony which
recognizes that "all conceptualizations are limited" (Fischer, 1986, p. 224). This postmodern position evokes what Connor (1989)
calls "an absolute weightlessness in which anything is imaginatively possible because nothing really matters" (p. 227). Clarke
(1991) dubs it a "playfulness that produces emotional and/or political disinvestment: a refusal to be engaged" (p. 103). The luxury
of being able to muse about what constitutes the self is a posture in keeping with a critical venue that divorces language from
material objects and bodily subjects.
at language k: generic

Suppressing language because it is offensive preserves its injurious meaning – using the words in new ways makes them more
humane.

Kurtz and Oscarson 03 (Anna and Christopher, Members of National Council of Teachers of English Conference on College
Composition and Communication, “BookTalk: Revising the Discourse of Hate,” ProQuest)

However, Butler also argues that the daily, repeated use of words opens a space for another, more empowering kind of
performance. This alternative performance, Butler insists, can be "the occasion for something we might still call agency, the
repetition of an original subordination for another purpose, one whose future is partially open" (p. 38). To think of words as
having an "open" future is to recognize that their authority lies less in their historical than in their present uses; it is to
acknowledge that people can revise the meaning of words even as we repeat them; it is to embrace the notion that the instability
of words opens the possibility that we can use them to (re)construct a more humane future for ourselves and others. Because
words can be revised, Butler contends that it would be counterproductive simply to stop using terms that we would deem
injurious or oppressive. For when we choose not to use offensive words under any circumstance, we preserve their existing
meanings as well as their power to injure. If as teachers, for instance, we were simply to forbid the use of speech that is hurtful to
LGBT students we would be effectively denying the fact that such language still exists. To ignore words in this way, Butler insists,
won't make them go away. Butler thus suggests that we actually use these words in thoughtful conversation in which we work
through the injuries they cause (p. 1.02). Indeed, Butler insists that if we are to reclaim the power that oppressive speech robs
from us, we must use, confront, and interrogate terms like "queer."

Censorship will be co-opted by conservative elements to destroy minority rights – instead language should be used to subvert
the conventional meanings of the words.

Nye 99 (Andrea, Professor of Philosophy at the University of Wisconsin Whitewater, “Excitable Speech: A Politics of the
Performative; In Pursuit of Privacy: Law, Ethics, and the Rise of Technology,” Jstor)

Once the state has the power to legislate what can be said and not said, she argues, that power will be coopted by conservative
elements to defeat liberal causes and minority rights. State power will also curtail the freedom of speech of private individuals
that is the very basis for effective antidotes to derogatory name calling. DeCew, however, painstakingly reviews the legal and philosophical history of privacy rights as
well as current debates about its scope and status before she takes on the question of whether feminists have any interest in preserving a private sphere. For DeCew, too, a major target is MacKinnon, specifically her
argument that leaving alone the privacy of home and family means leaving men alone to abuse and dominate women. DeCew argues that decisions that protect the use of sexually explicit materials in the home,
consensual sex practices in private, and personal decisions about abortion are in the interest of women as well as men, even though in some cases, such as wife beating, there may be overriding considerations that
, the danger is that the state becomes
justify state intervention. Both authors argue persuasively for a more careful look at the dangers lurking behind calls for state action. For Butler
arbiter of what is and is not permissible speech, allowing rulings that the erection of burning crosses by the Ku Klux Klan is
protected speech but that artistic expressions of gay sexuality or statements of gay identity are actions rather than speech and
so are not protected. The danger DeCew sees is that once the right to privacy is denied or narrowly defined, the state can, on the grounds of immorality, move into women's personal lives to interfere
with sexual expression, whether homosexual or heterosexual, or with the right to choose an abortion established in Roe v. Wade. Both DeCew and Butler, however, provide alternative remedies for the admitted harm
that state action is intended to redress. For DeCew, the right to privacy is not absolute; like freedom, it can be overridden by other rights —thus the state can intervene in domestic abuse cases because of the
. Given the postmodern view that the subject
physical harm being done. Butler's remedy for harmful hate language is more deeply rooted in postmodern theories of the speaking subject
can never magisterially use a language with fixed meanings according to clear intentions, it is always pos sible to subvert the
conventional meanings of words. What is said as a derogatory slur—"nigger," "chick," "spic," or "gay," for example —can be
"resignified," that is, returned in such a manner that its conventional mean ing in practices of discrimination and abuse is
subverted. Butler gives as examples the revalorization of terms like "black" or "gay," the satirical citation of racial or sexual slurs,
reappropriation in street language or rap music, and expressions of homosexual identity in art depicting graphic sex. These are
expressions that any erosion in First Amendment rights might endanger.
at language k: generic

Placing representations and discourse first trades off with concrete political change and makes no difference to those engaged in
political struggles.

Taft-Kaufman, 95 Jill Speech prof @ CMU, Southern Comm. Journal, Spring, v. 60, Iss. 3, “Other Ways”, p pq
The postmodern passwords of "polyvocality," "Otherness," and "difference," unsupported by substantial
analysis of the concrete contexts of subjects, creates a solipsistic quagmire. The political sympathies of the
new cultural critics, with their ostensible concern for the lack of power experienced by marginalized people,
aligns them with the political left. Yet, despite their adversarial posture and talk of opposition, their
discourses on intertextuality and inter-referentiality isolate them from and ignore the conditions that have
produced leftist politics--conflict, racism, poverty, and injustice. In short, as Clarke (1991) asserts,
postmodern emphasis on new subjects conceals the old subjects, those who have limited access to good jobs,
food, housing, health care, and transportation, as well as to the media that depict them. Merod (1987) decries this
situation as one which leaves no vision, will, or commitment to activism. He notes that academic lip service to
the oppositional is underscored by the absence of focused collective or politically active intellectual
communities. Provoked by the academic manifestations of this problem Di Leonardo (1990) echoes Merod and
laments: Has there ever been a historical era characterized by as little radical analysis or activism and as
much radical-chic writing as ours? Maundering on about Otherness: phallocentrism or Eurocentric
tropes has become a lazy academic substitute for actual engagement with the detailed histories and
contemporary realities of Western racial minorities, white women, or any Third World population. (p. 530)
Clarke's assessment of the postmodern elevation of language to the "sine qua non" of critical discussion is
an even stronger indictment against the trend. Clarke examines Lyotard's (1984) The Postmodern Condition
in which Lyotard maintains that virtually all social relations are linguistic, and, therefore, it is through the
coercion that threatens speech that we enter the "realm of terror" and society falls apart. To this assertion, Clarke
replies: I can think of few more striking indicators of the political and intellectual impoverishment of a
view of society that can only recognize the discursive. If the worst terror we can envisage is the threat not
to be allowed to speak, we are appallingly ignorant of terror in its elaborate contemporary forms. It may
be the intellectual's conception of terror (what else do we do but speak?), but its projection onto the rest of
the world would be calamitous....(pp. 2-27) The realm of the discursive is derived from the requisites for
human life, which are in the physical world, rather than in a world of ideas or symbols.(4) Nutrition, shelter, and
protection are basic human needs that require collective activity for their fulfillment. Postmodern emphasis on the discursive without an accompanying analysis of how the discursive
emerges from material circumstances hides the complex task of envisioning and working towards concrete social goals (Merod, 1987). Although the material conditions that create
the situation of marginality escape the purview of the postmodernist, the situation and its consequences are not overlooked by scholars from marginalized groups. Robinson (1990) for
"the justice that working people deserve is economic, not just textual" (p. 571). Lopez (1992)
example, argues that
states that "the starting point for organizing the program content of education or political action must be
the present existential, concrete situation" (p. 299). West (1988) asserts that borrowing French post-
structuralist discourses about "Otherness" blinds us to realities of American difference going on in front of us (p.
170). Unlike postmodern "textual radicals" who Rabinow (1986) acknowledges are "fuzzy about power and the
realities of socioeconomic constraints" (p. 255), most writers from marginalized groups are clear about how
discourse interweaves with the concrete circumstances that create lived experience. People whose lives form the
material for postmodern counter-hegemonic discourse do not share the optimism over the new recognition
of their discursive subjectivities, because such an acknowledgment does not address sufficiently their
collective historical and current struggles against racism, sexism, homophobia, and economic injustice.
They do not appreciate being told they are living in a world in which there are no more real subjects.
Ideas have consequences. Emphasizing the discursive self when a person is hungry and homeless
represents both a cultural and humane failure.
at language ks: censorship hurst the left

Free speech is critical—censorship will backfire on the left.

Wilson, 2000 – Editor and Publisher of Illinois Academe – 2000 (John K. Wilson, “How the Left can Win Arguments and Influence
People” p. 163)

The left must stand strongly on the side of free speech. Because there is so much censorship of progressive ideas, it can be
tempting for the left to turn the tables and to try to silence far right advocates. But repression is unnecessary: progressive ideas
are more popular than conservatives ones, and all the left needs to do is get a fair and open hearing. Censorship is not only
wrong, it’s also a losing strategy for the left. Progressive attract more attention if they’re the censors. Because conservative
censorship is largely taken for granted, news about intolerance on the right isn’t usually publicized. But when someone on the left
seems guilty of censorship, the rights publicity machine quickly starts up. That’s why the right was able to push the myth of
political correctness in the 1990s and invent the idea of a wave of left-wing oppression sweeping college at a time when there
was more freedom of thought than ever before, and infringement of free speech on campuses by conservative forces was more
prevalent than anything committed by the left. Progressives certainly need to better publicize incidents of censorship, but the left
must also realize that the right will always win the suppression battles. It has all the resources and the media on its side. When
some leftists are willing to make exceptions to the First Amendment to silence conservative hate mongers, it becomes even more
difficult for progressive to draw attention to the censorship of left-wing ideas. The only winning strategy is to maintain a consistent
commitment to freedom of speech.
*** Representations ***

A2: Representations

Privileging representations locks in violence --- policy analysis is the best way to challenge
power
Taft-Kaufman ‘95 (Jill, Professor of Speech – CMU, Southern Communication Journal, Vol. 60, Issue 3,
Spring)
The postmodern passwords of "polyvocality," "Otherness," and "difference," unsupported by substantial analysis of the concrete
contexts of subjects, creates a solipsistic quagmire. The political sympathies of the new cultural critics, with their ostensible concern for the lack
of power experienced by marginalized people, aligns them with the political left. Yet, despite their adversarial posture and talk of opposition,
their discourses on intertextuality and inter-referentiality isolate them from and ignore the conditions that
have produced leftist politics--conflict, racism, poverty, and injustice. In short, as Clarke (1991) asserts, postmodern
emphasis on new subjects conceals the old subjects, those who have limited access to good jobs, food, housing, health care, and
transportation, as well as to the media that depict them. Merod (1987) decries this situation as one which leaves no vision, will, or
commitment to activism. He notes that academic lip service to the oppositional is underscored by the absence of
focused collective or politically active intellectual communities. Provoked by the academic manifestations of this
problem Di Leonardo (1990) echoes Merod and laments: Has there ever been a historical era characterized by as little radical
analysis or activism and as much radical-chic writing as ours? Maundering on about Otherness: phallocentrism or
Eurocentric tropes has become a lazy academic substitute for actual engagement with the detailed
histories and contemporary realities of Western racial minorities, white women, or any Third World population. (p. 530)
Clarke's assessment of the postmodern elevation of language to the "sine qua non" of critical discussion is an even
stronger indictment against the trend. Clarke examines Lyotard's (1984) The Postmodern Condition in which Lyotard
maintains that virtually all social relations are linguistic, and, therefore, it is through the coercion that threatens speech that we enter the "realm of
terror" and society falls apart. To this assertion, Clarke
replies: I can think of few more striking indicators of the
political and intellectual impoverishment of a view of society that can only recognize the
discursive. If the worst terror we can envisage is the threat not to be allowed to speak, we are appallingly ignorant of terror in its elaborate
contemporary forms. It may be the intellectual's conception of terror (what else do we do but speak?), but its projection onto the rest of the world
would be calamitous....(pp. 2-27) The realm of the discursive is derived from the requisites for human life, which are in the physical world, rather
than in a world of ideas or symbols.(4) Nutrition, shelter, and protection are basic human needs that require collective activity for their
fulfillment. Postmodern emphasis
on the discursive without an accompanying analysis of how the discursive
emerges from material circumstances hides the complex task of envisioning and working towards concrete
social goals (Merod, 1987). Although the material conditions that create the situation of marginality escape the purview of
the postmodernist, the situation and its consequences are not overlooked by scholars from marginalized groups. Robinson (1990)
for example, argues that "the justice that working people deserve is economic, not just textual" (p. 571). Lopez (1992)
states that "the starting point for organizing the program content of education or political action must be the present existential, concrete
situation" (p. 299). West (1988) asserts that borrowing French post-structuralist discourses about "Otherness" blinds us to realities of American
difference going on in front of us (p. 170). Unlike postmodern "textual
radicals" who Rabinow (1986) acknowledges are "fuzzy
about power and the realities of socioeconomic constraints" (p. 255), most writers from marginalized groups are clear
about how discourse interweaves with the concrete circumstances that create lived experience. People whose lives form the material for
postmodern counter-hegemonic discourse do not share the optimism over the new recognition of their discursive
subjectivities, because such an acknowledgment does not address sufficiently their collective historical and current
struggles against racism, sexism, homophobia, and economic injustice. They do not appreciate being told
they are living in a world in which there are no more real subjects. Ideas have consequences. Emphasizing the discursive
self when a person is hungry and homeless represents both a cultural and humane failure. The need to look beyond
texts to the perception and attainment of concrete social goals keeps writers from marginalized groups ever-mindful of the specifics of how power
works through political agendas, institutions, agencies, and the budgets that fuel them.
A2: Representations

Representations don’t influence reality


Kocher ‘2K (Robert L, Author and Philosopher, http://freedom.orlingrabbe.com/lfetimes/reality_sanity1.htm)
While it is not possible to establish many proofs in the verbal world, and it is simultaneously possible to make many uninhibited
assertions or word equations in the verbal world, it should be considered that reality is more rigid and does not abide
by the artificial flexibility and latitude of the verbal world. The world of words and the world of human
experience are very imperfectly correlated. That is, saying something doesn't make it true. A
verbal statement in the world of words doesn't mean it will occur as such in the world of consistent human
experience I call reality. In the event verbal statements or assertions disagree with consistent human experience, what proof is
there that the concoctions created in the world of words should take precedence or be assumed a greater truth than the world of
human physical experience that I define as reality? In the event following a verbal assertion in the verbal world produces pain or
catastrophe in the world of human physical reality or experience, which of the two can and should be changed? Is it wiser to live
with the pain and catastrophe, or to change the arbitrary collection of words whose direction produced that pain and catastrophe?
Which do you want to live with? What proven reason is there to assume that when doubtfulness that can be constructed in verbal
equations conflicts with human physical experience, human physical experience should be considered doubtful? It becomes a
matter of choice and pride in intellectual argument. My personal advice is that when verbal contortions lead to chronic confusion
and difficulty, better you should stop the verbal contortions rather than continuing to expect the difficulty to change. Again, it's a
matter of choice. Does the outcome of the philosophical question of whether reality or proof exists
decide whether we should plant crops or wear clothes in cold weather to protect us from freezing?
Har! Are you crazy? How many committed deconstructionist philosophers walk about naked in subzero temperatures or don't
eat? Try creating and living in an alternative subjective reality where food is not needed and where you can sit
naked on icebergs, and find out what happens. I emphatically encourage people to try it with the stipulation that
they don't do it around me, that they don't force me to do it with them, or that they don't come to me complaining about the
consequences and demanding to conscript me into paying for the cost of treating frostbite or other consequences. (sounds like
there is a parallel to irresponsibility and socialism somewhere in here, doesn't it?). I encourage people to live subjective reality. I
also ask them to go off far away from me to try it, where I won't be bothered by them or the consequences. For those who haven't
guessed, this encouragement is a clever attempt to bait them into going off to some distant place where they will kill themselves
off through the process of social Darwinism — because, let's face it, a society of deconstructionists and counterculturalists
filled with people
debating what, if any, reality exists would have the productive functionality of a
field of diseased rutabagas and would never survive the first frost. The attempt to convince people to
create and move to such a society never works, however, because they are not as committed or sincere as they claim to be.
Consequently, they stay here to work for left wing causes and promote left wing political candidates where there are people who
live productive reality who can be fed upon while they continue their arguments. They ain't going to practice what they profess,
and they are smart enough not to leave the availability of people to victimize and steal from while they profess what they pretend
to believe in.
A2: Fear of Death

Fear is necessary to check extinction – provides an active consciousness which sustains


peace
J.A.H. Futterman 94, Former US nuclear weapons scientist, 1994, “Obscenity and Peace: Meditation on the Bomb,” Virtual Church of
the Blind Chihuahua, www.dogchurch.org/books/nuke.html

But the inhibitory effect of reliable nuclear weapons goes deeper than Shirer's deterrence of adventurer-conquerors. It changes
the way we think individually and culturally, preparing us for a future we cannot now imagine. Jungian psychiatrist Anthony J.
Stevens states, [15] "History would indicate that people cannot rise above their narrow sectarian concerns without some
overwhelming paroxysm. It took the War of Independence and the Civil War to forge the United States, World War I to create
the League of Nations, World War II to create the United Nations Organization and the European Economic Community. Only
catastrophe, it seems, forces people to take the wider view. Or what about fear? Can the horror which we all experience when we
contemplate the possibility of nuclear extinction mobilize in us sufficient libidinal energy to resist the archetypes of war?
Certainly, the moment we become blasé about the possibility of holocaust we are lost. As long as horror of nuclear exchange
remains uppermost we can recognize that nothing is worth it. War becomes the impossible option. Perhaps horror, the experience
of horror, the consciousness of horror, is our only hope. Perhaps horror alone will enable us to overcome the otherwise invincible
attraction of war." Thus I also continue engaging in nuclear weapons work to help fire that world-historical warning shot I mentioned above,
namely, that as our beneficial technologies become more powerful, so will our weapons technologies, unless genuine peace precludes it. We must
build a future more peaceful than our past, if we are to have a future at all, with or without nuclear weapons — a fact we had better learn before
worse things than nuclear weapons are invented. If you're a philosopher, this means that I regard the nature of humankind as mutable rather than
fixed, but that I think most people welcome change in their personalities and cultures with all the enthusiasm that they welcome death — thus, the
fear of nuclear annihilation of ourselves and all our values may be what we require in order to become peaceful enough to survive
our future technological breakthroughs.[16] Collective fear of death is key to check genocide and extinction Louis Rene
Beres, PhD Princeton, 1996, “No Fear, No Trembling Israel, Death and the Meaning of Anxiety,”
www.freeman.org/m_online/feb96/beresn.htm Fear of death, the ultimate source of anxiety, is essential to human survival.
This is true not only for individuals, but also for states. Without such fear, states will exhibit an incapacity to confront nonbeing
that can hasten their disappearance. So it is today with the State of Israel. Israel suffers acutely from insufficient existential dread. Refusing
to tremble before the growing prospect of collective disintegration - a forseeable prospect connected with both genocide and war - this state is
now unable to take the necessary steps toward collective survival. What is more, because death is the one fact of life which is not relative but
absolute, Israel's blithe unawareness of its national mortality deprives its still living days of essential absoluteness and growth. For states, just as
for individuals, confronting death can give the most positive reality to life itself . In this respect, a cultivated awareness of nonbeing is
central to each state's pattern of potentialities as well as to its very existence. When a state chooses to block off such an awareness, a choice
currently made by the State of Israel, it loses, possibly forever, the altogether critical benefits of "anxiety." There is, of course, a distinctly ironic
resonance to this argument. Anxiety, after all, is generally taken as a negative, as a liability that cripples rather than enhances life.
But anxiety is not something we "have." It is something we (states and individuals) "are." It is true, to be sure, that anxiety, at the
onset of psychosis, can lead individuals to experience literally the threat of self-dissolution, but this is, by definition, not a
problem for states. Anxiety stems from the awareness that existence can actually be destroyed, that one can actually become
nothing. An ontological characteristic, it has been commonly called Angst, a word related to anguish (which comes from the
Latin angustus, "narrow," which in turn comes from angere, "to choke.") Herein lies the relevant idea of birth trauma as the
prototype of all anxiety, as "pain in narrows" through the "choking" straits of birth. Kierkegaard identified anxiety as " the dizziness
of freedom," adding: "Anxiety is the reality of freedom as a potentiality before this freedom has materialized." This brings us back to Israel. Both
individuals and states may surrender freedom in the hope of ridding themselves of an unbearable anxiety. Regarding states, such surrender can
lead to a rampant and delirious collectivism which stamps out all political opposition. It can also lead to a national self-delusion which augments
enemy power and hastens catastrophic war. For the Jewish State, a lack of pertinent anxiety, of the positive aspect of Angst, has already
led its people to what is likely an irreversible rendezvous with extinction.
A2: Fear of Death

Death imagery affirms life


Michael Allen Fox, Assoc. Prof Phil. @ Queens, 1985, “Nuclear War: Philosophical Perspectives,” ed. Fox and
Groarke, p. 127

There remains but one choice: we must seek a reduction of world tensions, mutual trust, disarmament, and peace.35
Security is not the absence of fear and anxiety, but a degree of stress and uncertainty with which we can cope and
remain mentally healthy. For security, understood in this way, to become a feature of our lives, we must admit our
nuclear fear and anxiety and identify the mechanisms that dull or mask our emotional and other responses. It is
necessary to realist that we cannot entrust security to ourselves, but, strange as it seems and however difficult to
accept, must entrust it to our adversary Just as the safety and security of each of us, as individuals, depends upon the
good will of every other, any one of whom could harm us at any moment, so the security of nations finally depends
upon the good will of other nations, whether or not we willingly accept this fact. The disease for which we must
find the cure also requires that we continually come face to face with the unthinkable in image and thought and
recoil from it. 36 In this manner we can break its hold over us and free ourselves to begin new initiatives. As
Robert J. Lifton points out, “confronting massive death helps us bring ourselves more in touch with what we care
most about in life. We [will then] find ourselves in no way on a death trip, but rather responding to a call for
personal and professional actions and commitments on behalf of that wondrous and fragile entity we know as
human life.
A2: Fear of Death

We do not fear death, we discuss our impacts for the purpose of confronting death and
engaging in the idea, it is only by discussing death that we can define its purpose and the
true purpose and value of life, this robs all fear from death
Gunaratna Buddhist 1982 (V.F. “Buddhist Reflections on Death” http://www.accesstoinsight.org/lib/authors/gunaratna/wheel102.html)
To the average man death is by no means a pleasant subject or talk for discussion. It is something dismal and oppressive — a
veritable kill-joy, a fit topic for a funeral house only. The average man immersed as he is in the self, ever seeking after the
pleasurable, ever pursuing that which excites and gratifies the senses, refuses to pause and ponder seriously that these very
objects of pleasure and gratification will some day reach their end. If wise counsel does not prevail and urge the unthinking pleasure-seeking man
to consider seriously that death can knock at his door also, it is only the shock of a bereavement under his own roof, the sudden and untimely death of a parent, wife or
child that will rouse him up from his delirious round of sense-gratification and rudely awaken him to the hard facts of life. Then only will his eyes open, then only will
he begin to ask himself why there is such a phenomenon as death. Why is it inevitable? Why are there these painful partings which rob life of its joys? To most of us,
at some moment or another, the spectacle of death must have given rise to the deepest of thoughts and profoundest of questions. What is
life worth, if able bodies that once performed great deeds now lie flat and cold, senseless and lifeless? What is life worth, if eyes
that once sparkled with joy, eyes that once beamed with love are now closed forever, bereft of movement, bereft of life?
Thoughts such as these are not to be repressed. It is just these inquiring thoughts, if wisely pursued, that will ultimately unfold
the potentialities inherent in the human mind to receive the highest truths. According to the Buddhist way of thinking, death,
far from being a subject to be shunned and avoided, is the key that unlocks the seeming mystery of life. It is by understanding
death that we understand life; for death is part of the process of life in the larger sense. In another sense, life and death are two
ends of the same process and if you understand one end of the process, you also understand the other end. Hence, by
understanding the purpose of death we also understand the purpose of life. It is the contemplation of death, the intensive thought
that it will some day come upon us, that softens the hardest of hearts, binds one to another with cords of love and compassion,
and destroys the barriers of caste, creed and race among the peoples of this earth all of whom are subject to the common destiny
of death. Death is a great leveler. Pride of birth, pride of position, pride of wealth, pride of power must give way to the all-consuming thought of inevitable
death. It is this leveling aspect of death that made the poet say: "Scepter and crown Must tumble down And in the dust be equal made With the poor crooked scythe
and spade." It is the contemplation of death that helps to destroy the infatuation of sense-pleasure. It is the contemplation of death
that destroys vanity. It is the contemplation of death that gives balance and a healthy sense of proportion to our highly over-
wrought minds with their misguided sense of values. It is the contemplation of death that gives strength and steadiness and direction to the erratic
human mind, now wandering in one direction, now in another, without an aim, without a purpose. It is not for nothing that the Buddha has, in the very highest terms,
commended to his disciples the practice of mindfulness regarding death. This is known as "marananussati bhavana." One who wants to practice it must at stated times,
and also every now and then, revert to the thought maranam bhavissati — "death will take place." This contemplation of death is one of the classical
meditation-subjects treated in the Visuddhi Magga which states that in order to obtain the fullest results, one should practice this
meditation in the correct way, that is, with mindfulness (sati), with a sense of urgency (samvega) and with understanding (ñana). For example, suppose a
young disciple fails to realize keenly that death can come upon him at any moment, and regards it as something that will occur in old age in the distant future; his
contemplation of death will be lacking strength and clarity, so much so that it will run on lines which are not conducive to success. How great and useful isthe
contemplation of death can be seen from the following beneficial effects enumerated in the Visuddhi Magga: — "The disciple
who devotes himself to this contemplation of death is always vigilant, takes no delight in any form of existence, gives up
hankering after life, censures evil doing, is free from craving as regards the requisites of life, his perception of impermanence
becomes established, he realizes the painful and soulless nature of existence and at the moment of death he is devoid of fear, and
remains mindful and self-possessed. Finally, if in this present life he fails to attain to Nibbana, upon the dissolution of the body he is bound for a happy
destiny." Thus it will be seen that mindfulness of death not only purifies and refines the mind but also has the effect of robbing death of
its fears and terrors, and helps one at that solemn moment when he is gasping for his last breath, to face that situation with
fortitude and calm. He is never unnerved at the thought of death but is always prepared for it. It is such a man that can truly
exclaim, "O death, where is thy sting?" In the Anguttara Nikaya the Buddha has said, "Oh Monks, there are ten ideas, which if made
to grow, made much of, are of great fruit, of great profit for plunging into Nibbana, for ending up in Nibbana." Of these ten, one is death. Contemplation on
death and on other forms of sorrow such as old age, and disease, constitutes a convenient starting point for the long line of
investigation and meditation that will ultimately lead to Reality. This is exactly what happened in the case of the Buddha . Was it not
the sight of an old man followed by the sight of a sick man and thereafter the sight of a dead man that made Prince Siddhattha, living in the lap of luxury, to give up
wife and child, home and the prospect of a kingdom, and to embark on a voyage of discovery of truth, a voyage that ended in the glory of Buddhahood and the bliss of
Nibbana? The marked disinclination of the average man to advert to the problem of death, the distaste that arouses in him the desire
to turn away from it whenever the subject is broached, are all due to the weakness of the human mind, sometimes occasioned by
fear, sometimes by tanha or selfishness, but at all times supported by ignorance (avijja). The disinclination to understand death, is
no different from the disinclination of a man to subject himself to a medical check-up although he feels that something is wrong
with him. We must learn to value the necessity to face facts. Safety always lies in truth. The sooner we know our condition the
safer are we, for we can then take the steps necessary for our betterment. The saying, "where ignorance is bliss it is folly to be wise" has no
application here. To live with no thought of death is to live in a fool's paradise.
at fear of death: mobilizes people/compassion

Fear spurs compassion, mobilizing people to protect each other and giving meaning to life.

Greenspan, 03 (Miriam Greenspan – Pioneer in the Area of Women’s Psychology – 2003 (“An Excerpt from Healing through the
Dark Emotions: The Wisdom of Grief, Fear, and Despair by Miriam Greenspan,”
www.spiritualityhealth.com/newsh/excerpts/bookreview/excp_5513.html)

"Fear is a very powerful emotion. When you feel fear in your body, it's helpful to relate to it as an energy that can be mobilized for
life. It may feel like a constriction in your chest, throat, or abdomen. Breathe through it without judgment and allow yourself to feel
it as a very strong force. If you pray for help, you can begin to expand this energy we call 'fear' and use it for healing and
transformation. "In this regard, we can take our model from the heroes of Flight 93 who. realizing that they were bound for death,
stormed the plane and brought it down without hitting a civilian target. One cannot even imagine being able to do this without
fear. Fear for the lives of others was the energy that mobilized them to do something meaningful with their last moments of life.
Some of these people said good-bye to their husbands and wives and wished them happiness before they left this earth. They
had found some peace in their last moments, peace in the midst of turbulence. And they found it through their last wish, which
they heroically put into action: to help others live. "Perhaps there is nothing that can redeem the dead but our own actions for the
good. This is a time to find out what we want to do for the world and do it. And, as every trauma survivor knows, this is the way to
make meaning out of pain, perhaps the most effective way: to draw something good out of evil. The heroes of September 11
point us to the choice we each have: to help create a state of global peace and justice that we, like they, will not see before we
die. It is in giving ourselves to this vision, out of love for this world that we inhabit together, that we stand a chance of
transcending the human proclivity to damage life. And that we honor those we have brought into this world and who must inherit
it. . . . "Our only protection is in our interconnectedness. This has always been the message of the dark emotions when they are
experienced most deeply and widely. Grief is not just "my" grief; it is the grief of every motherless child, every witness to horror in
the world. Despair is not just "my" despair; it is everyone's despair about life in the twentyfirst century. Fear is not just 'my' fear; it
is everyone's fear — of anthrax, of nuclear war, of truck bombs, of airplane hijackings, of things falling apart, blowing up,
sickening and dying. "If fear is only telling you to save your own skin, there's not much hope for us. But the fact is that in
conscious fear, there is a potentially revolutionary power of compassion and connection that can be mobilized en masse. This is
the power of fear. Our collective fear, which is intelligent, is telling us now: Find new ways to keep this global village safe. Find
new forms of international cooperation that will root out evil in ways that don't create more victims and more evil. Leap out of the
confines of national egos. Learn the ways of peace. Find a ceremony of safety so that not just you and I but all of us can live
together without fear."
at fear of death: Fear Key to Value to Life

Fear of death is key to value to life – recognizing death is inevitable allows us to create a world of love.

Kelsang 99 (Geshe, internationally renowned teacher of Buddhism (, http://www.tharpa.com/background/fear-of-death.htm)

A healthy fear of death would be the fear of dying unprepared, as this is a fear we can do something about, a danger we can
avert. If we have this realistic fear, this sense of danger, we are encouraged to prepare for a peaceful and successful death and
are also inspired to make the most of our very precious human life instead of wasting it. This "sense of danger" inspires us to
make preparations so that we are no longer in the danger we are in now, for example by practicing moral discipline, purifying our
negative karma, and accumulating as much merit, or good karma, as possible. We put on a seat belt out of a sense of danger of
the unseen dangers of traffic on the road, and that seat belt protects us from going through the windshield. We can do nothing
about other traffic, but we can do something about whether or not we go through the windscreen if someone crashes into us.
Similarly, we can do nothing about the fact of death, but we can seize control over how we prepare for death and how we die.
Eventually, through Tantric spiritual practice, we can even attain a deathless body. In Living Meaningfully, Dying Joyfully, Geshe
Kelsang says: Dying with regrets is not at all unusual. To avoid a sad and meaningless end to our life we need to remember
continually that we too must die. Contemplating our own death will inspire us to use our life wisely by developing the inner refuge
of spiritual realizations; otherwise we shall have no ability to protect ourself from the sufferings of death and what lies beyond.
Moreover, when someone close to us is dying, such as a parent or friend, we shall be powerless to help them because we shall
not know how; and we shall experience sadness and frustration at our inability to be of genuine help. Preparing for death is one
of the kindest and wisest things we can do both for ourself and others. The fact of the matter is that this world is not our home.
We are travelers, passing through. We came from our previous life, and in a few years, or a few days, we shall move on to our
next life. We entered this world empty-handed and alone, and we shall leave empty-handed and alone. Everything we have
accumulated in this life, including our very body, will be left behind. All that we can take with us from one life to the next are the
imprints of the positive and negative actions we have created. If we ignore death we shall waste our life working for things that
we shall only have to leave behind, creating many negative actions in the process, and having to travel on to our next life with
nothing but a heavy burden of negative karma. On the other hand, if we base our life on a realistic awareness of our mortality, we
shall regard our spiritual development as far more important than the attainments of this world, and we shall view our time in this
world principally as an opportunity to cultivate positive minds such as patience, love, compassion, and wisdom. Motivated by
these virtuous minds we shall perform many positive actions, thereby creating the cause for future happiness.
at fear of death: key to survival

Fear of Death is key to human survival – confronting death is key to state and individual existence

Beres 96 (Louis Rene, Professor of Political Science and International Law at Purdue University, Feb.,
http://www.freeman.org/m_online/ feb96/ beresn.htm).

Fear of death, the ultimate source of anxiety, is essential to human survival. This is true not only for individuals, but also for
states. Without such fear, states will exhibit an incapacity to confront nonbeing that can hasten their disappearance. So it is today
with the State of Israel. Israel suffers acutely from insufficient existential dread. Refusing to tremble before the growing prospect
of collective disintegration - a forseeable prospect connected with both genocide and war - this state is now unable to take the
necessary steps toward collective survival. What is more, because death is the one fact of life which is not relative but absolute,
Israel's blithe unawareness of its national mortality deprives its still living days of essential absoluteness and growth. For states,
just as for individuals, confronting death can give the most positive reality to life itself. In this respect, a cultivated awareness of
nonbeing is central to each state's pattern of potentialities as well as to its very existence. When a state chooses to block off
such an awareness, a choice currently made by the State of Israel, it loses, possibly forever, the altogether critical benefits of
"anxiety."

Fear is key to value to life, survival and transcending evil.

Greenspan 03 (Miriam, Pioneer in the Area of Women’s Psychology, Healing Through the Dark Emotions: The Wisdom of
Grief, Fear, and Despair, Excerpt of Chapter Three - How Dark Emotions Become Toxic,
http://www.miriamgreenspan.com/excerpts/chapterThreeEx.html)

Grief, fear, and despair are primary human emotions. Without them, we would be less than human, and less likely to survive.
Grief arises because we are not alone, and what connects us to others and to the world also breaks our hearts. Grieving our
losses allows us to heal and renew our spirits. Fear alerts us to protect our survival, extending beyond our instinct for self-
preservation to our concern for others. Despair asks us to find meaning in the midst of apparent chaos or meaninglessness.
Making meaning out of suffering is the basis of the human capacity to survive evil and transcend it. The purposefulness of these
dark emotions is evident when we can experience them mindfully, tolerate their intense energies, and let them be.
at fear of death: deterence good

Fear of nuclear weapons has prevented their use – deterrence has checked conflict.

Rajaraman 02 (Professor of Theoretical Physics at JNU, 2002 [R., “Ban battlefield nuclear weapons,” 4/22/2, The Hindu,
http://www.hinduonnet.com/thehindu/2002/04/22/stories/2002042200431000.htm[

There were a variety of different reasons behind each of these examples of abstinence from using nuclear weapons. But one
major common factor contributing to all of them has been an ingrained terror of nuclear devastation. The well documented
images of Hiroshima and Nagasaki, the awesome photographs of giant mushroom clouds emerging from nuclear tests in the
Pacific and the numerous movies based on nuclear Armageddon scenarios have all contributed to building up a deep rooted fear
of nuclear weapons. This is not limited just to the abhorrence felt by anti-nuclear activists. It permeates to one extent or another
the psyche of all but the most pathological of fanatics. It colours the calculations, even if not decisively, of the most hardened of
military strategists. The unacceptability of nuclear devastation is the backbone of all deterrence strategies. There is not just a
fear of being attacked oneself, but also a strong mental barrier against actually initiating nuclear attacks on enemy populations,
no matter how much they may be contemplated in war games and strategies. As a result a taboo has tacitly evolved over the
decades preventing nations, at least so far, from actually pressing the nuclear button even in the face of serious military crises.
at fear of nukes: fear Key to Peace and Survival

Fear of nuclear war is key to stopping WMD use and prevents military adventurism.
Futterman, 91 (JAH, Livermore lab researcher, 1995, Mediation of the Bomb, online,
http://www.dogchurch.org/scriptorium/nuke0

I could say that if I didn't do it, someone else would, but that answer was rejected at Nuremberg. (It's also a better reason to
leave the weapons program than to stay.) I continue to support the u business with my effort for many reasons, which I discuss
throughout this piece. But mostly, I do it because the fear of nuclear holocaust is the only authority my own country or any other
has respected so far when it comes to nationalistic urges to make unlimited war. As William L. Shirer states in his preface to The
Rise and Fall of the Third Reich (Touchstone Books, New York, 1990), "Adolf Hitler is probably the last of the great adventurer-
conquerors in the tradition of Alexander, Caesar, and Napoleon, and the Third Reich the last of the empires which set out on the
path taken earlier by France, Rome and Macedonia. The curtain was rung down on that phase of history, at least, by the sudden
invention of the hydrogen bomb, of the ballistic missile, and of rockets which can be aimed to hit the moon." Now this contrasts
with the argument of those who would "reinvent government" by putting up bureaucratic roadblocks to maintaining the reliability
of the US nuclear arsenal through research and testing. They reason that if the reliability of everyone's nuclear arsenals declines,
everyone will be less likely to try using them. The problem is that some "adventurer-conqueror" may arise and use everyone's
doubt about their arsenals to risk massive conventional war instead. An expansionist dictatorship might even risk nuclear war
with weapons that are simpler, cruder, less powerful, much riskier (in terms of the possibility of accidental detonation) but much
more reliable than our own may eventually become without adequate "stockpile stewardship."[14] But the inhibitory effect of
reliable nuclear weapons goes deeper than Shirer's deterrence of adventurer-conquerors. It changes the way we think
individually and culturally, preparing us for a future we cannot now imagine. Jungian psychiatrist Anthony J. Stevens states, [15]
"History would indicate that people cannot rise above their narrow sectarian concerns without some overwhelming paroxysm. It
took the War of Independence and the Civil War to forge the United States, World War I to create the League of Nations, World
War II to create the United Nations Organization and the European Economic Community. Only catastrophe, it seems, forces
people to take the wider view. Or what about fear? Can the horror which we all experience when we contemplate the possibility
of nuclear extinction mobilize in us sufficient libidinal energy to resist the archetypes of war? Certainly, the moment we become
blasé about the possibility of holocaust we are lost. As long as horror of nuclear exchange remains uppermost we can recognize
that nothing is worth it. War becomes the impossible option. Perhaps horror, the experience of horror, the consciousness of
horror, is our only hope. Perhaps horror alone will enable us to overcome the otherwise invincible attraction of war." Thus I also
continue engaging in nuclear weapons work to help fire that world-historical warning shot I mentioned above, namely, that as our
beneficial technologies become more powerful, so will our weapons technologies, unless genuine peace precludes it. We must
build a future more peaceful than our past, if we are to have a future at all, with or without nuclear weapons — a fact we had
better learn before worse things than nuclear weapons are invented. If you're a philosopher, this means that I regard the nature
of humankind as mutable rather than fixed, but that I think most people welcome change in their personalities and cultures with
all the enthusiasm that they welcome death — thus, the fear of nuclear annihilation of ourselves and all our values may be what
we require in order to become peaceful enough to survive our future technological breakthroughs.
at fear of nukes: Peace and Survival

Fear motivates people to pursue constructive means to sustain peace and prevent large-scale catastrophe.
1.
2. Lifton 01 (Robert Jay, Distinguished Professor of Psychiatry and Psychology at John Jay College,
Illusions of the second nuclear age, World Policy Journal. New York: Spring 2001. Vol. 18, Iss. 1;  pg. 25, 6 pgs)
3.
The trouble is that in other ways the dangers associated with nuclear weapons are greater than ever: the continuing weapons--
centered policies in the United States and elsewhere; the difficulties in controlling nuclear weapons that exist under unstable
conditions (especially in Russia and other areas of the former Soviet Union);2 and the eagerness and potential capacity of
certain nations and "private" groups to acquire and possibly use the weapons. In that sense, the nuclear quietism is perilous. Or,
to put the matter another way, we no longer manifest an appropriate degree of fear in relation to actual nuclear danger. While
fear in itself is hardly to be recommended as a guiding human emotion, its absence in the face of danger can lead to
catastrophe. We human animals have built-in fear reactions in response to threat. These reactions help us to protect ourselves-
to step back from the path of a speeding automobile, or in the case of our ancestors, from the path of a wild animal. Fear can be
transmuted into constructive planning and policies: whether for minimizing vulnerability to attacks by wild animals, or for more
complex contemporary threats. Through fear, ordinary people can be motivated to pursue constructive means for sustaining
peace, or at least for limiting the scope of violence. Similarly, in exchanges between world leaders on behalf of preventing large-
scale conflict, a tinge of fear-sometimes more than a tinge- can enable each to feel the potential bloodshed and suffering that
would result from failure. But with nuclear weapons, our psychological circuits are impaired. We know that the weapons are
around-and we hear talk about nuclear dangers somewhere "out there" -but our minds no longer connect with the dangers or
with the weapons themselves. That blunting of feeling extends into other areas. One of the many sins for which advocates of
large nuclear stockpiles must answer is the prevalence of psychic numbing to enormous potential suffering, the blunting of our
ethical standards as human beings. In the absence of the sort of threatening nuclear rhetoric the United States and Russia
indulged in during the 1980s, we can all too readily numb ourselves to everything nuclear, and thereby live as though the
weapons pose no danger, or as though they don't exist.
at nucelar numbing: plan solves impact

Advocating a plan to address harms of nuclear war overcomes impact of numbing.

Sandman and Valenti 86 (Peter and JoAnn, Professor of Human Ecology at Rutgers and Preeminent Risk Communications
Expert published over 80 articles and books on various aspects of risk communication, Scared stiff — or scared into action, ,
Bulletin of the Atomic Scientists, January 1986, pp. 12–16, http://www.psandman.com/articles/scarstif.htm)

WHEN THE MOVEMENT against nuclear weapons celebrates its heroes, a place of honor is reserved for Helen Caldicott, the
Australian pediatrician who revived Physicians for Social Responsibility (PSR) in 1978 and made it the vehicle for her
impassioned antinuclear crusade. In countless communities since then, Caldicott has briskly narrated the devastation that would
result if a small nuclear warhead exploded right here and now. Thousands of activists trace their movement beginnings to a
Helen Caldicott speech, wondering if it wouldn't help reverse the arms race just to make everyone sit through that speech — and
each week hundreds of activists do their best to give the speech themselves. Nonetheless, PSR Executive Director Jane Wales,
while acknowledging a huge debt to Caldicott, said in 1984 that the time for the “bombing runs” (as insiders call the speech) was
past. “We knew it was past when someone interrupted the speech one evening, actually interrupted it, and said, ’We know all
that, but what can we do?’” In a 1985 newsletter, similarly, Sanford Gottlieb of United Campuses to Prevent Nuclear War warned
that many students were “being numbed by the emphasis on nuclear blast, fire and radiation” in courses on nuclear war and
were therefore “feeling more impotent and depressed than before the class began.”(1) Perhaps the first broad awareness that
shock therapy may not be the best therapy came, ironically, in 1983 in the weeks preceding the broadcast of the television film
The Day After, when Educators for Social Responsibility and others worried that the program might do children more harm than
good. The Day After turned out to be less frightening than expected, but other films (Threads, Testament, and Caldicott’s own
The Last Epidemic) raise the same worry — and not just for children. In the following analysis of the fear of nuclear Armageddon
and its implications for antinuclear advocacy, we will argue that most people are neither apathetic about nuclear war nor actively
terrified of it but rather, in Robert Jay Lifton’s evocative phrase, “psychically numbed”; that it is ineffective to frighten audiences
who have found a refuge from their fears in numbness; and that there exist more effective keys to unlocking such paralysis. THE
CENTRAL ENIGMA of antinuclear activism is why everyone is not working to prevent nuclear war. Activists who can understand
those who disagree about what should be done are bewildered and frustrated by those who do nothing. Such inaction is
objectively irrational; as Caldicott asked in a 1982 cover article in Family Weekly, “Why make sure kids clean their teeth and eat
healthy food if they’re not going to survive?”(2) Advocates of all causes chafe at their neighbors’ lack of interest. When the issue
is something like saving whales or wheelchair access to public buildings, the problem is usually diagnosed as apathy.
Psychiatrist Robert Winer argues that the same is true of the nuclear threat, which most of us experience as remote, impersonal,
and vague. For Winer, “one of the genuinely tragic aspects of the nuclear situation is that immediacy may be given to us only
once and then it will be too late to learn.”(3) There is obviously some truth to this view. When asked to describe their images of
nuclear war, people do tend to come up with abstractions — and those with more concrete, immediate images are likely to be
antinuclear activists.(4)
at non-violence: could not solve the holocaust

Examples of non-violent resistance to the Nazis too small scale to be considered as an effective historical example.

Futterman, 91 (JAH, Livermore lab researcher, 1995, Mediation of the Bomb, online,
http://www.dogchurch.org/scriptorium/nuke0.html)

The Nazis, who with their "Master Race" ideology admitted only so-called "Aryans" to the category of human, provide an example
counter to that of the British. There were some successful acts of non-violent confrontation against the Nazis, like King Christian
of Denmark's public declaration that he would wear the yellow star if it were introduced in his country. He did so in response to
the Nazi practice of ordering Jews to wear yellow-starred armbands so that the Nazis could more easily isolate them from their
surrounding society. That many Danes followed their king's example helped camouflage many Jews until they could escape to
Sweden in fishing boats. [5] Now this resistance worked partly because the Nazis considered the Danes to be "Aryans" like
themselves. Had the Poles tried the same thing, the Nazis would have been perfectly happy to use the event as an excuse for
liquidating more Poles. Rather than awaken the Nazis' moral sense, non-violent confrontation on the part of the Poles would
probably have enabled the Nazis to carry out their agenda in Poland more easily. The other reason these acts succeeded was
that overwhelming violence of the Allies had stretched the Nazi forces too thin to suppress massive action by a whole populace,
and eventually deprived the Nazis of the time they needed to find other ways to carry out their "final solution." In other words,
non-violence resistance alone would have been very slow to work against the Nazis, once they had consolidated their power.
And while it slowly ground away at the evil in the Nazi soul, how many millions more would have died, and how much extra time
would have been given to Nazi scientists trying to invent atomic bombs to go on those V-2 rockets? The evil of Nazism may well
have expended itself, but perhaps after a real "thousand-year Reich," leaving a world populated only by blue-eyed blondes. In
other words, if the world had used non-violence alone against the Nazis, the results may have been much worse those of the
war.[6]
at non-violence: Violence Key to Peace

Non violence does not work against most enemies—genocide and mass murder will result.

Rummel, 81 (R.J., professor of political science at the University of Hawaii, The Just Peace,
http://www.hawaii.edu/powerkil1s/TJP.CHAP 10.HTM)

Now, peacemaking is not necessarily the best and most immediate response to conflict. Doubtlessly, some conflicts are
unnecessary, some needlessly intense and long-lasting. But some also are a real and unavoidable clash, the only means
through which one, as a partisan, can protect or further vital interests and achieve a more satisfactory and harmonious just
peace. For example, war against Hitler’s Germany from 1939 to 1945 cost millions lives, but it prevented the greater misery, the
terror, the executions, the cold-blooded murders which probably would have occurred had Hitler consolidated his control of
Europe and subjugated the Soviet Union. We always can end a conflict when we want by surrender. But some ideas are more
important than peace: Dignity. Freedom. Security. That is, peace with justice--a just peace. There is another relevant
qualification. The term "peacemaking" is well established, and I used it accordingly. Unfortunately, the verb "make" can imply that
peace is designed and constructed, as a house is planned and erected brick by brick or a road engineered and built. This
implication is especially seductive in this age when society is seen as manmade (rather than having evolved),9 and many believe
that communities should be centrally planned and managed. But peace is not constructed like a bridge. Peace emerges from the
balancing of individual mental fields. What the leaders of a group or nation honestly believe, actually want, truly are willing to get,
are really capable of achieving are unknown to others--and perhaps only partially to themselves. Nonetheless only they can best
utilize the information available to them to justly satisfy their interests. For a third party to try to construct and enforce an abstract
peace imposed on others is foolhardy. Such a peace would be uncertain, forestall the necessary trial-and-error balancing of the
parties themselves, and perhaps even create greater conflict later. The best peace is an outcome of reciprocal adjustments
among those involved. At most, peacemaking should ease the process. A final qualification. Pacifists believe that violence and
war cannot occur if people laid down their arms and refused to fight. But this ignores unilateral violence. Under threat, a state or
government may try to avoid violence by submission. The result may be enslavement, systematic execution, and elimination of
leaders and "undesirables." The resulting genocide and mass murder may ultimately end in more deaths than would have
occurred had people fought to defend themselves. I agree that in some situations nonviolence may be an effective strategy for
waging conflict,10 as in the successful Black civil rights demonstrations of the 1960s in America; or the successful nonviolent,
civil disobedience movement for Indian independence from Britain begun by Mahatma Gandhi in 1922. In some situations refusal
to use violence may avoid unnecessary escalation and ease peacekeeping. However, there are also conflicts, especially
involving actual or potential tyrants, despots, and other such oppressors, in which nonviolence cannot buy freedom from violence
by others or a just resolution of a dispute. Then a down payment on such a peace requires public display of one's capability and
a resolve to meet violent aggression in kind.
at cuomo: Negative Peace Key to Positive Peace

Preventing nuclear war is the absolute prerequisite to positive peace.

Folk, 78 Professor of Religious and Peace Studies at Bethany College, 78 [Jerry, “Peace Educations – Peace Studies :
Towards an Integrated Approach,” Peace & Change, volume V, number 1, Spring, p. 58]

Those proponents of the positive peace approach who reject out of hand the work of researchers and educators coming to the
field from the perspective of negative peace too easily forget that the prevention of a nuclear confrontation of global dimensions
is the prerequisite for all other peace research, education, and action. Unless such a confrontation can be avoided there will be
no world left in which to build positive peace. Moreover, the blanket condemnation of all such negative peace oriented research,
education or action as a reactionary attempt to support and reinforce the status quo is doctrinaire. Conflict theory and resolution,
disarmament studies, studies of the international system and of international organizations, and integration studies are in
themselves neutral. They do not intrinsically support either the status quo or revolutionary efforts to change or overthrow it.
Rather they offer a body of knowledge which can be used for either purpose or for some purpose in between . It is much more
logical for those who understand peace as positive peace to integrate this knowledge into their own framework and to utilize it in
achieving their own purposes. A balanced peace studies program should therefore offer the student exposure to the questions
and concerns which occupy those who view the field essentially from the point of view of negative peace.
threat construction: peace

Debates about threats in the academic world result in better policy-making—real threats can be confronted and risks can be
weighed.
Walt 91 – Professor of Political Science, University of Chicago – 1991 (Stephen, INTERNATIONAL STUDIES QUARTERLY, p.
229-30)

A recurring theme of this essay has been the twin dangers of separating the study of security affairs from the academic world or
of shifting the focus of academic scholarship too far from real-world issues. The danger of war will be with us for some time to
come, and states will continue to acquire military forces for a variety of purposes. Unless one believes that ignorance is
preferable to expertise, the value of independent national security scholars should be apparent. Indeed, history suggests that
countries that suppress debate on national security matters are more likely to blunder into disaster, because misguided policies
cannot be evaluated and stopped in time. As in other areas of public policy, academic experts in security studies can help in
several ways. In the short term, academics are well placed to evaluate current programs, because they face less pressure to
support official policy. The long-term effects of academic involvement may be even more significant: academic research can help
states learn from past mistakes and can provide the theoretical innovations the produce better policy choices in the future.
Furthermore, their role in training the new generation of experts gives academics an additional avenue of influence.

Risk in the international system is inevitable—the goal should be to weigh the impacts of action vs inaction in the face of a
particular threat.
Harvard Nuclear Study Group 83 (Living with Nuclear Weapons, p.16-7)

When President John F. Kennedy was shown irrefutable evidence of the Soviet missile emplacement – U-2 photographs of the
missile bases in Cube – he and his advisors discussed the matter for six days before deciding on an American response to the
challenge. The decision, to place a naval blockade around the island, was not a risk-free response. This, Kennedy honestly
admitted to the nation the night of October 22, 1962: My fellow citizens, let no one doubt this is a difficult and dangerous effort on
which we have set out. No one can foresee precisely what course it will take… But the great danger of all would be to do nothing.
Why did the president believe that “to do nothing” about the missiles in Cuba would be an even greater danger than accepting
the “difficult and dangerous” course of the blockade? He accepted some risk of war in the long run, by discouraging future Soviet
aggressive behavior. Inaction might have led to an even more dangerous future. This the president also explained that night in
his address to the nation: [This] sudden, clandestine decision to station weapons for the first time outside Soviet soil – is a
deliberate provocative and unjustified change in the status quo which cannot be accepted by this country if our courage and our
commitments are ever to be trusted by either friend or foe. The 1930’s taught us a clear lesson: Aggressive conduct, if allowed to
grow unchecked and unchallenged, ultimately leads to war. The American government managed the 1962 crisis with skill and
restraint – offering a compromise to the Soviets and giving them sufficient time to call back their missile-laden ships, for example
– and the missiles were withdrawn from Cuba. The president carefully supervised American military actions to ensure that his
orders were not misunderstood. He did not push his success too far or ignore the real risks of war. The point here is not, to make
the blockade a model for American action in the future: different circumstances may call for different policies. Rather the point is
to underline the persistence of risk in international affairs. Every proposed response to the Soviet action – doing nothing,
enforcing the blockade, or invading Cuba – entailed some risk of nuclear war. Kennedy’s task – and we think his success – was
to weigh accurately the risks entailed in each course and decide on policy accordingly.
threat construction: prevents escalation

Confronting threats early prevents escalation—WWII proves.

Yoon 03 – Professor of International Relations at Seoul National University; former Foreign Minister of South Korea –
2003(Young-Kwan, “Introduction: Power Cycle Theory and the Practice of International Relations”, International Political Science
Review 2003; vol. 24; p. 7-8)

In history, the effort to balance power quite often tended to start too late to protect the security of some of the individual states. If
the balancing process begins too late, the resulting amount of force necessary to stop an aggressor is often much larger than if
the process had been started much earlier. For example, the fate of Czechoslovakia and Poland showed how non-intervention or
waiting for the “automatic” working through of the process turned out to be problematic. Power cycle theory could also
supplement the structure-oriented nature of the traditional balance of power theory by incorporating an agent-oriented
explanation. This was possible through its focus on the relationship between power and the role of a state in the international
system. It especially highlighted the fact that a discrepancy between the relative power of a state and its role in the system would
result in a greater possibility for systemic instability. In order to prevent this instability from developing into a war, practitioners of
international relations were to become aware of the dynamics of changing power and role, adjusting role to power. A
statesperson here was not simply regarded as a prisoner of structure and therefore as an outsider to the process but as an agent
capable of influencing the operation of equilibrium. Thus power cycle theory could overcome the weakness of theoretical
determinism associated with the traditional balance of power. The question is often raised whether government decision-makers
could possibly know or respond to such relative power shifts in the real world. According to Doran, when the “tides of history”
shift against the state, the push and shove of world politics reveals these matters to the policy-maker, in that state and among its
competitors, with abundant urgency. (2) The Issue of Systemic Stability Power cycle theory is built on the conception of changing
relative capabilities of a state, and as such it shares the realist assumption emphasizing the importance of power in explaining
international relations. But its main focus is on the longitudinal dimension of power relations, the rise and decline of relative state
power and role, and not on the static power distribution at a particular time. As a result, power cycle theory provides a
significantly different explanation for stability and order within the international system. First of all, power cycle theory argues that
what matters most in explaining the stability of the international system or war and peace is not the type of particular international
system (Rosecrance, 1963) but the transformation from one system to another. For example, in the 1960s there was a debate on
the stability of the international system between the defenders of bipolarity such as Waltz (1964) and the defenders of multi-
polarity such as Rosecrance (1966), and Deutsch and Singer (1964). After analyzing five historical occasions since the origin of
the modern state system, Doran concluded that what has been responsible for major war was not whether one type of system is
more or less conducive to war but that instead systems transformation itself led to war (Doran, 1971). A non-linear type of
structural change that is massive, unpredicted, devastating to foreign policy expectation, and destructive of security is the trigger
for major war, not the nature of a particular type of international system.
threat construction: threats real

Some states are genuine threats.

Kydd 97 – Professor of Political Science of California, Riverside, SECURITY STUDIES, Autumn 1997 p. 154

As for the Second World War, few structural realists will make a sustained case the Hitler was genuinely motivated by a rational
pursuit of security for Germany and the other German statesmen would have responded in the same way to Germany’s
international situation. Even Germen generals opposed Hitler’s military adventurism until 1939; it is difficult to imagine a less
forceful civilian leader overruling them and leading Germany in an oath of conquest. In the case of the cold war, it is again
difficult to escape the conclusion that the Soviet Union was indeed expansionist before Gorbachev and not solely motivated by
security concerns. The increased emphasis within international relations scholarship on explaining the nature and origins of
aggressive expansionists states reflects a growing consensus that aggressive states are at the root of conflict, not security
concerns.
threat construction: reps irrelevant

Representations of state action cannot change realism, and even if they could, we have no way of knowing if they new system
would be any better.

Mearsheimer, 95 – Professor of Political Science, University of Chicago – 1995 (John, INTERNATIONAL SECURITY, p. 91-2)

The most revealing aspect of Wendt’s discussion is that he did not respond to the two main charge leveled against critical theory
in “False Promise.” The first problem with critical theory is that although the theory is deeply concerned with radically changing
state behavior, it says little about how change comes about. The theory does not tell us why particular discourses become
dominant and other fall by the wayside. Specifically, Wendt does not explain why realism has been the dominant discourse in
world politics for well over a thousand years, although I explicitly raised the question in “False Promise” (p. 42). Moreover, he
shed no light on why the time is ripe for unseating realism, nor on why realism is likely to be replaced by a more peaceful,
communitarian discourse, although I explicitly raised both questions. Wendt’s failure to answer these questions has important
ramifications for his own arguments. For example, he maintains that if it is possible to change international political discourse and
alter state behavior, “then it is irresponsible to pursue policies that perpetuate destructive old orders [i.e., realism], especially if
we care about the well-being of future generation.” The clear implication here is that realists like me are irresponsible and do not
care much about the welfare of future generations. However, even if we change discourses and move beyond realism, a
fundamental problem with Wendt’s argument remains: because his theory cannot predict the future, he cannot know whether the
discourse that ultimately replaces realism will be more benign than realism. He has no way of knowing whether a fascistic
discourse more violent than realism will emerge as the hegemonic discourse. For example, he obviously would like another
Gorbachev to come to power in Russia, but a critical theory perspective, defending realism might very well be the more
responsible policy choice.
A2: Love Alternative

Love is impossible as long as domination exists, only the plan solves the root cause
Crass anarchist/activist against racism 2k3 (Chris, “Going To Places That Scare Me: Personal Reflections On
Challenging Male Supremacy” http://colours.mahost.org/articles/crass15.html)

This isn’t a confessional so that I will be forgiven. This is an on-going struggle to be honest about how
deeply shaped I am by patriarchy and these systems of oppression. Patriarchy tears me up. I have so many
fears about whether or not I’m capable of being in healthy loving relationships. Fears about whether or not I can be
genuinely honest and connected with myself so that I can then open up and share with others. Fears about organizing
to genuinely build and share power with others The scars of patriarchy are on every single person I interact with and
when I push myself to see it, to really look and take the time to think about it, I’m filled with sadness and rage. bell
hooks, in her book All About Love, writes that love is impossible where the will to dominate exists.
Can I genuinely love? I want to believe. I want to believe in a political practice for gendered privileged men forged
in opposition to patriarchy. I do believe that as we struggle against oppression, as we practice our
commitments, we actualize and express our humanity. There are moments, experiences and events
when I see patriarchy challenged by all genders and it shows what we can do. I believe that this is
our lives’ work and that at its core it’s a fight for our lives. And in this fight we realize that even in the face
of these systems of oppression, our love, beauty, creativity, passion, dignity and power grows.
We can do this. No alt – hooks has zero political strategy Samuel author/reviewer of books 1996 (Julian,
Review of “The Fact of Blackness: Frantz Fanon and Visual Representation” http://www.indiastar.com/samuel.htm)
Here is the thinking of the completely delirious American bell hooks -- another super-salaried anti-
colonialist: "In love. I was thinking a lot about the place of empathy in any kind of ethic of care and the notion that
part of how one embraces that larger you -- that you that Fanon uses -- is through the capacity to embrace the other
in some way. What does it mean if Fanon is unable to embrace the black female -- what part of himself remains
unembraced? How does the possibility of love or an ethic of care chart the path to this humanism that he poses as
redemptive?"pp. 106 Are these consequential and serious psychological insights? Is there anything
at all to be gained from "thinking" about bell hook's words? No. (This passage reminds me of the smell
of an epoch when people used to smear on patchouli oil). Need one really embrace questions of academic
freedom of speech and tenure? These passages offer sufficient proof that activists who have
anything contestory to say are not permitted anywhere near the university or art institutions.
Tenure protects complacent luminaries. Read's book is a quintessential dead end. There is no human
liberation here. It begins where Fanon began, not where Fanon left off. It is boring to see sloppy professors and
artists toying with Fanon's bones in the old-fashioned world of sexual politics, and in the wordy flatulence of
"Theory" devoted to more "Theory" and to more "Theory".
A2: Localism (Nayar)

Zero alternative --- breaking down ‘global orders’ fails and results in cataclysmic violence
Balakrishnan ‘03 (Uma, Department of Government and Politics – St. John’s University, “Taking Charge of the Future”, International
Studies Review, 5)

Re-Framing the International provides a perfect starting point for debates on the construction of the future. It raises a
number of interesting questions that need to be explored. Is it possible to create a global community without losing the focus on
the individual within this group? How does one balance the interests of larger actors like transnational corporations with those of
the community so that we do not exchange one set of absolute rules (embodied in static sovereignty) for another? Where do we
locate the norms that will underlie the new order, given the variety and seeming incoherence of demands from across the globe?
In spite of the great sense of hope that underlies Re-Framing the International, the nagging
question of how this can be accomplished without upheaval remains. Although the arguments for a peaceful
transition are logical, the contributors are unable to show how power can be transcended. Given the

current intransigence of the U nited S tates and the United Kingdom with respect to Iraq, it is difficult to
envision the triumph of logic without the thrust provided by cataclysmic events like those that have
characterized the past century. Even Nayar concedes Nayar ‘99 (Jayan, Critical Theorist, 9
Transnat'l L. & Contemp. Probs. 599, Lexis) And so, what might I contribute to the present collective exercise
toward a futuristic imaging of human possibilities? I am unsure. It is only from my view of the "world," after
all, that I can project my visions. These visions do not go so far as to visualize any "world" in its totality;
they are uncertain even with regard to worlds closer to home, worlds requiring transformatory actions all the
same. Instead of fulfilling this task of imagining future therefore I simply submit the following two "poems." Retreating into
localism siphons off and deflects crucial activist energies from centers of power --- the result is depoliticization
and uncontested rule by elites Boggs ‘97 (Carl, Professor of Political Science – National University, Theory &
Society 26, December, p. 760-1) Grassroots politics, of course, remains a significant part of any transformative agenda; clearly there is no iron law
favoring an enclave outcome will be difficult to avoid
, but in a depoliticized culture it . In many ways the dilemmas of local activism go
back to the origins of the American political system, which was set up to allow space for local participation apart from federal structures so that no amount of grass-
even where oppositional groups were able to carve out a local
roots mayhem would disturb the national political system. Thus,
presence, their influence on the national state was likely to be minimal owing to the complex maze of checks and
balances, overlapping forms of representation, legislative intricacies, and a cumbersome winner-take-all electoral system that pushes the two
main parties toward moderation. Over time, too, the national government became stronger and more bureaucratized, further reducing the scope of
local decision-making and rendering much local empowerment illusory. Meanwhile, the federal state, with its expanded role in the military,
foreign policy, and global economy, assumed ever greater control over people's lives. Such realities, along with constitutional and legal
obstacles to securing a national foothold, often compelled progressive movements to stress local organizing. At
the same time, as Mark Kann observes, community radicalism could actually serve elite interests by
siphoning off discontent and deflecting it away from the real centers of power.40 Like spiritual
politics, enclave activism can be understood as a reaction against the chaos of urban life and the eclipse of public space,
along with a rejection of normal politics itself. The globalizing pressures exerted on the economy and political system reinforce this trend.
Collective action within the enclave has less to do with rejuvenating public discourse, making policy, and
gaining levers of institutional power than with erecting barriers against outside intrusions, just as city-dwellers may look to gated
communities as a way of protecting themselves against the Hobbesian features of civil society. The end result of this type of populism is a
widespread turning-away from the concerns of power, governance, and citizen participation within the general community –
one of the hallmarks of a depoliticized society.
A2: Localism (Nayar)

Global thinking promotes local action and checks violence


Sachs ‘92 (Wolfgang, Green Movement Activist and Prof Science – Penn St., Development Dictionary, p. 109-
113)
But recognizing the pitfalls of global eco-managernent does not solve the dilemma which will stay with us in the decades to come. Both
alternatives— to think in categories of one world as well as not to think in such categories—are equally self-destructive. On the one hand, it is a
sacrilege in our age of cultural evaporation to apprehend the globe as a united, highly integrated world. On the other hand , a
vision of the
globe as a multitude of different and only loosely connected worlds cannot dispense with the
idea of ecumenism in the face of lurking violence and the devastation of nature. Not surprisingly,
calls for global consciousness abound. Given that local events can affect the conditions of life in remote places, these
calls aim at bringing into congruence the range of our responsibility with the range of our effects.
[Continues] People are seldom residents of only one mental space. They have the ability to change their point of view and to look with the other’s
eyes at themselves. in fact, people often hold multiple loyalties at one and the same time. In many instances they combine rootedness in a place
with affiliation to a larger community. An inhabitant of medieval Cologne knew how to be a member of the Christian Church; a villager in
Rajasthan was aware of Bharat, Mother india; and Croatian peasants as well as the citizens of Cracow were part of the Habsburg empire. In a
similar vein, the one world may be thought of in terms of a meta-nation instead of a super-nation. It constitutes the horizon within which places
live out their density and depth. In
this perspective. ‘one world’ is not a design for more global planning,
but an ever present regulative idea for local action. Cosmopolitan localism seeks to amplify the
richness of a place while keeping in mind the rights of a multi-faceted world. It cherishes a particular
place, yet at the same time knows about the relativity of all places. Global thinking key to check environmental collapse ---
impact is extinction Held ‘98 (David, Professor of Politics and Sociology – Open U., “Democracy and
Globalization”, Re-Imagining Political Community, Ed. Archibugi, p. 19-20) Contemporary
environmental problems are perhaps the clearest and starkest examples of the global shift in
human organization and activity, creating some of the most fundamental pressures on the efficacy of
the nation-state and state-centric politics. There are three types of problems at issue. First, there are shared
problems involving the global commons, that is, fundamental elements of our ecosystem. The clearest
examples of the environmental commons are the atmosphere, the climate system and the oceans
and seas. And among the most fundamental challenges here are global warming and ozone
depletion. A second category of global environmental problems involves the interlinked challenges of
demographic expansion and resource consumption. An example of the profoundest importance under this
category is desertification. Other examples include questions of biodiversity and challenges to the very existence of certain
species. A third category of problems is transboundary pollution of various kinds, such as acid rain or
river pollutants. More dramatic examples arise from the siting and operation of nuclear power plants, for instance,
Chernobyl. In response to the progressive development of, and publicity surrounding,
environmental problems, there has been an interlinked process of cultural and political
globalization as illustrated by the emergence of new cultural, scientific and intellectual networks;
new environmental movements with transnational organizations and transnational concerns; and
new institutions and conventions like those agreed in 1992 at the Earth Summit in Brazil. Not all
environmental problems are, of course, global. Such an implication would be quite false. But there has been a striking
shift in the physical and environmental circumstances - that is, in the extent and intensity of
environmental problems - affecting human affairs in general. These processes have moved
politics dramatically away from an activity which crystallizes simply around state and interstate
concerns.It is clearer than ever that the political fortunes of communities and peoples can no
longer be understood in exclusively national or territorial terms.
at global/local: perm

Local resistance must be combined with larger struggles to effectively resist.

Gills – 02 [Barry K, Chair of the World Historical Systems theory group of the International Studies Association and a faculty
affiliate of the Globalization Research Center of the University of Hawaii, “Democratizing Globalization and Globalizing
Democracy, May, Annals of the American Academy of Political and Social Science, May]

In this sense, we may conclude that we are living through the (gradual or sudden?) demise of the old world order and the (slow
or sudden?) birth of a new one. Economically, this new order is based on an increased level of global economic integration and
unison. Politically, however, it is premised on the need to translate grassroots participatory political action into increasingly
popular democratic forms of governance at local, national, regional, and global levels (Gills 2000c; 2001). Moreover, it is also
based on a real need to combine the peoples and social forces of North and South in new ways, bringing together new coalitions
drawn from movements around the world. The governments and the corporations of the world must now listen to and
accommodate the demands of the peoples of the whole world, who represent the voice of the governed. This new reality, which
in my view is an objective one and not mere idealism, therefore requires a new [*169] paradigm. This new paradigm of world
order must be based profoundly on multicivilizational dialogue and universal inclusion. Rather than a political order based on one
nation, we are moving toward the need for a political order based on one humanity, and only democratic norms can
accommodate such a form of governance.
at global/local: global resistance key

Local struggles are not enough—global impacts require coordinated, large scale responses too.

Best and Kellner, 01 - Assoc. Prof Phil. and Human. U Texas and Phil. Of Ed. Chair – 2001 (Steven and Douglas, “Postmodern
Politics and the Battle for the Future,” Illuminations, http://www.uta.edu/huma/illuminations/kell28.htm)

The emphasis on local struggles and micropower, cultural politics which redefine the political, and attempts to develop political
forms relevant to the problems and developments of the contemporary age is extremely valuable, but there are also certain
limitations to the dominant forms of postmodern politics. While an emphasis on micropolitics and local struggles can be a healthy
substitute for excessively utopian and ambitious political projects, one should not lose sight that key sources of political power
and oppression are precisely the big targets aimed at by modern theory, including capital, the state, imperialism, and patriarchy.
Taking on such major targets involves coalitions and multi-front struggle, often requiring a politics of alliance and solidarity that
cuts across group identifications to mobilize sufficient power to struggle against, say, the evils of capitalism or the state. Thus,
while today we need the expansion of localized cultural practices, they attain their real significance only within the struggle for the
transformation of society as a whole. Without this systemic emphasis, cultural and identity politics remain confined to the margins
of society and are in danger of degenerating into narcissism, hedonism, aestheticism, or personal therapy, where they pose no
danger and are immediately coopted by the culture industries.
A2: Disease K

Securitization of AIDS is a key factor in motivating those ignoring it into action


Singer, Senior Fellow at the Brookings Institution, 2002 (Peter W., “AIDS and International Security”, Survival, vol.44,
no.1, Spring, p. 145-158)

AIDS is a daunting threat, but not an unbeatable foe. It is a disease that is still preventable. The present
challenge is to support those programmes and leaders who are facing the hard issues of AIDS, while
encouraging those now shirking their duties to respond. Thinking about AIDS as a security threat
helps clarify that this scourge reaches beyond individual lives and deaths into the realm of violence
and war – and thus strengthens the case for serious action. Fighting AIDS is not just a matter of
altruism, but enlightened self interest.
A2: Orientalism (Said)

Said’s discourse and alternative links to his own criticism


Cliford 80[James, University of California at Santa Cruz. “Review: [Untitled]”. History and Theory, Vol. 19, No. 2. (Feb., 1980), pp. 204-
223.]

Discourse analysis is always, in a sense, unfair to authors. It is interested not in what they have to say
or feel as subjects, but is concerned merely with statements as related to other statements in a
fie1d.l1 Escaping an impression of unfairness and reductionism in this kind of analysis is a matter of
methodological rigor and stylistic tact. Foucault, at least, seldom appears unfair to authors because he never appeals
to any individual intentionality or subjectivity. "Hybrid perspectives" like Said's have considerably more
difficulty in escaping reductionism.1 Indeed, Said's methodological catholicity repeatedly blurs
his analysis. If he is advancing anthropological arguments, Orientalism appears as the cultural
quest for order. When he adopts the stance of a literary critic, it emerges as the processes of writing, textualizing,
and interpreting. As an intellectual historian Said portrays Orientalism as a specific series of influences
and schools of thought. For the psychohistorian Orientalist discourse becomes a representative series of
personal/historical experiences. For the Marxist critic of ideology and culture it is the expression of definite
political/economic power interests. Orientalism is also, at times, conflated with Western positivism;
with general definitions of the Primitive, with evolutionism, with racism. One could continue the list.
Said's discourse analysis does not, itself, escape the all-inclusive "Occidentalism" he specifically
rejects as an alternative to Orientalism (328). The privileging of the Orient as the oppressed culture ignores
other oppressed groups Halliday 93[Fred, Prof of IR at the London School of Economics. “Orientalism and its
Critics.” British Journal of Middle Eastern Studies 20:2] Secondly, the category of the 'Orient' is
rather vague, since in Orzentalzsm its usage implies that the Middle East is in some ways special, at
least in the kind of imperialist or oppressive writing produced about it. Racist or oppressive
writing is found about all subject peoples, whether they are Islamic or not, and there is nothing to choose between
then] The claim of a special European animosity to Arabs-let alone Palestinians-or to Muslims
does not bear historical comparison. Such ideas of persecution rest on some implicit yardstick, a comparative
massacrology in which the wrongs done to one people are greater. Such an approach is best avoided, but it may be pointed out
that the fate of the native people of the Americas, whose conquest was also presented as a crusade, was far worse than that of the
peoples of 'Islam'. Equally spurious is the implication that the hypostatization and reification of the
Middle East are specific, whether by those writing from outside, or from within: anyone familiar with
the writing on Japan entitled Nzhonjiron and books such as Ruth Benedict's The Chrysanthemum and the Sword will be familiar
with similar themes-the special place of language studies, the search for the unchanging national character, the stress on the
specificities of the Japanese mind, the search for the true 'Japanese' position on women, or the emperor, or flower arrangement
or whatever;36 Russia too has had its share of such ahistorical analysis. Here again external authority and internal nationalism
collude to create a timeless, and particularist, discourse. About what people has it not been said 'They are like that, 'They will
never change' etc? If any people in the Middle East believe that in some way they have been singled
out by the West--but in its historic or contemporary forms, this is an unsustainable idea. The
thesis of some enduring, trans-liistorical, hostility to the Orient, the Arabs, the Islamic world, is
a myth-albeit one, as already indicated, which many in the region and the West find it
convenient to sustain.
***Gender***

at gender ir: no alt

Critiques of gender relations that do not pose concrete alternatives are destined to fail.

Caprioli, 04 (“Feminist IR Theory and Quantitative Methodology: A Critical Analysis” Mary Caprioli, Dept. of Political Science,
University of Tennessee. International Studies Review. Volume 42 Issue 1 Page 193-197, March 2004. http://www.blackwell-
synergy.com/links/doi/10.1111/0020-8833.00076).

If researchers cannot add gender to an analysis, then they must necessarily use a purely female-centered
analysis, even though the utility of using a purely female centered analysis seems equally biased. Such
research would merely be gendercentric based on women rather than men, and it would thereby provide
an equally biased account of international relations as those that are male-centric. Although one might
speculate that having research done from the two opposing worldviews might more fully explain international relations, surely an
integrated approach would offer a more comprehensive analysis of world affairs. Beyond a female-centric analysis,
some scholars (for example, Carver 2002) argue that feminist research must offer a critique of gender as a set
of power relations. Gender categories, however, do exist and have very real implications for individuals,
social relations, and international affairs. Critiquing the social construction of gender is important, but it
fails to provide new theories of international relations or to address the implications of gender for what
happens in the world.
at gender ir: no link

IR feminists vastly over simplify the diverse field of international relations literature—they need a specific link our aff.

Caprioli, 04 (“Feminist IR Theory and Quantitative Methodology: A Critical Analysis” Mary Caprioli, Dept. of Political Science,
University of Tennessee. International Studies Review. Volume 42 Issue 1 Page 193-197, March 2004. http://www.blackwell-
synergy.com/links/doi/10.1111/0020-8833.00076).

Conventional feminist IR scholars misrepresent the field of international relations in arguing that IR
scholarship as popularly accepted excludes alternative explanations of state behavior, including feminist
inquiry, that go beyond structural, state-focused models. Feminist IR theorists, among others, critique the IR field for
its state-centric approach and argue that ‘‘a world of states situated in an anarchical international system leaves little room for
analyses of social relations, including gender relations’’ (Tickner 2001:146). As a result, they appear to set up a straw man
by refusing to recognize the variety within ‘‘conventional’’ IR research . Indeed, as Jack Levy (2000) has
observed, a significant shift to societal-level variables has occurred, partly in response to the decline in the
systemic imperatives of the bipolar era. Certainly the democratic peace literature, particularly its normative explanation
(Maoz and Russett 1993; Dixon 1994), among other lines of inquiry, recognizes the role of social relations in explaining state
behavior. The normative explanation for the democratic peace thesis emphasizes the societal level values of
human rights, support for the rule of law, and peaceful conflict resolution in explaining the likelihood of
interstate conflict. Furthermore, dyadic tests of the democratic peace thesis rely ‘‘on an emerging
theoretical framework that may prove capable of incorporating the strengths of the currently predominant
realist or neorealist research program, and moving beyond it’ ’ (Ray 2000:311). In addition, theorizing and research
in the field of ethnonationalism has highlighted connections that domestic ethnic discrimination and violence have with state
behavior at the international level (Gurr and Harff 1994; Van Evera 1997; Caprioli and Trumbore 2003a, 2003b).

Arguing that any IR theory overwhelms the specifics of the situation is an over simplification re-creates the very hierarchies
feminists are trying to overcome.
Caprioli, 04 “Feminist IR Theory and Quantitative Methodology: A Critical Analysis” Mary Caprioli, Dept. of Political Science,
University of Tennessee. International Studies Review. Volume 42 Issue 1 Page 193-197, March 2004. http://www.blackwell-
synergy.com/links/doi/10.1111/0020-8833.00076).

There is little utility in constructing a divide if none exists. As Thomas Kuhn (1962) argues, common measures do exist across
paradigms that provide a shared basis for theory. It seems overly pessimistic to accept Karl Popper’s ‘‘Myth of Framework,’’
which postulates that ‘‘we are prisoners caught in the framework of our theories, our expectations, our past experiences, our
language, and that as a consequence, we cannot communicate with or judge those working in terms of a different paradigm’’
(Neufeld 1995:44). Some feminists (for example, Tickner 1996, 2001; Peterson 2002; Steans 2003) appear to embrace
this ‘‘Myth of Framework’’ by accentuating the differences between the perspectives of feminist and IR
theorists based on their past experiences and languages and criticize IR theorists for their lack of
communication with feminist IR scholars. Ironically, the ‘‘Myth of Framework’’ shares a number of
assumptions with Hobbes’s description of the state of nature that feminists routinely reject. The ‘‘Myth
of Framework’’ assumes no middle ground scholars are presumably entrenched in their own worldviews
without hope of compromise or the ability to understand others’
worldviews. If this is the case, scholars are doomed to discussions with likeminded individuals rather than having a
productive dialogue with those outside their own worldview. Scholars who accept the ‘‘Myth of Framework’’ have
essentially created a Tower of Babel in which they choose not to understand each other’s language. The
acceptance of such a myth creates conflict and establishes a hierarchy within international relations
scholarship even though conventional feminists theoretically seek to identify and eradicate conflict and
hierarchy within society as a whole.

at gender ir: perm


The perm solves best: IR criticism is only effective when it is combined with practical policy making.

Keohane, 98 (“Beyond Dichotomy: Conversations Between International Relations and Feminist Theory” Robert O. Keohane,
Duke University. International Studies Quarterly 42, 193-198. http://www.blackwell-synergy.com/action/showPdf?
submitPDF=Full+Text+PDF+%2889+KB%29&doi=10.1111%2F0020-8833.00076

The problem with Tickner’s dichotomies, however, goes much deeper . The dichotomies should be replaced by
continua, with the dichotomous characterizations at the poles. Each analyst of world politics has to locate herself or himself
somewhere along the dimensions between critical and problem-solving theory, nomothetic and narrative epistemology, and a
social or structural conception of international relations. In my view, none of the ends of these continua are the optimal places to
rest one’s perspective. Criticism of the world, by itself, becomes a jeremiad, often resting implicitly on a
utopian view of human potential. Without analysis, furthermore, it constitutes merely the opinion of one
or a number of people. On the other hand, implicit or complacent acceptance of the world as it is would rob the study of
international relations of much of its meaning. How could one identify “problems” withough criticism at some level? The issue is
not problem-solving vs. critical theory- a convenient device for discarding work that one does not wish to
accept- but how deeply the criticism should go. For example, most students of war study it because they
hope to expose its evils or to control it in some way: few do so to glorify war as such. But the depth of
their critique varies. Does the author reject certain acts of warfare, all warfare, all coercion, or the system of states itself?
The deeper the criticism, the more wide-ranging the questions. Narrowly problem-solving work, as in
much policy analysis, often ignores the most important causal factors in a situation because they are not
manipulable in the short run. However, the more critical and wide-ranging an author’s perspective, the
more difficult it is to do comparative empirical analysis. An opponent of some types of war can compare the causes
of different wars, as a way to help to eliminate those that are regarded as pernicious; but the opponent of the system of states
has to imagine the counterfactual situation of a system without states.
at gender ir: 3rd world feminism

Feminism that prioritizes theory over material experience excludes the voices of third world feminists.

Oloka-Onyango and Tamale, 95 “The Personal is Political” or Why Womens Rights are Indeed Human Rights. J. Oloka-Onyango
and Slyvia Tamale. Human Rights Quarterly 17.4, 691-731 . Joe Oloka-Onyango is a Senior Lecturer at the Faculty of Law,
Makerere University, Uganda, and spent the 1994-1995 academic year as a Visiting Professor at the University of
Minnesota.Sylvia Tamale holds law degrees from Makerere University (Uganda) and Harvard Law School. She is currently a
doctoral student in Sociology and Feminist Studies at the University of Minnesota, Projest Muse).

In tandem with such an approach, feminists in third world contexts must be wary of cooptation and
exploitation--a trait of western societies that appears to not respect boundaries of sex--particularly
because the dominant mode of international feminism reflects the dominant character and color of
international relations, Bourgeois/white, often predatory, and paternalistic. 26 As Maivân Lâm has recently
pointed out in an article aptly entitled, Feeling Foreign in Feminism, the agenda of Western feminism appears not only to
be off target, but also "filmic." 27 According to Lâm, Western feminism is "too cleanly and detachedly
representational, with little connection to the ongoing lives of women who have experienced racial or
colonial discrimination. . . ." 28 Vasuki Nesiah is even more critical of the transposition of Western feminism onto the
international scene because it ignores "global contradictions" 29 by emphasizing the commonality of
women's experience. Instead, she urges theorists to look at gender identities as being "continually
reconstituted through social processes."

Feminists that prioritize theory over reform continue the marginalization of third world women.

Oloka-Onyango and Tamale, 95 “The Personal is Political” or Why Womens Rights are Indeed Human Rights. J. Oloka-Onyango
and Slyvia Tamale. Human Rights Quarterly 17.4, 691-731 . Joe Oloka-Onyango is a Senior Lecturer at the Faculty of Law,
Makerere University, Uganda, and spent the 1994-1995 academic year as a Visiting Professor at the University of
Minnesota.Sylvia Tamale holds law degrees from Makerere University (Uganda) and Harvard Law School. She is currently a
doctoral student in Sociology and Feminist Studies at the University of Minnesota, Projest Muse).

In a succinct treatment of the issue elsewhere, Hilary Charlesworth points out that feminists "should aim not for respectability and
acceptance through developing a specialized branch of women's international law because this would leave the international
legal system unchanged. We must work to change the heartland of international law and its institutions ." 51 However, in her
essay in Women's Rights, Charlesworth devotes a scant paragraph to the issue of third world feminism
and even then, only in its relationship to first world feminism. 52 A more inclusive examination would have
incorporated the views of Southern feminists on the international legal and political regime. 53 Third world discourse must
be integrated directly into the critique of dominant structures of knowledge and power in academia, rather than "added in
and stirred" as an afterthought. This is particularly necessary in light of the assault on southern institutions of advanced learning
and intellectual culture by the International Monetary Fund (IMF) and World Bank structural adjustment policies (SAPs). 54 Of
course, internationalist works that include and are sensitive to the concerns of third world scholars are far
better than those which presume to speak to and for them. Unfortunately, the latter are in far greater
abundance. Such imbalance imports a special duty among those who experience similar conditions of
exclusion in academia to allow for the expression of marginalized voices beyond the "particularities" of
their geographical contexts. 55 In short, the "gates" must be opened even wider to ensure that international feminist theory
is truly decolonized and thematically internationalized. Otherwise, we remain with the same problem as the debacle
of WID--nominal participation and continuing marginalization--or just lip-service to multiculturalism and
universal human rights.
*** Ethics ***

A2: Obligation To Other (Levinas)

Infinite obligation to the other produces political deadlock and philosophical irrelevance
Rorty ‘98 (Richard, Professor of Philosophy, Achieving Our Country, p. 94-7)

These futile attempts to philosophize one's way into political relevance are a symptom of what
happens when a Left retreats from activism and adopts a spectatorial approach to the problems of its
country. Disengagement from practice produces theoretical hallucinations. These result in an intellectual
environment which is, as Mark Edmundson says in his book Nightmare on Main Street, Gothic. The cultural Left is haunted by ubiquitous
specters, the most frightening of which is called "power." This is the name of what Edmund- son calls Foucault's "haunting agency, which is
everywhere and nowhere, as evanescent and insistent as a resourceful spook."10 In its Foucauldian usage, the term "power" denotes an agency
which has left an indelible stain on every word in our language and on every institution in our society. It is always already there, and cannot be
spotted coming or going. One might spot a corporate bagman arriving at a congressman's office, and perhaps block his entrance. But one cannot
block off power in the Foucauldian sense. Power is as much inside one as outside one. It is nearer than hands and feet. As Edmundson says: one
cannot "confront power; one can only encounter its temporary and generally unwitting agents . . . [it] has capacities of motion and transformation
that make it a preternatural force."1' Only interminable individual and social self-analysis, and perhaps not even that, can help uses- cape from the
infinitely fine meshes of its invisible web. The ubiquity of Foucauldian power is reminiscent of the ubiquity of Satan, and thus of the ubiquity of
original sin— that diabolical stain on every human soul. I argued in my first lecture that the repudiation of the concept of sin was at the heart of
Dewey and Whitman's civic religion. I also claimed that the American Left, in its horror at the Vietnam War, rein-vented sin. It reinvented the old
religious idea that some stains are ineradicable. I now wish to say that, in committing itself to what it calls "theory," this Left has gotten
something which is entirely too much like religion. For the cultural Left has come to believe that we must place our country within a theoretical
frame of reference, situate it within a vast quasi-cosmological perspective. Stories about the webs of power and the insidious influence of a
hegemonic ideology do for this Left what stories about the Lamanites did for Joseph Smith and what stories about Yakkub did for Elijah
Muhammad. What stories about blue-eyed devils are to the Black Muslims, stories about hegemony and power are to many cultural leftists—the
only thing they really want to hear. To step into the intellectual world which some of these leftists inhabit is to move out of a world in which the
citizens of a democracy can join forces to resist sadism and selfishness into a Gothic world in which democratic politics has become a farce. It is
a world in which all the day lit cheerfulness of Whitmanesque hypersecularism has been lost, and in which "liberalism" and "humanism" are
synonyms for naivete—for an inability to grasp the full horror of our situation. I have argued in various books that the philosophers most often
cited by cultural leftists—Nietzsche, Heidegger, Foucault, and Derrida—are largely right in their criticisms of Enlightenment rationalism. I have
argued further that traditional liberalism and traditional humanism are entirely compatible with such criticisms. We can still be old- fashioned
reformist liberals even if, like Dewey, we give up the correspondence theory of truth and start treating moral and scientific beliefs as tools for
achieving greater human happiness, rather than as representations of the intrinsic nature of reality. We can be this kind of liberal even after we
turn our backs on Descartes, linguistify subjectivity, and see everything around us and within us as one more replaceable social construction. But
I have also urged that insofar as these antimetaphysical, anti-Cartesian philosophers offer a quasi-religious form of spiritual pathos, they should
be relegated to private life and not taken as guides to political deliberation. The
notion of "infinite responsibility,"
formulated by Emmanuel Levinas and sometimes deployed by Derrida—as well as Derrida's own frequent
discoveries of impossibility, unreachability, and unrepresentability— may be useful to some of us in our individual
quests for private perfection. When we take up our public responsibilities, however, the infinite
and the unrepresentable are merely nuisances. Thinking of our responsibilities in these terms is
as much of a stumbling-block to effective political organization as is the sense of sin.
Emphasizing the impossibility of meaning, or of justice, as Derrida sometimes does, is a temptation to
Gothicize—to view democratic politics as ineffectual, because unable to cope with preternatural
forces. Whitman and Dewey, I have argued, gave us all the romance, and all the spiritual uplift, we Americans need to go about our public
business. As Edmundson remarks, we should not allow Emerson, who was a precursor of both Whitman and Dewey, to be displaced by Poe, who
was a pre- cursor of Lacan. For purposes of thinking about how to achieve our country, we do not need to worry about the correspondence theory
of truth, the grounds of normativity, the impossibility of justice, or the infinite distance which separates us from the other. For those purposes, we
can give both religion and philosophy a pass. We can just get on with trying to solve what Dewey called "the problems of men." To think about
those problems means to refrain from thinking so much about otherness that we begin to acquiesce in what Todd Gitlin has called, in the title of a
recent book, "the twilight of common dreams." It means deriving our moral identity, at least in part, from our citizenship in a democratic nation-
state, and from leftist attempts to fulfill the promise of that nation.
A2: Obligation To Other (Levinas)

Constant ethic for the other allows for the West to assert our identity upon the other
without being seen as racist allowing for oppression.
Zizek, 2005 (Slavoj, “Smashing the neighbors face,” http://www.lacan.com/zizsmash.htm)
Levinas therefore secretly imputes to Spinoza an egotistic "subjectivist" notion of (my) existence - his anti-
Spinozean questioning of my right to exist is inverted arrogance: as if I am the center whose existence threatens all
others. So the answer should not be an assertion of my right to exist in harmony with and tolerance of others, but a
more radical claim: do I exist in the first place? Am I not, rather, a hole in the order of being? This brings
us to the ultimate paradox on account of which Levinas's answer is not sufficient: I am a threat to
the entire order of being not insofar as I positively exist as part of this order, but precisely insofar
as I am a hole in the order of being - as such, as nothing, I "am" a striving to reach out and
appropriate all (only a Nothing can desire to become Everything) - it was already Schelling who defined the
subject as the endless striving of the Nothing to become Everything. On the contrary, a positive living being
occupying a determinate space in reality, rooted in it, is by definition a moment of its circulation
and reproduction. Recall the similar paradox of that structures the Politically Correct landscape: people far
from the Western world are allowed to fully assert their particular ethnic identity without being
proclaimed essentialist racist identitarians (native Americans, blacks...); the closer one gets to the notorious
white heterosexual males, the more problematic this assertion is: Asians are still OK, Italians and Irish maybe, with
Germans and Scandinavians it is already problematic... However, such a prohibition of asserting the particular
identity of White Men (as the model of oppression of others), although it presents itself as the admission of
their guilt, nonetheless confers on them a central position: this very prohibition to assert their
particular identity makes them into the neutral medium, the place from which the truth about the
others' oppression is accessible. Political engagement to mediate with the other creates the
ethical obligation to politics displacing our true connection to the original second party
allowing for desrtruction of it because it disturbs the natural violence Zizek, 2005 (Slavoj,
“Smashing the neighbors face,” http://www.lacan.com/zizsmash.htm) One should therefore assume the risk of
countering Levinas's position with a more radical one: others are primordially an (ethically) indifferent
multitude, and love is a violent gesture of cutting into this multitude and privileging a One as the neighbor, thus
introducing a radical imbalance into the whole. In contrast to love, justice begins when I
remember the faceless many left in shadow in this privileging of the One. Justice and love are thus
structurally incompatible: justice, not love, has to be blind, it has to disregard the privileged One whom I “really
understand." What this means is that the Third is not secondary: it is always-already here, and the
primordial ethical obligation is towards this Third who is NOT here in the face to face
relationship, the one in shadow, like the absent child of a love-couple. This not simply the Derridean-
Kierkegaardian point that I always betray the Other because toute autre est un autre, because I have to make a
CHOICE to SELECT who my neighbor is from the mass of the Thirds, and this is the original
sin-choice of love. The structure is similar to the one described by Emile Benveniste apropos verbs: the
primordial couple is not active-passive, to which the neutral form is then added, but active and neutral (along the
axis of engaged-disengaged). The primordial couple is Neutral and Evil (the choice which disturbs the
neutral balance), or, grammatically, impersonal Other and me - "you" is a secondary addition. 4
A2: Obligation To Other (Levinas)

Instead of a face to face contact with the other we should suspend the hold of the face in
order to refocus it to uproot justice liberating it from this umbrella link that always
renders them beneath us. The affirmative’s engagement is impossible because of our
limited capacity to relate to the other through the “third”.
Zizek, 2005 (Slavoj, “Smashing the neighbors face,” http://www.lacan.com/zizsmash.htm)

This brings us to the radical anti-Levinasian conclusion: the trueethical step is the one BEYOND the face
of the other, the one of SUSPENDING the hold of the face: the choice AGAINST the face, for
the THIRD. This coldness IS justice at its most elementary. Every preempting of the Other in the guise of his face
relegates the Third to the faceless background. And the elementary gesture of justice is not to show respect for
the face in front of me, to be open for its depth, but to abstract from it and refocus onto the
faceless Thirds in the background. It is only such a shift of focus onto the Third that effectively
uproots justice, liberating it from the contingent umbilical link that renders her »embedded« in a
particular situation. In other words, it is only such a shift onto the Third that grounds justice in the
dimension of universality proper. When Levinas endeavors to ground ethics in the Other's face, is he not still
clinging to the ultimate root of the ethical commitment, afraid to accept the abyss of the rootless Law as the only
foundation of ethics? Justice as blind thus means that, precisely, it cannot be grounded in the relationship to the
Other's face, i.e., in the relationship to the neighbor: justice is emphatically NOT justice for - with regard to - the
neighbor. This structure is irreducible: it is not that, while, in our empirical lives, the Third is irreducible, we
should maintain as a kind of regulative Idea the full grounding of ethics in the relationship to the Other's Face. Such
a grounding is not only empirically impossible, it is a priori impossible, since the limitation of
our capacity to relate to Other's faces is the mark of our very finitude. In other words, the limitation of
our ethical relation of responsibility towards the Other's face which necessitates the rise of the Third (the domain of
regulations) is a positive condition of ethics, not simply its secondary supplement. If we deny this - i.e., if we stick
to the postulate of a final translatability of the Third into a relation to Other's face -, we remain
caught in the »understanding«: one can »understand« everything, even the most hideous crime
has an »inner truth and beauty« when observed from within: recall the refined spiritual meditations of
the Japanese warriors. There is a weird scene in Hector Babenco's The Kiss of a Spider-Woman: in the German-
occupied France, a high Gestapo officer explains to his French mistress the inner truth of the Nazis, how they are
guided in what may appear brutal military interventions by an inner vision of breath-taking goodness - we never
learn in what, exactly, this inner truth and goodness consist; all that matters is this purely formal gesture of asserting
that things are not what they seem (brutal occupation and terror), that there is an inner ethical truth which redeems
them... THIS is what the ethical Law prohibits: justice HAS to be blind, ignoring the inner truth. Recall the famous
passage from Graham Greene's The Power and the Glory:
A2: Badiou

Badiou has no way to separate between different types of ethics, causing him to ignore
human rights
Brown 04 [Nicholas, Professor at University of Illinois at Chicago. “{∅} ∈ {$} ? Or, Alain Badiou and Slavoj Žižek, Waiting for
Something to Happen” CR: The New Centennial Review 4.3]

This apparatus is a powerful lens, and there can be no doubt that Badiou is describing something important; perhaps it is even an aspect of evil.
But is it really Evil (Mal) itself? Badiou's evil, like his truth, is indifferent to content, a merely formal label. In its
formalism, its insistence on fidelity to any Event whatever—on "ethical consistency" itself as a value— Badiou's
good is almost an
aesthetic rather than an ethical category. (At one point, in an echo of Kant's purposeless purpose, ethical consistency is even
described as "disinterested interest.") While there is something undeniably attractive in ethical consistency (and
something ugly in its lack), the most important thing for a modern ethics may be to push these
sentimental considerations aside. The value of ethical consistency is authorized by Lacan's well-known dictum not to give up on
one's desire [ne pas céder sur son désir]. But we should not forget that this maxim derives from the reading of Antigone in Séminaire VII. Yes,
Sophocles' Antigone, in her awful ethical consistency, is a captivating figure. Brecht's Galileo, on the other hand, in his opportunism and
wavering inconsistency, is a bit distasteful. But Antigone is a reactionary, and Galileo invents physics. Further, Badiou
has no way of
sorting out different evils beyond his tripartite division. Ethics tells us what Nazism and
scientific obscurantism [End Page 300] have in common. But an ethics would have to be able to tell
them apart. The distinction between, say, the abandonment of a social movement by its leader
and the abandonment of a poem by its author cannot be made without some kind of qualitative
supplement. Since, as we shall see, Badiou's philosophy is predicated precisely on the subtraction from consideration of all qualitative
predicates, this supplement can only be vulgar, nonphilosophical. Perhaps the supplement it requires is the language of
human rights, which, whatever its faults, can tell the difference between a concentration camp
and a creationist textbook. (What if, as Žižek suggests, the international war-crimes tribunal were simply to refuse the de facto
bifurcation of the subject of human rights which is currently written into its constitution: "arrest Kissinger or shut up!" [Revolution, 266]?) Or
perhaps, genuinely spurning such a supplement, it is really no different than Pauline faith. Since
Badiou himself uses the
language of grace when speaking of the Event, he cannot regard it as very damning that his
conception of the Event shares something with religious revelation. But can we be satisfied with
an Ethics that remains in the "category of pious discourse"?
A2: Badiou

Perm solves --- Badiou doesn’t criticize state action


Hallward 02[Peter, Lecturer in French Department at King’s College “BADIOU'S POLITICS: EQUALITY AND
JUSTICE” http://culturemachine.tees.ac.uk/Cmach/Backissues/j004/Articles/hallward.htm]

We know that Badiou’s early and unequivocally hostile attitude to the state has considerably evolved. Just how far it has evolved remains a little
unclear. His conception of politics remains resolutely anti-consensual, anti-‘re-presentative’, and thus anti-democratic (in the ordinary sense of
the word). ‘A philosophy today is above all something that enables people to have done with the "democratic" submission to the world as it is’
(‘Entretien avec Alain Badiou’, 1999: 2). But he seems more willing, now, to engage with this submission on its own terms. La Distance politique
again offers the most precise points de repère. On the one hand, the OP remains suspicious of any political campaign – for instance, electoral
contests or petition movements – that operates as a ‘prisoner of the parliamentary space’ (LDP, 19-20.04.96: 2). It remains ‘an absolute necessity
[of politics] not to have the state as norm. The separation of politics and state is foundational of politics.’ On the other hand,
however, it is now equally clear that ‘their separation need not lead to the banishment of the state from the field
of political thought’ (LDP, 6.05.93: 1).24 The OP now conceives itself in a tense, non-dialectical ‘vis-à-vis’ with the state, a stance that
rejects an intimate cooperation (in the interests of capital) as much as it refuses ‘any antagonistic conception of their operation, any conception
that smacks of classism.’ There
is to no more choice to be made between the state or revolution; the ‘vis-
à-vis demands the presence of the two terms and not the annihilation of one of the two’ (LDP,
11.01.95: 3-4). Indeed, at the height of the December ‘95 strikes, the OP recognised that the only contemporary movement of ‘désétatisation’
with any real power was the corporate-driven movement of partial de-statification in the interests of commercial flexibility and financial mobility.
The
Unsurprisingly, ‘we are against this withdrawal of the state to the profit of capital, through general, systematic and brutal privatisation.
state is what can sometimes take account of people and their situations in other registers and by
other modalities than those of profit. The state assures from this point of view the public space
and the general interest. And capital does not incarnate the general interest’ (LDP, 15.12.96: 11). Coming
from the author of Théorie de la contradiction, these are remarkable words. The next question is whether the very possibility of such prescription
according to the general interest does not itself presuppose that same liberal-parliamentary realm upon whose systematic vilification its own
critical distance depends. What kind of state can respond ‘responsibly’ to political prescriptions, if not one closely responsible to electoral
pressure? Badiou maintains that the old socialist states, as states, were ‘more sensitive’ to workers’ strikes than are today’s parliamentary states –
the great example being the Solidarity campaign in Poland (Letter to the author, 9.12.98).25 But when the OP ventures into the vexing domain of
constitutional reform, it is to propose very explicitly parliamentary procedures: an end to a separately elected president (and so an end to the
possibility of cohabitation); a purely cosmetic head of state; only one major forum for elections (a legislative chamber of deputies); assurance that
the head of government is always the head of the dominant party; and finally, a guarantee ‘that there is always a dominant party’, thanks to some
kind of first-past-the-post electoral system. The whole package is to be softened with calls for more open government and the rule of law
(‘Proposition de réforme de la Constitution’, LDP, 12.02.95: 5-6). The once Maoist Organisation Politique now recommends something almost
exactly like the British Constitution! At this point, the reader has to wonder if the OP’s policy of strict non-participation in the state really stands
up. The OP declares with some pride that ‘we never vote’, just as ‘in the factories, we keep our distance from trade unionism’ (LDP, 12.02.95:
1).26 The OP consistently maintains that its politics of prescription requires a politics of ‘non-vote’. But
why, now, this either/or?
Once the state has been acknowledged as a possible figure of the general interest, then surely it
matters who governs that figure. Regarding the central public issues of health and education, the OP maintains, like most
mainstream socialists, that the ‘positive tasks on behalf of all are incumbent upon the state’ (LDP, 10.11.94: 1).27
That participation in the state should not replace a prescriptive externality to the state is obvious
enough, but the stern either/or so often proclaimed in the pages of La Distance politique reads today like a displaced
trace of the days when the choice of ‘state or revolution’ still figured as a genuine alternative.
A2: Badiou

Badiou wrongly universalizes, destroy any chance for a successful alternative


Rothberg 01[Michael, University of Illinois, Urbana-Champaign. “Ethics: An Essay on the Understanding of
Evil” Criticism 43.4 (2001) 478-484]

Another sort of problem emerges when we consider Badiou's attempt to [End Page 482] surpass the discourse of victimization that he and many
others see as defining the contemporary moment. While this critique of victim-centered ethics is crucial, and works well with
respect to many situations, it risks overgeneralization. In his laudable insistence that humanity "does not coincide with the identity
of the victim" (11; emphasis in original), Badiou leaves out of his system the possibility that a human being could be
reduced precisely to the status of victim. Such a case has been investigated by the Italian philosopher Giorgio Agamben in Remnants of
Auschwitz under the heading of the "Muselmann." Muselmann, or "Muslim," was the name given in certain Nazi camps to prisoners who had been so
overcome by hunger, beatings, etc. that they became zombie-like, incapable of human communication or response, trapped in an indeterminate zone
between life and death. While surely the product of an extremity not conducive to generalization, the Muselmann nevertheless constitutes the unthought of
Badiou's own project: the potential of a victimization so radical that it really does exceed the possibility of any human project or truth-process. Whether this
case is at all conducive to ethical or political elaboration must remain open here, but what the counter-example of the Muselmann suggests is the limit of
Badiou's will to universality. The problem with universality surely also returns in the insistence on ignoring questions of
cultural difference. Badiou's absolute commitment to the ethical value of the Same—the fact that truths are addressed equally to all—
demonstrates a provocative and radically democratic spirit. In presenting truths as simultaneously multiple and universal, Badiou poses an
imaginative answer to what may be the most intractable antinomy of contemporary left social theory: the difficulty
of adjudicating claims for universality and particularlity. (For other attempts to think through this problem, see the contributions to the
recent collective volume by Judith Butler, Ernesto Laclau, and Slavoj Z;akiz;akek, Contingency, Hegemony, Universality [London and New York: Verso,
2000]. And yet, is his notion that the universality of truths is premised on the simultaneous local nature of truth —its
immanence to a particular situation with which it breaks—sufficientto ward off fears of homogenization, if not cultural
imperialism? How can we differentiate between the Sameness of truth and the homogenization produced by
capitalist commodification? Is there an alternative formulation that would respect the universal address of truths
while still allowing for a valorization of or commitment to difference? The unease that Badiou's dismissal of cultural
difference provokes, despite the freshness of his formulation, suggests that the antinomy of the universal and the particular is
as much a symptom of the post-Cold War historical moment as a problem solvable in theory .
A2: Badiou

Badiou has no alt – even waiting for the event ruins it


Brown 04 [Nicholas, Professor at University of Illinois at Chicago. “Or, Alain Badiou and Slavoj Žižek, Waiting
for Something to Happen” CR: The New Centennial Review 4.3]

Badiou's ontology cannot usefully displace the dialectic. Because the Event must descend like a grace, Badiou's ontology can only describe situations and
never History. Since the event emerges from outside of the state of the situation, it is rigorously untheorizable: as we
saw above, it is theorized as untheorizable. Despite every protestation to the contrary, Badiou's system cannot address the
question "What is to be done?" because the only thing to do is to wait for the Event. What happens when the precipitation of
the Event is precisely what needs to be done? Yes, we can be faithful to a previous event, as Badiou says Lenin was to the
Paris Commune. But surely this solution mitigates the power of the Event as the irruption of the void into this situation.
The dialectic, on the other hand, conceives the void as immanent contradiction. While both contradiction and void are immanent to the situation,
contradiction has the tremendous advantage of having movement built in, as it were: the Event does not appear out of an immanent nowhere, but is already
fully present in itself in the situation, which it explodes in the movement to for-itself. Meanwhile, the question of the dialectic leads us back to the twofold
meaning of "state": both the law and order that govern knowledge, and law and order in the everyday sense. This identification authorizes Badiou's
antistatism, forcefully reflected in his own political commitment, the Organisation Politique (whose members do not vote), which has made limited [End
Page 306] but effective interventions into the status of immigrant workers. In Badiou's system, nothing can happen within the state of a situation;
innovation can only emerge from an evental site, constitutively excluded from the state. But can a principled indifference to the state ground a politics? The
state surely has the function of suppressing the anarchic possibilities inherent in the (national) situation. But it can also suppress the possibilities exploited
by an anarchic capitalism. It is well known that the current rightist "small-government" movement is an assault on the class compromise represented by the
Keynesian state. To be sure, one should be suspicious of that compromise and what it excluded. But it also protected workers against some of capitalism's
more baleful effects. As with Ethics, Badiou is certainly describing something: the utopian moment of a total break with the state may
be a part of any genuine political transformation. But, unless we are talking about the sad old interplay of
transgression and limit—which posited the state as basically permanent, with transgression as its permanent suspension—this anarchic
moment says nothing about the new state of affairs that will ultimately be imposed on the generic set it constructs.
Surely the configuration of that state will be paramount—in which case state power has to be fought for, not merely
evaded. L'Organisation Politique, it must be said, has come to recognize this. But doesn't this recognition imply a philosophical problem as well—one
which the dialectic, in Lukács's writings on party organization (on what amounts to the imposition of a state on the evental site itself) is called upon to
answer?
at badiou: ethics

Badiou’s concept of ethics fails because it is impossible to make qualitative distinctions between different sorts of evil—leading to
absurd results.

Brown, 04 (Nicholas, University of Illinois at Chicago, Or, Alain Badiou and Slavoj Žižek, Waiting for Something to Happen, CR:
The New Centennial Review 4.3 (2004) 289-319).

This apparatus is a powerful lens, and there can be no doubt that Badiou is describing something important; perhaps it is even
an aspect of evil. But is it really Evil (Mal) itself? Badiou's evil, like his truth, is indifferent to content, a merely formal label. In its
formalism, its insistence on fidelity to any Event whatever—on "ethical consistency" itself as a value—Badiou's good is almost an
aesthetic rather than an ethical category. (At one point, in an echo of Kant's purposeless purpose, ethical consistency is even
described as "disinterested interest.") While there is something undeniably attractive in ethical consistency (and something ugly
in its lack), the most important thing for a modern ethics may be to push these sentimental considerations aside. The value of
ethical consistency is authorized by Lacan's well-known dictum not to give up on one's desire [ne pas céder sur son désir]. But
we should not forget that this maxim derives from the reading of Antigone in Séminaire VII. Yes, Sophocles' Antigone, in her
awful ethical consistency, is a captivating figure. Brecht's Galileo, on the other hand, in his opportunism and wavering
inconsistency, is a bit distasteful. But Antigone is a reactionary, and Galileo invents physics. Further, Badiou has no way of
sorting out different evils beyond his tripartite division. Ethics tells us what Nazism and scientific obscurantism have in common.
But an ethics would have to be able to tell them apart. The distinction between, say, the abandonment of a social movement by
its leader and the abandonment of a poem by its author cannot be made without some kind of qualitative supplement. Since, as
we shall see, Badiou's philosophy is predicated precisely on the subtraction from consideration of all qualitative predicates, this
supplement can only be vulgar, non-philosophical. Perhaps the supplement it requires is the language of human rights, which,
whatever its faults, can tell the difference between a concentration camp and a creationist textbook.
at badiou: politics fail

Badiou is not politically useful because his alternative is too vague—he says that the event side steps the state but any
alternative politics must be able to reform the state to succeed.

Brown, 04 (Nicholas, University of Illinois at Chicago, Or, Alain Badiou and Slavoj Žižek, Waiting for Something to Happen, CR:
The New Centennial Review 4.3 (2004) 289-319).

Badiou's ontology cannot usefully displace the dialectic. Because the Event must descend like a grace, Badiou's ontology can
only describe situations and never History. Since the event emerges from outside of the state of the situation, it is rigorously
untheorizable: as we saw above, it is theorized as untheorizable. Despite every protestation to the contrary, Badiou's system
cannot address the question "What is to be done?" because the only thing to do is to wait for the Event. What happens when the
precipitation of the Event is precisely what needs to be done? Yes, we can be faithful to a previous event, as Badiou says Lenin
was to the Paris Commune. But surely this solution mitigates the power of the Event as the irruption of the void into this situation.
The dialectic, on the other hand, conceives the void as immanent contradiction. While both contradiction and void are immanent
to the situation, contradiction has the tremendous advantage of having movement built in, as it were: the Event does not appear
out of an immanent nowhere, but is already fully present in itself in the situation, which it explodes in the movement to for-itself.
Meanwhile, the question of the dialectic leads us back to the twofold meaning of "state": both the law and order that govern
knowledge, and law and order in the everyday sense. This identification authorizes Badiou's antistatism, forcefully reflected in his
own political commitment, the Organisation Politique (whose members do not vote), which has made limited [End Page 306] but
effective interventions into the status of immigrant workers. In Badiou's system, nothing can happen within the state of a
situation; innovation can only emerge from an evental site, constitutively excluded from the state. But can a principled
indifference to the state ground a politics? The state surely has the function of suppressing the anarchic possibilities inherent in
the (national) situation. But it can also suppress the possibilities exploited by an anarchic capitalism. It is well known that the
current rightist "small-government" movement is an assault on the class compromise represented by the Keynesian state. To be
sure, one should be suspicious of that compromise and what it excluded. But it also protected workers against some of
capitalism's more baleful effects. As with Ethics, Badiou is certainly describing something: the utopian moment of a total break
with the state may be a part of any genuine political transformation. But, unless we are talking about the sad old interplay of
transgression and limit—which posited the state as basically permanent, with transgression as its permanent suspension—this
anarchic moment says nothing about the new state of affairs that will ultimately be imposed on the generic set it constructs.
Surely the configuration of that state will be paramount—in which case state power has to be fought for, not merely evaded.
at badiou: politics fail

Badiou’s system fails—he has no way to overcome the enormous power he attributes to capitalism.

Brown, 04 (Nicholas, University of Illinois at Chicago, Or, Alain Badiou and Slavoj Žižek, Waiting for Something to Happen, CR:
The New Centennial Review 4.3 (2004) 289-319).

But what is strange is the vehemence with which Badiou maintains his distance from the economic—from what classical Marxism
called the "base," the elements of a situation that pertain to its own reproduction. It is perfectly orthodox to say that there can be
no purely economic intervention in the economy: even with the best intentions, the World Bank could not solve the problem of
Third World poverty. However, in Badiou's system the economy is not merely reduced to one aspect among many, but actively
dismissed from consideration. Material reproduction is reduced to the sneering Lacanian contempt for "le service des biens," the
servicing of goods which pertains to the human animal beneath good and evil. Why should Badiou fully endorse Marx's analysis
of the world economy ("there is no need for a revision of Marxism itself," [Ethics, 97]) while keeping Marx's entire problematic at
arm's length? In fact, capitalism is the point of impasse in Badiou's own system, the problem which cannot be actively thought
without grave danger to the system as a whole. Capital's great power, the tremendous ease with which it colonizes (geographic,
cultural, psychic) territory, is precisely that it seizes situations at their evental site. In their paraphrase of a brilliant but much-maligned passage in
Marx's Grundrisse, Deleuze and Guattari insist that "capitalism has haunted all forms of society, but it haunts them as their terrifying nightmare, it is the dread they feel of a flow that
would elude their codes."2 Is this flow that eludes every society's codes not identical with generic multiplicity, the void which, eluding every representation, nonetheless haunts every
situation? Does not capitalism make its entry at a society's point of impasse—social relations already haunted by variously dissimulated exploitation—and revolutionize them into the
capital-labor relation? A safely non-Orientalist version of this would be the eruption from modernist art's evental site—the art market, which belonged to the situation of modernism while
being excluded from its represented state—of what we might call the "Warhol-event," which inaugurates the transition from the formal to the real subsumption of (artistic) labor under
Capital. It makes perfect sense to say that this transition is the truth of the [End Page 308] Warhol-event. As we saw earlier, the real subsumption of labor under Capital, the
conversion of every relation into a monetary relation, is the origin of formal equality: that is, the foundation of universalism. And far from pertaining to mere animal life beneath the level
capitalism itself fits perfectly the form of the revolutionary Event. It would then appear that capitalism is, like
of the truth-procedure,
religion, eliminated from the art-politics-science-love series only by fiat. And why is this? Because the economic, the "servicing of
goods," cannot enter Badiou's system without immediately assuming the status of a cause. Excluded from direct consideration,
capitalism as a condition of set theory is perfectly innocuous; its preconditional status belongs to a different order than what it
conditions. It opens up a mode of presentation, but what is presented existed all along: look at Paul, for example. But included as
the product of a truth-procedure, capitalism immediately appears as the basis for all the others: it is, in fact, the revolutionary
irruption of Capital (in whatever society) that conditions any modern process of science, art, love, or politics. If Badiou's system
were to consider capitalism directly, some elements, those pertaining to the "base," would appear to have more weight than
others—the "superstructure." The effects of such an inclusion of capitalism in Badiou's system—an inclusion which nothing
prevents—would be catastrophic. Radical universality (as opposed to the historically conditioned universality imposed by the
emergence of capitalism) would become unthinkable. The "eternity" of truth would yield to historicism.
at badiou: politics fail

Unfortunately for Badiou, his great enemy of capitalism fits perfectly within what he considers a truth event—the alternative
merely re-creates the status quo.

Brown, 04 (Nicholas, University of Illinois at Chicago, Or, Alain Badiou and Slavoj Žižek, Waiting for Something to Happen, CR:
The New Centennial Review 4.3 (2004) 289-319).

Badiou cannot think Capital precisely because Capital has already thought Badiou. And let's face it: despite Badiou's inspiring
presentation, nothing is more native to capitalism than his basic narrative matrix. The violent seizure of the subject by an idea,
fidelity to it in the absence of any guarantee, and ultimate transformation of the state of the situation: these are the elements of
the narrative of entrepreneurial risk, "revolutionary innovation," the "transformation of the industry," and so on. In pushing away
material reproduction, Badiou merely adapts this narrative to the needs of intellectuals, who, in Badiou's conception, have a
monopoly over much of the field of truth.

Failure to cope with the power of capitalism dooms any ethical system to failure.

Brown, 04 (Nicholas, University of Illinois at Chicago, Or, Alain Badiou and Slavoj Žižek, Waiting for Something to Happen, CR:
The New Centennial Review 4.3 (2004) 289-319).

The problem with this ethics—as Brecht showed us, with ethics in general—is that, under capitalism, the only fully consistent
ethical position is ruthless self-interest. There is no ethical position that is both minimally compassionate and fully ethically
consistent. Mauler in Saint Joan is doomed to make money from all of his generous impulses; the good woman of Szechwan can
only help her neighbors by taking advantage of them. In fact, this split constitutes part of capitalism's dynamism. The ideological
force of capitalism is that so many people are given a subjective interest in maintaining the stability of capitalism, even if this
interest involves competing with neighbors who share an "objective" interest in ending it. Any "opting out" is at present simply
quixotic, and only possible on the basis of substantial privilege. Plainly, professors want tenured positions, for the same reason
the unemployed want jobs: because they exist. (As for playing the stock market, this criticism buys neoliberal rhetoric hook, line,
and sinker: most academics who "play the stock market" do so because universities, like many other U.S. employers, have
shifted the burden of risk from their own retirement systems onto the individual employees.)
at badiou: permutation

The state and the revolutionary political subject can cooperate in Badiou’s conception of the alternative.

Hallward, 03 (Badiou: a subject to truth, Peter Hallward, University of Minnesota Press Minneapolis / London 2003, Professor of
Modern European Philosophy, Centre for Research in Modern European Philosophy, Middlesex Univeristy).

We know that Badiou's early and unequivocally hostile attitude to the state has considerably evolved. Just how far it has evolved
remains a little unclear. His conception of politics remains resolutely anticonsensual, anti–“re-presentative, ” and thus
antidemocratic (in the ordinary sense of the word). Democracy has become the central ideological category of the neo-liberal
status quo, and any genuine “philosophy today is above all something that enables people to have done with the 'democratic'
submission to the world as it is.” 66 But he seems more willing, now, to engage with this submission on its own terms. La
Distance politique again offers the most precise points de repère. On the one hand, the OP remains suspicious of any political
campaign—for instance, an electoral contest or petition movement—that operates as a “prisoner of the parliamentary space.” 67
It remains “an absolute necessity [of politics] not to have the state as norm. The separation of politics and state is foundational of
politics.” On the other hand, however, it is now equally clear that “their separation need not lead to the banishment of the state
from the field of political thought.” 68 The OP now conceives itself in a tense, nondialectical “vis-à-vis” with the state, a stance
that rejects an intimate cooperation (in the interests of capital) as much as it refuses “any antagonistic conception of their
operation—a conception that smacks of classism.” There is no more choice to be made between the state and revolution; the
“vis-à-vis demands the presence of the two terms and not the annihilation of one of the two.” 69
at badiou: alternative unworkable and communist

Badiou’s alternative of radical egalitarianism is unworkable and is based on a failed model of communism.

Hallward, 03 (Badiou: a subject to truth, Peter Hallward, University of Minnesota Press Minneapolis / London 2003, Professor of
Modern European Philosophy, Centre for Research in Modern European Philosophy, Middlesex Univeristy).

Badiou's politics have always been about “collective emancipation, or the problem of the reign of liberty in infinite situations” (DO,
54; cf. TC, 60). His political goals have remained consistent over the years, since “every historical event is communist, to the
degree that 'communist' designates the transtemporal subjectivity of emancipation, the egalitarian passion, the Idea of justice,
the will to break with the compromises of the service des biens, the deposition of egoism, an intolerance of oppression, the wish
to impose a withering away of the state. The absolute preeminence of multiple presentation over representation.” 84 What has
changed is communism's mode of existence. In Badiou's earlier work, the practical (if ultimately unattainable) goal was always to
effect the actual, historical achievement of stateless community. Today, in order to preserve politics' “intrinsic relation to truth”
(DO, 48), Badiou has had to let go of almost any sort of political engagement with the economic and the social. He continues to
declare a wholly egalitarian politics, but as reserved for a strictly subjective plane. The unqualified justice of a generic
communism, first proposed in Marx's 1844 Manuscripts and conceived in Badiou's own terms as the advent of “pure
presentation, ” as the “undivided authority of the infinite, or the advent of the collective as such” (AM, 91), remains the only valid
subjective norm for Badiou's political thought. This subjective norm has become ever more distant, however, from the day-to-day
business of “objective” politics: the programmatic pursuit of the generic ideal is itself now dismissed as a “Romantic” dream
leading to “fraternity terror” (AM, 101).
A2: Nietzsche

The eternal recurrence is the worst form of nihilism – the attempt to find meaning through
repetition conveys a dread at the possibility of a lack of meaning, it is only through denial of
meaning can it be possible
Loy prof phil @ Bunkyo U, Japan 1996 (David, “A Buddhist critique of Nietzsche” Asian Philosophy Vol. 6, No. 1,
March http://ccbs.ntu.edu.tw/FULLTEXT/JR-ADM/loy.htm)

The answer is complex, of course, and there is much that Buddhists can learn from Nietzsche, the first post-modernist and still the most important
one. In order to reach that answer, however, it will first be necessary to gain some understanding of anatman, the 'no self' doctrine central to
Buddhism and to the still-widespread misunderstanding of Buddhism as nihilistic. Of the various ways for us to approach anatman, one of the
most insightful is through modem psychology. Buddhism anticipated its reluctant conclusions: guilt and anxiety are not
adventitious but intrinsic to the ego. That is because our dissatisfaction with life derives from a repression even more
immediate than death-terror: the suspicion that 'I' am not real. For Buddhism, the sense-of-self is not some self-
existing consciousness but a mental construction which experiences its own groundlessness as a lack. On this
account, our most problematic dualism is not so much life fearing death as a fragile sense-of-self dreading its own
nothingness. By accepting and yielding to that groundlessness, however, I can discover that I have always been
grounded, not as a self-present being but as one manifestation of a web of relationships which encompasses
everything.     What does this understanding of self-as-lack imply about ethics, truth, and the meaning of life for us?
That is the question which motivates this paper, for to raise these issues in the Western tradition is to find ourselves in a dialogue with Nietzsche,
whose own texts resonate with many of the same insights: for example, his critiques of the subject ("The 'subject' is not something given, it is
something added and invented and projected behind what there is." WP 481) and substance ("The properties of a thing are effects on other 'things'
... there is no 'thing-in-itself.'" WP 557). From this critique, Nietzsche also drew some conclusions quite similar to those of
Buddhism: in particular, that morality, knowledge and meaning are not discovered but constructed -- internalised games we learn from each
other and play with ourselves. Perhaps the history of his own psyche reveals how momentous these discoveries were; and inevitably his insights
were somewhat distorted.     Nietzsche understood how the distinction we make between this world and a higher spiritual
realm serves our need for security, and he saw the bad faith in religious values motivated by this need. He did not understand how his
alternative, more aristocratic values, also reflects the same anxiety. Nietzsche ends up celebrating an impossible ideal, the heroic-ego which
overcomes its sense of lack, because he does not see that a heroic ego is our fantasy project for overcoming lack.     Nietzsche realised how
the search for truth is motivated by a sublimated desire for symbolic security ; his solution largely reverses our usual dualism
by elevating ignorance and 'untruth' into conditions of life. Philosophy's attempt to create the world reflects the tyrannical will-to-power,
becoming the most 'spiritualised' version of the need to impose our will Insofar as truth is our intellectual effort to grasp being symbolically,
however, those who no longer need to ground themselves can play the truth-versus-error game with lighter feet. Nietzsche overlooks a different
reversal of perspective which could convert the bad-infinite of the heroic will-as-truth into the good infinite of truth-as-play.     What he
considered the crown of his system -- eternal recurrence -- is actually its denouement. Having seen through the
delusion of Being, Nietzsche could not let it go completely, for he still sought a Being within Becoming. 'To impose
upon becoming the character of being -- this is the supreme will to power' (WP 617). Having exposed the bad faith
of believing in eternity, Nietzsche is nonetheless able to affirm the value of this moment only by making it recur
eternally. In place of the neurotic's attempt to rediscover the past in the future he tries to rediscover the present in the
future, yet the eternal recurrence of the now can add something only if the now in itself lacks something.     Rather
than the way to vanquish nihilism, Nietzsche's will-to-power turns out to be pure nihilism, for nihilism is not the
debacle of all meaning but our dread of that debacle and what we do to avoid it. This includes compulsively seizing
on certain meanings as a bulwark against that form of lack. If so, the only solution to the dread of meaninglessness is
meaninglessness itself: only by accepting meaninglessness, by letting it devour the meanings that we use to defend
ourselves against our nothingness, can we realise a meaning-freeness open to the possibilities that arise in our
world.    In sum, when the lack-driven bad infinite transforms into a lacking-nothing good-infinite, the dualisms of
good-versus-evil, truth-versus-error, and meaningfulness-versus-meaninglessness are realised to be games. Do I play
them or do they play me? As long as we do not understand what is motivating us, we play with the seriousness of a
life-versus-death struggle, for that is what the games symbolise for a self preoccupied with its lack. We are trapped
in games which cannot be escaped yet cannot be won, since playing well does not resolve one's sense-of-lack. When
there is no need to get anything from the game or gain cloture on it, we can play with the seriousness of a child
absorbed in its game. [3
A2: Nietzsche

The will to power re-creates the slave morality it intends to criticize – both are attempts to make
up for a lack in the self, the only truly ethical action is to view oneself as distinct from the
world and view the self as interconnected to all other beings
Loy prof phil @ Bunkyo U, Japan 1996 (David, “A Buddhist critique of Nietzsche” Asian Philosophy Vol. 6, No.
1, March http://ccbs.ntu.edu.tw/FULLTEXT/JR-ADM/loy.htm)

The will-to-power cannot be separated from its sublimation (or 'spiritualisation'), for Nietzsche discovered them
together. He was one of the first classicists to realise that the original Olympic games were a sublimated form of
war. Nietzsche contended that Greek civilisation was noble and sublime precisely because it had been so cruel and
bloodthirsty; the 'golden age' was created by bringing this original ferocity under control. "The thought seems to be:
where there is 'the sublime' there must have been that which was made sublime -- sublimated -- after having been for
a long time not sublime." [14] Having detected this phenomenon in ancient Greece, Nietzsche began to notice
sublimated 'base' impulses in many kinds of activity; for example, Wagner's ferocious will sublimated into the
Bayreuth festival. This makes Nietzsche the first, as far as I know, to undertake a systematic study of repression.
Nietzsche sees the sublimity of Greek culture as the sublimation of its original ferocity, yet here perhaps the
genealogist of morals does not trace his genealogy back far enough. What makes man so ferocious? Can even the
will to power, irreducible for Nietzsche, be deconstructed? What, after all, does power mean to us? All power is in
essence power to deny mortality. Either that or it is not real power at all, not ultimate power, not the power that
mankind is really obsessed with. Power means power to increase oneself, to change one's natural situation from one
of smallness, helplessness, finitude, to one of bigness, control, durability, importance. (Becker) [15] We feel we are
masters over life and death when we hold the fate of others in our hands, adds Becker; and we feel we are real when
the reality of others is in our hands, adds Buddhism. From that perspective, however, desire for power is little
different from the slave morality Nietzsche criticises. Both become symptoms of our lack, equally frustrating
inasmuch as we are motivated by something that cannot be satisfied in the way we try to satisfy it. No wonder
Nietzsche's will-to-power can never rest, that it needs to expand its horizons, and that for most of us morality has
been a matter of collecting religious brownie points. In both cases we think that we have found the way to get a grip
on our eligibility for immortality -- or being. The whole basis of the urge to goodness is to be something that has
value, that endures... Man uses morality to try to get a place of special belongingness and perpetuation in the
universe... Do we wonder why one of man's chief characteristics is his tortured dissatisfaction with himself, his
constant self-criticism? It is the only way he has to overcome the sense of hopeless limitation inherent in his real
situation. (Becker) [16] When I realise that I am not going to attain cloture on that diabolical part of myself, it is
time to project it. "The Devil is the one who prevents the heroic victory of immortality in each culture -- even the
atheistic, scientific ones." [17] As long as lack keeps gnawing, we need to keep struggling with the Devil, and as we
all know the best devil is one outside our own group. Evil is whatever we decide is keeping us from becoming real,
and since no victory over any external devil can yield the sense of being we seek, we have become trapped in a
paradox of our own making: evil is created by our urge to eliminate evil. Stalin's collectivisation programme was an
attempt to build a more perfect socialist society. The Final Solution of the Nazis was an attempt to purify the Earth
of its vermin.     The Buddhist critique of such ressentiment includes understanding the self-deception involved in
such dualistic thinking, when I identify with one pole and vainly try to eliminate its interdependent other. [18]
Buddhism gets beyond good and evil not by rebaptising our evil qualities as our best, but with an entirely different
perspective. As long as we experience ourselves as alienated from the world, and society as a set of separate selves,
the world is devalued into a field-of-play wherein we compete to fulfill ourselves. That is the origin of the ethical
problem we struggle with today: without some transcendental ground such as God, what will bind our atomised
selves together? When my sense-of-self lets-go and disappears, however, I realise my interdependence with all other
phenomena. It is more than being dependent on
Card Continues
Card Continued
them: when I discover that I am you, the trace of your traces, the ethical problem of how to relate to you is
transformed. [19]     Of course, this provides no simple yardstick to resolve knotty ethical dilemmas. Yet more
important, I think, is that this absolves the sense of separation between us which usually makes those dilemmas so
difficult to resolve, including the conceit that I am the one who has privileged access to transcendental principles, or
who embodies more fully the will-to-power. Loss of self-preoccupation entails the ability to respond to others
without an ulterior motive which needs to gain something from that encounter. Buddhist ethical principles
approximate the way of relating to others that nondual experience reveals. As in Christianity, I should love my
neighbour as myself -- in this case because my neighbour is myself. In contrast to the 'Thou shalt not -- or else!'
implied in Mosaic law, the Buddhist precepts are vows one makes not to some other being but to one's to-be-
realised-as-empty self: "I vow to undertake the course of training to perfect myself in non-killing," and so forth. If
we have not developed to the degree that we spontaneously experience ourselves as one with others, by following
the precepts we endeavour to act as if we did feel that way. Yet even these precepts are eventually realised not to
rest on any transcendental, objectively-binding moral principle. There are, finally, no moral limitations on our
freedom -- except the dualistic delusions which incline us to abuse that freedom in the first place.
A2: Nietzsche

The alternative enables genocide– the idea of supreme power for a person at the expense of others
legitimates actions of violence against the weak
Simpson scholar in human rights/democracy 1995 (Christopher “The Splendid Blond Beast”
http://www.thirdworldtraveler.com/Genocide/SplendidBlondeBeast.html)

Friedrich Nietzsche called the aristocratic predators who write society's laws "the splendid blond beast" precisely
because they so often behave as though they are beyond the reach of elementary morality. As he saw things, these
elites have cut a path toward a certain sort of excellence consisting mainly of the exercise of power at the expense of
others. When dealing with ordinary people, he said, they "revert to the innocence of wild animals.... We can imagine
them returning from an orgy of murder, arson, rape and torture, jubilant and at peace with themselves as though they
had committed a fraternity prank-convinced, moreover, that the poets for a long time to come will have something to
sing about and to praise.'' Their brutality was true courage, Nietzsche thought, and the foundation of social order.
Today genocide-the deliberate destruction of a racial, cultural, or political group-is the paramount example of the
institutionalized and sanctioned violence of which Nietzsche spoke. Genocide has been a basic mechanism of
empire and the national state since their inception and remains widely practiced in "advanced" and "civilized" areas.
Most genocides in this century have been perpetrated by nation-states upon ethnic minorities living within the state's
own borders; most of the victims have been children. The people responsible for mass murder have by and large
gotten away with what they have done. Most have succeeded in keeping wealth that they looted from their victims;
most have never faced trial. Genocide is still difficult to eradicate because it is usually tolerated, at least by those
who benefit from it. The Splendid Blond Beast examines how the social mechanisms of genocide often encourage
tacit international cooperation in the escape from justice of those who perpetrated the crime... According to
psychologist Ervin Staub, who has studied dozens of mass crimes, genocidal societies usually go through an
evolution during which the different strata of society literally learn how to carry out group murder. In his book The
Roots of Evil, Staub contends that genocidal atrocities most often take place in countries under great political,
economic, and often military stress. They are usually led by authoritarian parties that wield great power yet are
insecure in their rule, such as the Nazis in Germany or the Ittihad (Committee of Union and Progress) in Turkey.
The ideologies of such parties can vary in important respects, but they are nonetheless often similar in that they
create unity among "in-group" members through dehumanization of outsiders. Genocidal societies also show a
marked tendency toward what psychologists call "justworld" thinking: Victims are believed to have brought their
suffering upon themselves and, thus, to deserve what they get. But the ideology of these authoritarian parties and
even their seizure of state power are not necessarily enough to trigger a genocide. The leading perpetrators need
mass mobilizations to actually implement their agenda. For example, the real spearheads of genocide in Germany-
the Nazi party, SS, and similar groups- by themselves lacked the resources to disenfranchise and eventually murder
millions of Jews. They succeeded in unleashing the Holocaust, however, by harnessing many of the otherwise
ordinary elements of German life-of commerce, the courts, university scholarship, religious observance, routine
government administration, and so on-to the specialized tasks necessary for mass murder. Not surprisingly, many of
the leaders of these "ordinary" institutions were the existing notables in German society. The Nazi genocide
probably would not have been possible without the active or tacit cooperation of many collaborators who did not
consider themselves Nazis and, in some cases, even opposed aspects of Hitler's policies, yet nonetheless cooperated
in mass murder. Put bluntly, the Nazis succeeded in genocide in part through offering bystanders money, property,
status, and other rewards for their active or tacit complicity in the crime.
A2: Nietzsche

Permutation solves best – we must protect society by preventing our impacts before the will to
power is possible
Golomb Hebrew U of jeruselum 2k6 (Jacob, “Can One Really Become a "Free Spirit Par Excellence"or an
Übermensch?” The Journal of Nietzsche Studies 32 Muse)

In On the Genealogy of Morals, Nietzsche embarks on the genealogical inquiry also to examine whether the
emphasis on the immanence, autarchy, and extreme individuality of authentically powerful persons is compatible
with a social context. The genealogical account shows that the moral patterns of positive power (detailed below)
were occasionally manifested within this or that social and historical context, though not always in their most perfect
or distinct forms. It also shows that factors external to these patterns (like Christianity) were responsible for their
disappearance. Nietzsche's affirmation of society as the necessary condition for the materialization of positive
power attenuates the radical stance of his extreme individualism. And because Nietzsche affirms "a community"
(e.g., GM II:9) and does not seek to destroy it, he had to explain how the übermenschlich patterns of behavior or the
morality of positive power are possible within the social context. He analyzes the nature of the interaction among
members of society and maintains that genuine justice is possible only within a social fabric composed of equally
powerful members: "Justice . . . is the good will among parties of approximately equal power to come to terms with
one another, to reach an 'understanding' by means of a settlement—and to compel parties of lesser power to reach a
settlement among themselves" (GM II:8). Nietzsche argues that the powerful individual is characterized by egoism.
Avoidance of any altruistic activity and ideology would seemingly contradict any possible moral system. This
emphasis on the egoism of genuine power, however, does not prevent Nietzsche from continuing to describe the
moral and social network of powerful individuals who would willingly and freely enter the restrictive social fabric:
The noble soul accepts this fact of its egoism without any question mark. . . . [U]nder certain circumstances there are
some who have rights equal to its own. . . . [I]t moves among these equals with their equal privileges, showing the
same sureness of modesty and delicate reverence that characterize its relation with itself. . . . [E]very star is such an
egoist. . . . [I]t honors itself in them and in the rights it cedes to them; it does not doubt that the exchange of honors
and rights is of the nature of all social relations and thus also belongs to the natural condition of things. (BGE 265)
Nietzsche declares here that recognition of the value and freedom of others originates in egoism. Only an individual
possessing an abundance of positive [End Page 27] power and a firm selfhood is able to grant similar rights and
freedoms to all those who are recognized as equals. This individual is not afraid that this might diminish or destroy
her or his own power. It is a self-affirmation and a confidence in one's power and virtues that enable the affirmation
of "others" and their uniqueness. For Nietzsche, human egoism and the emphasis on selfhood do not contradict the
social and moral order; they actually create the ideal conditions for its proper functioning.
A2: Nietzsche

AFF: Perm
Use of the state is critical to Nietzsche’s exercise of the will to power—Self-cultivation is an
agonistic process that democracy can enhance
Hatab 2002 (professor at Old Dominion University, The Journal of Nietzsche Studies 24 (2002) 132-147,
Prospects For A Democratic Agon: Why We Can Still Be Nietzscheans, Project Muse)

How can we begin to apply the notion of agonistics to politics in general and democracy in
particular? First of all, contestation and competition can be seen as fundamental to self-
development and as an intrinsically social phenomenon. Agonistics helps us articulate the social
and political ramifications of Nietzsche's concept of will to power. As Nietzsche put it in an 1887
note, "will to power can manifest itself only against resistances; it seeks that which resists it"
(KSA 12, p.424). Power, therefore, is not simply an individual possession or a goal of action; it
is more a global, interactive conception. For Nietzsche, every advance in life is an overcoming of
some obstacle or counterforce, so that conflict is a mutual co-constitution of contending forces.
[End Page 134] Opposition generates development. The human self is not formed in some internal sphere and then
secondarily exposed to external relations and conflicts. The self is constituted in and through what it opposes and what opposes it; in other words ,
the self is formed through agonistic relations. Therefore, any annulment of one's Other would be
an annulment of one's self in this sense. Competition can be understood as a shared activity for
the sake of fostering high achievement and self-development, and therefore as an intrinsically
social activity. 10
In the light of Nietzsche's appropriation of the two forms of Eris, it is necessary to distinguish between agonistic conflict and sheer violence. A
radical agonistics rules out violence, because violence is actually an impulse to eliminate conflict
by annihilating or incapacitating an opponent, bringing the agon to an end. 11 In a later work
Nietzsche discusses the "spiritualization of hostility (Feindschaft)," wherein one must affirm
both the presence and the power of one's opponents as implicated in one's own posture (TI
"Morality as Antinature," 3). And in this passage Nietzsche specifically applies such a notion to
the political realm. What this implies is that the category of the social need not be confined to
something like peace or harmony. Agonistic relations, therefore, do not connote a deterioration
of a social disposition and can thus be extended to political relations.
How can democracy in general terms be understood as an agonistic activity? Allow me to quote
from my previous work. Political judgments are not preordained or dictated; outcomes depend
upon a contest of speeches where one view wins and other views lose in a tabulation of votes;
since the results are binding and backed by the coercive power of the government, democratic
elections and procedures establish temporary control and subordination—which, however, can
always be altered or reversed because of the succession of periodic political contests. . . .
Democratic elections allow for, and depend upon, peaceful exchanges and transitions of
power. . . . [L]anguage is the weapon in democratic contests. The binding results, however,
produce tangible effects of gain and loss that make political exchanges more than just talk or a
game. . . . The urgency of such political contests is that losers must yield to, and live under, the
policies of the winner; we notice, therefore, specific configurations of power, of domination and
submission in democratic politics. 12
A2: Nietzsche

Nietzsche rejected the repudiation of institutions. He saw the state as a place to realize his
alternative.
Hatab 2002 (professor at Old Dominion University, The Journal of Nietzsche Studies 24 (2002) 132-147,
Prospects For A Democratic Agon: Why We Can Still Be Nietzscheans, Project Muse)

Those who take Nietzsche to be diagnosing social institutions as descendants of slave morality should
take note of GM II,11, where Nietzsche offers some interesting reflections on justice and law. He
indicates that the global economy of nature is surely not a function of justice; yet workable conceptions of
justice and injustice are established by the historical force of human law. Nietzsche does not indict such
forces as slavish infirmities. Legal arrangements are "exceptional conditions" that modulate natural forces
of power in [End Page 136] social directions, and that are not an elimination of conflict but an instrument
in channeling the continuing conflict of different power complexes. Surprisingly, Nietzsche attributes the
historical emergence of law not to reactive resentment but to active, worldly forces that check and redirect
the "senseless raging of revenge," and that are able to reconfigure offenses as more "impersonal"
violations of legal provisions rather than sheer personal injuries. Here Nietzsche analyzes the law in a
way analogous to his account of the Greek agon and its healthy sublimation of natural impulses for
destruction. A legal system is a life-promoting cultural force that refashions natural energies in less
savage and more productive directions.
Finally, those who read Nietzsche as an anti-institutional transgressor and creator should heed TI
("Skirmishes of an Untimely Man," 39), where Nietzsche clearly diagnoses a repudiation of institutions as
a form of decadence. Because of our modern faith in a foundational individual freedom, we no longer
have the instincts for forming and sustaining the traditions and modes of authority that healthy institutions
require. The whole of the West no longer possesses the instincts out of which institutions grow, out of
which a future grows: perhaps nothing antagonizes its "modern spirit" so much. One lives for the day, one
lives very fast, one lives very irresponsibly: precisely this is called "freedom." That which makes an
institution an institution is despised, hated, repudiated: one fears the danger of a new slavery the moment
the word "authority" is even spoken out loud. That is how far decadence has advanced in the value-
instincts of our politicians, of our political parties: instinctively they prefer what disintegrates, what
hastens the end.
Card Continues
Card Continued
In the light of these remarks, a Nietzschean emphasis on power and agonistics offers significant
advantages for political philosophy. In some respects we are freed from the modern project of "justifying"
the force of social institutions because of a stipulated freedom from constraint in the "state of nature."
With a primal conception of power(s), we can retrieve an Aristotelian take on social institutions as fitting
and productive of human existence. Forces of law need not be seen as alien to the self, but as modulations
of a ubiquitous array of forces within which human beings can locate relative spheres of freedom. And an
agonistic conception of political activity need not be taken as a corruption or degradation of an idealized
order of political principles or social virtues. Our own tradition of the separation of powers and an
adversarial legal system can be taken as a baseline conception of the nature, function, and proper
operation of government offices and judicial practice.
The founders of the Constitution inherited from Montesquieu the idea that a division of powers is the best
check on tyranny. In other words, tyranny is avoided not by some project of harmony, but by multiplying
the number of power sites in a government and affirming their competition through mutual self-assertion
[End Page 137] and mistrust. 16 Our common law tradition is agonistic in both conception and practice.
Most procedural rules are built around the idea of coequal competition in open court before a jury who
will decide the outcome, where the judge in most respects plays the role of an impartial referee. And the
presumption of innocence is fundamentally meant to contest the government's power to prosecute and
punish. 17 I think that both notions of separation of powers and legal adversarialism are compatible with
Nietzsche's analysis of the law noted previously—that a legal order is not a means of preventing struggle,
but "a means in the struggle between power-complexes" (GM II,11).
A2: Nietzsche

Agonistic practices would limit out the possibility of authoritarian violence. The overall
practice teaches individuals to rid themselves of the traditional notion of democracy and
instead, recognize different ideas and notions as equally important.
Hatab 2002 (professor at Old Dominion University, The Journal of Nietzsche Studies 24 (2002) 132-147,
Prospects For A Democratic Agon: Why We Can Still Be Nietzscheans, Project Muse)

Assuming that politics should not be restricted and reserved for an elite, but open to the
participation of all citizens, can we retain a sense of respect and political rights in appropriating
Nietzsche for democracy? I think so. In fact, Nietzschean conceptions of agonistics and
nonfoundational openness can go a long way toward articulating and defending democratic
practices without the problems attaching to traditional principles of equality. If political respect implies
inclusiveness and an open regard for the rightful participation of others, an agonistic model of politics can underwrite
respect without the need for substantive conceptions of equality or even something like "equal
regard." I have already mentioned that agonistics can be seen as a fundamentally social phenomenon.
Since the self is formed in and through tensional relations with others, then any annulment of my
Other would be an annulment of myself. Radical agonistics, then, discounts the idea of sheer
autonomy and self-constitution. Such a tensional sociality can much more readily affirm the
place of the Other in social relations than can modern models of subject-based freedom.
Moreover, the structure of an agon conceived as a contest can readily underwrite political
principles of fairness. Not only do I need an Other to prompt my own achievement, but the
significance of any "victory" I might achieve demands an able opponent. As in athletics,
defeating an incapable or incapacitated competitor winds up being meaningless. So I should not only will
the presence of others in an agon, I should also want that they be able adversaries, that they have opportunities and
capacities to succeed in the contest. And I should be able to honor the winner of a fair contest.
Such is the logic of competition that contains a host of normative features, which might even
include active provisions for helping people in political contests become more able participants.
25 In addition, agonistic respect need not be associated with something like positive regard or equal worth, a dissociation that can go further in
facing up to actual political conditions and problematic connotations that can attach to liberal dispositions. Again allow me to quote my previous
Democratic respect forbids exclusion, it demands inclusion; but respect for the Other as
work.
other can avoid a vapid sense of "tolerance," a sloppy "relativism," or a misplaced spirit of
"neutrality." Agonistic respect allows us to simultaneously affirm our beliefs and affirm our
opponents as worthy competitors [End Page 142] in public discourse. Here we can speak of respect
without ignoring the fact that politics involves perpetual disagreement, and we have an adequate answer to the
about what is best (aristos) can be coordinated
question "Why should I respect a view that I do not agree with?" In this way beliefs
with an openness to other beliefs and a willingness to accept the outcome of an open competition among the full
citizenry (demos). Democratic respect, therefore, is a dialogical mixture of affirmation and negation, a political
bearing that entails giving all beliefs a hearing, refusing any belief an ultimate warrant, and perceiving one's own
viewpoint as agonistically implicated with opposing viewpoints. In sum, we can combine 1) the historical tendency
of democratic movements to promote free expression, pluralism, and liberation from traditional constraints, and 2) a
Nietzschean perspectivism and agonistic respect, to arrive at a postmodern model of democracy that provides both a
nonfoundational openness and an atmosphere of civil political discourse. 26
A2: Nietzsche

Nietzsche leads to Violence


Nietzschean views allow genocide to happen.
Kelly Ross 2003. (“Friedrich Nietzsche”, July 5, 2007, http://www.friesian.com/NIETZSCH.HTM)

One could hardly say that either Nietzsche or Hitler "love" the Jews the way that an eagle loves a small mammalian
meal. Some have wondered, however, how much of Hitler's hatred was heartfelt and how much merely cynical.
Albert Speer said that Hitler never talked about the Jews in private conversation. Was he really obsessed with them,
or were they merely a device in his larger schemes of predation, in which whole nations could be thoughtlessly
consumed and expended in the interest of Germany and himself? So much the better would this be, for Nietzsche.
One thing must always be kept in view here: Nietzsche provides a feelgood philosophy for predators.
There is going to be no fault to find with Hitler if he merely destroys, uses, tortures, kills, etc.
Nietzsche himself seems more at fault if the only real sin is impotent resentment and inactive
rancor. Nietzsche did not live to see the Nazis, but he knew of another power that had to deal with the Jews as an
alien, hostile, and disruptive force: Rome viewed Israel as a monstrosity; the Romans regarded the Jews as
convicted of hatred against the whole of mankind -- and rightly so if one is justified in associating the welfare of the
human species with absolute supremacy of aristocratic values.... The Romans were the strongest and most noble
people who ever lived. [p.185-186, boldface added] The Romans, of course, killed many Jews, and expelled them
from Jerusalem and their Temple, but they did not actually try to exterminate them. Perhaps genocide would
have been too much for Nietzsche. But exactly how would he object to it? He could not say that
mass murder was intrinsically unjust, since that is absurd. The most he could do would be to say,
"You're letting them get to you too much." But, Hitler might object, after 2000 (or 3000, who knows?)
years of damage done by these people, why not just get rid of them? Couldn't Nietzsche just say,
"Why not?" Is it really something to worry about so much? No. And, as Nietzsche says, the
"welfare of the human species" may be at stake.

Nietzsche’s ideas of good and evil justify all atrocities, and the people who commit murder
and genocide, in Nietzsche’s views, can’t be blamed for their actions.
Kelly Ross 2003. (“Friedrich Nietzsche”, July 5, 2007, http://www.friesian.com/NIETZSCH.HTM)

At the end of the passage above, before the ones about Rome and Napoleon, what more we get is the idea that
strength cannot but manifest itself as strength, i.e. there was no choice about the noble terror
inflicted by Hitler, or any other predator. “A quantum of strength is equivalent to a quantum of
urge, will, activity, and it is only the snare of language (of the arch-fallacies of reason petrified in
language), presenting all activity as conditioned by an agent -- the "subject" -- that blinds us to this
fact. ....so does popular morality divorce strength from its manifestation, as though there were behind the strong a
neutral agent, free to manifest its strength or contain it. But no such agent exists; there is no "being" behind the
doing, acting, becoming; the "doer" has simply been added to the deed by the imagination -- the doing is everything.
[pp.178-179]So there is no self, no "neutral agent," of the predator, that is free to choose good or
evil. The "doing" is all that there is. So not only cannot Hitler be blamed for being "evil," since
that term is only used by the miserable, impotent, and mean, but he cannot even be said to have
had a choice in the matter, since the idea of choice itself is an "arch-fallacy" perpetrated by the
miserable, impotent, and mean just so that they can blame the strong for acting in their
instinctively strong way.
A2: Nietzsche

Nietzsche leads to Violence


Nietzsche’s view of life and the human race is violent, justifying and praising oppression
and violence.
Kelly Ross 2003. (“Friedrich Nietzsche,” July 5, 2007, http://www.friesian.com/NIETZSCH.HTM)

But, one might think, violence and oppression are unjust! How could any progressive person not
see that expoitation and abuse are wrong! We have Nietzsche's answer: No act of violence, rape,
exploitation, destruction, is intrinsically "unjust," since life itself is violent, rapacious,
exploitative, and destructive and cannot be conceived otherwise. Even more disturbingly, we have to
admit that from the biological [i.e. Darwinian] point of view legal conditions are necessarily exceptional
conditions, since they limit the radical life-will bent on power and must finally subserve, as means, life's collective
purpose, which is to create greater power constellations. To accept any legal system as sovereign and universal -- to
accept it, not merely as an instrument in the struggle of power complexes, but as a weapon against struggle (in the
sense of Dühring's communist cliché that every will must regard every other will as its equal) -- is an anti-vital
principle which can only bring about man's utter demoralization and, indirectly, a reign of nothingness. [p.208,
boldface added]
Nietzsche is certainly life affirming, but then violence, rape, exploitation, and destruction are
intrinsic to his view of life. Attempts to protect the weak, see that justice is done, and mitigate
suffering are "anti-vital" projects that, being adverse to life itself, actually tend towards "a reign
of nothingness." Thus, if we actually care about others and are not just interested in asserting
power over them and using them for our own pleasure, then we can look forward to extinction.
The delicacy -- even more, the tartufferie -- of domestic animals like ourselves shrinks from imagining clearly to
what extent cruelty constituted the collective delight of older mankind, how much it was an ingredient of all their
joys, or how naïvely they manifested their cruelty, how they considered disinterested malevolence (Spinoza's
sympathia malevolens) a normal trait, something to which one's conscience could assent heartily.... To behold
suffering gives pleasure, but to cause another to suffer affords an even greater pleasure. [pp.197-
198, boldface added]
A great part of the pleasure that we get, according to Nietzsche, from injustice to others is simply
the pleasure of inflicting suffering. In this it is worth recollecting the feminist shibboleth that rape is not about
sex, it is about power. Nietzsche would heartily concur. So much the better! And what is more, the value of rape is
not just power, it is the chance to cruelly inflict suffering. The rapist who beats and mutilates, perhaps
even kills, his victim, has done no evil, he is instead one of the heroes of true historic nobility.
And people think that the droit de seigneur represents some "abuse" of power! No! It is the truly noble man as
heroic rapist! Nietzsche would turn around Susan Brownmiller, who said that all men are rapists. No, it is just the
problem that they are not. Nietzsche would regard most men as virtual castrati (domestic oxen, geldings)
for not being rapists.
A2: Nietzsche

Nietzsche leads to Violence


Nietzsche encourages rape, domination, murder, political oppression and violence, saying
they are all noble traits.
Kelly Ross 2003. (Friedrich Nietzsche, July 5, 2007, http://www.friesian.com/NIETZSCH.HTM)

The lack of rights for the dark underclasses brings us to the principal theme of The Genealogy of Morals: The
morality of "good and evil" has been invented out of hatred and resentment by the defeated and
subjugated races, especially the Jews. People who love Nietzsche for his celebration of creativity and
his dismissal of the moralism of traditional religion, mainly meaning Christianity, usually seem to think of
going "beyond good and evil" as merely legitimizing homosexuality, drugs, abortion,
prostitution, pornography, and the other desiderata of progressive thinking. They don't seem to
understand that Nietzsche wasn't particularly interested in things like that, but, more to the point,
legitimizing rape, murder, torture, pillage, domination, and political oppression by the strong. The
only honest Nietzschean graduate student I ever met frankly stated, "To be creative, you must be evil." We get something similar in the recent
Sandra Bullock movie, Murder by Numbers [2002], where the young Nietzschean student simply says, "Freedom is crime." The story of the
movie is more or less that of Leopold and Loeb, the Chicago teenagers who in 1924 murdered a young boy (Bobby Franks) to prove that they
And we are
were "beyond good and evil." Leopold and Loeb understood their Nietzsche far better than most of his academic apologists .
the first to admit that anyone who knew these "good" ones [nobility] only as enemies would find
them evil enemies indeed. For these same men who, amongst themselves, are so strictly constrained by custom, worship, ritual,
gratitude, and by mutual surveillance and jealousy, who are so resourceful in consideration, tenderness, loyality, pride and friendship, when once
Once abroad in the wilderness, they revel in
they step outside their circle become little better than uncaged beasts of prey.
the freedom from social constraint and compensate for their long confinement in the quietude of
their own community. They revert to the innocence of wild animals: we can imagine them
returning from an orgy of murder, arson, rape, and torture, jubilant and at peace with themselves
as though they had committed a fraternity prank -- convinced, moreover, that the poets for a long time to come will
have something to sing about and to praise. Deep within all the noble races there lurks the beast of prey,
bent on spoil and conquest. This hidden urge has to be satisfied from time to time, the beast let
loose in the wilderness. This goes as well for the Roman, Arabian, German, Japanese nobility as for the
Homeric heroes and the Sandinavian vikings. The noble races have everywhere left in their wake the
catchword "barbarian." .....their utter indifference to safety and comfort, their terrible pleasure in
destruction, their taste for cruelty -- all these traits are embodied by their victims in the image of
the "barbarian," and "evil enemy," the Goth or the Vandal. The profound and icy suspicion which the German arouses as soon as
he assumes power (we see it happening again today [i.e. 1887]) harks back to the persistent horror with which Europe for many centuries
witnessed the raging of the blond Teutonic beast (although all racial connection between the old Teutonic tribes and ourselves has been lost).
The "noble races" are thus ennobled by no restraint or consideration shown
[pp.174-175, boldface added]
for the persons or possessions, let alone feelings, of those helpless strangers who come within
their power. "Spoil and conquest," rape and torture, are fun. Kaiser Wilhelm got in the spirit of things by telling
German troups to act like the "Huns of Attila" on their mission to Peking in 1900. No Nietzschean has any business, for example, damning
Christopher Columbus for enslaving the Caribs. While Nietzsche actually seems to think that the "blond Teutonic beast" was gone from
Germany, and Hitler, as noted, hardly fills the bill, there is actually no lack of blonds in the "Nordic" nations, and Nietzsche himself here seems
to have a relatively expansive notion of racial superiority. While he apparently thought of the Roman nobility as themselves of Aryan extraction,
he can hardly have thought the same of the Arabians or Japanese. This acknowledgment would have been of material advantage in World War II,
when many Arabs preferred the Germans to the British (or to the Zionist Jews of Palestine) -- while the Japanese, even today, often think of
themselves as a pure and superior race. As actual German Allies in World War II, the Japanese where in close competition with Germany for
atrocities against civilians and prisoners-of-war (though the Germans were relatively considerate of American and British prisoners, while brutal
to Russians and others, as the Japanese were to all).
A2: Nietzsche

Nietzsche leads to Violence


Nietzsche rationalized genocide as noble--the “strong” simply overcoming the “weak”
Christopher Simpson, author and professor of journalism, 1995 [“The Splendid Blond Beast,”
http://www.thirdworldtraveler.com/Genocide/SplendidBlondeBeast.html

Friedrich Nietzschecalled the aristocratic predators who write society's laws "the splendid blond
beast" precisely because they so often behave as though they are beyond the reach of elementary
morality. As he saw things, these elites have cut a path toward a certain sort of excellence
consisting mainly of the exercise of power at the expense of others. When dealing with ordinary
people, he said, they "revert to the innocence of wild animals.... We can imagine them returning from an orgy of
murder, arson, rape and torture, jubilant and at peace with themselves as though they had committed a fraternity
prank-convinced, moreover, that the poets for a long time to come will have something to sing about and to praise.''
Their brutality was true courage, Nietzsche thought, and the foundation of social order. Today
genocide-the deliberate destruction of a racial, cultural, or political group-is the paramount
example of the institutionalized and sanctioned violence of which Nietzsche spoke. Genocide has
been a basic mechanism of empire and the national state since their inception and remains widely
practiced in "advanced" and "civilized" areas. Most genocides in this century have been perpetrated by
nation-states upon ethnic minorities living within the state's own borders; most of the victims have been children.
The people responsible for mass murder have by and large gotten away with what they have done. Most have
succeeded in keeping wealth that they looted from their victims; most have never faced trial. Genocide is still
difficult to eradicate because it is usually tolerated, at least by those who benefit from it.

Nietzsche’s politics caused the Holocaust


Darrell J. Fasching, professor of religious studies at South Florida, in 1993 [The Ethical Challenge of Auschwitz
and Hiroshima : Apocalypse or Utopia?, p. 25-6]

Scarcely more than half a century after Nietzsche's madman had unleashed his prophecy the
Nazis came along to embrace his vision of a normless will to power. Nietzsche had offered a
vision of a new type of individual who would have to take charge of human history after the
death of God; namely, the Übermensch or self-transcending person. Such individuals would have the
courage to "transvalue all values” and remake the world in their own image. Nietzsche, of course, had
a somewhat aristocratic vision of these new individuals. But his vision was easily usurped by the Nazis
who imagined themselves, the pure Aryan race, as the natural embodiment of this superior
human being who would recreate the world through a will to power. The Nazi program of
attempted genocide of the Jews is a logical outcome of this new normless situation expressed in
Nietzsche's parable of "the Death of God." In a world where power is the final arbiter of values and
might makes right, deicide is inexorably followed by genocide.
A2: Nietzsche

Alt Fails
Nietzsche’s alternative is completely outdated and could never be achieved in modern
society. We must apply Nietzsche politically for his philosophy to be effective
William Connolly, Professor of Political Science at John Hopkins University, 1991 [Identity/ Difference:
democratic negotiations of political paradox]

'But the collapse of two types called "man" and "overman" also results from disappearance of the social space in which this figure of solitude was supposed to reside.
The overman, remember, rises; above the reactive politics of society, both by cultivating certain dispositions while residing within society and by clearing a space on
the edge of social life. In this marginal space projected by Nietzsche, one could not stifle the definitions others gave one, but one could avoid extensive implication in
a dense web of relations that would render it necessary either to accept those identifications or to struggle against them politically. The
Nietzschean
overman, in its dominant presentation in Thus Spoke Zarathustra, lives a life of relative solitude—one that escapes, for instance, the
hold of the state, that "superfluous new idol," that "coldest of cold monster" that "tells lies in all the tongues of good and evil: "Only where the state ends,
there begins the human being who is not superfluous: there begins the song of necessity, the unique and inimitable tune. . . . Where the state ends—look there my
brother! Do you not see it, the rainbow and the bridges of the overman?
But this picture of a marginal space the effective reach of the tentacles of the state no longer refers to any
discernable place in late-modern time. The avoidances it counsels are no longer available, if they ever
were. The clean air it seeks is polluted at low altitudes and too thin to breathe at the highest . Exactly what
late modern life renders inescapable is the intensive entanglement of everyone with everyone else . No one is
left alone anymore, though too many are compelled to fend for themselves as they respond to the violent impositions of state and society. The social fabric of
interdependencies and conflicts is now too tightly woven; the gaps between the lines of regulation and surveillance have tightened up. This tightening of the social
fabric cannot be measured-by ascertaining whether more or fewer people now live on the wrong side of officially defined norms—a mistake critics of the theme of
"disciplinary society" repeatedly make when they support the thesis they seek to refute by pointing to those who resist, evade, elude, and disrupt social practices of
discipline and normalization. It can be measured by pointing to the enlarged network of intrusions and regulations the army, of misfits face as the standards of
normality are extended and intensified; it can be discerned in the resistances they require in order to sustain themselves amid these demands, and in the extension of
disciplinary techniques to overcome those resistances. Those
who want aggregate measures can count the number of people
today whose primary job is to control, observe, confine, reform, discipline, treat, or correct other people
(think of the police, military personnel, welfare agents, therapists, state security agents, private security agents, advertising firms, prison officials, parole boards,
nursing home attendants, licensing agents, tax officials, and so on) and
the various clients, patients, delinquents, misfits, troubled souls, losers,
subversives, and evaders who provide the primary objects of these practices. And they can compare this index —after
sorting out the complex dimensions that make every aggregate comparison extremely coarse—with its counterpart a hundred (or even fifty) years
ago.
Perhaps we can today listen to enunciations by Nietzsche hundred years ago with ears attuned to a century of social intensification: Do you have courage, O my
brothers? Are you brave? Not courage before witnesses, but the courage of hermits and eagles, which is no longer watched even by a god." Avoid all such
unconditional people! They are a poor sick sort, a sort , of mob: they look, sourly at this life, they have the evil eye for this earth. Avoid all such unconditional people!
They have heavy feet and sultry hearts: they do not know how to dance. How should the earth be light for them?19 One dimension in Zarathustra's message can still
be heard by those with ears, but the metaphors of wildness, hermits, eagles, snakes, caves, silence, deep wells, high mountains, solitude, mob, flight, and earth that
populate Nietzsche's invocations of the overman no longer do double duty today. The "hermit" has become an anonymous member of a regulated multitude who are
homeless; the "eagle" has become a protected species; the "mob" has become a criminal network entangled with official intelligence agencies; the "deep well"
accumulates pollutants from road maintenance, toxic wastes, and fertilizer runoffs; urban "caves" have become nightly residences for homeless outcasts who restlessly
haunt the streets by day; the "earth" has become a deposit of finite resources for late-modern production. The
Nietzschean metaphors now refer
to a spiritual disposition disjoined from topographical space; they are drained of reference to identifiable
sites between the lines of social organization .. Even the metaphors have become infiltrated by the signification they would rise above.
These changes in the signifying power of Nietzsche's nineteenth-century metaphors point to the collapse
of social space for the overman as an independent, solitary type . The distinction between types now gives way to struggle within
and between selves. The elevation to a fictive space above the muck of reactive politics must be translated into
political engagement with institutionalized practices . Put another way: the overman must either become a
beautiful soul or be dismantled as an apolitical type: either Nietzschean critique of ressentiment becomes
an anachronism or it is refigured into a political philosophy.
A2: Nietzsche

Extinction O/W
Nietzsche’s philosophy ignores the character of modern politics--The possibility of
extinction precludes his alternative
William Connolly, Professor of Political Science at John Hopkins University, 1991 [Identity/ Difference:
democratic negotiations of political paradox, p. 186]
Zarathustra says: "The most concerned ask today, 'How is man to be preserved?' But Zarathustra is the first and only one to ask: `How is man to
be overcome?"16 The
idea is to stop worrying about the preservation of man, to strive to create a few
overmen. Leave to their own devices those who insist upon being consumed by resentment, so
that a few can cultivate another type of humanity. The new type to be cultivated consists of a few free spirits who fend
off the resentment against the human condition that wells up in everyone, a few who rise above the insistence that there be symmetry between
evil and responsibility, who live above the demand that some guilty agent worthy of punishment be located every time they themselves suffer,
who recognize that existential suffering is a precondition of wisdom.
But this typological differentiation between man and overman no longer makes much sense, if it
ever did. For the overman— constituted as an independent, detached type—refers simultaneously to a spiritual disposition and to the
residence of free spirits in a social space relatively insulated from reactive politics. The problem is that the disappearance of the relevant
social preconditions confounds any division of humanity into two spiritual types.
If there is anything in the
type to be admired, the ideal must be dismantled as a distinct caste of solitary individuals and
folded into the political fabric of latemodern society. The "overman" now falls apart as a set of distinctive
dispositions concentrated in a particular caste or type, and its spiritual qualities migrate to a set of dispositions that
may compete for presence in any self. The type now becomes (as it already was to a significant degree) a voice in the
self contending with other voices, including those of ressentiment.
This model is implicitly suggested by Foucault when he eschews the term "overman" (as well as "will to power")
and shifts the center of gravity of Nietzschean discourse from heroes and classical tragic figures to everyday misfits
such as AlexiAlexina and Pierre Riviere. These textual moves are, I think, part of a strategy to fold Nietzschean
agonism into the fabric of ordinary life by attending to' the extraordinary character of the latter. I seek to pursue this
same trail.
The Nietzschean conception of a few who overcome resentment above politics while the rest
remain stuck in the muck of resentment in politics is not today viable on its own terms. Today
circumstances require that many give the sign of the overman a presence in themselves and in the
ethicopolitical orientations they project onto the life of the whole. But this break with the spirit of Nietzsche
requires further elucidation. The shift results partly from the late-modern possibility of self-extinction. In
this new world the failure to "preserve man" could also extinguish the human basis for the
struggle Nietzsche named "overman." Preservation and overcoming are now drawn closer
together so that each becomes a term in the other: the latter cannot succeed unless it touches the
former. But the entanglement of each with the other in sociopolitical relations means, when the
logic of this entanglement is worked out, that the "overman"' as a type cannot eliminate from its
life some of the modalities definitive of the "human." If the overman was ever projected as a distinct type—and this is
not certain—it now becomes refigured into a struggle within the self between the inclination to existential resentment and an affirmation of life
that rises above this tendency.
*** Security ***

A2: Security K

Realism may be bad but its inevitable, we must take action to do the best we can inside this
framework for evaluating the world
Hixson 2005 University of Akron 2k5 (Walter, Review of The Tragedy of Great Power Politics, Journal of Cold War Studies 7.3 (2005)
149-151 Muse)

To call this
argument bold and deterministic would be an understatement. With a few minor exceptions, it
explains everything that has happened in great-power politics since the French Revolution.
Individual state actors, whether Winston Churchill or Adolf Hitler, Franklin Roosevelt or Josef Stalin, do not matter
in the slightest. National culture, ideology, and domestic politics are nearly irrelevant to understanding foreign
policy, past and future. Diplomacy and engagement between states will not work. Offensive realism alone explains
world politics; answers will not be found in archives or in any other discourse or theoretical construct. Many will
judge such an uncompromising level of deterministic realism as offensive indeed. Theorists from
other schools have stressed that language itself must be carefully examined for the meaning it conveys. The term
"realism" seized the linguistic [End Page 150] upper hand from the start by positing as its binary
opposite "idealism," typically associated with Woodrow Wilson's failed quest to establish a lasting peace after
World War I. Mearsheimer assails not only idealism but liberalism, defensive realism, institutionalism, and any
other ism that might get in the way of his own. The impetus for Mearsheimer's book was the post–Cold
War euphoria, best encapsulated by Francis Fukuyama's "end of history" thesis on the putative triumph of global
liberalism. Mearsheimer anxiously argues that the post–Cold War order is not new and promises to be no less
conflictual than before. The best way to handle statecraft, he argues, is to pursue power rather than peaceful
understanding among states in a global community. Mearsheimer acknowledges that this situation is
"genuinely tragic" (p. 3) but only because it is an inevitable product of the unrelenting will to
power that is the nature of the human beast. The fact that states pursue power is decidedly not
tragic. What would be tragic, and highly destabilizing, would be the adoption of cooperation,
demilitarization, and diplomacy as models of state behavior, rather than the aggressive nation-
centered pursuit of power. Mearsheimer thus emphasizes that his book does not just focus on the past but offers
as well a "prescriptive theory" advising that "states should behave according to the dictates of offensive realism,
because it outlines the best way to survive in a dangerous world" (p. 11). It would be unfair to
dismiss Mearsheimer's work in its entirety. Realism is an important concept, and it would be
futile to attempt to understand modern world politics without due consideration of the theory and
its influence over the minds of statesmen. Mearsheimer clearly knows his subject well, and his book is a product of
years of research and study. Much can be learned from some of its themes, such as the linkage between wealth and
power and the factors that motivate states to intervene on some occasions and "pass the buck" on others. The book is
certainly provocative, which in itself is praiseworthy.
A2: Security K

Discourse of security prevents extinction – it justifies the taking of action by the US against
terrorist organization that have the capability to destroy the world
Noorani 2005 ass’t prof near east studies @ U Arizona, Tuscon, 2k5 (Yaseen, “The Rhetoric of Security” The New Centennial Review
5.1 (2005) 13-41 Muse)

The Bush administration perpetually affirms that the war against terrorism declared in response to the attacks of September 2001 is
"different from any other war in our history" and will continue "for the foreseeable future."1 This affirmation, and indeed
the very declaration of such a war, belongs to a rhetoric of security that predates the Bush administration and which this
administration has intensified but not fundamentally altered. Rhetorically speaking, terrorism is the ideal enemy of the United
States, more so than any alien civilization and perhaps even more so than the tyrannies of communism and fascism, terrorism's defeated sisters.
This is because terrorism is depicted in U.S. rhetoric not as an immoral tactic employed in political struggle,
but as an immoral condition that extinguishes the possibility of peaceful political deliberation. This
condition is the state of war, in absolute moral opposition to the peaceful condition of civil societ y. As a state
of war, terrorism portends the dissolution of the civil relations obtaining within and among nations, particularly liberal nations, and thus portends
the dissolution of civilization itself. [End Page 13] Terrorism is therefore outside the world order, in the sense that it
cannot be managed within this order since it is the very absence of civil order. For there to be a world order at all,
terrorism must be eradicated. In prosecuting a world war against the state of war, the United States puts itself outside the world order
as well. The Bush administration affirms, like the Clinton administration before it, that because the identity of the United States lies in
the values that engender peace (freedom and democracy), the national interests of the United States always
coincide with the interests of the world order. The United States is the animus of the world order and the
power that sustains it. For this reason, any threat to the existence of the United States is a threat to world peace
itself, and anything that the United States does to secure its existence is justified as necessary for the
preservation of world peace. In this way, the existence of the United States stands at the center of world peace and liberal values, yet
remains outside the purview of these values, since when under threat it is subject only to the extra-moral necessity of self-preservation. I will
argue that the symmetrical externality of the United States and terrorism to the world order lies at the
foundation of the rhetoric of security by which the U.S. government justifies its hegemonic actions and
policies. This rhetoric depicts a world in which helpless, vulnerable citizens can achieve agency only
through the U.S. government, while terrorist individuals and organizations command magnitudes of destructive power previously held
only by states. The moral-psychological discourse of agency and fear, freedom and enslavement invoked by this rhetoric is rooted in both
classical liberalism and postwar U.S. foreign policy. The war of "freedom" against "fear" is a psychic struggle with no
specific military enemies or objectives. It arises from the portrayal of the United States as an autarkic, ideally impermeable
collective agent that reshapes the external world in its own image. The war of freedom against fear thereby justifies
measures said to increase the defenses and internal security of the United States as well as measures said
to spread freedom and democracy over the world. Now that the destructive capacity of warlike
individuals can threaten the world order, the power of the United States must be deployed in equal
measure to neutralize this threat throughout the world . The world as a [End Page 14] whole now comes within the purview of
U.S. disciplinary action. Any manifestation of the state of war, terrorist activity, anywhere in the world, is now a threat to the existence of the
United States and to world peace. There is no "clash of civilizations," but the Middle East, as the current site of the state of war, is the primary
danger to the world and must be contained, controlled, and reshaped. The symmetrical externality of the United States and
terrorism to the world order, then, allows its rhetoric to envision a historic opportunity for mankind—the
final elimination of the state of war from human existence, and fear from the political psyche . This will be
achieved, however, only by incorporating the world order into the United States for the foreseeable future
A2: Security K

We must act in realist fashion towards Africa – its essential to our interest
Lake 2006 Professor in the Practice of Diplomacy @ Georgetown 2k6 (Anthony, “More than humanitarianism”
http://www.cfr.org/publication/9302/more_than_humanitarianism.html

First, Americans must pause and reflect on how Africa has become a region of growing vital
importance to U.S. national interests. It is outdated and counterproductive to assume that Africa is simply the
object of humanitarian concerns or a charity cause. The need for a broader approach exists even while the
United States should and does stand ready to answer Africa’s urgent humanitarian needs.
Nevertheless, steadily in recent years, and with an accelerating pace post-9/11, other newly emergent U.S.
stakes in Africa have become apparent: energy, terror, and HIV/AIDS. As these interests have grown
in importance, Africa has become a more competitive environment, in particular with China’s
rapidly escalating engagement and quest for Africa’s energy and other natural resources. These new realities
challenge our thinking and our policies. Second, a more comprehensive policy is needed. Such a policy is
essential for the United States to operate effectively in the increasingly competitive environment
in Africa. A broader policy framework is needed to correct U.S. intelligence and diplomatic
weaknesses. Such an approach would bind the diverse and promising recent U.S. initiatives—in
counterterrorism, HIV/AIDS, and the reward of good governance and economic reform—that today
operate in relative isolation of one another into a coherent, dynamic policy. It would recognize the
growing capacity of African leaders and institutions working to improve economic performance and governance, to
promote democracy, and to resolve conflicts. Finally, this more comprehensive approach will strengthen
the U.S. response to Africa’s humanitarian needs, not weaken it. The results will not end poverty in
Africa, but they will raise hope within the bounds of realism.
A2: Security K

Current stances towards Africa are hollow – only embracing realism can infuse activism to
solve Africa’s biggest problems with life
Rothchild 2001 Professor of Political Science at the University of California, Davis 2k1 (Donald, SAIS Review 21.1, “The U.S. Foreign
Policy Trajectory on Africa” Muse)

U.S. policymakers have broadened their African agenda in the wake of the Cold War. No longer concerned about stemming Soviet expansion or maintaining a global
Global strategic priorities have been
strategic balance, those formulating U.S. policy can now accept and work with Africa's uniqueness.
replaced by a diffuse set of purposes, which involve limited risks and costs. The Clinton
administration has attempted to deal with a complex array of challenging African issues--
security, conflict resolution, democratization, human rights, trade, AIDS, and the environment.
Its agenda is broad but deficient in depth and obligation. With limited interests at stake in the post-Cold
War era, U.S. leaders perceive no compelling reason to rally the world community for a genuine
African Renaissance. The result is a tendency toward activism without follow-through. The Clinton team
reacted to immediate challenges but lacked an overarching policy framework for coping effectively with the continent's long-term problems of conflict, disease, and
1
poverty. The Clinton administration's record on Africa is, on balance, a mixed one. Early enthusiasm led to increased trade, offers of debt relief, and actions to
enlarge democracy. However, since the Somali debacle, the administration has cautiously resisted commitments to UN peacekeeping initiatives and allowed aid levels
achievements never matched rhetoric. With the American
as a percent of GNP to sink to inappropriate levels. On the whole,
public mesmerized by the country's prosperity and largely uninformed about pressing problems
abroad, there has been little public pressure to close the gap between promise and performance. Rather than
develop a coherent strategy for coping with African challenges, the policy problem, as John Clark
remarks, "has been left to the foreign policy bureaucracies [End Page 179] to work out the priorities, as
well as to translate vague presidential concerns into concrete initiatives." 2 Such insufficient concern and
commitment make the United States appear indifferent toward many of the major issues swirling
about the continent. In fact, the United States retains a limited capacity to influence African actions, but this
applies mainly to policy issues that are in line with the preferences of African leaders and publics. When it comes
to disagreeable matters such as genocide in eastern Congo, electoral manipulation, or the plight of internally
displaced persons, however, the partially autonomous African state lies outside the reach of the
United States. Why has the United States come to appear irrelevant to some of Africa's most troubling problems in the twenty-first century? Certainly, such
factors as the weakness of the African state and state system, Africa's partial integration into the global capitalist system, and the limited autonomy of African states
from external influences contribute to this outcome. This essay will examine the tendency toward a low-profile U.S. policy that has been manifest, with some
, after
prominent exceptions, over the post-colonial period, and the contribution of this tendency to the distance at which the United States keeps Africa. Then
analyzing the gap between the executive branch and both the Congress and general public on
African-related issues in the Clinton period, it will focus on the need to build a U.S. constituency
that is supportive of Africa. Only when such a concerned and informed constituency emerges
will an overarching U.S. strategy commensurate Africa's needs and aspirations materialize.
***A2: Borders***

at borders: inevitable

Borders are inevitable because states use them for security.

Starr, 06 (Harvey, Dag Hammarskjold Professor in International Affairs and Chair of the Department of Political Science at the
University of South Carolina. “International Borders: What They Are, What They Mean, and Why We Should Care” SAIS review
vol. XXVI no.1, Winter-Spring,Project Muse).

In a recent article on the nature of borders and their relationship to international conflict, this author noted:
The location of states, their proximity to one another, and especially whether or not they share "borders," emerge time and again
as key variables in studies of international conflict phenomena: from major power general war, to the diffusion of international
conflict, to the analysis of peace between pairs of democracies... From Boulding's (1962) ideas of "behavior space," "loss-of-strength gradient" and "critical boundary" to the
simple but profound concern of geographers that humans interact most with those to whom they are closest (Zipf 1949), there are powerful theoretical reasons to be interested in borders and how they affect
international relations.1 Broadly, the concept of "border" has been an important one throughout world history. The concept of a border as the demarcation of two sovereign states was essential to the Westphalian
state system that developed following the Thirty Years War. This example illustrates two related aspects of borders derived from realism's approach to international relations: borders as legal phenomena and borders
, territoriality is a central component of state security
as related to security. [End Page 3] International law and legal matters have never been key concerns of realism. However
and is fundamental to the (more or less deterministic) geopolitical setting that also affects the security of states.

Borders have an enormous and inevitable impact on international affairs.

Starr, 06 (Harvey, Dag Hammarskjold Professor in International Affairs and Chair of the Department of Political Science at the
University of South Carolina. “International Borders: What They Are, What They Mean, and Why We Should Care” SAIS review
vol. XXVI no.1, Winter-Spring,Project Muse).

Borders matter. Even in today's "turbulent," post-Cold War world of growing democracy, ever-extensive interdependence, and
globalization, borders still serve a wide variety of functions across the areas of security, economics, politics, and social
interactions. Even as some aspects of international law challenge or erode traditional notions of sovereignty, borders delineate
areas of legal competence. Borders provide one key element in the structure of the global system: mapping the number and
arrangement of the territorial units upon which all humans live. Borders permit a spatial approach to international or global
politics by setting out the location of states and their absolute and relative distances from each other. Borders act as factors of
constraint on human interaction, as well as factors that facilitate human interaction. Borders have significant effects on
international politics, both by their presence and by their meaning to humans (either peoples, policymakers, or scholars). In turn,
the internal and external politics of peoples, sub-state organizations, and states affect the creation, dissolution, and meaning of
borders. As I have argued in earlier work, analysts of international politics cannot ignore the spatial dimension of human
relations.
at borders: solves war

Respect for borders has prevented major wars.

Zacher, 01 (Mark, International Organization, Vol. 55, No. 2. (Spring, 2001), pp. 215+ Jstor).

The decline of successful wars of territorial aggrandizement during the last half century is palpable. In fact, there has not been a
case of successful territorial aggrandizement since 1976. Furthermore there have been important multilateral accords in support
of the norm and frequent interventions by international organizations to force states to withdraw from foreign countries. Clearly, a
central source of the norm has been the industrialized world’s fear that territorial revisionism could ignite a major war that would
cause great human suffering. Several scholars have observed that this revision against the imposition of physical pain has been
central to the strengthening of a variety of security and human rights regimes. The experiences of the two world wars, a general
understanding of territorial revisionism’s encouragement of major wars, and a fear of nuclear weapons drove the development of
the territorial integrity norm at key points in its multilateral legitimization.
at borders: africa

Their description of African borders as only products of colonialism ignores numerous other factors that went into their
construction.

Mbembe, 2000 (Achille, Public Culture 12.1 (2000) 259-284, Visitng Professor of History at Yale University “At the Edge of the
World: Boundaries, Territoriality, and Sovereignty in Africa” Project Muse).

Moreover, to state that current African boundaries are merely a product of colonial arbitrariness is to ignore their multiple
geneses. In fact, their establishment long antedated the Congress of Berlin held in 1884, whose objective was to distribute
sovereignty among the different powers engaged in dividing up the continent. Their protogenesis goes back to the period of the
trading-post economy, when Europeans set up agencies on the coasts and began to trade with the natives. The establishment of
this economy explains, in part, some of the physical characteristics of African states, and first of all the distinction between the
littoral areas and the hinterland that so deeply marks the geographical structure of various countries, or again the enclosure of
vast enclaves situated far from the oceans. Boundaries gradually crystallized during the period of "informal empire" (from the
abolition of the slave trade up to the repression of the first resistance movements), thanks to the combined action of traders and
missionaries. The rise of boundaries took a military turn with the construction of forts, the penetration of the hinterland, and the
repression of local revolts. Far from being simple products of colonialism, current boundaries thus reflect commercial, religious,
and military realities, the rivalries, power relationships, and alliances that prevailed among the various imperial powers and
between them and Africans through the centuries preceding colonization proper. From this point of view, their constitution
depends on a relatively long-term social and cultural process. 19 Before the conquest, they represented spaces of encounter,
negotiation, and opportunity for Europeans and Africans. 20 At the time of conquest, their main function was to mark the spatial
limits that separated colonial possessions from one another, taking into account not ambitions but the actual occupation of the
land.

Even if African borders are arbitrary, they have become part of the social landscape and are not going to change.

Atzili, 07 (Boaz, Research Fellow in the Belfer Center for Science and International Affairs at the John F. Kennedy School of
Government at Harvard University. “When Good Fences Make Bad Neighbors: Fixed Borders, State Weakness, and
International Conflict”, International Security, 31.3 page 139-173 Project Muse).

Africa's borders are particularly intriguing. Despite the arbitrariness with which many state borders in Africa were drawn, they
have remained largely fixed.25 From its inception in 1963, the Organization of African Unity (OAU) has endorsed the norm in
accordance with the principle of preserving the colonial territorial status quo. 26 In practice, as Jeffery Herbst notes, "the vast
majority of [African borders] have remained virtually untouched since the late 1800s, when they were first demarcated." The
OAU's determination to uphold the norm was demonstrated, for instance, in the 1967–70 civil war in Nigeria, when the
organization sought to prevent Biafra's attempts to secede.27
***RANDOM CARDS***

at critiques of science

Science is the opposite of domination – scientific knowledge liberates and improves lives.
Bronner 04 Stephen Eric Bronner, Professor of Political Science at Rutgers University, 2004, Reclaiming the Enlightenment:
Toward a Politics of Radical Engagement, p. 21-23

Something will always be missing: freedom will never become fully manifest in reality. The relation between
them is asymptotic. Therefore, most philosophes understood progress as a regulative ideal, or as a postulate,13
rather than as an absolute or the expression of some divine plane or the foundation for a system.’4 Even in
scientific terms, progress retained a critical dimension insofar as it implied the need to question established
certainties. In this vein, it is misleading simply to equate scientific reason with the domination of man and
nature.15 All the great figures of the scientific revolution —Bacon, Boyle, Newton—were concerned with
liberating humanity from what seemed the power of seemingly intractable forces. Swamps were everywhere;
roads were few; forests remained to be cleared; illness was rampant; food was scarce; most people would never
leave their village. What it implied not to understand the existence of bacteria or the nature of electricity, just to
use very simple examples, is today simply inconceivable. Enlightenment figures like Benjamin Franklin, “the
complete philosophe,”’6 became famous for a reason: they not only freed people from some of their fears but
through inventions like the stove and the lightning rod they also raised new possibilities for making people’s
lives more livable. Critical theorists and postmodernists miss the point when they view Enlightenment
intellectuals in general and scientists in particular as simple apostles of reification. They actually constituted
its most consistent enemy. The philosophes may not have grasped the commodity form, but they empowered
people by challenging superstitions and dogmas that left them mute and helpless against the whims of
nature and the injunctions of tradition. Enlightenment thinkers were justified in understanding knowledge as
inherently improving humanity. Infused with a sense of furthering the public good, liberating the
individual from the clutches of the invisible and inexplicable, the Enlightenment idea of progress required
what the young Marx later termed “the ruthless critique of everything existing.” This regulative notion of
progress was never inimical to subjectivity. Quite the contrary: progress became meaningful only with
reference to real living individuals.
at said: no alternative

Post-colonialism essentializes oppression and makes resistance impossible.

Ong 99 Aihwa Ong, Professor of Anthropology at UC Berkeley, Flexible Citizenship: The Cultural Logic of Transnationality, 1999,
p. 33-34

More broadly, postcolonial theorists focus on recovering the voices of subjects silenced by patriarchy and
colonial rule (The Empire Writes Back is the title of one popular collection); they assume that all
contemporary racial, ethnic, and cultural oppressions can all be attributed to Western colonialisms.
American appropriations of postcolonial theory have created a unitary discourse of the postcolonial that
refers to highly variable situations and conditions throughout the world; thus, Gayatri Spivak is able to talk about “the paradigmatic subaltern woman,” as well as
“New World Asians (the old migrants) and New Immigrant Asians (often ‘model minorities’) being disciplinarized together?” Other postcolonial feminists also have been eager to seek
structural similarities, continuities, conjunctures, and alliances between the postcolonial oppressions experienced by peoples on the bases of race, ethnicity, and gender both in formerly
colonized populations in the third world and among immigrant populations in the United States, Australia, and England.16 Seldom is there any attempt to link these assertions of unitary
postcolonial situations among diasporan subjects in the West to the historical structures of colonization, decolonization, and contemporary developments in particular non-Western
countries. Indeed, the term postcolonial has been used to indiscriminately describe different regimes of economic, political, and cultural domination in the Americas, India, Africa, and
other third-world countries where the actual historical experiences of colonialism have been very varied in terms of local culture, conquest, settlement, racial exploitation, administrative
postcolonial theory can represent a kind of theoretical
regime, political resistance, and articulation with global capitalism. In careless hands,
imperialism whereby scholars based in the West, without seriously engaging the scholarship of faraway places,
can project or “speak for” postcolonial situations elsewhere. Stuart Hall has warned against approaches
that universalize racial, ethnic, and gender oppressions without locating the “actual integument of
power…in concrete institutions.” A more fruitful strand of postcolonial studies is represented by subaltern scholars such as Partha Chatterjee, who has criticized
the Indian national projects, which are based on Western models of modernity and bypass “many possibilities of authentic, creative, and plural development of social identities,” including
the marginalized communities in Indian society. He suggests that an alternative imagination that draws on “narratives of community” would be a formidable challenge to narratives of
capital. This brilliant work, however, is based on the assumption that both modernity and capitalism are universal forms, against which non-Western societies such as India can only
mobilize “pre-existing cultural solidarities such as locality, caste, tribe, religious community, or ethnic identity.” This analytical opposition between a universal modernity and non-Western
culture is rather old-fashioned it is as if Chatterjee believes the West is not present in Indian elites who champion narratives of the indigenous community. Furthermore, the concept of a
universal modernity must be rethought when, as Arif Dirlik observes, “the narrative of capitalism is no longer the narrative of the history of Europe; non-European capitalist societies now
The loose use of the term “the postcolonial,” then, has had the bizarre
make their own claims on the history of capitalism.”20
effect of contributing to a Western tradition of othering the Rest; it suggests a postwar scheme whereby “the
third world” was followed by “the developing countries,” which are now being succeeded by “the postcolonial.”
This continuum seems to suggest that the further we move in time, the more beholden non-Western
countries are to the forms and practices of their colonial past. By and large, anthropologists have been
careful to discuss how formerly colonized societies have developed differently in relation to global economic
and political dominations and have repositioned themselves differently vis-a-vis capitalism and late modernity.
By specifying differences in history, politics, and culture, anthropologists are able to say how the postcolonial
formation of Indonesia is quite different from that of India, Nicaragua, or Zaire.
at butler: perm

Perm solves best – even Butler agrees strategically using gender categories is politically effective

Baldwin, 97 Margaret A. Baldwin, Assoc. Prof Law @ FSU, Spring 1997, “Public Women and the Feminist State,” 20 Harv.
Women’s L.J. 47, p ln

However salutary the postmodern goal of de-essentializing women, postmodern theory ultimately effaces
the specific situation of public women, and forfeits altogether any account of gender along the way. This
difficulty, and its implications for political strategy, is often spoken of but rarely addressed seriously within
postmodern feminism. Denise Riley offers the diktat that at such junctures women can know amongst
themselves "that 'women' don't exist -- while maintaining a politics of [*160] 'as if they existed' -- since
the world behaves as if they unambiguously did." 434 Judith Butler makes the same tactical concession
when she affirms the continued necessity of asserting "a generally shared conception of 'women'" 435 as a
political strategy: Within feminism, it seems as if there is some political necessity to speak as and for
women, and I would not contest that necessity. Surely, that is the way in which representational politics operates,
and . . . lobbying efforts are virtually impossible without recourse to identity politics. So we agree that
demonstrations and legislative efforts and radical movements need to make claims in the name of women.
436
identity politics: cede the political

Identity politics only fracture the left—a universal goal of using reform to fight oppression will be more effective.

Wilson, 2000 – Editor and Publisher of Illinois Academe – 2000 (John K. Wilson, “How the Left can Win Arguments and Influence
People” p. 124- 126)

Many philosophers of the left propose solving all the problems posed to progressives by uniting the left behind some single
universal principle of class, abandoning its diverse obsessions with race, gender, sexual orientation, environmentalism, and all
the other "isms" of the left. A left that works for the working class is not incompatible, however, with a left that campaigns for
equality on the basis of race, gender, sexual orientation, disability, age, and so on. If progressives ignore the fact that African
Americans or women or gays and lesbians face inequality and discrimination, no one will take them seriously when they argue
that the poor are treated unjustly. A theory of oppression that challenges the American presumption of equality must encompass
all these oppressions, not just the ones that seem most politically palatable. Conservative forces developed the term special-interest groups to attack progressive
causes. For Republicans, it became the way to attack the best-organized leftist parts of the Democratic Party. For Democrats, denouncing "special interests" has been the favorite tactic of centrist candidates who
want to eliminate progressive forces with their own party. The special-interest groups are not nearly as special as the corporate interests. The "special interests" have relatively little power, even within the Democratic
Party, and they represent a far larger part of America than do the corporations that often oppose them. Progressives are tagged with the label of being the servants of "special interests”-blacks, Latinos, women, gays
and lesbians, labor unions, the poor, the disabled, and so forth. Of Course, if you add up all the "special interests" supposedly beholden to the left, they represent more than 90 percent of the population. Obviously,
. There is no inherent conflict between the so-called
it's the right and the neoliberals who serve the truly “special” interests: rich straight white men and the corporations they run
special interests on the left and the ideal of common progressive ideals. The goal of liberty and equality requires addressing the
injustices affecting these so-called special interests. No progressive movement can be true to its name unless it speaks out
against racial discrimination, gender inequality, homophobia, and any other forms of oppression. Progressives must demand that
class be added to these issues, but class analysis alone cannot adequately address all inequalities. Class alone does not explain
why the wealthy African American driving a BMW gets pulled over by the police. Class alone does not explain why women face a
glass ceiling (or, even more often, a glass door). Exclusively class-based solutions may help some poor white males who
legitimately need assistance, but class ignores too many other important factors. It's the unequal treatment of women and
minorities that helps create many of these class inequalities, and the politics of racism and sexism that prevents the public from
looking at the inequities created by class. Both the Republican and Democratic Parties are notorious for using race to distract
poor whites from the poverty they face and to prevent them from organizing to achieve greater equality. The answer to this
problem is not to abandon the cause of racial equality in hopes of building a larger progressive movement based on appealing to
racists. The left can succeed only if it has principles. What the left needs to do is communicate the fact that these principles
promote an ideal of equality that includes advancing the cause of poor people of all races. Race and gender analysis does not distract us from
understanding an overarching principle of class. On the contrary, no one can understand how class inequalities exist in America without comprehending how racial bias and gender discrimination create many of
these class inequities. Class is not a magic word capable of bringing the masses to progressive causes, any more than race or gender has brought success to the left. Progressives need to cast a wide net of
interlinked causes that can be pursued separately while still contributing to the ultimate goal of justice and equality. The left needs intellectuals and activists, academics and anarchists, special-interest groups and
. The
broad coalitions, Marxist ideologies and crazed hippies, atheists and religious freaks, reformers and revolutionaries, and everybody else. This is the left. It cannot be controlled, but it can be nudged
diverse left cannot be led down a single path, but perhaps it can be guided in a general direction. Or at the very least, perhaps
the left can be persuaded to give up its internal bickering and recognize its common foes.
specific solvency outweighs the k link

Specific solvency outweighs general theory—theories are only as good as their applications and excessively generic arguments
are a very weak form of reasoning.

Zournazi and Massumi, 02 - , PhD in cultural theory, philosophy, and politics & professor of communications/literature at the
University of Montreal – 2002 (Mary Zournazi and Brian Massumi, “Navigating Movements,” Hope: new philosophies for change)

Critical' practices aimed at increasing potentials for freedom and for movement are inadequate, because in order to critique
something in any kind of definitive way you have to pin it down. In a way it is an almost sadistic enterprise that separates
something out, attributes set characteristics to it, then applies a final judgment to it - objectifies it, in a moralising kind of way. I
understand that using a 'critical method' is not the same as 'being critical'. But still I think there is always that moralising
undertone to critique. Because of that, I think, it loses contact with other more moving dimensions of experience. It doesn't allow
for other kinds of practices that might not have so much to do with mastery and judgment as with affective connection and
abductive participation. The non-judgmental is interesting, you know, because you are always somehow implicated in trying to
make judgments ... To not make judgments in critical thought is a very hard thing to do. It takes a lot courage to move in that
direction, because othetwise ... Well, it requires a willingness to take risks, to make mistakes and even to come across as silly. A
critical perspective that tries to come to a definitive judgment on something is always in some way a failure, because it is
happening at a remove from the process it's judging. Something could have happened in the intervening time, or something
barely perceptible might have been happening away from the centre of critical focus. These developments may become
important later. The process of pinning down and separating out is also a weakness in judgment, because it doesn't allow for
these seeds of change, connections in the making that might not be activated or obvious at the moment. In a sense, judgmental
reason is an extremely weak form of thought, precisely because it is so sure of itself. This is not to say that it shouldn't be used.
***08/09 TOPIC SPECIFIC***
***Land K***
SCIENCE GOOD
Economist can use cost/benefit analysis to advances to solve environmental problems in an
eco-friendly way
Grove-White 1997 (Parliamentary Forestry Commissioner,researcher @ Centre for the Study of Environmental
Change. )Robin .The Environmental Valuation Controversy:Observations on its recent history and significance.
Valuing Nature? Ethics, Economics and the Environment 1997 p. 21

Modern environmentalism has tended to embody a deep suspicion of economists. It is not hard to
see why. At the most obvious level, economic theory has provided—indeed continues to provide
—the underpinning for many environmentally destructive practices. Moreover, in the 1970s and
1980s, the environmental movement gained popular strength partly as an implicit counter to
political-economy doctrines about the imperatives of undifferentiated economic growth and
about the supposedly central role of markets as sources of human welfare.

But these days many economists are on the side of the angels. They are keen to develop new
theoretical tools and methods of analysis which can help solve environmental problems. They
argue that, since it is economic realities which dominate the imperatives of modern government,
environmental priorities too need to be expressed in such terms. Only so, it is urged, will these
priorities be taken seriously within government. Such economists see themselves,
understandably, as friends of the environment. The burgeoning school of economic theorists
concerned to develop methods for capturing in monetary terms the ‘value’ of environmental
goods exemplifies this trend. With the most benign of intentions, such individuals have been
advancing new concepts and techniques for use within cost-benefit analyses: ‘contingent
valuation’, ‘existence value’, ‘hedonic pricing’ and the like. In what follows, I refer to these
innovations collectively as ‘surrogate valuation’ methods.
PERM
Perm solves- Best way to combine values.
Agar 1997 (prof of ethics, senior lecturer @ Victoria University of Wellington)Nicholas. Biocentrism and the
Concept of Life. Ethics, Vol. 108, No. 1 (Oct., 1997),

An obvious option would be to supplement biocentric worth with more familiar brands of value.
On this account the value of the merely alive would be outweighed by the greater value of the rational and
conscious. In this way we avoid the absurd consequences of monistic theories whilst still assigning value to all
living things. Goodpaster leaves this option open by distinguishing between moral considerability and moral
significance." Though a human and an ant cannot be distinguished in terms of their moral considerability, being folk
psy- chological will make the human more morally significant.'2 There are problems for this type of theory. An
amalgam made up of both biocentric value and familiar sentience or rationality-based value
requires some mechanism for systematically comparing these lower and higher goods. Anyone
who takes biocentric value seriously would surely hope that there would be situations in which nonessential human
interests can sometimes be trumped by those of nonsentient nature. Yet without some additional theory this type
of value pluralism cannot rule out there being a very large gap between the values real- ized in
any human being and those in any nonhuman individual. Even the most fleeting and trivial
human desire may deserve more attention than the life of a nonsentient being. Humans are finite
beings whose actions are capable of subserving only a finite number of goals. If the choice is
between courses of action that impact only on nonsentient beings, our value pluralist may urge us
to decide in a nature-respecting way. How often will this be the case? Advancing technology opens an
increasing array of environment-consuming goals to ever growing numbers of humans. Failure to
adequately calibrate human and non- human value may mean that there will almost always be
more worthy recipients of moral concern than nonsentient nature. Familiar human- centered value
threatens, for all practical purposes, to drive this new biocentric value out of existence.
PERM
Perm solves, we can combine the negative's ethics with a responsible use of technologyLeiss
94 (PhD, Scientist @ McLaughlin center for pop. Health risk assessment, U of Ottawa. Author/
collaborator/editor of fifteen books.) William .The Domination of Nature, McGill-Queen's
University Press, 1994,

The liberation of science and technology from the thrall of this dynamic is a task that primarily
involves the reconstruction of social institutions. Theoretical reflection plays a part in this
process by suggesting changes in the categories through which individuals and groups interpret
the significance of their activity. To dethrone science and technology as the guiding forces in the
mastery of nature is a necessary step not only for them, but for the idea as well. Freed from a
historical association that has become anachronistic, the idea of the mastery of nature would be
open to new meanings. Considered as moral progress, it would indicate more forcefully the most
demanding challenge that confronts us: not to conquer external nature, the moon, and outer
space, but to develop the ability -- widely dispersed among all individuals in society -- to use
responsibly the technical means presently available for the enhancement of life, together with an
institutional framework which will nourish and preserve that ability.
PERM
Perm solves best- Integrating ideas about the environment ensures sustainability
Dernbach 2003, John C .Achieving Sustainable Development: The Centrality and Multiple Facets of Integrated
Decisionmaking Indiana Journal of Global Legal Studies - Volume 10, Issue 1, Winter 2003,

To operationalize sustainable development, we need to recognize that one principle—


integrated decisionmaking—holds the other principles together. Integrated decisionmaking
would ensure that environmental considerations and goals are integrated or incorporated
into the decisionmaking processes for development, and are not treated separately or
independently. Of all the principles contained in the sustainable development framework, integrated
decisionmaking is perhaps the principle most easily translated into law and policy tools. We
also need to recognize that integrated decisionmaking has multiple facets, not a single meaning. When we
see the many facets or types of integrated decisionmaking, we find a major way to operationalize sustainable
development. Each facet of integrated
decisionmaking can be implemented by applying or broadening the application of tools that are already
receiving some use. These tools also provide practical ways to move toward sustainable development.
***Famine K***
PERM

The permutation’s contradictions make it the undesirable choice that edkins concludes is
the only ethical action.
EDKINS, 2000 Jenny, Whose Hunger?: Concepts of Famine, Practices of Aid pg. 147 – 148
Following through the deconstructive analysis that I am attempting here, we could perhaps argue that, on the
contrary, famine relief as supplement is undecidable: whether it solves or exacerbates the famine is undecidable—
and hence political. We have here an example of the “double contradictory imperative.”80 On the
one hand, famine relief must be given; food cannot be withheld from the starving. On the
other hand, famine relief must be withheld; it is the relief aid that is causing the famine.81
This leads to an aporia, which takes the form of a contradiction. But “ethics, politics and
responsibility, if there are any, will only ever have begun with the experience and
experiment of the aporia.”82 In other words, it is only through the logic of the aporia, where a
decision has to be made, that we will arrive at something that can be called “political.”
Without this, what we are doing is following a program, claiming a priority for knowledge, and an epistemological
certainty: “when a path is clear and given, when a certain knowledge opens up the way in advance, the decision is
already made, it might as well be said that there is none to make: irresponsibly, and in good conscience, one simply
applies or implements a programme.”83 It may not be possible to escape the program, but if this is what is
happening, then moral or political responsibility does not come into it: “the condition of possibility of this thing
called responsibility is a certain experience and experiment of the possibility of the impossible: the testing of the
aporia from which one may invent the only possible invention, the impossible invention.”84 It is through the
experience of this contradiction or aporia, to which no answer can be found, that ethical
responsibility becomes possible. By accepting the question of famine relief as undecidable,
in the sense that an answer cannot be found through knowledge, the way is opened to the
process of ethico-political decision.
Edkins Indict

Edkins contradicts herself and offers no Alt.


MOISO AND BURGE, 1
Aimee, Kimberly, senior writer and editor at Bread for the World “Whose Hunger? Concepts of Famine, Practices of Aid. - Review - book
review” Christian Century, 16 May, http://findarticles.com/p/articles/mi_m1058/is_16_118/ai_75197309

Edkins, a lecturer in the department of international politics at the University of Wales-Aberystwyth, does
manage to dissect some traditional and modern responses to famine, including country-to-country
global food aid and long-term development programs, but her results are muddled. In seeming
contradiction, she both argues for a rethinking of theories of famine relief and claims their
futility, effectively discounting any potential solutions. "It is not a question of finding better early
warning systems, more participatory development projects, or faster methods of delivering relief. Nor is it a question
of seeking the deeper, more structural causes of famines, nor its complexities," she writes. She identifies
international power relations and violence as sources of famine, but her conclusion begs the
question: If power relations are the cause, is not the remedy to examine their structures
and complexities for solutions? Not that Edkins claims to offer answers. "The practical political
aim of this book is neither to understand famine nor to provide a solution," she writes. Unfortunately, the result is a
digressive book of little practical political use to those able to respond tangibly to famine. Bombarded with images
of severe hunger and famine in the 1980s and early '90s, people grew weary of hearing about the seeming
hopelessness of global poverty. It's refreshing to see new books appearing on this old subject, and heartening to
know that solutions are at hand.
Repoliticization bad.
Attempts to repoliticize that only reverse this vies in the context of famine relief fail to
reconstitute modernity and risks institutionalization of further technologization
EDKINS, 2000
Jenny, Whose Hunger?: Concepts of Famine, Practices of Aid pg. 129-130

I examine some recent attempts to do this—to repoliticize famines—particularly those that introduced the notion of
famine as a complex political emergency. Much of this work began as an endeavor to reintroduce an explicit
consideration of violence and the political missing in Sen’s work. As I argued in chapter 3, violence is always
already present in the violence of the law and the state even in the absence of military
conflict. Efforts to repoliticize run the risk of constituting no more than a struggle to
reverse the view of relief or humanitarian aid as a solution to famine. Such a reversal is
problematic because it does not move outside the frame of modernity. It lays itself open to a
further technologization, and I will discuss how that is taking place. What is needed is a displacement or
resituating of the problematics of humanitarianism and aid. Humanitarianism can be a deeply conservative activity
in the face of the emergence of new political formations, or it can be an emancipatory move. Which it is depends on
the specific configuration in which it is located and the political substance with which it is articulated or linked at
any particular moment. These articulations can be contested: they are not natural or given.
Tech (s) Imperialism

Edikins’ rejection of technological thought allows for incalculable ethics that make it
impossible to criticize imperialism.
DeFAZIO, 3 Kimberly, The Imperialism of "Eating Well" The Red Critique,
http://www.redcritique.org/Spring2003/theimperialismofeatingwell.htm

The suspension of judgment, law, determination, etc., while masquerading as the height of
freedom, is really a means of justifying deeply contradictory practices. When for instance
Chris Barker extends Derrida's and Edkins' theoretical framework to a discussion of the possibility of any social
change, arguing that "ethics do not require to be grounded in anything outside our beliefs
and desires" (13), what he is really saying is that the ethical subject, not determined by any
laws or objectives, can be ethical in one place, and not in another. Post-al ethics is a "politics
without guarantees". But what are "guarantees" on this logic except the ability to connect one's
actions to the larger social forces of which one is a part, and what does the suspension of
"guarantees" do except eliminate the means of understanding the world outside of
experience and immediacy? On the terms of the ethical, there is no basis on which to
critique the U.S. imperialist practices of on the one hand dropping food for the hungry in
its "humanitarian" missions, and, on the other, using food and nutrition as a weapon
against people in the economic blockade against Iraq. The ethical subject can simply say
that because our knowledge is always limited, it is impossible to determine the effects of all
of our practices, let alone their causes. To once again return to Barker—whose "introductory" writings on the
analysis of culture for beginning students serve as an index of the institutionalization of the deeply conservative
trend in cultural studies today—we engage in so many contradictory practices in such a "complex" world, "the
justification of ethics becomes an increasingly complex matter that depends at its best on dialogue and at its worst
on a descent into violence" (13-14). Indeed, for post-al ethics, there is only "indeterminacy" and contradiction, and
violence is thus inevitable. By reducing the subject to the effects of textual oscillations, there is
no principle upon which one acts. Far from serving as an intervention into the domination
of the West, post-al ethics becomes an alibi for U.S. imperialist practices of dropping food
for the hungry, while simultaneously using food and nutrition as a weapon against people
in order to secure U.S. economic interests in oil, labor and other resources. Post-al ethics is a pretext for
opportunism.
***Deep Eco K***
AFF ANSWERS

Deep ecology is Malthusian, Etc


Bookchin, 2006 Murray, Social Ecology versus Deep Ecology: A Challenge for the Ecology Movement, January 4th,
http://libcom.org/library/social-versus-deep-ecology-bookchin

Let us face these differences bluntly: deep


ecology, despite all its social rhetoric, has virtually no real
sense that our ecological problems have their ultimate roots in society and in social problems.
It preaches a gospel of a kind of "original sin" that accurses a vague species called humanity---as though people
of color were equatable with whites, women with men, the Third World with the First, the poor with the rich,
and the exploited with their exploiters.
Deep ecologists see this vague and undifferentiated humanity essentially as an ugly
"anthropocentric" thing---presumably a malignant product of natural evolution---that is "overpopulating"
the planet, "devouring" its resources, and destroying its wildlife and the biosphere--- as though some vague
domain of "nature" stands opposed to a constellation of nonnatural human beings, with their
technology, minds, society, etc. Deep ecology, formulated largely by privileged male white
academics, has managed to bring sincere naturalists like Paul Shepard into the same company
as patently antihumanist and macho mountain men like David Foreman of Earth First! who
preach a gospel that humanity is some kind of cancer in the world of life.
It was out of this kind of crude eco-brutalism that Hitler, in the name of "population control,"
with a racial orientation, fashioned theories of blood and soil that led to the transport of millions
of people to murder camps like Auschwitz. The same eco-brutalism now reappears a half-
century later among self-professed deep ecologists who believe that Third World peoples
should be permitted to starve to death and that desperate Indian immigrants from Latin
America should be exclude by the border cops from the United States lest they burden "our"
ecological resources.
This eco-brutalism does not come out of Hitler's Mein Kampf. It appeared in Simply Living, an Australian
periodical, as part of a laudatory interview of David Foreman by Professor Bill Devall, who co-authored Deep
Ecology with Professor George Sessions, the authorized manifesto of the deep ecology movement. Foreman,
who exuberantly expressed his commitment to deep ecology, frankly informed Devall that
"When I tell people who the worst thing we could do in Ethiopia is to give aid---the best thing
would be to just let nature seek its own balance, to let the people there just starve---they think
this is monstrous. . . . Likewise, letting the USA be an overflow valve for problems in Latin America is not
solving a thing. It's just putting more pressure on the resources we have in the USA."
PERM – COTTON AFF

Case is the net-benefit to the perm: ending cotton subsidies key to hemp popularity, which
is at the root of bioregional movement.
Andrews, 1997 David U., Mother Earth News, “Grown in the USA?”,
http://www.globalhemp.com/Archives/Magazines/grown_in_the_usa.html

“All paper companies in North America are looking for fibers other than wood,” says Patrick
Girouard, economic analyst with Resource Efficient Agricultural Production (REAP)-Canada. Though
agricultural subsidies of other fiber crops like cotton economically militate against hemp for certain
applications, it can find a niche in the short term replacing soft woods for paper pulp. “Over the next 20 years the
world demand for paper products will double, especially in Asia,” says Girouard. He too sees cultural factors
drawing attention to hemp. “When you talk about hemp it’s catchy. Just because it’s illegal, people show more
interest in the beginning.” This, coupled with a global movement to remove price supports from
agricultural commodities should augur well for hemp’s prospects. Within a few years programs in
British Columbia, Saskatchewan and Ontario will begin to yield the data to run bona fide economic analyses, he
says.
The U.S. Department of Agriculture spokesperson calls hemp a good fiber, but “institutional constraints” pose
“overwhelming” obstacles to studying it.
USDA’s Jeffrey Gain, chairman of the board of the Alternative Agriculture Research and Commercialization
Corp., has said, “Anybody who comes to the conclusion that hemp is not viable is probably not
fully aware…”
The state of Wisconsin angered regional drug warriors last spring by organizing a conference on the commercial
cultivation of hemp. Erwin Sholts of the state’s Department of Agriculture figured hemp acreage to be worth “in
the neighborhood of a couple hundred dollars more than corn.” And its root system is “good for fragile
soils,” he said.
Hemp roots, nine to fourteen inches long, condition the soil and provide the plant good water
access. It is highly pest resistant and the crop requires minimal treatment with farm chemicals.
Advocates of hemp commerce look to the day when hemp grown in the US. can support value-
added economies in rural areas, where the commodity will be grown, processed, and sold in the same
community, providing jobs and supporting the local tax base. They refer to this concept as “bioregionalism.”
It should sound familiar to patrons of farmers markets and organics enthusiasts.
AFF ANSWERS

No brightline on which species it is acceptable to eliminate


Bookchin, 2006 Murray, Social Ecology versus Deep Ecology: A Challenge for the Ecology Movement, January 4th,
http://libcom.org/library/social-versus-deep-ecology-bookchin

The problems that deep ecology and biocentrism raise have not gone unnoticed in more thoughtful press in
England. During a discussion of "biocentric ethics" in The New Scientist 69 (1976), for example, Bernard
Dixon observed that no "logical line can be drawn" between the conservation of whales,
gentians, and flamingoes on the one hand and the extinction of pathogenic microbes like
the small pox virus on the other. At which point God's gift to misanthropy, David Ehrenfeld, cutely
observes that the smallpox virus is an "endangered species" in his The Arrogance of Humanism, a
work that is so selective and tendentious in its use of quotations that it should validly be renamed "The
Arrogance of Ignorance." One wonders what to do about the AIDS virus if a vaccine or therapy should
threaten its survival. Further, given the passion for perpetuating the ecosystem of every species,
one wonders how smallpox and AIDS virus should be preserved. In test tubes? Laboratory
cultures? Or to be truly ecological, in their native habitat, the human body? In which case, idealistic acolytes
of deep ecology should be invited to offer their own bloodstreams in the interests of "biocentric equality."
Certainly, if "nature should be permitted to take its course," as Foreman advises for Ethiopians and
Indian peasants, then plagues, famines, suffering, wars, and perhaps even lethal asteroids of the
kind that exterminated the great reptiles of the Mesozoic should not be kept from defacing the purity
of first nature by the intervention of second nature. With so much absurdity to unscramble, one can
indeed get heady, almost dizzy, with a sense of polemical intoxication.
AFF ANSWERS

No Solvency: Alt doesn’t challenge social hierarchies at the root of the ecology crisis
Bookchin, 2006 Murray, Social Ecology versus Deep Ecology: A Challenge for the Ecology Movement, January 4th,
http://libcom.org/library/social-versus-deep-ecology-bookchin

Unless there is a resolute attempt to fully anchor ecological dislocation in social


dislocations, to challenge the vested corporate and political interests known as capitalist
society---not some vague "industrial/technological" society that even Dwight D. Eisenhower attacked with
a more acerbic term---to analyze, explore and attack hierarchy as a reality, not only as a sensibility, to
recognize the material needs of the poor and of Third World people, to function politically, not simply as a
religious cult, to give the human species and mind their due in natural evolution, not simply
to regard them as cancers in the biosphere, to examine economies as well as souls and freedom as
well as immerse ourselves in introspective or scholastic arguments about the rights of pathogenic viruses---
unless in short North American Greens and the ecology movement shift their focus toward
a social ecology and let deep ecology sink into the pit it has created, the ecology movement
will become another ugly wart on the skin of society.
What we must do today is return to nature, conceived in all its fecundity, richness of potentialities, and
subjectivity---not to supernature with its shamans, priests, priestesses, and fanciful deities that are merely
anthropomorphic extensions and distortions of the human as all-embracing divinities. And what we must
enchant is not only an abstract nature that often reflects our own systems of power,
hierarchy, and domination, but rather human beings, the human mind, and the human spirit
that has taken such a beating these days from every source, particularly deep ecology.
Deep ecology, with its Malthusian thrust, its various centricities, its mystifying Eco-la-la, and its
disorienting eclecticism degrades this enterprise into a crude biologism that deflects us from
the social problems that underpin the ecological ones and the project of social
reconstruction that alone can spare the biosphere from virtual destruction.
AFF ANSWERS

Deep ecology denies humanity any unique place in nature


Bookchin, 1990 Murray, Dean for the Institute for Social Ecology, Remaking Society, Pg. 194
Modern ecology, specifically social ecology, is also in need of presuppositions if it is to become a coherent
outlook that tries to explain humanity’s place in the natural world. A number of frivolous ecological
theories have emerged that essentially deny humanity any unique place in nature, say, one
that is different from the “intrinsic worth” of a snail. This view, as I have observed, has a name –
“biocentricity” – and it advances the view that human beings are neither more nor less
“worthy” than snails in the natural world (hence the myth of a “biocentric democracy”). In the
natural scheme of things the two are merely “different”. That they are “different” is a rather trite
fact, but one that tells us nothing whatever about the way they are different and the
significance of that difference in the natural world.
AFF ANSWERS

No solvency because deep ecology fails to tackle the nation-state


Purchase, 1994
Graam, Professor of Metaphysics and Epistemology at University of Sydney, Anarchism and Environmental Survival, pg. 101

If we are to reintegrate humanity with nature, it is critical that we first dismantle the nation
state which ignores ecological regions, and instead allows communities to reconstruct social political
and economic life based upon local environmental exigencies. This would eliminate both corporate
interference and bureaucratic dictates from distant political bodies.
Social anarchists argue that, in the absence of the nation state, the free and ecoregionally
integrated commune (or, in more modern terms, community) will again become the primary unit of
social identification. The idea of the commune stems from revolutions in medieval Europe and 19th-
century France, when the people in many cities attempted to regain political independence, first from the
aristocracy and later from the bureaucratic and bourgeois classes. While French communes ranged from
small hamlets to the mighty Paris Commune, they were all based on the belief that individual’s political
identification should be with the autonomous village, town, city, or ecological region (or some combination
thereof).
AFF A2: ECONOMY LINK

Turn: Growth is key to the environment


Lomberg, 2001
Bjorn, Director of Denmark’s National Environmental Assessment Institute and Associate
Professor of Statistics in the Department of Political Science at the University of Aarhus, The
Skeptical Environmentalist, pg. 32-33

In general we need to confront our myth of the economy undermining the environment. We
have grown to believe that we are faced with an inescapable choice between higher economic welfare and a
greener environment. But surprisingly and as will be documented throughout this book, environmental
development often stems from economic development – only when we get sufficiently rich
can we afford the relative luxury of caring about the environment. On its most general level, this
conclusion is evident in Figure 9, where higher income in general is correlated with higher
environmental sustainability.
AFF ANSWERS

Wrong: Concern for persons and social justice outweighs concern for the non-human
Pepper, 1993
David, Principal Lecturer in Geography at Oxford Brooks University, Eco-socialism: From Deep
Ecology to Social Justice, pg. xi-xii

For I think that this is what the green movement no needs, rather than its current ‘biocentric’ and
politically diffuse approach, in order to appeal to the concerns of the many who are still alienated by or
indifferent to it. Furthermore, and pragmatism aside, I think it’s important not to allow our concern
for non-human nature to become a substitute for, or a priority over, concern about people.
Some greens believe that we should protect and respect nature for its ‘intrinsic worth,
whatever that is, rather than its worth for all people. I am not comfortable about this. Social
justice, I think, or the increasingly global lack of it, is the most pressing of all environmental
problems. And the Summit showed clearly that attaining more social justice is the prerequisite
for combating ozone depletion, global warming, and the rest.
AFF ANSWERS

The alt leads to Nazism and genocide


Bobertz, 1997
Bradley, Assistant Professor of Law at the University of Nebraska College of Law, Columbia Journal of Environmental Law, “Of Nature
and Nazis”, Lexus-Nexus

  ... According to French philosopher and essayist Luc Ferry, this statute expresses ideas we associate only with recent
trends in ecological thinking. ... But how can we be so sure? When we see chimpanzees using twigs to fish termites from
their mounds, must we dismiss the idea that they teach and transmit this skill between generations to form what Ferry
calls "culture" and "history"? More than a generation has passed since a Japanese macaque was observed initiating a new
potato-washing behavior "which other members of her group learned by imitation, and subsequently taught to succeeding
generations ...." This seems to fit within Ferry's criteria for human culture and history. ... To illustrate the perils of deep
ecology, Ferry turns to animal protection and environmental laws passed during the Nazi regime. ... Must one be a deep
ecologist to conclude that an environmental philosophy will accomplish little if it depends on the recognition of "human
signs in nature?" In the end, Ferry tries to sew the seeds of a new environmentalism, but all he ends up with is barren
earth. ...   I. Introduction: The Left Bank Tilts Right In June 1935 the Reich government of Germany passed a law
calling for the protection of plants and animals, "nature monuments," and national parks. n1 According to French
philosopher and essayist Luc Ferry, n2 this statute expresses ideas we associate only with recent 8540*354 trends in
ecological thinking. He presents the Nazi law as a cautionary tale for environmentalists and animal rights supporters, and
more generally for those who proclaim the virtues of diversity or call for a reawakening of communitarian values. Among
these groups, Ferry detects a new breed of ideas that form a dangerous political orthodoxy. Today's reigning philosophy
of nature - that humans are not fundamentally separate from the biosphere but are merely a part of it - portends darker
times ahead, Ferry warns, claiming that a willingness to condemn industrial technology while at the same time glorifying
the purity of nature mirrors 1930s-style utopian romanticism, a sentimental yearning for an idealized past that in Ferry's
view leads to neo-fascist political models. n3
It might be tempting to write a polemic equating the holistic norms of environmental philosophy with the mystical,
totalizing elements of German fascism. When then-Representative James Inhofe likened the United States Environmental
Protection Agency to the Nazi Gestapo, n4 he was speaking the language of the antienvironmental right, a growing
segment of the electorate that blames fuzzy-thinking nature lovers and their friends in Washington for undermining
individual liberty and threatening American growth. n5 Presumably, a book claiming that what environmentalists profess
is just a greener shade of fascism would find a comfortable niche in today's marketplace of ideas. But The New
Ecological Order is not that kind of book. Instead of launching populist broadsides against government overreaching,
Ferry offers a thoughtful critique of the foundations of Western environmentalism. He also attempts to create a new
philosophy of nature that protects environmental values without rejecting human ones, thus "integrating ecology within a
democratic framework." n6 8540*355 This is a worthy endeavor, and The New Ecological Order has been praised on
both sides of the Atlantic. n7 The book carries forward the tradition of the New Philosophers, a group of conservative
educators and essayists who split from prevailing trends in Continental thought during the late 1970s and 1980s. n8
Ironically, just as French socialists gained power in government during the 1980s n9 they lost influence in intellectual
circles, their traditional strongholds. The Nouveaux Philosophes, as they came to be called, disavowed the pantheon of
writers associated with the Hegelian/Marxist and structuralist schools. Targets of New Philosophers included Jean-Paul
Sartre (existential philosophy), Claude Levi-Strauss (structural anthropology), Jacques Derrida (structural linguistics),
Roland Barthes (literary criticism), Jacques Lacan (psychoanalysis), and Michel Foucault (social history/criticism) n10 -
in short, the very thinkers that American scholars were discovering and championing in the 1970s and 1980s. Thus, the
same writers whom many Americans were upholding as 8540*356 the quintessence of modern French thought were
simultaneously losing luster in their home country. n11 As he explains more fully in previous works, Ferry criticizes the
postwar generation of philosophers for promoting the idea that humans are governed by forces outside their control (such
as the unconscious workings of the psyche or impersonal structures of power). n12 By rejecting the concept of human
sovereignty, Ferry believes, these philosophers also repudiated modernity itself and rendered democracy problematic.
n13 They profess ideas Ferry calls "antihuman" because they fail to recognize the unique power of human reason to
control individual behavior and collective destiny. The task Ferry and his colleagues set for themselves involves saving
the essential "humanism" of Enlightenment philosophy, a humanism that forms a bulwark against authoritarian tyranny,
recognizes inviolable human rights, and allows for the practice of democratic politics. n14
*** Heidegger K***
AFF ANSWERS: Perm
Perm – combining the ethics of environmentalism with the law is crucial to an embrasure
of democratization and rejection of strict utilitarianism.
Alyson Flournoy 03, Professor, University of Florida, Levin College of Law, Fall 20, Building Environmental Ethics from the Ground
Up”

Thus, ittakes a concerted effort to identify what values we are pursuing under our laws. It
requires that we wade through the analysis required under the relevant statutes and regulations
for a start. Then, we need to look at how the regulations are applied and interpreted by agencies and courts, to
determine whether some values are systematically favored, while others are protected in name only. Lawyers and
legal scholars already typically engage in this type of analysis, but only up to a point. We study how the laws,
regulations, and policies are interpreted and applied, and analyze whether agency and court
decisions are based on sound reasoning. What I am suggesting is that this analysis needs an ethical
dimension -- a translation of what happens under the law into the language of values. The challenge such a task
presents is that it demands work across the boundary that divides law and philosophy. Both philosophers willing to
delve into environmental law and legal scholars interested in environmental ethics will need to forge the path for this
work. The missing analysis would seek to determine what values and ethics are embedded in
Section 404. I have suggested that we need a detailed and systematic analysis. n16 Some may challenge the notion
that detailed work is really necessary. For example, those familiar with Section 404 or with environmental
philosophy might be willing to forgo a close analysis and offer as adequate the following general characterization:
that Section 404 reflects predominantly a human-centered and utilitarian ethic -- that, in general,
Section 404 employs a balancing designed to maximize human good. For purposes of considering
whether detailed analysis is really necessary, let us assume that this characterization is generally correct, in this
sense: that the values Section 404 advances are more consistent with a human-centered
utilitarianism than with any  [*61]  other coherent ethic (environmental or not) that we can identify.
Even if this is an accurate generalization, there is a fundamental problem with relying on this general
characterization as a statement of the ethic of Section 404. By virtue of its effort to capture Section 404 in the
abstract vocabulary of pure philosophy, this characterization is misleading. If we look more closely at Section 404,
it is quickly apparent, that as applied, the section incorporates a very incomplete calculation of
"the good." Rather than reflecting a pure and perfect utilitarianism, it reflects what I call a
bounded and imperfect utilitarianism. By these qualifiers, I mean that the analysis is demonstrably
inadequate on numerous scores. If one imagines a utilitarian calcPerulus that incorporates the insights of ecology as
perfectly as is humanly possible, that is not the utilitarian calculus we are currently performing. n17 Our assessment
of the benefits provided by wetlands is severely constrained by data gaps as well as by the limits of our
understanding of complex natural systems. This is no surprise to most who study environmental law. Critiques that
highlight the flaws of available analytic techniques are core contributions of legal scholarship. n18 But I submit
that we lack and need analysis that does more than identify the flaws in regulatory and judicial
decisions. We need to refocus our attention away from the consistency and completeness of
regulatory and legal analysis, and onto the values advanced by the flawed analytic techniques as
they exist. We need a language to express this, to describe the mix of values that actually emerges from the flawed utilitarian calculus that
our laws so often embrace. This demands a new vocabulary, one that belongs neither to philosophy nor to law. n19 Philosophers can make a
significant  [*62]  contribution by helping to develop this vocabulary, and lawyers and legal scholars can contribute by using it. n20 The work to
uncover and articulate the values embedded in our laws represents a significant challenge that will require thinking that transcends disciplinary
boundaries -- work and discussion fostered by symposia like this one. C. What Can We Gain from Unearthing the Ethics in
↓Card Continues↓
↑Card Continued↑
Environmental Law? Having described the type of work I advocate, let me turn to the question of its worth. Why do we need a better
understanding of the values embedded in our laws? Why do we need to be able to accurately describe the mix of values that Section 404 tends to
protect? I contend that the public and decisionmakers need a better sense of our current bearings in order to
validate or invalidate popular assumptions. Consider the portrait of Section 404 offered above: let us assume for a moment that
the most accurate description of the ethic embedded in it is a bounded and imperfect human-centered utilitarianism -- in other words, a
utilitarianism that does not fully account even for the total value to humans of wetlands. Contrast this with the public narrative we tell about
Section 404. Section 404 is widely viewed as one of the brightest stars of the environmental law constellation. And rightly so. Wetlands have
what we might call "most-favored-ecosystem" status under our legal regime, while many uplands ecosystems are left relatively unprotected. But
the fact that Section 404 is one of our stronger environmental laws does not mean that it reflects any
uniquely environmental values or a unique way of valuing the environment . Yet, the public narrative may
reinforce the assumption that environmental law is a pure reflection of some indistinct but noble set of
environmental values. This may contrast with the reality that our laws, like Section 404 ,  [*63]  often protect
a wide array of very traditional human values, and uniquely environmental values or ethics are at best
only partially reflected. What if this is true broadly? What if the values protected by our environmental laws are human values that differ
only slightly from the values protected by the common law of nuisance, cloaked in the appealing veil of environmentalism? It is possible that
n21
instead of an identity between the values in our laws and those held by the public, there is a significant disjuncture. At present, I contend that
it is difficult for an expert, let alone a lay person, to know if this is the case. Such
ignorance can interfere with democratic
participation. The easy equation of environmental laws and environmentalism, which is reinforced by our inability to
describe accurately the values of our laws, discourages serious public discourse about why we care about the
environment. If we do not ever consider or discuss as a society what we value and why, we are like the rowers I described at the start: backs
into the wind, rowing in unison with no idea where we are headed, but convinced it is where we all mean to go. A clearer picture of the
embedded values can correct any erroneous assumptions and validate accurate ones . Moreover, the discourse
involved in providing a more accurate account of the values protected by our laws may promote wider
attention to the ethical questions, challenging people to consider why they support environmental
protection. Thus, I propose that legal scholars and philosophers work to enhance public understanding of the values embedded in our laws.
When members of the public are confronted with a clearer picture of what values are advanced by our
current laws and policies, they can determine whether or not these laws comport with their ethical
intuitions. If they find that the laws are consistent with their ethical intuitions, we will have a stronger public
commitment to support existing laws. If not, the public can support efforts to reform the law in any
direction -- to enhance protection of non-environmental human values like autonomy or to enhance protection of the social or intrinsic  [*64] 
value of the environment, as they see fit. Beyond the democratic benefits of better public understanding of the values advanced under our laws,
this work is important to the long-term efficacy of environmental law and policy. Engagement with
environmental issues by the public and changes in individual and civic behavior will only result if we care
about something at stake. If, as I suggest, the American public lacks clarity about what it values about the
environment, then the public is less likely to be engaged and responsive on issues of environmental
policy. As philosopher James Rachels explains, in describing where ethical argument leads us: As Hume observed, when we come to the last
reason, we mention something we care about. Nothing can count as an ultimate reason for or against a course of conduct unless we care about
that thing in some way. In the absence of any emotional involvement, there are no reasons for action. The fact that the building is on fire is a
reason for me to leave only if I care about not being burned; the fact that children are starving is a reason for me to do something only if I care
n22
about their plight.  In short, unless we have a sufficient grasp both of our own values and of how a law or
decision or action affects something we care about, we will not respond . The process of gaining clarity, of discussing
n23
the values at stake, may itself promote more reasoned thinking. Deliberation may promote ethical development. n24 One
does not need to know how  [*65]  deliberation might affect Americans' views on environmental values, to believe that greater thought and
attention to these issues is a desirable end in itself. Because there is ultimately no reason for any act ion, nor for law that
regulates action, unless we care, environmental philosophy matters for environmental law. The steady support for
increased regulation to limit our impact on the environment over the past thirty years suggests that American society does care about some values
associated with the environment. If we are to change our current pattern of conduct with regard to the environment
-- including our action through government -- we need both information and motivation to deliberate. Several characteristics
of environmental problems -- their technical complexity, the scientific uncertainty and extremely long time horizons attending them, and the wide
array of values they engage -- discourage clear thinking about the relevant values. The possibility that a majority may wish to change our current
patterns makes the effort to clarify our thinking about why we care worthwhile. 
Perm EXT

Perm solves best -the alternative’s radical ethics are inapplicable to politics because they
are incompatible with an anthropocentric view of ethics.
Alyson Flournoy 03, Professor, University of Florida, Levin College of Law, Fall 20, Building Environmental Ethics from the Ground
Up”
Initially, the term environmentalism may have served as an adequate focus for our discourse. It captured and
expressed the public desire to embrace a new ethic, new values, and prompted deep thought about our
relationship with the environment. But the meaning of the term has been so diluted over time that
commentators have noted that it is now on a par with apple pie and motherhood , n36 something most people
embrace and only a few view unsympathetically. n37 Today, environmentalism seems to suggest a posture supportive of environmental
laws as they exist or with moderate reforms. It may be that environmentalism today  [*69]  lacks a core meaning distinct from the
dominant human-centered utilitarian ethic. n38 Use of the word "environmentalism" does not lead to
thoughtful engagement with the ethical and practical problems that arise under the current dominant ethic .
It is a question mark too often used as a period. One might argue that to cure this void, coherent alternative theories are needed and that the theory-building work
being done by philosophers is the most urgent need. However, it
seems possible that the leap required of people if they are to
understand and embrace a coherent environmental ethic is too difficult for most, given current attitudes
and the limited public discourse about underlying values. Coherent environmental ethics are
compartmentalized as "radical" and rejected, leaving a vast undefined realm of "mainstream"
environmentalism. Most people believe themselves concerned about the environment, even though that commitment may be one without well-defined
content. To challenge the public's comfortable self image as "environmentally friendly," we may need
concepts that are not so radically removed from utilitarianism but which frame the ethical and practical
shortcomings of our current ethics as applied to environmental problems . n39 In other words, concepts that show
the possibility and value  [*70]  of more ecologically enlightened ethics, but which do not require wholesale
acceptance of a radically altered worldview, may have value. n40 A. Towards a New Ethical Discourse: Stepping Stones This paper
emphasizes the value of an environmental ethics discourse that can reach a wide segment of the public. Concepts that can frame the ethical issues in a more accessible
form may help those who are not completely satisfied with the dominant bounded and imperfect, anthropocentric utilitarian ethics embedded in our policies and laws.
Therefore, I advocate developing concepts that can serve as points of departure from where the majority is today -- concepts that frame the ethical issues in an
n41
 Such
concepts may fill a gap
accessible form and offer a new direction for those whose ethical impulses diverge from current dominant norms.
that exists between legal scholars' work that is directed at improving decisionmakers' analytic techniques
and philosophers' work to develop coherent ethical theories . Concepts and vocabulary that draw on both
philosophy and law may be useful tools that will help members of the public to understand the full
implications of current laws in ethical terms , and to identify or envision practices and policies consistent with their evolving individual
ethical intuitions. Developing these concepts will require that we broaden the definition of appropriate work for lawyers and philosophers. Philosophers' contribution
cannot be limited to developing and justifying a coherent alternative completely apart from human-centered utilitarianism. And lawyers' contribution cannot be limited
to critiquing current legal standards or decisionmaking techniques. Philosophers must help us to create a discourse that describes
ways of valuing the environment that builds on people's current values, and lawyers must analyze the
extent to which existing and proposed laws are compatible with these value s. Ultimately, such concepts may prove more
radical in practice than ecocentric ethical theories, in that they may enable ethical transformation that would otherwise not occur. Metaphorically, we can think
of such concepts as stepping stones -- ideas that help people to find their way past some of the constraints
of  [*71]  traditional ethics. Such concepts should focus public attention on the constraints imposed by traditional utilitarian ethics and bring into view the
possibility of an ethic that addresses these constraints. These constraints include inadequate capacity to deal with long time horizons, uncertainty, integrated
decisionmaking, social equity, and values that are not easily monetized. Stepping stones, unlike a true environmental ethic, may not
provide coherent and complete responses to these constraints, but by making the issues salient for the
public, they may represent a necessary step in any widespread ethical evolution. Where an environmental ethic might be described as
requiring a leap from current dominant ethics, stepping stones require only a small step . They invite contemplation
of change by highlighting the constraints of current ethics, but they do not demand a complete ethical transformation. To be effective, a stepping stone must have
broad resonance with the public and provide a context for confronting some of the challenges that any environmental ethic will have to overcome, including long time
n42
horizons, scientific uncertainty and the limitations of the dominant economic framework.  
Aff – Management Good

The management of the environment is a necessary task

Mintz 4 (Joel, Professor of law, Boston college environmental affairs law review,
http://findarticles.com/p/articles/mi_qa3816/is_200401/ai_n9403748/pg_1?tag=artBody;col1

For Kelly Parker, the principal insight of environmental pragmatism is that "the human
sphere is embedded at
every point in the broader natural sphere, that each inevitably affects the other in ways that are
often impossible to predict, arid that values emerge in the ongoing transactions between humans
and environments."40 Parker defines environment as "the field where experience occurs, where my life and the
lives of others arise and take place."41 He believes that pragmatism commits us to treating all places
where experience unfolds, i.e., all environments, with "equal seriousness."42 Moreover, under
Parker's pragmatic approach, people are encouraged to "restructure our social institutions" so that the
public is afforded "a real voice in determining the kinds of environments we inhabit."43Like
Parker, Sandra B. Rosenthal and Rogene A. Buckholz also emphasize the organic unity of the individual embedded
in his or her environment.44 To them, human beings are biological creatures, part of, and continuous
with, nature.45 In light of this, the philosophical argument over anthropoceiitrism is meaningless since no real
line may be drawn between human and environmental well-being.46 Rosenthal and Buckholz see the "systematic
focus" of pragmatism as being on "science as method, or as lived through human activity, on what the scientist does
to gain knowledge."47 Humans exist in the world as active experimenters who create knowledge and
formulate ethical values by integrating "potentially conflicting values and viewpoints."48

We cannot only look at environmental concerns we must also look at other factors
Mintz 4 (Joel, Professor of law, Boston college environmental affairs law review,
http://findarticles.com/p/articles/mi_qa3816/is_200401/ai_n9403748/pg_1?tag=artBody;col1

In Farber's view, we need


to draw on both democratic public values and private economic interests
in formulating environmental policies.110 In his words:
Without appealing to public values environmental regulations could not long enjoy general support based purely on
the calculus of competing private interests. But without recognizing private interests as legitimate, environmental
regulations may provoke unmanageable resistance from those paying the price and are likely to be seen by society as
a whole as too draconian to be acceptable.111
Farber argues for the inclusion of an environmental "baseline" in policymaking, i.e., a rebuttable
presumption in favor of environmental protection.112 He advocates a "feasibility approach" to
regulation, noting that:
Although feasible in some sense of the word, achievement of an environmental goal may sometimes
involve costs that are grossly disproportionate to any plausible benefit. Thus, cost-benefit
analysis may serve as a useful backstop for feasibility analysis to handle these situations. We
should always begin, however, with a presumption in favor of protecting the environment except
when infeasible or grossly disproportionate to benefit.113
Aff – Technology Good
Rejecting all technology makes life meaningless, culminating in extinction
Hicks 3 (Steven V., Professor and chair of philosophy at Queens College of the CUNY, “Nietzsche, Heidegger, and Foucault: Nihilism and Beyond,” Foucault
and Heidegger: Critical Encounters, Ed. Alan Milchman and Alan Rosenberg, p. 109, Questia)<

Heidegger's insightful reading of


Why a “philosophical shock”? The answer, in part, may be that from Foucault's perspective ,
Nietzsche and the problem of nihilism is itself too ascetic. Heidegger's emphasis on “silence” as
proper to Dasein's being, his frequent use of quasireligious (even Schopenhauerean) terms of “grace” and “call of
conscience, ” his many references to the destiny of the German Volk, his avoidance of politics
and the serious “quietistic” tone of Heideggerian Gelassenheit are all reminiscent of the life-
denying ascetic ideal Nietzsche sought to avoid. 65 Moreover, Foucault seems to join with Derrida
and other “neo-Nietzscheans” in regarding Heidegger's idea of “letting Being be”—his vision of
those who have left traditional metaphysics behind and with it the obsession with mastery and
technology that drives contemporary civilization—as too passive or apathetic a response to the
legitimate problems of post-Nietzschean nihilism that Heidegger's own analysis uncovers. 66 Here
we have arrived at a key difference between Heidegger and Foucault: for Foucault, Heidegger takes insufficient account of the playful and even irreverent elements in
Nietzsche and of Nietzsche's critique of the dangers of the ascetic ideal. Foucault joins with other new Nietzscheans in promoting, as an alternative to Heideggerian
Gelassenheit, the more Nietzschean vision of “playing with the text”—which in Foucault's case means promulgating active and willful images of resistance and
struggle against particular practices of domination, rebellion against “micro-powers, ” and blatant disregard for tradition (cf. DP, 27). 67 This context-specific,
unambiguously confrontational nature of Foucault's critique of the forms of domination and technologies of power lodged in modern institutions offers a more
the lessons Heidegger
Nietzsche-like response than the one Heidegger offers to the nihilistic problems of Western civilization. As Foucault sees it,
would have us draw from Nietzsche throw us back to the passive “nihilism of emptiness” that
Nietzsche feared. While not predicting the emergence of better times, Foucault tries to offer a better (less passive, less ascetic) model for reforming our
“background practices” and for cultivating an affirmative attitude toward life that he and other neo-
Nietzscheans think may be “our only chance to keep from extinguishing life on earth altogether.”
Aff – Technology Good

The kritik ignores the benefits of modern progress, new technology allows us to fully
understand ecological problems and solve them
Zimmerman 3 ("On Reconciling Progressivism and Environmentalism" http://www.integralworld.net/zimmerman7.html)
In the process of dissociating himself from nature, modern man also gradually dissociated himself from the divine. The emergence of modern
science's mechanical worldview, along with the idea that human freedom involves rational self-law giving without interference from any outside
force, be it human or divine, led progressives to conclude that God is either a non-intervening creator or else altogether absent. Alone
as the
only form of consciousness in an inert cosmos; stricken with the anxiety that accompanies
modern egoic subjectivity; dissociated from his feelings, body, nature, and divine; and alienated
from other people, audacious European man colonizes the planet, subjecting native peoples to his
will and transforming entire mountain ranges into industrial raw materials. Without denying
modernity's drawbacks, Wilber argues that it has brought genuine progress. In many places,
democratic institutions have replaced despotic regimes; the right to own property has become
recognized as an important dimension of owning oneself, i.e., of being a person; scientific
knowledge enables us to understand both the fact of and the causes for today's ecological
problems; extraordinary productive forces have ended scarcity for vast numbers of people, and
could in principle end scarcity for all. The task for an authentic "postmodern" humanity is to
further the evolution of consciousness, the next stage of which involves reintegrating what has
been dissociated: body and nature "below" and divine "above." Such integration does not mean giving up the
gains associated with individuated selfhood, anxiety-ridden though it may be, by regressing to earlier stages of consciousness and
collective-authoritarian social formations. Instead, integration involves an Aufhebung, in which the individuated selfhood of
rational-egoic subjectivity is both included and transcended in a more comprehensive form of awareness that is open both to
nature and divine. Wilber argues that the achievement of rational bourgeois selfhood living in constitutional-governed societies is
only one stage of human history, not its culminating moment. Humankind is only about halfway along in a lengthy and often
painful evolutionary process. Although emphasizing the importance of reintegrating the divine into human life, Wilber
nevertheless insists that the greatest possible contemporary revolution would be the worldwide achievement of responsible
personhood, constitutional democracies, and market economies, since grave social and ecological problems arise in countries
where premodern, autocratic ways impede the material productivity and social institutional change necessary for the rise of
modern individuals. In other words, only when the positive political and economic achievements of modernity are consolidated
on a planetary basis can large numbers of people begin exploring the next, integrative phase of human history. There is a
dialectical relation between material circumstances and consciousness development. Scarcity prevents people from developing
their human potential, yet regressive social structures impede the development of the productive forces needed to overcome
scarcity.
Aff – Technology Good/AT: Tech Links
The plan is not the dangerous technological understanding of being that Heidegger
criticizes, he has no problem with using technology
Dreyfus 96 (Hubert, “Being and Power: Heidegger and Foucault” University of California Berkeley http://socrates.berkeley.edu/~hdreyfus/html/paper_being.html)

Heidegger and Foucault are clear, then, that what is uniquely oppressive in our current practices
is not that they are illegitimate nor that they cause ecological devastation. According to Foucault
legitimacy is a red herring; our current society becomes more oppressive as it becomes more protective of rights and
more permissive, and productive. Heidegger, on his part, distinguishes the current problems of
technology -- ecological destruction, urbanization, nuclear danger, etc. -- from the devastation that would
result if technology solved all our problems. What threatens man in his very nature is the ...
view that man, by the peaceful release, transformation, storage, and channeling of the energies of
physical nature, could render the human condition, man's being, tolerable for everybody and
happy in all respects. Their common critique of techno/bio-power does not, however, lead
Heidegger or Foucault to oppose the use of technological devices, nor specific welfare practices.
Heidegger is clear that it is the essence of technology -- the technological understanding of being
-- not technology, that causes our distress. That the technological understanding of being can be
disassociated from technological devices is clear if one looks at contemporary Japan where a traditional, non-
technological understanding of being -- or, perhaps better, no single understanding of being at all, but a pluralistic
understanding of multiple realities -- exists alongside the most advanced high-tech production and consumption.
Aff – Resource Mindset Good/AT: Resources Links
Technology can be used without assigning meaning
Dreyfus 96 (Hubert, “Being and Power: Heidegger and Foucault” University of California Berkeley http://socrates.berkeley.edu/~hdreyfus/html/paper_being.html)

In the end, however, he seemsclearly to hold that technology can treat people and things as
resources to be enhanced without setting meaning-giving subjects over against objectified things.
A year after the previous remark about subjects and objects reaching extreme dominance
Heidegger appears to retract his view about objects at least, in his observation that nature has
become "a system of information" and a modern airliner is not an object at all, but just a flexible
and efficient cog in the transportation system. Passengers are presumably not autonomous
subjects either, but resources recruited by the tourist industry to fill the planes. Heidegger
concludes: "Whatever stands by in the sense of standing-reserve no longer stands over against us
as object."

Modern technology doesn’t objectify and manipulate nature


Dreyfus 96 (Hubert, “Being and Power: Heidegger and Foucault” University of California Berkeley http://socrates.berkeley.edu/~hdreyfus/html/paper_being.html)

In his final analysis of technology, Heidegger is critical of those who, still caught in the
subject/object picture, think that technology is dangerous because it embodies instrumental
reason. Modern technology, he insists, is "something completely different and therefore new."
Heidegger describes the hydroelectric power station on the Rhine as his paradigm technological
device because for him electricity is the paradigm technological stuff. He says: The revealing that rules
throughout modern technology has the character of a setting-upon, in the sense of a challenging-forth. That challenging happens
in that the energy concealed in nature is unlocked, what is unlocked is transformed, what is transformed is stored up, what is
stored up is, in turn, distributed, and what is distributed is switched about ever anew. But we can see now that electricity is not a
perfect example of technological stuff because it ends up finally turned into light, heat, or motion to satisfy some subject’s desire.
Heidegger’s intuition is that treating everything as standing reserve or, as we might better say,
resources, makes possible endless disaggregation, redistribution, and reaggregation for its own
sake. As soon as he sees that information is truly endlessly transformable Heidegger switches to computer manipulation of
information as his paradigm. The goal of technology, Heidegger then tells us, is more and more
flexibility and efficiency simply for its own sake. There is no longer, as there was in Kant, an onto-
theological center that provides a goal for all activity. There is ordering but no orderer. Heidegger
says: Everywhere everything is ordered to stand by, to be immediately at hand, indeed to stand there just so that it may be on
call for a further ordering. Whatever is ordered about in this way has its own standing. We call it the standing-reserve [Bestand,
i.e. resources].
Aff – Permutation Solves – Alternative Energy Key
Perm do both solves- In already noting the destructive nature of our lifestyle and
promoting alternative energies we thus begin to question subject-object relations, solving
the K.
Sabatiano 7 (Charles J., Daemen College. “A Heideggerian Reflection on the Prospects of Technology” reprinted
in Janus Head 10.1. p. 63-76. PDF accessed July 3, 2008)

There is evidence of such questioning. For example, even as our medical technologies become
increasingingly capable of controling life and death processes, we hear questions raised concerning the kind of care
that is generally available, or perhaps not available, especially in the end stages of life where we apparently view
death mostly as failure and defeat. Learning to accept the living—dying process as an essential aspect
of our lives is certainly humbling; but it offers a good example of acknowledging limits to how
much we can control and manuplate, how much we can use technology to distance ourselves
from who we are as human (humus: of the earth, thus mortal). Along the same lines, with regard to energy
policies and uses, we can find hope from those who note the destructive nature of our lifestyles;
those who recognize the impact of global warming, depleted resources, waste, etc; those who
warn us of the unsustainability of it all. Even as the problems brought with technology are
becoming global in nature, we are being asked more than ever to attend to what we are doing to
the environment, and therefore to one another. We hear calls for greater reli- ance on renewable
resources that find us accepting the natural limits within which we live rather than become
dependent on other technologies such as nuclear, that demand control for thousands of years,
increasing risk factors at many levels. Further, with respect to genomic and stem cell research, there is the
differentiation being made between techniques that are therapeutic, and thus consistent with the
healing energies that sustain life versus those that are strictly reproductive and serve mostly
what we see as a right and prerogative to do what we will.
Aff – Permutation Solves – General
The plan’s pragmatism is absolutely essential to the success of the alternative

Margolis 6 (Joseph, “Pragmatism’s Advantage” 2006, Temple University http://www.arsdisputandi.org/index.html?


http://www.arsdisputandi.org/publish/articles/000126/index.html)

It was, of course, originally a parochial success, though it did gain adherents abroad; and it began to attract a wider Eurocentric
interest in its short second life, despite a distinctly poor showing at home. We may even speculate about a third career. For
pragmatism has begun, possibly for the first time in its history, to be seriously treated as a distinct
alternative to – possibly more than an alternative, perhaps a connective tissue spanning the great
divide between – analytic and continental philosophy. At any rate, it now counts as a distinctly
strong constellation of doctrines and strategies potentially capable of contesting the hegemonies
of the day—within both the English-language analytic movement of the last half of the twentieth
century and the trailing forces of the Cartesian, Kantian, Husserlian, and Heideggerean movements of late
continental Europe. It would not be unreasonable to say that pragmatism’s promise at the present time is a function of the
fatigue of its principal competitors and of the economy and fluency with which it can coopt the principal strengths that remain
attractive among the many movements of Eurocentric philosophy, without betraying its own conviction. Rightly perceived,
pragmatism’s best feature lies with its post-Kantian ancestry coupled with its opposition to the extreme forms of analytic
scientism with which it has shared a gathering sense of conceptual rigor. It forms, for that reason, a natural bridge
between analytic and continental philosophy, for rigor is not inherently scientistic. In my opinion,
none of the three movements mentioned (hardly unified within themselves) is separately likely to overtake its own limitations or
incorporate the best work of the others in a compelling way. Still, within its own conceptual space , pragmatism favors a
constructive realism drawn in as spare a way as possible from post-Kantian resources, freed from
every form of cognitive, rational, and practical privilege, opposed to imagined necessities de re
and de cogitatione, committed to the continuities of animal nature and human culture, confined
to the existential and historical contingencies of the human condition, and open in principle to
plural, partial, perspectived, provisional, even non-converging ways of understanding what may
be judged valid in any and every sort of factual and normative regard. There may well be a touch of
reportorial distortion in going beyond these clichés; but, risking that, it would not be unreasonable to say that pragmatists believe
that the analysts are likely to favor scientism and the continentals, to exceed the bounds of naturalism, and that both tendencies
are more extreme or extravagant than their policies require. In this fairly direct sense, pragmatism’s strength lies in the possibility
of a rapprochement by way of the ‘corrections’ mentioned. It could never have claimed such an advantage earlier, had not the
main efforts of analytic and continental philosophy perseverated too long in their obviously vulnerable commitments.
Pragmatism has persevered as well, of course, but it seems poised now for a larger venture.
Aff – Permutation Solves – General
The perm is the only option that can fully solve for the environment because it allows for a
combativeness between ideas
Mintz 4 (Joel, Professor of law, Boston college environmental affairs law review,
http://findarticles.com/p/articles/mi_qa3816/is_200401/ai_n9403748/pg_1?tag=artBody;col1)

One particularly provocative aspect of environmental pragmatic thought is its desire for
compatibilism, i.e., a philosophical framework within which competing environmental theories
may be compatible in practice.53 Andrew Light is an advocate for this view.54 Light contrasts the views of
social ecologists and materialists, such as Murray Bookchin and Herbert Marcuse,55 who view environmental
degradation as presupposed by a capitalist economy, and ontologists, including “deep ecologists” like Arne Naess,56
whose focus is on reform of the self, and one’s relationship with the non-human world, as expressed in individual
identity.57 To harmonize these mutually antagonistic schools of environmental thought, Light
proposes a pragmatic “principle of tolerance.”58 Under it, theorists and practitioners are required to
communicate a “straightforward public position” that endorses the considerations on which they agree, and the
practices best suited to meeting their mutually desired goals, while leaving some questions that divide them to
private dispute.59
As Light sees it, “environmental pragmatists are not wedded to any particular theoretical framework
from which to evaluate specific problems, but [they] can choose the avenue which best protects
the long-term health and stability of the environment, regardless of its theoretical origin.”60 For Light and
other environmental pragmatists, the “truth” of various environmental theories is thus not always
important in environmental practice.61 Instead, “the appropriateness of any one theory in a
particular case is contingent on historical, cultural, social and resource conditions.”62

The Negatives stubborn resistance to any other form of alternative destroys any chance for
effective change. The perm is the way to make change and find compromise to determine
the best form of action
Mintz 4 (Joel, Professor of law, Boston college environmental affairs law review,
http://findarticles.com/p/articles/mi_qa3816/is_200401/ai_n9403748/pg_1?tag=artBody;col1)

For Hirokawa, each of these alternative paradigms is too sweeping and impractical to provide a sound normative
basis for environmental laws. he states: "[u]nfortunately, in attacking the accepted tenets of ownership,
proponents of radical environmental critiques may argue themselves off the negotiating table and
render their insights ineffective."131
In lieu of these defective radical paradigms, Hirokawa proposes a pragmatic approach based upon "a little
flexibility toward what might be termed 'truth,'" and "persuasion, not stubborn dogmatism."132
Arguing that "revolutionary ideals can be presented in light of dominant beliefs, rather than in
spite of them,"133 Hirokawa states that "pragmatism offers a means by which paradigm opponents
can find common ground and potentially agree on environmental policies and laws."134 In
conclusion, he claims:
The challenge is to continue the progress and find better environmental solutions that both effect a
change in the way we treat the environment and are practical enough to be adopted by our legal system. In taking up
this challenge, it is imperative that loyalties to the goals of environmental protection include a
willingness to modify, or even discard, radical environmental theories in an effort to secure far-
reaching results.135
Aff – Permutation Solves – Technology
Totalizing discourses on the dangers of technology ignore its utility, and only by working
with technology can we prevent powerlessness
Feenburg 3 (Andrew, “Pragmatism and Critical Theory of Technology,” 2003. Techne)
Hickman’s comments make me reflect on my own path to philosophy of technology. I arrived at my approach starting
out from Martin Heidegger and the Frankfurt School, that is, from doctrines keenly sensitive to the failure
and indeed the threat of modern technology, the famous “dialectic of Enlightenment.” The dystopian sense
of oppression by technology, or more precisely, by technically mediated institutions, has never
entirely left me. On the other hand, I have learned through the experience of working with actual
technology that the logic of dystopia is too totalizing and does not correspond to the rather
chaotic realities of technical life. Human beings in a technological society are far from powerless
and the future is wildly unpredictable. Constructivist sociology of technology has recognized these realities and so helps me to articulate
my own sense of the workings of technical life and culture. How to mediate these two different views, a dystopian critique and a constructivist analytic? This is the
background agenda of my work.

The healing power of the alternative must spawn from technology, the plan and alt
function together to restore the world
Beckman 0( Tad, Emeritus Professor of Philosophy, Humanities and Social Sciences at Harvey Mudd College,
“Martin Heidegger and Environmental Ethics,” http://thuban.ac.hmc.edu/~tbeckman/personal/Heidart.html)
While we may now glimpse something of the human essence and the legitimate role of human life and art in all revealing, we are still left with the problem of how we
the
can make contact with art as a saving power from within our epoch of enframing. In "The Question Concerning Technology" Heidegger suggested that
disaster that technology is becoming compels us to look more closely at the genus and essentia of
the whole process in which we have become involved. In fact, the very character of the disaster
brings out the specific nature of that genus; for the disaster is a human disaster, having the scope
of affecting the whole human role and destiny on earth. Technology is a destining that has involved the human role in
revealing, that is, in bringing-to-presence truth; and consequently, it raises the issue of why and for what wider cause humans possess their particular station as
conscious agents in the whole process of truth happening. We must stare into the depths of all that is and was and can be and recognize, above all, that what humans
essentially are is, in some mysterious way, a "grant." So Heidegger says. "Only what is granted endures. That which endures primally out of the earliest beginning is
If technology is seen as an imminent threat to humans, it comes to focus human
what grants." {[7], p. 31}
attention upon that which is granted to human life since what is granted is precisely what is most
threatened. As we view the specific history of that grant in the development of technology from
antiquity onward, we can begin to understand that which is granted to humans in a deeper sense.
This is why Heidegger could suggest that the saving power begins to grow precisely within the
greatest danger. "The essence of technology is in a lofty sense ambiguous. Such ambiguity points to the mystery of all revealing, i.e., of truth." {[1}, p. 33}
In this way, technology may share some of the privilege that we have already discussed for art. The artist, we should recall, is specially stationed with respect to
"homecoming," because the artist has a propensity to journey forth. Having seen the ordinary objects of the world in many different lands, the artist is specially
prepared to recognize that home is not merely those particular objects that make up one's familiar world. The essence of home lies in the special way that humans
make their way in the world, that is, the special way in which life comes into being in the homeland. In the case of technology, we mirror the poet's propensity to
journey far afield though, in this case, it is journeying away from a more passive and cooperative relationship with other beings. In the epoch of technology, we are
well prepared, then, to understand that the human essence is not merely invested in being one-among-beings, any more than home is merely the people and the place.
We are, indeed, better prepared for the mystery involved in the human essence and, in particular, the grant that we humans live. Technology's aggressive venturing
forth in the world eventually calls us to cast our glance back on what is granted to us in the world. That world is, afterall, our home. Technology too, in its perilous
ways, calls us to a homecoming. What is it to save? Heidegger suggested that "saving" is more than hauling something back to its original form; instead, it should be
the saving power that arises through art and within the
construed as bringing something back into its essence. Thus,
danger of modern technology must be a power to bring humans back into their essence. "Might there
not perhaps be a more primally granted revealing that could bring the saving power into its first shining forth in the midst of the danger, a revealing that in the
technological age rather conceals than shows itself?" {[7], p. 34} Yes, art, the poetic in particular, stands in this position. What we have known as art has been
merely a shadow of the poetic that could remain within the truth of enframing, something "derived from the artistic," "enjoyed aesthetically," merely "a sector of
cultural activity." {[7], p. 34} But art in its essential nature holds forth a mode of revealing in which humans themselves can come to presence in that characteristic
role that is their grant. In conclusion, Martin Heidegger rested an amazing assertion of hope upon art and, in particular, upon the poetic. "Could it be that the fine arts
are called to poetic revealing? Could it be that revealing lays claim to the arts most primally, so that they for their part may expressly foster the growth of the saving
power, may awaken and found anew our look into that which grants and our trust in it?" {[7], p. 35}
Aff – Permutation Solves – Technology
Perm do the aff and embrace the danger of technology in an whole and authentic
affirmation of our duty as safekeepers of the world
Sabatiano 7 (Charles J., Daemen College. “A Heideggerian Reflection on the Prospects of Technology” reprinted
in Janus Head 10.1. p. 63-76. PDF accessed July 3, 2008)

Nevertheless, in spite of what at times appears to be a severe criticism of technology, Heidegger does not
sound an ultimately despairing note about the fate of the modern world. In the latter part of his
reflections, he addresses what he refers to xas the possibility of a saving that might turn things
around and offer hope. To be saved, he says, is to be brought back to an awareness of who we are:
as those to whom has been entrusted the safekeeping of world. This will occur not by trying to
escape the danger; but rather by reaching into the danger; for where the danger is, there precisely is the
saving.8 Hope exists not in backing away from the danger out of fear, condemning the reach of
technology; but rather in entering more fully and embracing the danger for what it is. We would
be saved from the danger by the danger itself. Harking back to the themes of authenticity
originally struck in Being and Time (eigentlicheit: owning as pertaining to oneself), it would entail owning
responsibility as those to whom the world has indeed been given over and into whose safekeeping
it now rests.
Aff – Permutation Solves/Ontology Not First
The negatives focus on ontology prevents any change, only the ethic offered by the perm can
fully solve for environmental problems
Hirokawa 2(Keith, Stanford Environmental Law Review, June 2002, 21 Stan. Envtl. L.J. 225)
However, Delgado's critique is not aimed at the way in which environmental stewards approach their duties; his
attack raises a philosophical objection to the very possibility of stewardship. Yet, Delgado's position is undermined
by his insistence that the primordial human impulse compels us to dominate. That is, if Delgado is correct, and
humans cannot transcend their natural impulses when adopting the position of a trust administrator, then no other
theory of environmental protection should fare any better. In this sense, his critique of the public trust theory
leaves the distinct feeling that no legal tool or theory could counteract such destructive impulses.
Moreover, the problem with applying the flaws of paternalistic decisionmaking to the environment
is an epistemological problem. Paternalistic impulses drive the desire to protect others, usually upon the
judgment that the "others" are unable to rationally discern harmful from harmless activity. n198 The problem of
paternalism is typically raised as the inevitability of value assumptions, made by the actor on
behalf of the "other." n199 The rejection of environmental paternalism necessitates an assertion
of a truer understanding of nonhuman values; or, in other words, a (likely unjustified)  [*271] 
claim to an epistemological privilege. This does not imply that any comprehensible form of right ascribed to
an animal is senseless because of the inability of an animal to assert such a right. n200 But the rejection of the
human perspective ignores a means to implement environmental rights, especially when an
accurate understanding of nonhuman interests is a contested subject. As an epistemological matter,
access to such notions is laden with obscurity and leads us into the Kantian antinomy of the human
condition - that we too often focus on questions that we are unable to answer. n201 Delgado asks us to
throw out the baby with the bath water.
What must be observed here, from the pragmatic perspective, is that "environmentalistsneed to be able to
enter the public arena armed with genuine and defensible moral principles so that they can assert
the priority of their goals over the mere preferences of the consumer society." n202 The
pragmatist rejects the notion that the human perspective on environmental law inevitably leaves
us feeling "somewhat disingenuous," n203 and results in "two distortions: First, an inchoate sense of
obligation toward natural objects is flattened into an aspect of self-interest; second, value discontinuities tend to be
foreshortened." n204 These two "distortions," like Delgado's criticisms, are diversions that obscure any real
solutions to environmental problems. The pragmatic response to the problem of the human land
manager is that the human perspective is not as constricting as Delgado proposes. Quite simply, the law
has not yet found the limits of the human perspective.
Recent decades have witnessed a resurgence of experiments in testing these limits of anthropocentrism. The
pragmatists, in particular, have set out to defend a human perspective that is not limited by
anthropocentric interests. n205 They have found that human insights create a context in which
environmentally protective doctrines come to light in common terms and concepts. Besides
avoiding some of the more agitating epistemological debates, the  [*272]  human perspective
limits dialogue only to the extent of our ability to empathize and theorize. It is because humans are
able to seek both economic and non-use values, while reconciling the competing ends, that the human
perspective is demands attention. Our philosophical underpinnings may in fact be confined by
our distinctively human perspective, but they are concurrently liberated by our sense of selves, "characterized
not so much by [our] ability to produce a culture but by [our] ability to transcend old cultural forms." n206
Aff – Technology/Calculation Permutation Solves
We must engage in calculative think and meditative thinking.
Kockelmans 85(Joseph, Penn State philosopher, HEIDEGGER AND SCIENCE , p.254.)
This ambivalent attitude In regard to modem science and technology, which says at the same time yes and no,
corresponds to the two modes of thinking we have referred to earlier. Calculative thinking will help us to
use our resources effectively; meditative thinking will help us in making certain that technicity
will not overpower us. Meditative thinking will thus make it possible for us to come to a freedom
in regard to things that lets beings be ( Gelassenheit), by maintaining an openness to the mystery
that is hidden in modem technicity.

Human should neither totally identify with or repudiate technology


Lovitt 77(William, Professor of German, Cal State-Sacramento, introduction to Martin Heidegger's THE QUESTION CONCERNING
TECHNOLOGY AND OTHER ESSAYS, p.xxxiii.)

Man needs above all in our age to know himself as the one who is so claimed. The challenging summons of
Enframing "sends into a way of revealing" (QT 24). So long as man does not know this, he cannot know
himself; nor can he know himself in relation to his world. As a consequence he becomes trapped
in one of two attitudes, both equally vain: either he fancies that he can in fact master technology
and can by technological means - by analyzing and calculating and ordering - control all aspects of his
life; or he recoils at the inexorable and dehumanizing control that technology is gaining over
him, rejects it as the work of the devil, and strives to discover for himself some other way of life apart
from it. What man truly needs is to know the destining to which he belongs and to know it as a
destining, as the disposing power that governs all phenomena in this technological age.
Aff – Environmental Pragmatism Good
Radical environmentalism must reconcile itself with the progessivism of the plan in order
to create a new modern awareness and real change
Zimmerman 3 ("On Reconciling Progressivism and Environmentalism" http://www.integralworld.net/zimmerman7.html)
Modernity has contributed to this evolutionary process by making possible greater individuation, but modernity's understandable reaction against
religion's based on revelation and supportive of reactionary regimes went too far. Denying the divine altogether, whether as the source of matter-
energy or as the end toward which cosmic evolution strives, modernity ends up defining progress as the ever greater control of the physico-
biological plane, and thus as ever great capacity for satisfying human desire, instead of as an evolutionary process that makes possible a higher,
more both a integrated level of consciousness and the social institutions consistent with that level. Satisfying material desires is surely important,
but humanity's ultimate desire can never be satisfied, even by an infinite amount of material goods, for arguably what people really want is not
more consumer goods, not more power over nature, not a longer life span, but eternity in the form of union with the nondual divine. The
consumerist culture that wreaks social and ecological chaos arises because so many people are using material goods to fill a void that such goods
cannot fill. One can never get enough of what one doesn't really want. If
progressives must take environmental problems
seriously, environmentalists must recognize the validity of modernity's emancipatory economic
and political aims. Freeing people from material deprivation and political authoritarianism is a
noble goal, but one that must be furthered in ways that enhance all life on this planet, whenever
possible. Reconciling progressivism and environmentalism is central to a positive, postmodern
awareness, which transcends customary boundaries--physical, organic, mental, artificial--in
disclosing the creative principle at work in all phenomena. A prerequisite for the rise of this
more integrative awareness is that the majority of people need to develop modern consciousness
and institutions, although such development must avoid critical damage to the biosphere. Hence,
the need for environmentalists and progressives to cooperate in reminding each other of what the
other finds so important.

Environmental pragmatism has been empirically proven to solve for environmental


problems
Mintz 4 (Joel, Professor of law, Boston college environmental affairs law review,
http://findarticles.com/p/articles/mi_qa3816/is_200401/ai_n9403748/pg_1?tag=artBody;col1

Another potential environmental benefit of pragmatism-with its insistence on social justice and
the accomplishment of social ends-is that judicial adherence to its methods appears likely to
increase the likelihood that environmental statutes will be afforded pro-environmental
interpretations in the courts. Oliver Wendell Holmes, Jr.'s legal pragmatic idea of the judge as interstitial legislator, shaping the law
consistent with prevalent moral and political theories,142 is certainly consistent with this interpretative possibility, as is Benjamin Cardozo's
notion of the judge as the guardian of morality, reason, good conscience, and social justice.143 Richard Posner's staunch insistence on the
superiority of practical reasoning over the rigidity of legal formalism144 also appears to have this same benefit for environmental proponents.145
pragmatism places a high value on experimentation and innovative problem solving. In
Moreover,
view of the inherent complexity of environmental problems, as well as the legislative "gridlock" that has
characterized environmental law since the early 1990s-particularly at the federal level-these aspects of pragmatic theory seem especially well
Although environmental law contains notable examples of
suited to contemporary environmental policymaking.
bold, large-scale innovations-from technology-forcing requirements to emissions trading
regimes146-it is relatively devoid of small-scale pilot projects carefully designed to test the
efficacy of particular technologies or regulatory techniques under controlled conditions. Those
relatively inexpensive experiments, which hold the promise of eventual environmental
improvements on a broader scale, are very much consistent with the pragmatic method.147
Aff – Rejecting Management Bad/Impossible
It is impossible to do nothing in the environment. Our very existence impacts on the
environment and is a form of management. We must figure out ways of acting that crafts
action towards the environment and historical examination of ecosystems allows us to do
so. Their claim that it is too complex for predicting the effect of any of our actions is false.
Bowman 1 (D.M.J.S., Northern Territory University, Australia, 2001
Journal of Biogeography, Vol 28, No. 5, Future Eating and Country Keeping: what role has environmental history in the management of
biodiversity? JSTOR)

Land managers can use the enormous complexity of landscape change and the absence of clear
goals as an excuse not to act. But this strategy ignores a basic lesson from environmental history
that it is impossible to ‘do nothing’ in dynamic systems. I suggest that the most basic goal of land
management should be to minimize the rate of species extinction, particularly in largely intact landscapes and to
maintain or restore ecosystem services, such as potable water, in degraded landscapes. In this regard a historical
perspective provides .in important context to detect marked declines in species populations and
ecosystem function. For example, the widespread decline of granivorous bird species (Franklin, 1999) and the
population crash of the endemic conifer (lslli:ris sn:rarroprca R.T. Baker & HG. Sin. (Bowman & Panton, 1993) is
clear evidence that northern Australian tropical savamsas are undergoing rapid evolutionary adjustment in response
to ecological changes initiated by European colonization, The unpotability of many inland waters and collapse of
native fish populations because of eutrophication and salinity dramatically signals the unsustainability of past and
present agriculture practices in southern Australia (State of Environment Advisory Council. 1996.) CONCLUSION
The description and explanation of environmental change is of critical importance for land
management and the conservation of biodiversiry. However, practitioners of environmental
history must accept that their studies are politically charged and that their findings are bounded
by great uncertainty. It is inevitable that conflicting interpretations of essentially the same data will arise because
authors have different value systems. Dovers (2000. p. X) sensibly advises scientists and historians that they should
enter debates about environmental history with their ‘eyes wide open and other parts of their anatomy well covered’.
Many of these tensions will he most apparent in popular works that have a tendency to gloss-over uncertatnties and
over-generalize in order to make psychologically saris’ing stories. Some stories can be so powerhil that they assume
a life of their own. Rather than attempting to quash these stories, I suggest it is more productive for
ecologists to harness them to justify subsequent enquiry and to bolster land management
interventions. Although in the short term dogmatically interpreted stories can stifle public land
management debates and frustrate the incorporation of new research findings, in the long term
they will be replaced by new stories to accommodate changing knowledge and value systems. In
any case, on a practical level the rigid application of any single view across the diversity of landscapes is doomed to
failure. just as traditional ecological knowledge enables indigenous people to adapt to particular
environments I am confidetit that appropriate stories will he continually rehned to bolster
ecologically sustainable management of modern landscapes. Regardless of the ideological
struggles amongst different value systems, eventually natural selection will see the triumph and
ecological ‘fit’ stories over ecological flawed ones. Isn’t thdt Darwin’s law?
Aff – Rejecting Technology Bad
Heidegger’s philosophy has moral consequences and leads to paralysis – It justifies sitting
back and allowing for the Holocaust while criticizing the technology used to kill the Jews
Bookchin 95(Murray, Founder of the Institute for Social Ecology and Former Professor at Ramapo College, 1995 [Re-enchanting
Humanity, p. 168-170] Rein)

"Insofar as Heidegger can be said to have had a project to shape human lifeways, it was as an endeavor to resist, or should I say, demur from,
what he conceived to he an all-encroaching technocratic mentality and civilization that rendered human beings 'inauthentic' in their relationship to
Heidegger
a presumably self-generative reality, 'isness', or more esoterically, 'Being' (Sein). Not unlike many German reactionaries,
viewed ‘modernity' with its democratic spirit, rationalism, respect for the individual, and
technological advances as a 'falling' (Gefallen) from a primal and naive innocence in which
humanity once 'dwelled,’ remnants of which he believed existed in the rustic world into which he was born a century ago.
'Authenticity', it can be said without any philosophical frills, lay in the pristine Teutonic world of the tribal Germans who retained their ties with
‘the Gods’, and with later peoples who still tried to nourish their past amidst the blighted traits of the modern world. Since some authors try to
muddy Heidegger's prelapsarian message by focusing on his assumed belief in individual freedom and ignoring his hatred of the French
Revolution and its egalitarian, 'herd'-like democracy of the 'They', it is worth emphasizing that such a view withers m the light of his denial of
individuality. The individual by himself counts for nothing', he declared after becoming a member of the National Socialist party in 1933. 'The
fate of our Volk m its state counts for everything.' 22 As a member of the Nazi party , which he remained up to the defeat of
Germany twelve years later, his antihumanism reached strident , often blatantly reactionary proportions. Newly appointed as the
rector of the University of Freiburg upon Hitler's ascent to power, he readily adopted the Fuehrer-principle of German
fascism and preferred the title Rektor-Fuhrer, hailing the spirit of National Socialism as an antidote to 'the
darkening of the world, the flight of the gods, the destruction of the earth [by technology], the
transformation of men into a mass, the hatred and suspicion of everything free and creative His .’28

most unsavory remarks were directed in the lectures, from which these lines are taken, 'from a metaphysical point of view', against 'the pincers'
created by America and Russia that threaten to squeeze 'the farthermost corner of the globe ... by technology and ... economic exploitation.' 29
Technology, as Heidegger construes it, is 'no mere means. Technology is a way of revealing. If we give heed to this, then another whole realm for
the essence of technology will open itself up to us. It is the realm of revealing, i.e., of truth.’ 30 After which Heidegger rolls out technology's
transformations, indeed mutations, which give rise to a mood of anxiety and finally hubris, anthropocentricity, and the mechanical coercion of
things into mere objects for human use and exploitation. Heidegger's views on technology are part of a larger weltanschauung which is too
multicolored to discuss here, and demands a degree of interpretive effort we must forgo for the present in the context of a criticism of
technophobia. Suffice it to say that there is a good deal of primitivistic animism in Heidegger's
treatment of the 'revealing' that occurs when techne is a 'clearing' for the 'expression' of a crafted
material - not unlike the Eskimo sculptor who believes (quite wrongly, I may add) that he is 'bringing out' a hidden form that lies in the
walrus ivory he is carving. But this issue must be seen more as a matter of metaphysics than of a spir itually charged technique. Thus, when
Heidegger praises a windmill, in contrast to the 'challenge' to a tract of land from which the ‘hauling out of coal and ore' is
subjected, he is not being 'ecological'. Heidegger is concerned with a windmill, not as an ecological
technology, but more metaphysically with the notion that 'its sails do indeed turn in the wind;
they are left entirely to the wind's blowing'. The windmill 'does not unlock energy from the air
currents, in order to store it'. Like man in relation to Being, it is a medium for the 'realization' of
31

wind, not an artifact for acquiring power. Basically, this interpretation of a technological
interrelationship reflects a regression - socially and psychologically as well as metaphysically – into quietism.
Heidegger advances a message of passivity or passivity conceived as a human activity, an
endeavor to let things be and 'disclose' themselves. 'Letting things be' would be little more than a
trite Maoist and Buddhist precept were it not that Heidegger as a National Socialist became all
too ideologically engaged, rather than 'letting things be', when he was busily undoing
'intellectualism,' democracy, and technological intervention into the 'world'. Considering the time, the
place, and the abstract way in which Heidegger treated humanity's 'Fall' into
↓Card Continues↓
↑Card Continued↑
it is not hard to see
technological ‘inauthenticity’ – a ‘Fall’ that he, like Ellul, regarded as inevitable, albeit a metaphysical, nightmare -
why he could trivialize the Holocaust, when he deigned to notice it at all, as part of a techno-
industrial ‘condition’. 'Agriculture is now a motorized (motorsierte) food industry, in essence the
same as the manufacturing of corpses in the gas chambers and extermination camps,' he coldly
observed, 'the same as the blockade and starvation of the countryside, the same as the production
of the hydrogen bombs.’ In placing the industrial means by which many Jews were killed before
32

the ideological ends that guided their Nazi exterminators, Heidegger essentially displaces the
barbarism of a specific state apparatus, of which he was a part, by the technical proficiency he
can attribute to the world at large! These immensely revealing offhanded remarks, drawn from a speech he gave in Bremen m
1949, are beneath contempt. But they point to a way of thinking that gave an autonomy to technique that
has fearful moral consequences which we are living with these days in the name of the sacred, a
phraseology that Heidegger would find very congenial were he alive today . Indeed, technophobia,
followed to its logical and crudely primitivistic conclusions, finally devolves into a dark reactionism – and a
paralyzing quietism. For if our confrontation with civilization turns on passivity before a
‘disclosing of Being’, a mere ‘dwelling’ on the earth, and a ‘letting things be’ , to use Heidegger’s verbiage
– much of which has slipped into deep ecology’s vocabulary as well – the choice between supporting barbarism and
enlightened humanism has no ethical foundations to sustain it. Freed of values grounded in
objectivity, we are lost in a quasi-religious antihumanism, a spirituality that can with the same
equanimity hear the cry of a bird and ignore the anguish of six million once-living people who
were put to death by the National Socialist state.
Aff – Alternative Doesn’t Solve
The alternative’s reliance on theory will always preclude solvency because of its inherent
indeterminacy, only the affirmative plan’s rejection of theory is key to coalitions that can
form the best policy
Hirokawa 2(Keith, Stanford Environmental Law Review, June 2002, 21 Stan. Envtl. L.J. 225)

The turn to pragmatismthus symbolizes a rejection of the alleged  [*251]  relationship between
theory and answers to practical questions. To the pragmatist, this rejection comes for very good reason.
Competing conclusions can often be derived from the same incomplete set of premises, and divergent theories can
often produce the same conclusions. Pragmatists redirect human inquiry to avoid the indeterminacy of
theory, since the "knowledge of obstacles is not itself an obstacle unless it leads to defeatism; for pragmatists it
serves as a spur to seek a way to overcome those obstacles." n118 In the final analysis, although theories are
important, the pragmatist warns against theory commitments, because theories provide "no more
than commentary on practice, based on premises drawn from that practice itself." n119Accordingly,
the pragmatic position against theory is not a broad, sweeping dismissal of every idea derived from a theoretical
framework. n120 Rather, the pragmatist is free to consider a variety of ideas, approaches and
solutions without committing to particular theoretical foundations. The method and strength of problem
solving, n121 if not the purpose, is to ensure conversation participants  [*252]  that their theories are duly
considered. n122 The resulting formation of policy is "inclusive, treating current theories as
perspectives, each of which can add to the understanding of law." n123
Pragmatism, then, is a helpful tool (especially to environmental debate) because of its freedom from
any particular method of inquiry and any particular metaphysical "good" of society. For the
pragmatist, theories "are not Euclidean axioms or Kantian categorical imperatives, but graffiti, practical guidelines
to be noticed by the alertly street-wise when context makes them applicable." n124 When unbounded by
consistency with or loyalty to any particular theory, all relevant ideas become useful to the
resolution of a dilemma. The lesson from pragmatism is that to see the law as something more
than a refined, yet interminably eclectic conglomerate of ideas, taken from all forms of social
and cultural practices, would be to give too much credit to our insight into the nature of justice.
Aff – Alternative Doesn’t Solve
Major ontological paradigm shifts, produce conservative backlash, destroying any chance
at effective change
Hirokawa 2(Keith, Stanford Environmental Law Review, June 2002, 21 Stan. Envtl. L.J. 225)
Delgado's second criticism refers to the notion of paradigm shifts and normative analysis in legal theory. n91 He
notes that when dramatic paradigm shifts occur, a great deal of energy is directed against the old
paradigm. n92 Once a new dominant paradigm is in place, other alternatives, in addition to the
old dominant paradigm,  [*245]  are condemned and rejected. n93 Delgado's critique adopts the external
perspective: the law should have rejected the public trust doctrine in favor of a new paradigm that favored common
interests in property, such as a Marxist-type legal consciousness that eschewed the concept of private property
ownership altogether. n94 At least, the law should have adopted an understanding of natural resources that did not
institutionalize anthropocentric aims. n95 Based on this critique, Delgado offers alternative conceptions of the
environment - such as those discussed above - as alternatives deserving the attention of the legal community.
However, the alternatives Delgado identifies were not chosen as the foundations for environmental protection to the
detriment of the environmental debate. And, in a somewhat defeated tone, Delgado points to the public trust theory
as a lesson in futility: the legal community refuses to condemn and abandon old perspectives and
paradigms until "we are certain that the new one is not more discomforting than necessary." n96
At the same time, the legal community seeks to "condemn the more ambitious reform programs
as extreme and dangerous." n97 To be clear, Delgado does not argue that no change has taken place. Rather,
argues Delgado, when incommensurable paradigms compete, the exchange results in the acceptance of a
compromised and disconcerted effort, a facade designed to quell opposition and lead opponents to mistakenly
believe they have won the day. n98 The public trust doctrine is, according to Delgado, the product of such a
compromise. The only winner in such a case is the conservative proponent.
Delgado further argues that the principal effect of paradigm resolution is a death knell for radical
environmental perspectives since the resolution, a compromise from all but conservative points of view,.
diverts the intellectual community's attention away from anything that could sincerely be called a
solution By the time a movement regains composure and momentum, any solution offered
[*246]  comes too late. n99 For example, for many African Americans, changes such as abolition,
desegregation and equal rights came too late to be enjoyed. Likewise, for many women, modernization of status, job
opportunity and rape laws arrived too late. And most relevant to this discussion, for many forests, fish,
and other voiceless members of the ecological community, environmental protection has come
too late.
Aff – Alternative Doesn’t Solve
The Kritik, with its focus on the intrinsic value of beings, will never be able to succeed
because it cannot offer a policy alternative.
Hirokawa 2(Keith, Stanford Environmental Law Review, June 2002, 21 Stan. Envtl. L.J. 225)
The environmentalist must choose between two strategies. She can persist in the hope that courts will
determine that the current environmental regime is inherently faulty, which will lead to the destruction
of environmental law and property rights as we know them. Or, alternatively, she can recognize that the
perceived barriers to integrating environmental ethics and environmental law are themselves a
result of the rhetoric and persuasion from those opposing radical change. Such a recognition could
favor the environmentalist's position as it has her opponents. Inevitably, the choice  [*256]  wavers between
two types of thinking: the dogmatic and the pragmatic. The dogmatic "thinks of truth as a
vertical relationship between representations and what is represented," n140 whereas the
pragmatist "thinks of truth horizontally - as the culminating reinterpretation of our predecessor's reinterpretation of their
predecessor's reinterpretation." n141 The difference between the two perspectives, in essence, is "the difference between regarding truth,
goodness, and beauty as eternal objects which we try to locate and reveal, and regarding them as artifacts whose fundamental design we often
have to alter." n142 This, of course, is just another way of saying that adapting Kuhn's thesis to legal analysis is itself a paradigm, one that is
by attempting to advance radical theories of nature that lack a viable
principally pragmatic in nature. But
means of policy implementation, n143 environmentalists find themselves in an extremely
unpersuasive, dogmatic camp.
Recent philosophical scholarship on environmental ethics has begun to recognize the limits of
theorizing on intrinsic value in nature. While noble in cause, debates about intrinsic value may not
provide useful answers. n144 Instead, "many writers have begun to outline a pragmatic agenda for
environmental ethics by proffering approaches that attempt to shift the field's mode of inquiry to a more practical
dialogue about the multiplicity of values at play in specific matters of environmental policy." n145
Environmental ethics, it seems, is experiencing a "swelling wave" of support for the "pragmatic
turn" in environmental theory. n146
Aff – Alternative Doesn’t Solve

The Negative’s radicalized stance prevents its acceptance in society, the only way to solve is
the compromised facilitated by pragmatism
Hirokawa 2(Keith, Stanford Environmental Law Review, June 2002, 21 Stan. Envtl. L.J. 225)

James' observations recognize that the basic problem with new ideas is that they are contrary to
accepted beliefs. Although there is often a natural resistance to revolutionary ideas, integrating new ideas into
prior assumptions is a powerful form of persuasion. While persuading someone to give up firmly held
beliefs may be exceedingly difficult (if not hopeless), she may welcome being told that her prior
beliefs are basically correct, with slight modifications. A successful result of paradigm exchange,
in the environmental context as well as any other, may hinge on this simple lesson. Pragmatic proponents of
environmental protection have adopted such an approach, arguing that "a more democratic and pluralistic
approach to the countenance of ethical orientations regarding the natural world must be
accepted" to foster informed environmental policy. n151 Some adopt a more direct strategy aimed at softening the
property right that assumes "environmental protection is not at odds with property; it is part and parcel of the same relationship between society
and the individual that gives property its meaning." n152 Even Leopold, who was acutely aware of the barriers obstructing his project, came to
the pragmatic conclusion that "non-anthropocentrism raises issues too intractable to make it useful in management decisions." n153
It should now be obvious that the ways of thinking about argument and the methods of
recharacterization offered by pragmatism have application in the environmental debate. Certainly,
understanding deadlocked disputes in paradigm terms helps explain the basis for heartfelt
disagreement. In this context, pragmatism offers a means by which paradigm opponents can find
common ground and potentially agree on environmental policies  [*259]  and laws. In short, the
pragmatic view of environmental law serves as a promising negotiating strategy for those who
have grown weary of ongoing disputes over meta-theoretical, incommensurable principles
between rights to environmental quality and rights to property. n154
Aff – Ontology Doesn’t Come First – Pragmatism
Instead of a focus on ontology, we should look first to pragmatism
Putnam 4 (Hilary, Harvard University. http://evans-experientialism.freewebspace.com/ethics_without_ontology.htm)
When in the last of these four lectures I present an obituary on the project of Ontology, it will be
an obituary on all of these versions, the deflationary as well as the inflationary. In place of
Ontology (note the capital “O”), I shall be defending what one might call pragmatic pluralism, the
recognition that it is no accident that in everyday language we employ many different kinds of
discourses, discourses subject to different standards and possessing different sorts of
applications, with different logical and grammatical features—different “language games” in
Wittgenstein’s sense—no accident because it is an illusion that there could be just one sort of language game which
could be sufficient for the description of all of reality! My pragmatic pluralism may, perhaps, make it clear why I
reject eliminationism in both its materialist, or Democritean, form and its idealist, or Berkleyan, form; but why will I
be rejecting inflationary (for example, “Platonic”)metaphysics? My answer is that I hold, with the pragmatists and
again with Wittgenstein, that pragmatic pluralism does not require us to find mysterious and supersensible objects
behind our language games; the truth can be told in language games that we actually play when language is working,
and the inflations that philosophers have added to those language games are examples, as Wittgenstein said—using a
rather pragmatist turn of phrase—of “the engine idling.
Aff – Ontology Doesn’t Come First – Violence
Central to Heidegger’s philosophy is a violent creativity, which insures the construction of
a self-destructive ontology.
Habermas 98 (Jurgen, “Habermas on Heidegger” http://evans-experientialism.freewebspace.com/habermasonheidy.htm)

It is well known that, for Heidegger, the fate of the present is forgetfulness of Being
[Seinsvergessenheit]. [...] Europe lies in a great pair of pincers between Russia and America, which are, in their
essence, the same: "the same hopeless frenzy of technology unbound and the unparalleled organization of the
normalized man," for whom time means nothing more than speed. From both sides there is spreading over Europe
the darkening of the world, the flight of the gods, the destruction of the earth, the massification of man, and hatred,
suspicion toward all that is creative and free. [... Heidegger] is calling for a heroic existence in
opposition to the insipid, deteriorated condition of ordinary life. p. 192 It is "strength" that
elevates the aristocratic individual above the ordinary Many. The noble individual, who chooses fame,
will be ennobled by the rank and mastery that belong to Being itself, while the Many--who, according to
Heraclitus, whom Heidegger approvingly cites, are like well-fed cattle--the Many are the dogs and the asses.
What is worthy of rank is that which is stronger, for which reason Being eludes whoever is
concerned about evening out, reducing tension, leveling off: "The true is not for everyone, but
only for the strong." Moreover, it is "spirit" ["Geist"] that distinguishes the thinker vis- a-vis the
intellectual. Intelligent calculation is oriented towards objects and places them at man's disposal. Its leveling grasp
brings all things down to one level: extension and number are its predominant dimensions. For this thinking,
"ability" no longer means extravagant expenditure out of lofty abundance, but the sweaty
performance of a routine. This thinking, which follows the laws of traditional logic, cannot
understand the question about Being [nach dem Sein], let alone develop it, because logic is itself
grounded in an answer to the question about what it is [nach dem Seienden], an answer that closes
off Being from the very outset. [...] The degeneration of thinking to intelligence can only be
overcome by thinking that is more primordial. Finally, "courage" must be added to strength and
spirit, an ambiguous form of courage that does not even shrink back from violence and error.
Appearance, deception, illusion, errancy are all powers that are appropriated by Being itself; it is only everyday
reason [Verstand] that no longer experiences their numinous force and degrades them to mere error. The
courageous individual repeats the beginning, in pre-Platonic Greece, of our intellectual-historical
existence, saying Yes to all the disconcerting strangeness, the darkness, {p. 193} the uncertainty and
insecurity of the true beginning. In the final analysis, the heroic individual develops his full essence [Wesen] as
one who dares: he is the violent individual, the creative individual, who masters Being by placing the
unsaid under the spell of his speech, the unseen under the spell of his gaze, and the unoccurred [das Ungeschehene]
under the spell of his deed. In this context, violence is not to be taken to mean the banality of a "brute, arbitrary act."
On the other hand, it is the faint-hearted man who is concerned with agreement, compromise, and
mutual care and who is accordingly only able to experience violence as a disturbance of his life.
"Thus the violence-doer [der Gewalt-Taetige] does not know kindness and appeasement (in the
ordinary sense), nor is he soothed and quieted as a result of his successes or prestige." He despises the appearance of
completion. [...] The violent man is the towering individual, the towering solitary; he is, in the final analysis,
the man with no way out, for whom non-existence represents the highest victory over Being,
whose existence finds its tragic fulfillment "in the most profound and far-reaching Yes to his
own destruction," who, in willing what is extraordinary, casts aside all help.
Aff – Ethics Before Ontology
Ethics must always come before ontology
Cohen 1 (Richard A., the Isaac Swift Distinguished Professor of Judaic Studies at the University of North Carolina at Charlotte, pg. 5
“Ethics, Exegesis, and Philosophy: Interpretation after Levinas,” Cambridge University Press)

Philosophy as ethical exegesis – discovering the ethical in the ontological, seeing the lower in the light of the higher, not anthropology
but ethics – is attuned to this deeper, weightier, truer history that defies straightforward language and is refractory to the
light of publicity. Its commitments are not to visible history alone, the history of historiography, but rather to a more insecure but deeper history,
that of the humanity of the human. The human is not a biological or a rational category. Rather, the human emerges when and
where morality is at work. Humanity is not a given but an achievement , an accomplishment, an elevation. Moral
rectitude and justice are rare enough for philosophy also to miss them. Ethical exegesis is philosophy attentive to
responsibilities beyond epistemology, and higher than the aesthetic celebration of the spectacle of be-ing
or its language. It is thinking bound to the “difficult freedom” of moral responsibilities and obligations –
for fellow humans, for sentient life, and finally for all of creation in all its diversity. And as such it is wisdom, or the quest for wisdom –
philosophy.
Just as the aesthetic dimension is not by itself evil (or good), one cannot say that the aesthetic life is false
(or true). Like good and evil, truth and falsity are not its standard. They are standards of epistemology .
Epistemology need not refrain from judging aesthetics, but neither epistemology nor aesthetics has the right to the last word. Ethics, in contrast,
can and must remind us that the aesthetic life is inferior to the moral life. The aesthetic world – however
spectacular, grand, or beautiful – is too small a world. When aesthetics takes itself for a world it becomes precious ,
as in Huysmans, or both precious and precocious, as in Heidegger and Derrida, or fascist, its true moral face. And let there be no
doubt, the aesthetic life revolves around the self , is indeed its very cult. And thus it is essentially linked to deat h or, by
dialectical rebound, linked to youth, for the self by itself is a mortal being.
Regarding not the truth but the superiority of morality, of ethical commitment (“either/or”) over aesthetic
disengagement (“both/and”), Kierkegaard has written penetrating and moving tributes to this wisdom . The
great nineteenth-century German Orthodox rabbi and scholar, Samson Raphael Hirsch, in the Jewish tradition, commenting on Proverbs (chapter
two, “Wise Men and Fools”), notes that the word that text opposes to “wisdom” (Hebrew: chochmah), namely, “foolishness” (Hebrew: olat), “is
related to oulai, 'perhaps, ' and ahfal, 'darkness'. ” 6 Again, Levinas's “temptation of temptation, ” the perhaps, the maybe, the possible, opposed
to and by the actual, the here, the now, not the real but the moral “demands of the day. ” No one would oppose beauty, to be sure,
but when self-regard becomes disregard for others – and surely it tends in this direction – then aesthetic desires
become evils, hardening rather than softening the heart. There are worthier, nobler tasks. Ethical exegesis – penetrating through the
spectacle and its display of signs to its human dimension, the dimension of suffering and moral demand – articulates the fragile but
overpowering solidarity of a human community on the difficult road of redemption. It will say and say again the
rupture of the masks of being demanded by morality and of justice.
Beyond but through morality, ethical exegesis will also dare to suggest, obliquely, to be sure, the glimmer of another exigency – spirit,
inspiration, absolution – more intense, higher, brighter, illuminating and not illuminated by the light of sun, moon and stars. Micah 6:8: “For he
has told thee, oh humans, what is good, and what the Lord thy God does require of thee, but to do justice and to love mercy and to walk humbly
with thy God. ” Not a “proof text, ” to be sure, but a confirmation.
The difference putting ethics first makes is of no less consequence than that which, according to Husserl,
separates philosophy and psychology. That difference was at once the greatest chasm and yet barely discernible, in that the findings
of these two disciplines would be strictly parallel to one another, so parallel that a sentence from one could be transposed word for word into the
other, yet their significance would be entirely different. Philosophy – in this case ethics, what I am calling ethical exegesis – would be
the absolute source of all meaning, hence the ground of psychology and sociology and all the sciences, social or natural. While not
another epistemological grounding of epistemology, ethical exegesis nonetheless still has the pretension to provide the reason for philosophy.
But “reason” in the sense of “end, ” “purpose, ” “aim” – what is most important, most significant . Without
returning to pre-modern philosophy, without imposing one arbitrary ontotheo-logy or another, without making a fetish of science or of its
drifting, and most especially without the pretended “second innocence” of aesthetic celebration, ethical exegesis – in moral responsibilities and
obligations, and in the call to justice built upon these – supplies a reason for philosophy, a reason for knowledge and a reason for living. No doubt
this is a very large claim. And in this sense, this is an ambitious book. Very simply : nothing is more significant than serving
others. All other significations, in all other registers, derive from this deepest or highest significance.
Aff – Kritik Ethics Bad
TURN: The hierarchical indeterminacy of the kritik ethics actively affirms genocidal eco-
fascism and is politically useless.
Thomson 4 (Iain, Dept. of Philosophy, Univ. of New Mexico. “Ontology and Ethics at the Intersection of
Phenomenology and Environmental Philosophy”, published in Ecophenomenology: Back to the Earth Itself
copyright 03. PDF accessed July 2, 2008 p. 393-94)

What will naturalistic eco-phenomenologists say, then, about cases in which the needs of different
ecosystems collide, either at borders between systems (as, for example, when those systems expand or shift in
response to changing environmental conditions), or when new and seemingly dominant organisms begin to emerge
(by mutation or transplantation) within estab- lished ecosystems? They will point out that ecosystems tend to be
connected holistically, nested within one another (vertically) in relations of hierarchical
interdependence or joined (horizontally) in interlocking webs, which means (by the same logic as
above) that disputes between ecosystems should in principle be decidable by appeal to the needs
of a broader system encom- passing the competing organisms or systems. The problem here,
however, is not so much that such an appeal suggests an infinite regress (vertically, from ecosystem to meta-
ecosystem to meta-meta-ecosystem, and so on; as well as horizontally, from one ecosystem directly to the entire
holistic web of eco- systems), for we can halt this regress for almost all current practical purposes by treating the
planet earth itself as the outermost ecosystem within which all its interconnected ecosystems are nested (even while
acknowledging that all life in this earth ecosystem is in fact sustained by its precise place within the solar system –
its distance from our sun and moon, for example – and that roughly the same point can be made regarding the place
of our solar system within its galaxy, and so on). The more pressing conceptual problem is that the
further up in the hierarchy of nested ecosystems one ascends, the more difficult it is to say what
the needs of (and so the good for) that overarching ecosystem are, making it very difficult fairly
to adjudicate conflicts within large-scale ecosystems. For, how do we really know which of the
ecosystems that have come into conflict (the old or the new, say) better serve the needs of the meta-ecosystem
encompassing them, unless we know what these needs are? What, then, are the needs of the earth-as-ecosystem,
beyond the perpetuation of conditions capable of sustaining life? Gaia metaphysics notwithstanding, does the earth
ecosystem have an ‘interest’ in which of its competing sub-ecosystems wins out? Insofar as it does, is it not likely
that the earth ecosystem’s interests will conflict with the interests of humanity, that species most
likely (by several orders of magnitude) to extinguish life on earth, a species currently
responsible for a massive depletion of biodiversity?24 If, with Husserl, we should recognize
‘consciousness itself as a project of the world’, then, with Freud, we must also acknowledge that a powerful
thanatological drive seems to be at work within this project. Eco- phenomenologists might not agree with
Wood ‘that photographs from space argue that the earth, itself a living system, is dying’ (p. 226), but many share
Wood’s belief that the grave nature of our environmental crisis calls for desperate measures, up to
and including governmentally-controlled ‘active human population reduction’ (p. 224). Some of us
worry that we see ‘eco-fascism’ here, a worry which will not be assuaged by Wood’s suggestion that
‘[t]he argument that there are circumstances in which democratic societies might suspend
democracy [in order to enact unpopular but necessary environmental measures] is not as
totalitarian as it seems’ (p. 231).25 (Here we face another ‘empirical’ problem about what people
are willing to accept; the recurrence of such problems, I suggest, indicates the presence of a basic philosophical
conflict between eco-centric and humanistic eco-phenomenological approaches.)
Aff – Kritik Ethics Bad

TURN: Their ethical determination of being is infinitely regressive, and is framed upon the
same anthropocentric hierarchy they seek to kritik. The aff provides the necessary limit on
this system and prevent wasteful dilution of our ethical duty.
Thomson 4 (Iain, Dept. of Philosophy, Univ. of New Mexico. “Ontology and Ethics at the Intersection of
Phenomenology and Environmental Philosophy”, published in Ecophenomenology: Back to the Earth Itself
copyright 03. PDF accessed July 2, 2008 p. 401-03)

The suggestion, put provocatively, is that eco-phenomenologists should answer the question, ‘Which entities
deserve intrinsic rights?’, with: ‘All Dasein’, that is, all entities whose being s an issue for them, and only
these entities (although other entities could, of course, deserve rights instrumentally, in virtue of their relations to
Dasein, including relations of eco-systemic interdependence). On this neo-Heideggerian eco-
phenomenological view, what counts (in contrast to the naturalistic, neo-Nietzschean and Husserlian posi-
tions considered earlier) is not life per se, but rather a life that has a temporally-enduring world that
matters to it explicitly. Heidegger did well to escape the gravity of his age far enough to recognize that being
a Dasein is not an all-or-nothing affair, since there are degrees of ‘having a world’. Still, as
Llewelyn observes, ‘Heidegger’s phenomenology ... does not entail this ... thinking of the non-
human other. It only enables it’ (on Llewelyn’s view, by conceiving of Dasein broadly as ‘being’s oikos, the
ecologicality of being’ (pp. 58, 62)). The simple tripartite distinction Heidegger famously proposes in The
Fundamental Concepts of Metaphysics between the ‘worldless’ rock, the ‘world-poor’ animal, and the world-rich
‘Dasein’ really only inaugurates the more difficult labor of drawing fine-grained distinctions on a
much fuller continuum. We might imagine such a continuum of Dasein as stretching, for example, from: (1) ‘worldless’
inorganic matter; to (2) similarly ‘worldless’ invertebrate organisms (lacking a nervous-system and so physiologically incapable
of sensation); to (3) simple vertebrate organisms (possessing of the capacity to experience pleasure and pain, and so somewhere
between being ‘worldless’ and ‘world-poor’); to (4) ‘world- poor’ entities like the lizard on the rock and cow in the field
(sentient but not reflexive, apparently permanently immersed in perceptual immediacy); to (5) the ‘near-Dasein’ of such entities
as the chimpanzee (whose self-awareness is demonstrated, for example, by a remarkable capacity to incorporate an explicit
understanding of its role in a complex social group into a creative plan to accomplish difficult, temporally- distant goals); to (6)
the partial Dasein of such entities as gorillas (who conveniently demonstrate their possession of a world by learning our
languages); to (7) the potential Dasein of young children (who combine capacities like (6) with the potential for (8)); to (8) the
‘rich world’ of full Dasein (including not only normal adult human beings but also whatever other entities – be they organic,
android, or alien – possess a reflexive self-understanding making them capable of experiencing not merely pleasure and pain but
also immense suffering and sublime elevation, and of developing and pursuing a self-understanding which gives meaning to
their lives from within); and, perhaps, to (9) entities with even richer worlds than human Dasein – who could deny the
possibility? This suggested elaboration of a graded ‘continuum of Dasein’ remains too simplistic and
speculative, of course, and perhaps its implicit hierarchy is marked by a residual
anthropocentrism. Notice, however, that the same ‘criticisms’ hold even for extreme, eco-centric perspectives,
which yield such anthropomorphizing confusions as Klaver’s belief that a stone can have ‘being-in-the-world’ (pp.
159–62) and Diehm’s idea that, in all organisms, ‘the horizon opened by need is, minimally, a horizon of self-
concern, an openness to experience’, such that mere ‘being alive ... is ppropriately ... characterized as being-for-
itself’ (p. 181). Such implicitly anthropomorphic descriptions – which seek to bestow upon simple
organic (and even inorganic) entities reflexive capacities such as ‘self-concern’, ‘being-for-
itself’, and even ‘being-in-the-world’ (in other words, Dasein) – generate a hyperbolic, neo-Levinasian
extension of ethical concern in which ethical duties, multiplied to infinity, become uselessly
‘diluted’ (as Casey argues).48
Aff – Kritik Ethics Bad
Their environmental ethics is merely a vague rationalization of a world without a value
system—it can’t solve real crises at the point where the ethics cover non-human organisms.
Thomson 4 (Iain, Dept. of Philosophy, Univ. of New Mexico. “Ontology and Ethics at the Intersection of
Phenomenology and Environmental Philosophy”, published in Ecophenomenology: Back to the Earth Itself
copyright 03. PDF accessed July 2, 2008 p. 391)

Insofar as it is, should we adopt this ethics? Nietzsche argues that a naturalistic ethics of life will not resemble
our Judeo- Christian value system; some will go further and worry that it may not justify any value system
at all. This worry about the indeterminacy of an ethics of life is suggested by the fact that, the
more specific the values that are supposed to follow from ‘the intrinsic purposes and desires of
life’, the more dubious their derivations seem. To add another example: when Koha ́ k asserts that the
human attachment to a ‘homeland’ is justified by the fact that we are embodied entities (p. 28), it sounds as if he
is rationalizing a politically disastrous and thus ethically suspect attachment. We are not salmon,
after all. The underlying worry here is that the ‘axiological transcendental’ (which holds that
ultimately the objective conditions that generate life are good while those which diminish it are bad) may be too
general to be of much help in resolving real-world environmental disputes, especially once we
have made room at the ethical bargaining table for non-human organisms, as the view suggests we
must when it posits life in general as its fundamental value.
Aff – Kritik Efhics Bad

The kritik can never solve disputes of life and death


Thomson 4 (Iain, Dept. of Philosophy, Univ. of New Mexico. “Ontology and Ethics at the Intersection of
Phenomenology and Environmental Philosophy”, published in Ecophenomenology: Back to the Earth Itself
copyright 03. PDF accessed July 2, 2008 p. 391-92)

Perhaps many kinds of living beings can appreciate (after a fashion) ‘the goodness of water’ (Brown, p. 12), but
what should we do in those increasingly common situations in which water becomes so scarce
that human groups and non-human organisms develop what amount to competing and
incompatible claims to it? In the high desert region of central New Mexico, for example, a debate now
rages between those who, in effect, want to divert scarce water from the Rio Grande in order to
meet the needs and desires of the human population, and those environmentalists who argue that
any further depletion of this once-great river will devastate the ecosystem it now struggles to
support, driving to extinction the last of the silvery minnows, an endangered species (Hybognathus
amarus) in the Rio Grande which has given the environmentalists their only leverage in this political struggle.23
How will the eco-phenomenologists’ naturalistic axiological transcendental help decide such
pressing issues, when its appeal to those conditions that serve life seems to support both sides?
How should we adjudicate between the competing demands of human lives, on the one hand, and the lives of the
silvery minnow and other non-human animals, on the other? One ‘solution’ might be to point out that life
lives on life, that the needs of one group of organisms often come into conflict with the needs of
another, and conclude that we should leave it to life to work through these problems for itself.
Husserl recommended ‘recognizing consciousness itself as a project of the world’ (Klaver, p. 165),
and, as Langer shows, one can extend such a view all the way to Freya Mathews’ position that we should
accept nature’s ‘dark side and its exemplification in human destructiveness’ by recognizing that
‘bulldozers, chainsaws, bombs, missiles, and all our other technologies are part of “the natural
order” and contribute to “the moral order” as effectively as any forest’, so that we should ‘honor these
ecologically destructive technologies’ as nature’s own projects
Aff – Impact Inevitable
The neg naively imposes their kritik and its system of universal ethics upon a world already
overrun by selfish Western value—the alternative can never solve subjective perceptions of
nature.
Thomson 4 (Iain, Dept. of Philosophy, Univ. of New Mexico. “Ontology and Ethics at the Intersection of
Phenomenology and Environmental Philosophy”, published in Ecophenomenology: Back to the Earth Itself
copyright 03. PDF accessed July 2, 2008 p. 389)

Now, within the phenomenological tradition, there are at least two ways to take the claim that, as Lester Embree
puts it, ‘the world in which one finds oneself is always already fraught with values’ (p. 39). Taken as
an ontic claim (concerning our everyday experience of entities), the idea is that, for example, ripe apples show
up looking good to us, clear-cut forests looking bad. Taken as an ontological claim (concerning what
such ontic facts reveal about the structures conditioning the way reality is disclosed to ‘Dasein,’ that entity whose
being is an issue for it), the idea is rather that entities only show up for us at all because we care about
them in some way (as Heidegger argues in Being and Time). This latter, ontological claim
complicates the ethical realism assumed by the former, ontic one, however. For, if entities only show up
within the horizons of my concerns, then unless there is some universal concern, the values
entities possess when encountered pre-theoretically cannot be made the basis for a non-
relativistic ethics. The ripe apple, for example, may not look good to someone who has eaten too much, or who
simply dislikes the taste of apples, and Brown himself maintains that ‘[ c]learcutting large tracks of old-
growth forest may appear as good from the perspective of business and profit’ (p. 11).17 Thus,
although entities and events show up within our worlds as already mattering to us in various ways ( our pre-
reflexive experience is always-already infused with ‘value’, if you will), appealing to these ‘pre-
given values’ will not help us escape the charge of ethical relativism unless these values are both
universal and substantive. There may well be ‘no separation of factual information from
meaning and value’ in our ordinary experience of the lifeworld (Marietta, p. 122). Nevertheless, among
the values that show up in this ‘pre-thetic’ lifeworld, we still need to be able to distinguish between the
entrenched sedimentations of pernicious traditions, on the one hand, and truly universal values
(if there are any), on the other. Given the conflicting values embedded in our lifeworld, and the fact that pre-
reflexive experiences are often shaped by, and so tend to reinforce, all manner of pre-existing
prejudices, Brown’s own faith that the ethical wisdom ‘pre-given’ in the lifeworld will eventually
resolve all of our conflicts seems rather optimistic.
Aff – Impact Inevitable

Impact is inevitable—Heidegger never assumes our postmodern world of infinite human


capacity and manipulation.
Sabatiano 7 (Charles J., Daemen College. “A Heideggerian Reflection on the Prospects of Technology” reprinted
in Janus Head 10.1. p. 63-76. PDF accessed July 3, 2008)

Interestingly, Heidegger sounded his note of warning about the danger of this era of technology
even before some of the more astounding achieve- ments of recent years. Events during the past
half century would seem to confirm his perspective, as we find very little that escapes the power
of human manipulation. Not only have we split the atom, but we have managed to delve into the inner
workings of its most elementary particles. Nature, in its most minute dimensions has been penetrated.
But also laser and radio scopes have reached out into the beginnings of time itself to practically
catch up with the very origins of the universe. And so nature at its cosmically largest and distant has also
been pentrated as its countless galaxies are probed and made ready for observation. However, even those
accomplishments pale before more recent break- throughs in bio-technology. The encoding within
its double helix structure of the human genome has been deciphered; various possibilities for genetic engineering
(whether therapeutic or reproductive) are already available. Nuclear transfer techniques associated with cloning
open the door to incred- ible options with embryonic stem cell research. There is practically nothing,
whether far or near, large or small that has not been laid open to human scrutiny and
manipulation.
***Eco Fem K***
Connection of Nature and women is Bad

Ecofeminsts root women’s traits in biology, playing into the old stereotypes that are used to
oppress women

Biehl 1991
Janet, Co-author of Left Green Perspectives, social ecologist, “Rethinking Ecofeminst Politics” p. 11-12 1991
Connection of Nature and women is Bad

Ecofeminism attempts to build a system of ethics for dealing with the environment based
entirely on metaphors will fail—judgments become arbitrary
Biehl 1991
Janet, Co-author of Left Green Perspectives, social ecologist, “Rethinking Ecofeminst Politics” p. 24-25 1991
Connection of nature and women is Bad
Karen Warren Flawed

Warren is flawed, some domination is justified


Perm Political action

Ecofeminists are struggling for political reform and have the ability to spur change in the
political arena
McGuire and McGuire 1991
Ecofeminist Visions,revised 1993 and 2003

At the heart of that analysis is an understanding that for the past 5,000 years, the “male” has
dominated the “female.” Many ecofeminists believe the planet at this point needs massive
infusions of female energy to regain balance. Although ecofeminists are immersed in social
and political struggles for reform on a variety of fronts to achieve this balance, there remains a
certain sympathy for Ynestra King’s classic declaration, "We don’t want a piece of their rotten, carcinogenic pie."
A few inspiring examples of ecofeminist activism (from the early 1990s) intended to right
the balance include: Wangari Maathai's formation of the Green Belt Movement in Kenya
in which rural women planted trees as part of a soil conservation effort to avert
desertification of their land; The women of Greenham Common Peace Camp in England
whose over ten-year presence was instrumental in the removal of nuclear missiles there;
Mohawk women along the St. Lawrence River who established the Akwesasne Mother's Milk Project to monitor
PCB toxicity while continuing to promote breastfeeding as a primary option for women and their babies; Judi
Bari's bringing together labor and environmental groups in Northern California to save
the remaining five percent of old growth redwood forests from corporate logging; Artist
Helene Aylon's Sister Rivers performance ritual in which Japanese women placed rice, seeds, and soil from
Hiroshima and Nagasaki in pillowcases and then floated the artwork down the Kama River; Bernadette Cozart, a
gardener and founder of the Greening of Harlem, who organizes diverse community groups in Harlem to transform
vacant garbage-strewn lots into food and flower gardens; Lois Gibbs' exposure of Love Canal as a toxic waste site,
and her founding of the Citizens Clearinghouse for Hazardous Waste to share tactical skills with local environmental
groups.
Ecofeminism excludes Women of Color

Domination in the ecofeminist movement by white women exclude women of color


Taylor PhD., 1997
Dorceta E., Professor of Sociology, Program director for Minority Environmental Leadership development initiative” Women of Color,
Environmental Justice, and Ecofeminism” in Ecofeminism: Women, Culture, Nature pp.38-81

Ecofeminist is embedded with totalizing and dominating discourses that exclude women of
color
Taylor PhD., 1997
Dorceta E., Professor of Sociology, Program director for Minority Environmental Leadership development initiative” Women of Color,
Environmental Justice, and Ecofeminism” in Ecofeminism: Women, Culture, Nature pp.38-81
Ecofeminism excludes Women of Color

Ecofeminist cannot build coalitions with race neutral language and lack of adherence to
experiences of women of color
Taylor PhD., 1997
Dorceta E., Professor of Sociology, Program director for Minority Environmental Leadership development initiative” Women of Color,
Environmental Justice, and Ecofeminism” in Ecofeminism: Women, Culture, Nature pp.38-81
Ecofeminism excludes Women of Color

Women of color have a multilayered experience with oppression that ecofeminsim doesn’t
address
Taylor PhD., 1997
Dorceta E., Professor of Sociology, Program director for Minority Environmental Leadership development initiative” Women of Color,
Environmental Justice, and Ecofeminism” in Ecofeminism: Women, Culture, Nature pp.38-81
*** Bataille K***
Bataille is too utopian
Bataille’s approach is problematic since it reinforces the Hegelian model of historical
development and hopes on a utopian society that is solely absent from capitalism
Allan Stoekl 1990, Professor of French and Comparative Literature at Penn State University, Source: “Editor's
Preface “ Yale French Studies, No. 78, On Bataille, (1990), pp. 1-6
Bataille’s approach has no social importance
Bataille’s approach affirms the Hegelian dialectic and amounts to nothing
Allan Stoekl 1990, Professor of French and Comparative Literature at Penn State University Source: “Editor's
Preface “Yale French Studies, No. 78, On Bataille, (1990), pp. 1-6

BATaILLE’S WRITING IS UNDERDEVELOPED AND HIS CONCEPTS ARE INACCESSIBLE AND


UNWORKABLE TO OTHERS
DeBoer Jason 1999, literary and philosophical writer “Bataille versus Theory” “http://www.sauer-
thompson.com/essays/Bataille%20Versus%20Theory,%20an%20essay%20by%20Jason%20DeBoer.doc”

Georges Bataille organizes his writings around many core concepts or ideas, many of which remain diffuse and
somewhat underdeveloped in their definitions or meanings. Communication, sovereignty, heterology, inner
experience, the sacred, dépense or expenditure, transgression, excess, etc.; each concept appears in his texts as a
momentary connotation, a brief enunciation that creates an impact in the reader, then disappears before becoming
fully ensnared within the parameters of conceptualization. Perhaps it is this vagueness or ambiguity inherent in all of
Bataille’s concepts that prevents them from being appropriated by the theoretical mainstream and being put to work
in a dogmatic system. In order for an idea to be put to work, for it to be able to perform a function, perhaps it must
first have a proper definition... which many of Bataille’s concepts lack. The broadness of his terms (indeed,
Bataille’s move from a restrictive to a general economy shows a digression from the specific, from specialization)
may keep them from being utilized by others; this subversion of utility arises from the difficulty of pinpointing
where or when a Bataillean concept begins or ends. This sacrifice of clarity certainly is an intentional strategy,
Bataille’s own “employment” of unworkable concepts. It is within this arena of thought that I wish to examine the
contemporary state of theory.
Bataille’s Methodology of Sacrifice is Flawed

Batailies sacrificeable mode of resistance domestriates death


Elisabeth Arnould, 1996, Assistant Professor of French Literature at University of Colorado,”The Impossible Sacrifice of Poetry:
Bataille and the Nancian Critique of Sacrifice”

What exactly, however, does this sacrificial renunciation of words represent for Bataille and why did he choose to
load it with all the transgressive weight of the experience of nonknowledge ("non-savoir")? Why place the self-
sacrifice of poetry at the very center of a book that would be annulled by an immolation of speech? Indeed, why
make this final "supplice"--a sacrifice of words performed and dedicated "to" "nothing"--into the core of the book's
interior rapture? Isn't this figure of death and silence incurably equivocal? Does it not attribute the traditional traits
of a "nothingness" to this "rapturous heart" of finitude that Bataille always wanted to designate as the "impossible"
object of his experience? And is it not, therefore, condemned to appropriate, through this representation, the
nonmeaning of a finitude that Bataille, far from conceptualizing as simple nothingness, usually prefers to designate
ambiguously as "nonknowledge"?

Such are the questions I shall address in reevaluating Bataille's concept of sacrifice in light of Jean-Luc Nancy's
critique. In his article "The Unsacrificeable," Nancy was the first to point out the problematic nature of the
sacrificial model in Bataille's conceptualization of finitude. According to Nancy, sacrifice, including the self-
sacrifice of Rimbaud so important to Bataille, is the vehicle of an "ontotheological" appropriation. And it is through
sacrifice that Bataille's reflection on finitude attempts to domesticate death while claiming to abandon it to the
aporetic enunciation of a "nonknowledge."

Sacrifices for the sake for sacrificing is redundant


Elisabeth Arnould, 1996, Assistant Professor of French Literature at University of Colorado,”The Impossible Sacrifice of Poetry:
Bataille and the Nancian Critique of Sacrifice”

Let us return to the statement: the works, that is to say, "the goddess to whom we sacrifice is herself sacrifice, tears
wept to the point of dying" ("cette déesse à laquelle nous sacrifions est elle-même sacrifice, larmes pleurées jusqu'à
en mourir") [IE151, translation modified; EI175]. What are we to make of this tautology? What does it mean "to
perform a sacrifice to sacrifice"? One usually sacrifices to a God, to an ideal. One sacrifices, as Proust did, to a
work, to a meaning and a truth, in order to produce that meaning and that truth. According to the strange tautology
of an intention and a gesture that never coincide, to perform a sacrifice to sacrifice can mean nothing other than to
sacrifice for sacrifice. The sacrifice to sacrifice is a sacrifice that can never take place, a sacrifice that performs itself
as the impossibility, or as the indefinitely held-up possibility, of its realization. To offer a sacrifice to sacrifice is
also to sacrifice sacrifice itself. And the self-sacrifice of Proust not only repeats Rimbaud's negating gesture but also
emphasizes its nonmeaning. For this formulation clearly indicates that this last sacrifice is not held up to a sacrificial
truth, be it positive or negative, spiritual or immanent. Sacrifice does not sacrifice to the revelation of its
nothingness. It sacrifices simply to sacrifice, to the indefinite adjournment of its day of revelation. Like Rimbaud
turning away from poetry, it, in the classical sense of the term, apostrophizes 6 sacrifice, turns [End Page 94] away,
sacrifices to ("in order to" and "to") sacrifice.
Discourse is Inaccessible and unpragmatic
Racial Realism Supersedes the K

The silence of how white supremacy functions in the kritik perpetuates how it is an
invisible unnamed system that has made the modern world what it is today.
Charles Mills, a professor of philosophy at the University of Illinois writes in his book the Racial Contract in 1997 p.1
White supremacy is the unnamed political system that has made the modern world what it is today. You will
not find this term in introductory, or even advanced, texts in political theory. A standard undergraduate
philosophy will start off with Plato and Aristotle, perhaps say something about Augustine, Aquinas, and
Machiavelli, move on to Hobbes, Locke, Mill, and Marx, and then wind up with Rawls and Nozick. It will introduce
you to notions of aristocracy, democracy, absolutism, liberalism, representative democracy, socialism, welfare
capitalism, and libertarianism. But though it covers more than two thousand years of western political thought
and runs the ostensible gamut of political systems, there will be no mention of the basic political system that has
shaped the world for the past several hundred years. And this omission is not accidental. Rather, it reflects
the fact that standard textbooks and courses have for the most part been written and designed by whites, who
take their racial privilege so much for granted that they do not even see it as political, as a form of
domination. Ironically, the most important political system of recent global history—the system of
domination by which white people have historically ruled over and, in certain important ways, continue to
rule over nonwhite people-is not seen as a political system at all. It is jus taken for granted, it is the
background against which other systems, which we are to see as political, are highlighted.
Black Feminism Supersedes Postmodern Kritik

Turn: since postmodernism fails to provide alternative, black feminsim is hurt by post
modernist thought

Collins 1998, Distinguished University Professor of Sociology at the University of Maryland, College Park, 1998 (Patricia Hill,
Fighting words-Black women and the Search for Justice : pg 125)

On the other hand, postmodernism undercuts selected dimensions of African American women’s political activism.
For example, postmodernism rejects ethical positions that emerge from absolutes such as faith. It also eschews
social policy recommendations –to make such recommendations requires advancing truth claims and advocating
specific political actions stemming from those claims. The absence of responsibility grounded in some sort of ethical
stance is at odds with African American women’s long standing contributions to Black civil society. Thus, although
postmodernism provides a plausible response to dominant discourses and the politics they promote, it fails to
provide direction for constructing alternatives.

Turn: the exclusionary language used by postmodernists like Batonly continues to deny
black women from intellectual work
Collins 1998, Distinguished University Professor of Sociology at the University of Maryland, College Park, 1998
(Patricia Hill, Fighting words-Black women and the Search for Justice : pg 125)

Postmodern reliance on exclusionary language has tremendous implications for African American womens’ struggle
for self definition. As a result of a continual critique carried out in exclusionary language , little room remains to
construct an identity grounded in an authorial voice of Black women. Moreover, when coupled with the minimal
decentering of actual power relations in higher education, commodifying and exchanging the new language of
postmodernism has become a new form of cultural capital in the academic market-place. By performing a powerful
gatekeeping function for those who lack access to the exclusionary language of postmoderenism, the rhetoric of
deconstruction can be used to maintain the status quo. As is the case with any commodity, scarcity determines its
value . despite postmodern lip service to decentering, the intellectuals writing articles, giving papers, populating the
editorial boards of journals, and occupying positions of authority within academic disciplines seem remarkably
similar to those of the past. To me, this is the ultimate postmodern irony. The ability to manipulate exclusionary
language becomes yet another standard used to exclude Black women from legitmated intellectual work.
CSUS Debate Aff K Toolkit
TP
2008 - 2009

Transgression is Socially Irresponsible; Compassion is Better Alt


They are commodifying Bataille’s academic musings, identifying with the transgressive, to
win a debate. Refusing to imagine cooperative action with others is unacceptable given the
context.
Tauchert in 2008
Ashley, Head of the English Department in the School of Arts, Languages and Literatures at the University of
Exeter, Critical Quarterly, vol. 50, no. 1, April, pp. 17-18 (online at Wiley InterScience journal collection)

The intellectual engagement with questions of the final truth of human consciousness initiated by Bataille has
been subsequently reified into a commodified critical desire to identify with the anguished and transgressive
itself; a desire that has come to operate an unconscious fascination for academic thought, all the more
compulsive as it is unconscious. By making the conditions of this thought conscious we can face it and decide
what is to be done with it. Unconsciously motivated thought cannot be socially responsible thought, and we
would do well to remember our responsibilities to each other and to the younger generation of thinkers. So
this is also a book about power, the power of thought, and the power to generate and direct thought. The final
concern of this work is to begin a process of understanding how it is that academic theorists can be so excited by
transgressive thoughts, but dismayed by the effects of transgression in the social world.33 It is all very well and
good to celebrate Bataille’s condensed image of the exorbitated eye: quite another thing to have one’s own
eyes exorbitated. If this sounds naîve I am happy for it to be so, since to turn away from the fascinated,
morbid play of critical transgression is finally to turn towards a conscious innocence that might just be more
powerful in the end.

Compassion is a better alternative than transgression. It both enables us to deal with the
longing for substantiality and to imagine why the action of our advocacy is a good idea.
Tauchert in 2008
Ashley, Head of the English Department in the School of Arts, Languages and Literatures at the University of
Exeter, Critical Quarterly, vol. 50, no. 1, April, pp. 120-121 (online at Wiley InterScience journal collection)

Renunciation of desire is no simple thing in practice, however, and arguably more suited to the life of a well-
paid, highly reflective philosopher than of most subjects suffering the disorientating effects of virtual
capitalism in their daily lives. But we do have to start somewhere, and where we are is always the best place.
It is quite possible to accept insight into the longing expressed through transgression without making such a
song-and-dance about its more mutilated, anguished expressions. It is also possible (if not ultimately
inevitable) to experience this longed-for continuity without creating conditions of anguish and without
entering the states of moral transgression which are favoured by the Bataillean school. This is what Kathy
Acker realized in facing her own death consciously: I have become interested in languages which I cannot
make up, which I cannot create or even create in: I have become interested in languages which I can only
come upon (as I disappear), a pirate upon buried treasure. The dreamer, the dreaming, the dream.358
I think that Joy Davidman expresses the thought I am chasing here rather beautifully. She probably deserves, and would not expect to have, the
last word: O agony, burn at my heart;
Burn at my heart and keep me warm.
Deliver me from the harsh iron of winter,
Unclothe me of the silver fur of frost,
Pare away the ice from the ends of my fingers.
Set me free of cold idleness
And deliver me from the barrier across my tongue
For I will say my word.
This is winter and I am imprisoned in it
With the tips of my fingers slowly turning to ice
But I shall not forget words
And the beautiful ringing of words linked together,
And I shall remember compassion
And keep the heart lit with a fire of pain
And let the sound of suffering made music
Whistle and sing in my throat until I die.359
CSUS Debate Aff K Toolkit
TP
2008 - 2009

Compassion Alternative -- AT: Sentimental and sickly


Compassion alternative to transgression is not mere sentimentality. But it avoids the lazy
nihilism of transgressive thought.
Tauchert in 2008
Ashley, Head of the English Department in the School of Arts, Languages and Literatures at the University of
Exeter, Critical Quarterly, vol. 50, no. 1, April, pp. 118-119 (online at Wiley InterScience journal collection),

The mode of emptiness-compassion I want to offer up against transgression is not a sentimental, sickly
emphasis on suffering. It is the spontaneous selflessness that flows from an acknowledgement of the
‘emptiness’ of the ‘self’: and in this sense extends the thought of ‘continuity’ without linking it to its
transgressive concomitants, ‘sacrifice’ and ‘anguish’. This idea might accept and complete the transgression
of thought that Bataille attempted to put into motion through his work; acknowledging ‘the absence of reified
selfhood’ without falling from there into the lazy nihilism of much critical transgression.350 Emptiness-
compassion begins from the moment of realization that the self is not reifiable and does not exist ‘inherently’: The
conclusion that the self does not exist at all is not drawn, but rather that the self is utterly unreifiable, non-
inherently existent. Understanding this emphasizes the contingent, dependent, interconnected, and non-
autonomous nature of the self’s existence. The self apprehended in emptiness-compassion theory is
‘associated with the development of a compassionate sense of relatedness in which self and other are seen not
as oppositional but as relative designations, like the far and near banks of a river’ [Klein]351:
By becoming accustomed to the equality of self and other,
The spirit of enlightenment becomes firm.
Self and other are interdependent.
Like this side and the other side of a river, they are false.
The other bank is not in itself ‘other’;
In relation to someone else it is ‘this bank.’
Similarly, ‘self’ does not exist in its own right;
In relation to someone else it is ‘other’.352
CSUS Debate Aff K Toolkit
TP
2008 - 2009

Compassion Alternative – Transgression is Insufficient Experience


Compassion goes beyond the moment of emptiness that transgression produces to
liberation. It is a process of imaginative extension of feeling to the other.
Tauchert in 2008
Ashley, Head of the English Department in the School of Arts, Languages and Literatures at the University of
Exeter, Critical Quarterly, vol. 50, no. 1, April, pp. 118-119 (online at Wiley InterScience journal collection),

‘Emptiness’, however, once experienced, is for the practitioner of ‘ultimate liberation’, not an end in itself:
The insight into emptiness, facilitated by awareness, leads to an empowered compassion. The relationship
between such insight and concern for others is an important principle in Mahayana Buddhism. Insight, and
its affective counterpart, compassion, are crucial for the Buddhist enterprise of ultimate liberation. [Klein]346
The mode of liberation promised by transgression is always limited, momentary, fleeting, unsatisfactory:
Orgasm has possessed the libertine; during the irreducible timelessness of the moment of orgasm, the hole in the
world through which we fall, he has been as a god, but this state is as fearful as it is pleasurable and, besides, is lost
as soon as it is attained. He has burst into the Utopia of desire, in which only the self exists; he has not negotiated
the terms of his arrival there, as gentle lovers do, but taken Utopia by force. See, the conquering hero comes. And,
just as immediately, he has been expelled from it, a fall like Lucifer’s, from heaven to hell. [Carter]347 By
contrast, the ‘ultimate liberation’ offered through experience of ‘emptiness’, and the capacity for radical
compassion it engenders, is unlimited by external conditions. It cannot be thwarted by the counterdesire of
the other or the resistance of the object; it does not struggle to define itself against the constraints of necessity
(‘reality’); and it is not, according to Bataille, limited by ‘the idea of a personal God.’348

Perhaps most importantly for this discussion, emptiness-compassion does not remain bound to an ambivalent
desire for the (dead) maternal body as the ground of its becoming. Compassion is instead actively generated
by the practice of imagining the other as the subject’s living mother:

Imagine your mother clearly in front of you . . . As your mother, she


protected you from all harm and provided you all benefit and happiness.
Particularly in this lifetime she carried you for a long time in her womb.
Then, when you were a helpless, newborn infant, she held you to the
warmth of her flesh and bounced you on the tips of her ten fingers. She
suckled you at her breast . . . and used her hand to wipe your excrement.
So in various ways she nurtured you tirelessly . . . Even more, what she
gave to you were not things that she had obtained easily but that she had
received through great hardship . . . In short, contemplate one-pointedly
how your mother provided help and happiness and cleared away harm
and suffering to the best of her knowledge and ability.349

It’s not supposed to be an easy exercise. Just the first step of imagining your personal mother’s particular sacrifices
to generate, sustain and nurture the life of the subject you became is more difficult than it sounds on first reading.
Close your eyes and try it for a moment. It will take a few goes to settle into the practice, and the results can be
nauseating at first. Extending the feelings of compassion that result to the people closest to you is just about
possible. Taking this further to apply those feelings to those you dislike or ‘hate’ and to the millions to whom
you feel nothing much at all takes serious and sustained practice. The effects on the practising subject can be
very interesting, but vary from person to person, so it is best to experiment rather than be told what to feel.
CSUS Debate Aff K Toolkit
TP
2008 - 2009

Transgression cannot recognize difference in mind-body experiences


The range of extreme experiences in transgressive thought is too narrow to encompass the
mind-body continuum
Tauchert in 2008
Ashley, Head of the English Department in the School of Arts, Languages and Literatures at the University of
Exeter, Critical Quarterly, vol. 50, no. 1, April, pp. 116-117 (online at Wiley InterScience journal collection),

Transgression has revealed an abiding longing for ‘continuity’ glimpsed in ‘limit conditions’ outside of the range of
reason. This is an enduring and valuable insight, which has emerged from a profound intellectual reflection on the
mind-body continuum, demonstrating how ‘limit’ experiences of the body can produce exciting new states of mind.
It is conceivable, however, that the ‘limit’ states preoccupying critical transgression to date demonstrate only
a very small part of the story that might be told concerning the mind-body continuum: The main thrust of
Western intellectual history has avoided the discussion of the perfected human being. The focus has rather been on
the universal, not the exceptional . . . today’s Western thinkers [are] more open to discussing the exceptional than
were their predecessors, but, oddly, they examine mainly the negative side, that of the abnormal or diseased. Modern
Western neurophysiology and psychoanalysis have opened the door to the extraordinary in human life, but only, as it
were, the back door of the subnormal instead of the front door of the supernormal.340 Transgression is fixated on
the publicly abjected extremities of human potential: madness, orgies, murder, rape, incest, ecstacy, s-m,
death. Michel Foucault, according to James Miller, generated his most interesting ideas through
experimenting with ‘unusual bodily sensations and altered states of consciousness’.341 Miller identifies the
inspiration for Foucault’s mature intellectual work as an LSD trip in California. His companion on the trip recalls
that at its height, with tears ‘streaming down his cheeks’, Foucault suddenly exclaimed: ‘I now understand my
sexuality’.342 His sexuality was certainly very interesting, as was indeed Bataille’s. But there is no reason for
those very particular (and in the most practical sense ‘limited’) ‘limit’ experiences to become definitive of the
range and direction of critical thought to follow. The model of sovereign liberation forcing its way through
transgression may turn out in the long run to have been a retrograde step:
the conquest of morality and aesthetics, of shame, disgust and fear, the
pursuit of greater and greater sexual sophistication in terms of private
sensation lead them directly to the satisfactions of the child; transgression
becomes regression and, like a baby, they play with their own excrement. [Carter]343
CSUS Debate Aff K Toolkit
TP
2008 - 2009

Gender K of Transgression vs. Compassion Alternatives

Transgression is almost exclusively a masculine practice. It is a fantasy that excludes


women as reciprocal subjects.
Tauchert in 2008
Ashley, Head of the English Department in the School of Arts, Languages and Literatures at the University of
Exeter, Critical Quarterly, vol. 50, no. 1, April, pp. 113-114 (online at Wiley InterScience journal collection),

Transgression has revealed itself to be an almost exclusively masculine critical preoccupation. Transgression
as a respected intellectual tradition is the domain of male white Europeans. De Sade – Nietszche – Bataille –
Foucault; an auto-affective genealogy that has incorporated otherness in the form of female bodies and female-
embodied thought as the material resistance against which it forces its intellectual climax in a repetitive gesture that
returns to the point of departure without the possibility of what Luce Irigaray – for whom ‘woman represents the
emergence of the other with respect to Western tradition’ – would call ‘horizontal transcendence’.331 If
‘Transgression’ cannot be thought by the other, but only thinks of itself in violent domination or
incorporation of otherness, through which it reaches its climax of autoaffective intensity, it can only really
represent the logic of the same masquerading as a radical break with tradition. To make overt the core
metaphor on which the theory of transgression attempts to close itself as a system is quite possible. Schehr explains
that Lyotard, in his writing encounter with Bataille, had ‘a raging hard-on’; his ‘intensities gone wild’:

There is a homoerotic subtext of anal penetration . . . [c]ircles of men then


. . . [i]ntensities of prostitution, homosexuality, ultimately equivalent . . .
an orgy of exchange . . . homosexual fellation: a cosmic circle jerk.332

I wonder if the ‘intensities’ of prostitution really are equivalent to homosexuality? It seems very unlikely, unless
Schehr means the intellectual prostitution of homosexual desires. In any case, this intellectual figure of a ‘cosmic
circle jerk’ by necessity excludes women except as objects – rather than reciprocal subjects – of
representation or thought. Women would break the circle. Julia Kristeva has already noted that Bataille
explores openly the ‘subject/object relationship (and not subject/other subject)’ that is ‘rooted in anal
eroticism’.333 Now, I’ve nothing against ‘anal eroticism’ in itself, but wonder whether this is the best – or
only – way we might order our critical work.
CSUS Debate Aff K Toolkit
TP
2008 - 2009

AT: Do Both Transgression and Compassion


Compassion is intellectually competitive with transgression. Compassion asserts the
irreducible truth of the other and transgression centers on the sovereign imaginary.
Tauchert in 2008
Ashley, Head of the English Department in the School of Arts, Languages and Literatures at the University of
Exeter, Critical Quarterly, vol. 50, no. 1, April, pp. 115 (online at Wiley InterScience journal collection),

Compassion is the ‘enemy’ of transgression in the simple sense that compassion recognizes the irreducible
truth of the other in any encounter, critical or otherwise, while transgression centres on the inner experience
of a sovereign will-to-power; masterhood. Angela Carter’s searing analysis of de Sade exposes the particular
condition of Sadean transgression as the denial of the possibility of love: ‘The libertine’s perversions are the actings-
out of his denial of love’.337 The maternal body is the primary object of transgression because transgression is
always regressive in its tendencies, pushing back the ordering injunctions of the fundamental, maternal
‘architecture’ that structures the subject’s individuation. The excremental enthusiasm of the libertines transforms
the ordure in which they roll to a bed of roses. The pleasure of the libertine philosophers derives in a great
part from the knowledge they have overcome their initial disgust. By the exercise of the will, they have
overcome repugnance and so, in one sense, are liberated from the intransigence of reality. This liberation
from reality is their notion of freedom.338 [Carter]

Compassion avoids their dilemma between transgression and regression.


Tauchert in 2008
Ashley, Head of the English Department in the School of Arts, Languages and Literatures at the University of
Exeter, Critical Quarterly, vol. 50, no. 1, April, pp. 116-117 (online at Wiley InterScience journal collection),

Kathy Acker knew that for de Sade women can ‘transform (. . .) themselves into men by choosing to engage in
sodomy’: ‘In sodomy, the most delicious position is the passive one. In other words, a woman can know freedom by
choosing to counterfeit a man who selects the bottom power position.’335 I’m not convinced by this model of
freedom: it seems rather limiting for women to counterfeit the freedom of homosexual men. If criticism is
personal to the point of performing unconscious desires, it should also be possible to direct the process in
accordance with some form of moral consciousness (restraint) without capitulating to regressive tendencies or
reducing desire to instrumentalized productivity. This section takes the form of a ‘Preface to Compassion’
against ‘Transgression’ as an experiment in shifting the discussion beyond the transgression/regression impasse. If
transgression overcomes the other in its drive to achieve a sovereign state of continuity, intensified by
breaches in a fundamental moral ground for human individuation, compassion might just offer an equally
bracing model of radical liberation, and one less jarring to maternal subjectivity.
CSUS Debate Aff K Toolkit
TP
2008 - 2009

** Cap K***
CSUS Debate Aff K Toolkit
TP
2008 - 2009

Link Turn: Subsidies continue inequality


Subsidies unfairly interfere with the economy, by removing them the aff stops the
inequality of the SQ- solves your impacts

Karl Beitel policy analyst at Food First/Institute for Food and Development Policy. Food First Backgrounder
August 23rd, 2005 http://www.foodfirst.org/backgrounders/subsidies

Over the last five years, groups spanning the ideological spectrum have come out in opposition to US and EU
farm support payments, or subsidies. Critics of US and EU farm policy claim that subsidies are a major cause
of overproduction. Overproduction depresses global prices, leading to a loss of economic viability and the
destruction of small-scale agriculture, both in the US and globally. While US farm policy is highly
discriminatory against smaller farmers, the excessive focus on subsidies has served to
obscure the deeper forces underlying the long-term decline in global farm commodity
prices. This Backgrounder will argue that declining agricultural commodity prices are rooted in
the market’s lack of self-correcting mechanisms. Even in the absence of subsidies,
commodity markets do not tend to equilibrium or operate to ensure fair returns on farm
labor. Recognizing this reality is essential to any sound reform of US commodity policy.
On the surface, the argument against subsidies is quite compelling. Reforms in US farm policy
instituted after 1996 established subsidy programs in which payments to farmers are triggered once prices fall
below a floor price (the loan rate), which is set by Congress. While these subsidies shelter US farms
from risk, critics argue that the floor prices encourage overproduction, generating surpluses that are then
dumped on the international market at prices well below the cost of production. In fact, critics claim, the
main beneficiaries of subsidy payments are not farmers, but large agribusiness firms,
whose access to a steady supply of cheap farm commodities reduces their costs and boosts
their profits (as they don’t pass through full cost savings to consumers). This line of reasoning leads
to the assumption that reducing subsidies would curb overproduction and boost prices.
Critics further note, correctly, that US agricultural tariffs are higher than those levied by developing
countries, and call for their reduction.[1]
Without question, the current US subsidy system discriminates systematically against small
farmers in the US and globally. But two linked misconceptions pervade the present subsidy debate: that
subsidies are a principal—even the principal—cause of overproduction and falling prices; and, hence, that
removing subsidies (and cutting tariffs) will significantly boost incomes for poor farmers in the developing
world. Both these claims are inaccurate, and serve to obscure our understanding of the types of reforms that
are required to restore real equity and long-term sustainability to the US and global farm economy.
CSUS Debate Aff K Toolkit
TP
2008 - 2009

AT: Link – Subsidies facilitate Neoliberalism


They have it backwards- governments bow down to corporate domination via subsidies.
Bretton Woods Project 14 June 2000 The World Bank And The State: A Recipe For Change?
http://www.brettonwoodsproject.org/art.shtml?x=16242 pg 10

In Chile, too, liberalisation of the forestry sector has been characterised by government
hand-outs to the already rich. As policy analysts Joseph Collins and John Lear comment: "The
neoliberals’ stated goals were to curtail sharply the direct role of government in forestry
and to let market mechanisms determine the prices and direct the use of resources. Yet
government intervention and subsidies were in fact central to reorientating the benefits of
forestry production away from the rural population towards a handful of national and
foreign companies." 53 The new policies, argue Collins and Lear, directly or indirectly benefited Chile’s
largest conglomerates; some, such as Matte, already had significant investments in forestry, while others Vial
and Angelini, for instance used government concessions to create new forestry empires. All restrictions on
size and ownership of land holdings were lifted. The government also sold off its interests in the countries’
principal forestry processing plants. As in the privatisation of other areas of the economy: "these companies
were sold at a discount, according to one estimate at least 20 per cent below their value. They ended up,
together with privatised forestry land, in the hands of a few large conglomerates." 54 Government-
subsidised tree planting programmes also directly benefited the private sector. In 1988, 48
per cent of the area where planting was subsidised was owned by the ten largest forestry
companies in Chile. 55 Huge forestry empires have been created, often in alliance with
multinationals. Angelini, for example, owns large swathes of plantations with the New Zealand company
Carter Holt Harvey: together with Matte, it controls 40 per cent of the tree plantation area. Other companies
owning large areas of plantations include Shell, Bin Mahfouz of Saudi Arabia and Marubeni of Japan. In
addition to picking up subsidies from national governments, corporations are increasingly
availing themselves of a range of new subsidies from multilateral development banks
(MDBs). In the past, companies (particularly those from Northern countries) have benefited from
contracts for public sector development projects, such as roads, airports and irrigation
schemes, contracts which were awarded by governments but financed by MDBs. Increasingly,
however, the multilateral development banks are "moving to the private sector", with governments
acting as "facilitators rather than financiers". Instead of funding projects through states, the MDBs are
now funding private companies directly and underwriting investments through guarantees.
CSUS Debate Aff K Toolkit
TP
2008 - 2009

Alternative fails
The alternative fails – it can’t deal with state repression, it can’t develop social activities
outside of capitalism, and there is no way to organize self-determination.

John Holloway Ph.D Political Science-University of Edinburgh and Alex Callinicos Ph.D Philosophy
University of Oxford, former Professor of Politics- University of York August 16, 2005
http://www.zmag.org/znet/viewArticle/5616

For me, it is this second conception of revolution that we have to concentrate on. The fact
that we reject
the state-centred conception doesn't obviously mean that the non-state-centred conception
does not have its problems. I see three principal problems, none of which is an argument for
reverting to the idea of taking state power:
The first issue is how to deal with state repression. I do not think the answer is to arm ourselves
so that we can defeat the state in open confrontation: we would be unlikely to win, and
anyway it would involve reproducing precisely the authoritarian social relations we are
fighting against. Nor do I think that the answer is to take control of the state so that we can
control the army and the police forces: the use of the army and police on behalf of the people
obviously comes into conflict with the struggles of those who do not want anyone to act on
their behalf. This leaves us with trying to find other ways of dissuading the state from
exercising violence against us: this may have to involve some degree of armed resistance (as in the
case of the Zapatistas), but must surely rely above all on the strength of the integration of the rebellion into
the community.
The second issue is whether we can develop alternative doings (alternative productive activity)
within capitalism, and to what extent we can create an alternative social nexus between
activities, other than value. There are many experiments that point in the direction of some sort of
solution (the fábricas recuperadas, factories reopened by the workers, in Argentina, for example) and the
possibilities will obviously depend on the scale of the movement itself, but this remains a
major problem. How do we think of a social determination of production and distribution
that moves from the bottom up (from the interstitial revolts) rather than from a central planning
body?
The third issue is the organisation of social self-determination. How do we organise a
system of direct democracy on a scale that goes beyond the local level in a complex
society? The classic answer is the idea of councils linked by a council of councils to which the councils
send instantly recallable delegates. This seems basically correct, but it is clear that even in small
groups the operation of democracy is always problematic, so that the only way in which direct
democracy can be conceived is as a constant process of experimentation and self-education.
CSUS Debate Aff K Toolkit
TP
2008 - 2009

Alternative Fails- Holloway


Holloway’s alternative fails- he ignores the existence of state power, without radically
changing the oppressed’s position to the state nothing changes.

John Holloway Ph.D Political Science-University of Edinburgh and Alex Callinicos Ph.D Philosophy University
of Oxford, former Professor of Politics- University of York August 16, 2005
http://www.zmag.org/znet/viewArticle/5616

Whatever our differences, John and I stand for changing the world through a process of self-emancipation,
where there aren't leaders who tell people what to do but rather people who collectively liberate themselves. I
admire the honesty, clarity and consistency of John's work, which is evident in his presentation today. But I
also have to be honest and say that I find the ideal of changing the world without taking power
ultimately self-refuting.
I agree with John about uncertainty. There are lots of things we cannot know. But one thing I am certain
about. That is that it is impossible to change the world without addressing and solving the
question of political power.
I absolutely sympathise with one of the impulses behind the slogan 'Change the world without taking power'.
Among a lot of the traditions on the left worldwide there has been what has been called
'socialism from above'. Whether it is a Communist party with Stalinist traditions or a social democratic
party like the Workers Party in Brazil today, it involves the idea that the party changes things for
you and everyone else remains passive.
The political tradition I stand in is a very different one. It is that of socialism from below
summed up in Marx's definition of socialism as the self-emancipation of the working class.
Socialism is about the oppressed and exploited of the world effectively liberating
themselves.
My fundamental difference with John is that I believe this process of self-emancipation requires us to
confront and overthrow the existing state and replacing it with a radically different form of
state power.
John invites us essentially to turn our backs on the state. He says that we should carry out
what he calls an 'interstitial' revolution. It's been summed up by other thinkers sharing the
same ideas as John as life despite capitalism. We should all try and cultivate our
autonomous gardens despite the horrors of capitalism.
CSUS Debate Aff K Toolkit
TP
2008 - 2009

Alternative Fails- Total Rejection


Total rejection of capitalism fragments resistance
J.K. Gibson-Graham, feminist economist, 1996, End of Capitalism.

One of our goals as Marxists has been to produce a knowledge of capitalism. Yet as “that which is known,”
Capitalism has become the intimate enemy. We have uncloaked the ideologically-clothed,
obscure monster, but we have installed a naked and visible monster in its place. In return
for our labors of creation, the monster has robbed us of all force. We hear – and find it easy to
believe – that the left is in disarray. Part of what produces the disarray of the left is the vision of what the left
is arrayed against. When capitalism is represented as a unified system coextensive with the
nation or even the world, when it is portrayed as crowding out all other economic forms,
when it is allowed to define entire societies, it becomes something that can only be
defeated and replaced by a mass collective movement (or by a process of systemic dissolution that
such a movement might assist). The revolutionary task of replacing capitalism now seems
outmoded and unrealistic, yet we do not seem to have an alternative conception of class
transformation to take its place. The old political economic “systems” and “structures” that call forth a
vision of revolution as systemic replacement still seem to be dominant in the Marxist political imagination.
The New World Order is often represented as political fragmentation founded upon economic unification. In
this vision the economy appears as the last stronghold of unity and singularity in a world of diversity and
plurality. But why can’t the economy be fragmented too? If we theorized it as fragmented in the United
States, we could being to see a huge state sector (incorporating a variety of forms of appropriation of surplus
labor), a very large sector of self-employed and family-based producers (most noncapitalist), a huge
household sector (again, quite various in terms of forms of exploitation, with some households moving
towards communal or collective appropriation and others operating in a traditional mode in which one adult
appropriates surplus labor from another). None of these things is easy to see. If capitalism takes up the
available social space, there’s no room for anything else. If capitalism cannot coexist,
there’s no possibility of anything else. If capitalism functions as a unity, it cannot be
partially or locally replaced. My intent is to help create the discursive conception under
which socialist or other noncapitalist construction becomes “realistic” present activity
rather than a ludicrous or utopian goal. To achieve this I must smash Capitalism and see it
in a thousand pieces. I must make its unity a fantasy, visible as a denial of diversity and change.
CSUS Debate Aff K ToolKit
TP
2008 - 2009

Aff – Reform Possible/Key


Reform is not only possible, it’s key to avoiding inequality.

Mark Blyth associate Professor Dept. of Political Science Johns Hopkins University and Dr. Jonathan Hopkin
Lecturer in British and Comparative Politics American Political Science Association Sep 02, 200 4
http://www.allacademic.com/meta/p59770index.html 15-16

This paper has attempted to show two things…and tentatively suggest one more. First, and empirically, it
has confirmed the finders of other scholars that that globalization does not necessarily create more
problems for generous welfare states than for more limited welfare states. Instead, we have
argued that significant market-conforming reforms can be undertaken in advanced political
economies without necessarily giving up on equality. Unformatted Document Text: 17 order to
demonstrate this we examined the degree of structural reform, financial liberalization, labor market
flexibility, the regulation of entry, and the degree of legalistic intervention relative to existing levels of
inequality in the set of European advanced industrial states. We found that, in contrast to much of the
conventional wisdom, significant ‘business friendly’ structural (and other) reforms can be
undertaken without surrendering equality. In fact, those states that were most unequal to begin with
were precisely those that became comparatively more unequal under conditions of globalization. This applied
for both LMEs, and what might be termed the Southern European or Mediterranean ‘variety of capitalism’.
In contrast, the ‘most likely victim of globalization’, the Scandinavian welfare state, has shown
itself to be quite adaptive and able to undertake a variety of reforms, that while changing
the form of the model, do not (thus far) significantly alter its content. In sum, while there is
a tendency in the literature to conflate liberalization and welfare state retrenchment, the
two are distinct concepts, and distinct outcomes, which need to be measured separately.
Inequality in not an inevitable price to be paid for growth in the global economy. Second, and
analytically, to the extent that typologies remain useful, we argue that the over picture of welfare state
demography is still better captured by Esping-Andersen’s ‘three worlds of welfare capitalism’, in which
LMEs (liberal welfare states) are contrasted with two kinds of CME: the Scandinavian social democratic
welfare state, and a varient of the continental European Christian democratic welfare state. This outcome
pertains since economic reform, often portrayed as a one-way street towards free market
capitalism, is actually far more compatible with the institutions of the social democratic
welfare state than is generally acknowledged. The Northern European social democracies have
managed to combine market-friendly regulation of key areas of the economy with high
levels of state spending, which permit generous welfare provision and public services 21 .
As we have seen, despite significant reforms being undertaken in such states, the relatively good economic
performance of this group of economies through the difficult period of the 1990s has not been achieved at the
expense of social solidarity. Economic reform does seem to improve economic performance, but
nations can choose whether or not to accompany a liberal market regime with a
decommodificatory welfare state. One choice does not negate the other.
CSUS Debate Aff K ToolKit
TP
2008 - 2009

A2: Captialism Imperialism/War


Profit Motives and Capitalism don’t inherently result in imperialism or war- it’s the
twisted power dynamic of the U.S.
Gregory Bresiger assistant managing editor of Traders Magazine 4/6/2006 http://mises.org/story/2104

Charles Lewis, an official of a leftist think tank who believes the problem is not enough democracy
and too much capitalism, is given lots of time to make a quasi-Leninist case that imperialism is the
highest stage of capitalism. This doctrine among Marxists is seen as incontrovertible, with the Monthly
Review having asserted it yet again in its most recent issue ("In a world where everything has been turned
over to the market, that is, to capital accumulation, the fundamental problems dividing and endangering
human society and the planet are bound to worsen").
This is an idea that was destroyed many years ago by the great economist Joseph Schumpeter in his brilliant
essays "Imperialism" and "Social Classes." Capitalism wants peace, Schumpeter argued. Wherever
capitalism was the purest, wherever laissez-faire reigned, there were considerable peace
parties. But we have come a long way from laissez-faire. The producers of this documentary don't seem to
understand that.
This capitalism-is-imperialism idea is also supported through "Why We Fight" by historian Chalmers
Johnson. His book "Blowback" has documented American interventions around the world. But these
interventions are anything but the fault of unsubsidized elements of the American business
community.
Did the average American businessperson—often struggling to pay the huge costs of empire—
actually want the United States to embark on this path of empire? The producers have no answer
to that question.
Although "Why We Fight" sometimes attacks capitalism, we rarely hear from American
capitalists, who historically have opposed much of the inflation and disasters that have
been the result of the imperial policies of at least the last half-century. Their opposition
was well founded.
War, and its concomitant inflation, is bad for those capitalists who are not on the
government dole, bad for those who are not court intellectuals, bad for anyone who isn't an
enthusiastic part of Leviathan. Indeed, if one goes by the stock market, probably the worst extended
recent period was in the mid 1960s to mid 1970s during the height of the Vietnam War. That's when stocks
went through a very difficult time.
CSUS Debate Aff K ToolKit
TP
2008 - 2009

A2: Capitalism Inequality/Economic Stratification


Inequality is not the fault of capitalism- its Marxist propaganda. Capitalism actually
provides the best environment for equal economic success
Laurence M. Vance adjunct instructor in accounting at Pensacola Junior College, Mises 8/16/2005
http://mises.org/story/1887

The title of the book may initially seem to be an exercise in hyperbole, but such is not the case. How
Capitalism Saved America is indeed the untold history of our country. After a brief
introduction and two very crucial introductory chapters on the nature of capitalism and the perpetrators of
anticapitalism, DiLorenzo takes us through nine chapters of American history — from the Pilgrims to the
recent California energy "crisis" — and shows "how, from the very beginning, capitalism has
been vital to America's growth, and how excessive government interference in the
economy has only exacerbated economic problems and stifled growth." Although the book is
written chronologically, any of these nine chapters can be read independently. However, only one of them is
necessary to see that the book's title is not an overstatement.
Because it was Marx himself who coined the term, it is no surprise that capitalism has been
falsely thought to benefit only capitalists and the rich while exploiting workers and the
poor. DiLorenzo dismisses as Marxist propaganda the idea that capitalism is "a zero-sum
game in which 'somebody wins, somebody loses.'" Instead, "Capitalism succeeds precisely
because free exchange is mutually advantageous." And not only does it succeed, it is "the
source of civilizations and human progress." Capitalism has "brought to the masses
products and services that were once considered luxuries available only to the rich."
Capitalism is not only "the best-known source of upward economic mobility," it "actually
reduces income inequalities within a nation." In short, capitalism alleviates poverty, raises
living standards, expands economic opportunity, and enables scores of millions to live
longer, healthier, and more peaceful lives.
DiLorenzo makes great claims for capitalism. But consider the alternative: "The more regulations, controls,
taxes, government-run industries, protectionism, and other forms of interventionism that exist, the poorer a
country will be." Why then is capitalism blamed for causing monopolies, harming consumers,
endangering workers, damaging the environment, causing instability, exploiting the Third World, breeding
discrimination, and causing war? Why does "a careful review of our nation's history" reveal "a long series of
myths that demonize capitalism"? DiLorenzo believes that because there is "a widespread
misunderstanding of what capitalism is, our leaders — and also much of the general public
— incorrectly blame capitalism for any economic problems we face."
CSUS Debate Aff K ToolKit
TP
2008 - 2009

A2: Capitalism Corporate Abuse/Poverty


Capitalism isn’t the root of abusive corporations. Government regulation has reduced
property rights which are key to free market competition and poverty
Laurence M. Vance adjunct instructor in accounting at Pensacola Junior College, Mises 8/16/2005
http://mises.org/story/1887

The author spares no one in showing the tremendous anticapitalist bias in existence today:
the entertainment industry, universities, intellectuals, private foundations, journalists, radio and television
personalities, columnists, government regulators and agencies, environmentalists, politicians, egalitarians,
ministers, priests, religious leaders. Indeed, even "most businesspeople are not even capitalists."
DiLorenzo indicts large corporations, not because they are large or because they are corporations, but
because "many corporations support interventionist or anticapitalist policies like trade
protectionism or corporate welfare because they hope to benefit from the policies at
everyone else's expense."
Echoing Murray Rothbard, DiLorenzo carefully distinguishes between free-market capitalism
and state capitalism. Free-market capitalism — "true capitalism" — is based on secure
private property rights, the division of labor, social cooperation, freedom of contract,
freedom of association, and voluntary exchange on the free market. It is characterized by
an absence of "excessive government regulation and taxation." DiLorenzo considers private
property "the most important distinguishing feature of capitalism." In a system of private
property, "individuals are free to choose different occupations, consumption patterns, or
lifestyles as long as they don't interfere with the freedom of others to do the same." The
absence of "property rights protections is a major cause of world poverty." Another theme that
appears throughout the book is the crucial role of the entrepreneur in a capitalist society.
CSUS Debate Aff K ToolKit
TP
2008 - 2009

A2: Capitalism Starvation/Poverty


Early American colonies prove capitalism prevents starvation more so than communal
ownership.
Laurence M. Vance adjunct instructor in accounting at Pensacola Junior College, Mises 8/16/2005
http://mises.org/story/1887

True to the book's subtitle, DiLorenzo begins in chapter three with the Pilgrims. Why
did so many of the
early American settlers starve? They adopted communal ownership of land and property.
The absence of property rights — one of DiLorenzo's cornerstones of capitalism —
destroys the work ethic and leads ultimately to starvation. After the securing of private property
rights, the settlers began trading with the Indians and exploiting their comparative advantages. On the eve of
the American Revolution, capitalism had made the American colonists wealthier, taller, and in
better health than their British counterparts. In chapter four, "America's Capitalist Revolt,"
DiLorenzo examines the economic issues behind the Declaration of Independence and the Constitution. He
maintains that one must understand the economic context within which the Declaration of Independence was
written. King George "wanted to forcefully impose British mercantilism on the colonies." Beginning with the
Molasses Act of 1733, DiLorenzo recounts all the egregious attempts of Great Britain to tax and restrict the
trade of the colonists: the Navigation Acts, the Sugar Act, the Stamp Act, the Townsend Acts, and the Tea
Act. He continues with what he considers to be the "safeguards" built into the new constitution to protect
capitalism: the Contract Clause, the Commerce Clause, the Due Process Clause, the Uniform Taxation
Clause, and the General Welfare Clause. However, because of the later abuse of these clauses, DiLorenzo's
elaboration of these "safeguards" may be seen as a little too brief.
CSUS Debate Aff K ToolKit
TP
2008 - 2009

A2: Capitalism Oppresses Working Class/Child Labor


Capitalism doesn’t exploit the working class- it has lead to the introduction of better labor
laws including the need for child labor and sweatshops
Laurence M. Vance adjunct instructor in accounting at Pensacola Junior College, Mises 8/16/2005
http://mises.org/story/1887

The next myth exploded in How Capitalism Saved America is that capitalism exploits the
working class. The author considers this to be one of the most pervasive and pernicious
myths about capitalism. The truth is just the opposite: "Capitalism has continually
improved the lot of the working class." Capitalism results in more leisure time, provides
new, better, and cheaper goods, and increases workplace safety, productivity, and wages.
DiLorenzo defends "child labor" and "sweatshops" with an argument that should be quite
obvious: They were better than the alternatives of malnutrition, starvation, prostitution,
begging, and stealing. It is the increased productivity per adult worker brought about by
capitalism that eliminates the need for child labor-whether in manufacturing or agriculture.
The myth that labor unions are responsible for the long-term rise in wages and living standards in America is
similarly demolished.
CSUS Debate Aff K ToolKit
TP
2008 - 2009

A2: Capitalism caused the great depression/trade wars


Hoover and Roosevelt social policies collapsed our economy causing the trade war with
Japan. IF we stuck to true capitalism the situation would have turned out differently
Laurence M. Vance adjunct instructor in accounting at Pensacola Junior College, Mises 8/16/2005
http://mises.org/story/1887

Chapter nine is devoted to the Great Depression. Here our


author disposes of two myths: that the
Great Depression was caused by the breakdown of capitalism and that government
intervention was the remedy to save it. Using Austrian business cycle theory, and relying on Murray
Rothbard's American's Great Depression, DiLorenzo explains in simple terms what caused the
recession that Herbert Hoover turned into the Great Depression. Hoover was "an FDR-
style interventionist" with "a dangerously poor grasp of economics" who "did just about
everything wrong that could have been done." Hoover's misguided policies and socialistic
programs attacked free-market capitalism and exacerbated the Great Depression
Of special concern to the author is the Smoot-Hawley Tariff — signed into law by President Hoover despite
a letter of protest signed by more than a thousand economists. Under what DiLorenzo terms the "Smoot-
Hawley-Hoover tariff": "The average tariff rate soared to 59.1 percent, the highest in American
history." The result of these increased tariffs — the hallmark of Republican economic policy since
Lincoln — was a trade war that dried up the importation of many items completely, brought
about retaliatory tariffs, deepened the Depression, and contributed to the rise of Japan's
militaristic nationalism.
An even bigger myth than the cause of and remedy for the Great Depression is that FDR's
economic policies got us out of the Great Depression. DiLorenzo calls this the "biggest
economic myth of the twentieth century," and appropriately makes "How the New Deal
Crippled Capitalism" his longest chapter. Roosevelt's grasp of economics was even less than
that of Hoover. Because he believed that low prices caused the Depression, FDR paid
farmers to burn their crops and slaughter their livestock. This was followed by paying
farmers not to grow crops or raise livestock. Roosevelt's alphabet soup of new federal programs
and agencies (NRA, NIRA, AAA, CCC, WPA, TVA, Social Security, Fair Labor Standards Act, National
Labor Relations Act, etc.) destroyed jobs and hindered recovery.
FDR's "cure" for the economy was government spending on make-work projects; his
"cure" for unemployment was to conscript millions of men and send them to an overseas
war. DiLorenzo concludes that, "in reality, FDR's economic policies made the Great Depression
much worse; caused it to last much longer than it otherwise would have; and established
interventionist precedents that have been a drag on economic prosperity and a threat to
liberty to this day."
CSUS Debate Aff K ToolKit
TP
2008 - 2009

at capitalism K: cede the political

Capitalism is utterly inevitable—the left only looks crazy when they focus on Marxism over practical reforms.

Wilson, 2000 – Author of many books including ‘The Myth of Political Correctness’ – 2000 (John K. Wilson, “How the Left can
Win Arguments and Influence People” p. 7- 10)

Socialism is dead. Kaput. Stick a fork in Lenin's corpse. Take the Fidel posters off the wall. Welcome to the twenty-first century.
Wake up and smell the capitalism. I have no particular hostility to socialism. But nothing can kill a good idea in America so
quickly as sticking the "socialist" label on it. The reality in America is that socialism is about as successful as Marxist footwear
(and have you ever seen a sickle and hammer on anybody's shoes?). Allow your position to be defined as socialist even if it isn't
(remember Clinton's capitalist health care plan?), and the idea is doomed. Instead of fighting to repair the tattered remnants of socialism as a marketing slogan, the
left needs to address the core issues of social justice. You can form the word socialist from the letters in social justice, but it sounds better if you don't. At least 90 percent of America opposes socialism, and 90
percent of America thinks "social justice" might be a good idea. Why alienate so many people with a word? Even the true believers hawking copies of the Revolutionary Socialist Worker must realize by now that the
word socialist doesn't have a lot of drawing power. In the movie Bulworth, Warren Beatty declares: "Let me hear that dirty word: socialism!" Socialism isn't really a dirty word, however; if it were, socialism might have
a little underground appeal as a forbidden topic. Instead, socialism is a forgotten word, part of an archaic vocabulary and a dead language that is no longer spoken in America. Even Michael Harrington, the founder of
. The best reason for the left to abandon socialism is
the Democratic Socialists of America (DSA), didn't use the word socialism in his influential book on poverty, The Other America
not PR but honesty. Most of the self-described "socialists" remaining in America don't qualify as real socialists in any technical
sense. If you look at the DSA (whose prominent members include Harvard professor Cornel West and former Time columnist
Barbara Ehrenreich), most of the policies they urge-a living wage, universal health care, environmental protection, reduced
spending on the Pentagon, and an end to corporate welfare-have nothing to do with socialism in the specific sense of
government ownership of the means of production. Rather, the DSA program is really nothing more than what a liberal political
party ought to push for, if we had one in America. Europeans, to whom the hysteria over socialism must seem rather strange,
would never consider abandoning socialism as a legitimate political ideology. But in America, socialism simply isn't taken
seriously by the mainstream. Therefore, if socialists want to be taken seriously, they need to pursue socialist goals using
nonsocialist rhetoric. Whenever someone tries to attack an idea as "socialist" (or, better yet, "communist"), there's an easy answer: Some people think everything done by a government, from Social
Security to Medicare to public schools to public libraries, is socialism. The rest of us just think it's a good idea. (Whenever possible, throw public libraries into an argument, whether it's about good government
programs or NEA funding. Nobody with any sense is opposed to public libraries. They are by far the most popular government institutions.) If an argument turns into a debate over socialism, simply define socialism
as the total government ownership of all factories and natural resources--which, since we don't have it and no one is really arguing for this to happen, makes socialism a rather pointless debate. Of course, socialists
, abstract discussions about socialism are
will always argue among themselves about socialism and continue their internal debates. But when it comes to influencing public policy
worse than useless, for they alienate the progressive potential of the American people. It's only by pursuing specific progressive
policies on nonsocialist terms that socialists have any hope in the long term of convincing the public that socialism isn't (or
shouldn't be) a long-dead ideology.

Apocalyptic predictions about the ills of capitalism will not motivate activism—practical reforms are the only hope for the left.
Wilson, 2000 – Editor and Publisher of Illinois Academe – 2000 (John K. Wilson, “How the Left can Win Arguments and Influence
People” p. 14- 15)

Leftists also need to abandon their tendency to make apocalyptic predictions. It's always tempting to predict that environmental
destruction is imminent or the stock market is ready to crash in the coming second Great Depression. Arguments that the U.S.
economy is in terrible shape fly in the face of reality. It's hard to claim that a middle-class American family with two cars, a big-
screen TV, and a computer is oppressed. While the poor in America fell behind during the Reagan/Gingrich/Clinton era and the
middle class did not receive its share of the wealth produced during this time, the economy itself is in excellent shape. Instead,
the problem is the redistribution of wealth to the very rich under the resurgence of "free market" capitalism. Instead of warning
that the economy will collapse without progressive policies, the left should emphasize that the progressive aspects of American
capitalism have created the current success of the American economy after decades of heavy government investment in human
capital. But the cutbacks in investment for education and the growing disparity between the haves and the have-notes are
threatening the economy’s future success.
CSUS Debate Aff K ToolKit
TP
2008 - 2009

at capitalism K: cede the political

Capitalism is inevitable—reforms, not revolution, are the only option.

Wilson, 2000 – Editor and Publisher of Illinois Academe – 2000 (John K. Wilson, “How the Left can Win Arguments and Influence
People” p. 15- 16)

Capitalism is far too ingrained in American life to eliminate. If you go into the most impoverished areas of America, you will find
that the people who live there are not seeking government control over factories or even more social welfare programs; they're
hoping, usually in vain, for a fair chance to share in the capitalist wealth. The poor do not pray for socialism-they strive to be a
part of the capitalist system. They want jobs, they want to start businesses, and they want to make money and be successful.
What's wrong with America is not capitalism as a system but capitalism as a religion. We worship the accumulation of wealth and
treat the horrible inequality between rich and poor as if it were an act of God. Worst of all, we allow the government to
exacerbate the financial divide by favoring the wealthy: go anywhere in America, and compare a rich suburb with a poor town-the
city services, schools, parks, and practically everything else will be better financed in the place populated by rich people. The aim
is not to overthrow capitalism but to overhaul it. Give it a social-justice tune-up, make it more efficient, get the economic engine
to hit on all cylinders for everybody, and stop putting out so many environmentally hazardous substances. To some people, this
goal means selling out leftist ideals for the sake of capitalism. But the right thrives on having an ineffective opposition. The
Revolutionary Communist Party helps stabilize the "free market" capitalist system by making it seem as if the only alternative to
free-market capitalism is a return to Stalinism. Prospective activists for change are instead channeled into pointless discussions
about the revolutionary potential of the proletariat. Instead of working to persuade people to accept progressive ideas, the far left
talks to itself (which may be a blessing, given the way it communicates) and tries to sell copies of the Socialist Worker to an
uninterested public.

Overthrowing capitalism is a political non-starter—reforms are the only way that the left will be effective.

Wilson, 2000 Editor and Publisher of Illinois Academe of many books including ‘The Myth of Political Correctness’ – 2000 (John
K. Wilson, “How the Left can Win Arguments and Influence People” p. 123)

The left often finds itself stuck in a debate between revolution and reform. To self-described revolutionaries, any attempt to
reform the system is a liberal compromise that only delays the creation of a socialist utopia. The vision of workers casting off
their chains and embracing the overthrow of capitalism is pure fantasy. No one actually knows what it means to overthrow
capitalism, and it clearly isn't going to happen, anyway. Reforming American capitalism is not a halfhearted effort at modest
change; it is a fundamental attack on the reigning ideology of "free market" capitalism. Progressive reforms, taken seriously, are
revolutionary in every important sense. Reforms such as the New Deal were truly revolutionary for their time, and American
capitalism has been saved from its own flaws by these progressive reforms. The problem is that these progressive reforms have
not been carried far enough, in part because the revolutionary left has too often failed to support the progressives’ reformist
agenda. The only leftist revolution in America will come from an accumulation of progressive policies, and so the question of
revolution versus reform is irrelevant.
CSUS Debate Aff K ToolKit
TP
2008 - 2009

at capitalism K: peace

Capitalism promotes democratic peace.

Fukuyama 95 – Senior Social Scientist, Rand Corporation – 1995 (Francis, TRUST, p. 360-1)

The role that a capitalist economy plays in channeling recognition struggles in a peaceful direction, and its consequent
importance to democratic stability, is evident in post-communist Eastern Europe. The totalitarian project envisioned the
destruction of an independent civil society and the creation of a new socialist community centered exclusively around the state.
When the latter, highly artificial community, there were virtually no alternative forms of community beyond those of family and
ethnic group, or else in the delinquent communities constituted by criminal gangs. In the absence of a layer of voluntary
associations, individuals clung to their ascriptive identities all the more fiercely. Ethnicity provided an easy form of community by
which they could avoid feeling atomized, weak, and victimized by the larger historical forces swirling around them. In developed
capitalist societies with strong civil societies, by contrast, the economy itself is the locus of a substantial part of social life. When
one works for Motorola, Siemens, Toyota, or even a small family dry-cleaning business, one is part of a moral network that
absorbs a large part of one’s energies and ambitions. The Eastern European countries that appear to have the greatest chances
for success as democracies are Hungary, Poland and the Czech Republic, which retained nascent civil societies throughout the
communist period and were able to generate capitalist private sectors in relatively short order. There is no lack of divisive ethnic
conflicts in these places, whether over competing Polish and Lithuanian claims to Vilnius or Hungarian irredenta vis-à-vis
neighbors. But they have not flared up into violent conflicts yet because the economy has been sufficiently vigorous to provide an
alternative source of social identity and belonging. The mutual dependence of economy and polity is not limited to democratizing
states in the former communist world. In a way, the loss of social capital in the United States has more immediate consequences
for American democracy than for the American economy. Democratic political institutions no less than businesses depend on
trust for effective operation, and the reduction of trust in a society will require a more intrusive, rule-making government to
regulate social relations.
CSUS Debate Aff K ToolKit
TP
2008 - 2009

at capitalism K: can be liberatory

Capitalism is inevitable and can be turned into a force for liberation as long as progressives focus on practical reforms.

Wilson, 2000 – Editor and Publisher of Illinois Academe – 2000 (John K. Wilson, “How the Left can Win Arguments and Influence
People” p. 12- 14)

Progressive capitalism is not a contradiction in terms, for progressives support capitalism in many ways. Even nonprofit
organizations and cooperatives are not antithetical to capitalism and the market; these groups simply use capitalism for aims
different from the single-minded pursuit of profits. But the rules of supply and demand, the expenses and revenues, the idea of
entrepreneurship and innovation, and the need to adapt to the market are essential. Any progressive magazine or institution that
tries to defy the rules of capitalism won't be around for very long and certainly won’t have the resources to mount a serious
advocacy of progressive ideas. One of the most effective tactics of the environmental movement was encouraging consumers to
consider environmental values when making capitalist choices about what products to buy. Today, a manufacturer who ignores environmental issues puts
its profits at risk because so many people are looking for environmentally friendly products and packaging. Crusades against Coca-Cola for its massive output of non-recycled plastic bottles in America or against
companies supporting foreign dictatorships are part of the continuing battle to force companies to pay attention to consumer demands. Of course, consumer protests and boycotts are only one part of making
"capitalism for everyone." Many progressive groups are now buying stock in companies precisely to raise these issues at stockholder meetings and pressure the companies to adopt environmentally and socially
responsible policies. Unfortunately, the legal system is structured against progressive ideas. In 2000, Ben and Jerry's Ice Cream was forced to sell out to a big corporation that might ignore its commitment to many
. Imagine what would happen if
progressive causes. The company didn't want to sell, but the law demanded that the company's duty to stockholders was to consider only the money involved
our capitalist laws were designed to promote progressive ideas instead of impeding them. Instead of allowing a shareholder
lawsuit against any company acting in a morally, socially, and environmentally conscious way, American laws should encourage
these goals. The claim by some leftists that capitalism is inherently irresponsible or evil doesn't make sense. Capitalism is simply
a system of markets. What makes capitalism so destructive isn't the basic foundation but the institutions that have been created
in the worship of the "free market." Unfortunately, progressives spend most of their time attacking capitalism rather than taking
credit for all the reforms that led to America's economic growth. If Americans were convinced that social programs and
investment in people (rather than corporate welfare and investment in weaponry) helped create the current economic growth,
they would be far more willing to pursue additional progressive policies. Instead, the left allows conservatives to dismiss these
social investments as “too costly” or “big government.”
CSUS Debate Aff K ToolKit
TP
2008 - 2009

at capitalism K: no specifc alt = failure

The massive but failed WTO protests prove that protesting against capitalism without a specific alternative is doomed to failure.

Wilson, 2000 – Editor and Publisher of Illinois Academe – 2000 (John K. Wilson, “How the Left can Win Arguments and Influence
People” p. 110- 113)

Victory isn't easy for the left, even when it wins. One example in which progressives did almost everything right (but nevertheless
was widely attacked) was the 1999 World Trade Organization (WTO) hearings in Seattle. Thanks to the hard work of leftists around the country (and the
world), Seattle was overrun by more than 50,000 protesters who were determined to bring public attention to a powerful, secretive trade group. A huge rally organized by labor groups brought tens of thousands
marching through Seattle, complete with union workers and environmentalists in sea turtle costumes. Thousands of protesters linked arms and prevented the opening session of the WTO from meeting. Most of the
media coverage blamed the protesters for property damage that was planned and caused by anarchists and not stopped by the police. But the protesters did have a powerful effect on the scene, where the bias of the
American media was less important to the delegates, many of whom sympathized with some of the protests. President Clinton, the world's leading trend detector, expressed his support for listening to the peaceful
protesters, showing that he was more alert to the persuasive power of the anti-WTO forces than most of the media. Seattle and Washington left the left with many lessons. The first was never to let the media choose
what the issue would be. Unfortunately, journalists (and their editors) are trained to overlook an important point for the sake of a flashy image and to portray a dramatic confrontation rather than a moral cause. This
doesn't excuse the inaccurate reporting, biased attacks, and unquestioning defense of the authorities that filled most of the front pages and TV news about the WTO and IMF demonstrations. The
progressives failed to spin the issue beyond their simple anti-WTO message. The reasons for opposing the WTO got some
mention, but the idea of an alternative international organization built on genuine "free trade" and the protection of basic human
rights never was aired. The left has become so accustomed to being ignored that progressives have wisely refined the attention-
grabbing techniques of theatrical protest that can convey a simple message. Unfortunately, the left hasn't developed the difficult
techniques of bringing more complex arguments into the public debate, and the result is that progressive views seem shallow
and emotional compared with the more extensive coverage of the ideas of the right and the center in the mainstream media . Still,
Seattle was both a success and an opportunity lost. The left brought attention to an organization without many redeeming values, but it never was able to launch a serious debate about what the alternative global
values should be. Ignoring the massive evidence of police misconduct and brutality, the media served a well-defined role as gatekeepers of the truth. When the media criticized Seattle officials, it was for “permitting”
the peaceful protestors to exercise their right to protest instead of shutting down the city, as happened for the rest of the WTO meetings. Still, the inability of the left to unify their ideas as easily as they unified behind
the physical protest made it possible for many of the media errors to go unchallenged. Imagine if all the groups united behind the WTO protests had planned to meet after the initial melee and formulated a united
response. Imagine if they had declared,

We denounce all violence, whether it is the violence of smashing windows; the violence of shooting tear gas, concussion grenades, pepper spray, and rubber bullets at peaceful protestors; or the violence of regimes
anywhere in the world where political, human, or labor rights are violated and the environment is harmed.

We regret that the police chose to ignore the vandalism on the streets of downtown Seattle and instead attacked nonviolent protestors with tear gas and rubber bullets. As we informed police before the protests
began, a group of violent anarchists had announced their intention to try to disrupt our nonviolent protests and discredit our cause. although many peaceful demonstrators defended Seattle’s stores—some of which
we had previously protested in front of—against property damage and looting, we could no persuade these well-organized anarchists to stop, and we could not persuade the policy shooting tear gas at us to stop the
violence.

We remain united in our belief that the policies of the World Trade Organization are harmful to the people of the world and are designed instead to increase the profits of corporations and the politicians who serve
them. We will return to downtown Seattle to exercise our constitutional rights to assemble peacefully and express our ideas about the WTO.

Saying that the WTO should be abolished is a simply and perhaps desirable goal. But failing to present a comprehensive
alternative to international trade left the protesters open to accusations of being naïve or protectionist. The problem for the left
was that their efforts were so disorganized that no clear alternative emerged. There was no comprehensive solution offered for
the problems posed by the WTO, the World Band, and the IMF. No alternative institutions were proposed to take over the work of
helping the world rather than harming it. Progressives need an international approach to free trade that doesn’t seem like
protectionism. “America First” is not a progressive perspective, and it fails to help the rest of the world. Without a progressive
vision of globalism, the protests against free trade begin to merge with narrow-minded Buchananesque conspiracy theories
about the UN or the WTO taking over the world.
CSUS Debate Aff K ToolKit
TP
2008 - 2009

Depictions of Capitalism as Bad


Their attempt to blame corporate exploitation on capitalism is misinformed and continues
false teachings that result in corporate bureaucrats.
Laurence M. Vance adjunct instructor in accounting at Pensacola Junior College, Mises 8/16/2005
http://mises.org/story/1887

The all-too familiar circle of the government regulating an industry, creating a "crisis," and
then intervening even more to solve the crisis, thus making things worse, is no where more
apparent than DiLorenzo's examples from the energy industry.
The book concludes with a look at "the never-ending war on capitalism" by government
intervention, regulations, agencies, and bureaucrats. DiLorenzo also includes university
professors, politicians, and lawyers in his indictment. "American universities devote an
inordinate amount of time and resources to teach potential business leaders not how to be
capitalists but how to be corporate bureaucrats." Politicians "view businesses as cash cows
to be plundered for the benefit of their own political careers." "Lawyers now have
incentives to spend their lives digging up cases and evidence against corporations because
some consumers stupidly misused their products."
DiLorenzo also briefly reviews three anticapitalist but best-selling books: Eric Schlosser's Fast Food Nation:
The Dark Side of the All-American Meal, Barbara Ehrenreich's Nickel and Dimed: On (Not) Getting By in
America, and Michael Moore's Stupid White Men and Downsize This! He finds that the capitalism
attacked in these books is not capitalism at all, it is socialism, mercantilism, interventionism,
and assorted anticapitalist myths. Although these "reviews" are an added bonus to the book, they
would be even better if they were lengthened and made into a series of appendices.
CSUS Debate Aff K ToolKit
TP
2008 - 2009

Perm Solves/Reform Key (1/2)


Capitalism is not monolithic, but their K makes it so. Instead, we should affirm everyday
experiences of alterity and smaller actions like the plan as a way to create ethical
connections and cultivate subjectivities not already-interpolated by capital.
Po-Keung Hui, assistant professor dept of cultural studies at Lingnan University, Remaking Citizenship in Hong
Kong: Community, nation and the global city, 2004, pg. 217-223

Gibson-Graham and colleagues have engaged in such a project by bringing in the experience of ordinary
people in order to gain “a new positioning in the grammar of economy.” They have worked as a group since
1997 consisting of “members who hoped to become desiring economic subjects of a ‘socialist’ sort” (CEC
2001: 94). They do not see the economy and capitalism as a monotonous entity; for them to call the
economy capitalist is to engage in “categorical violence.” As a result, it is desirable to develop new
languages “to represent noncapitalist forms of economy (including ones we might value and desire) as
existing and emerging, and as possible to create” (ibid.: 95). Their cultivation of alternative economic
subjectivities is realized essentially through creating a new economic language and by rearticulating it with
existing economic processes. For them, many of the diverse everyday activities of ordinary people, such
as community and ecological services, household management, voluntary and religious works, can be
seen as diverse “economic practices” but they are disqualified as “non-economic” by mainstream
economic language and are thus marginalized as secondary or insignificant. In order to reclaim their
centrality in the economy, Gibson-Graham develops a typology that regards these practices as economic
(but not capitalist) activities.3 In other words, what Gibson-Graham advocates is to broaden and to open up the meaning of the
economy, instead of reducing everything into a narrowly defined economistic domain. To cultivate a “desiring economic subject” of a
“socialist sort” requires integrating two apparently contradictory ethical principles. The first is conventionally associated with the
economic domain: being an autonomous self that is independent, free and assertive. The second is considered to fall into the communal
domain: being a communal subject who is caring, willing to share and is concerned with collective welfare . To reconcile these
seemingly contradictory principles, the meanings of “economic” and “community” have to be
reconsidered. On the one hand, the homogenizing and exclusive tendencies that limit or even suppress community members’ freedom and autonomy
have to be avoided, and the meaning of “community” could just as well be understood in terms of difference. On the other hand, to balance the selfish,
indifferent, and atomizing tendencies of individualism, the economic subject can be reconceptualized as mutually respecting and supportive subjects who
are able to maintain feelings of common interest and sympathy but at the same time to keep a critical distance from communal cohesion and domination. In
Hong Kong, as elsewhere, unlike other keywords with contested meanings (perhaps the most notable one is “globalization”), the term community” is rarely
used unfavorably. Rebuilding community is an acceptable political agenda for almost all social forces differentially located along political spectrums, from
This is particularly true in this current recession period in which the
conservatives to liberals to the radicals.
community is increasingly accepted as an alternative to the malfunctioning market economy and the
retreating state. Yet in light of the not-always-positive experiences of various kinds of community projects in the past, it is still worthwhile to swim
against the current in order to rethink the meaning of community before endorsing its liberation potential. What is a “community”? The New Shorter Oxford
English Dictionary defines a community as: I. A body of individuals: 1. The commons as opposed to peers etc.; the common people. 2. An organized
political, municipal, or social body; a body of people living in the same locality; a body of people having religion, profession, etc., in common; a body of
nations unified by common interests. 3. A monastic, socialistic, etc. body of people living together and holding goods in common. II. A quality or state: 4.
The state of being shared or held in common; joint ownership or liability. 5. A common character, an agreement, an identity. 6. Social intercourse;
communion; fellowship, sense of common identity. 7. Commonness, ordinary occurrence. 8. Life in association with others; society; the social state. In
other words, in addition to its connotation of its detachment from the state and its difference from “peers” or those of rank, “community” often connotes
“commonness,~~ “sameness~~ or even “oneness.” From the nineteenth century onwards, “community” has become a term that implies “experiments in an
alternative kind of group-living,” whose constituency is always disadvantaged populations. The term has increasingly detached from national politics and
To many social activists,
official social welfare provision, and come closer to denote “working directly with people” (Williams 1976: 75—76).
the ideal “communal subject” is one who actively shapes his/her own future by engaging in various
communal relationships, promoting shared interests and constructing common identities . Yet in light of
past negative experiences of various kinds of community projects, such as the exclusive tendencies of the
community and its restriction of individual autonomy and freedom, the term “community” has increasingly
been rethought in recent socio-cultural studies. When community is understood as a geographically
bounded locality with the following characteristics: intimacy, immediacy, reciprocity, transparency,
assimilation, shared interests, shared identities and local autonomy, it is often used as a (utopian)
political model that could serve as an alternative to both the atomizing individualism and a panoptical
surveillant state. Yet in a cosmopolitan setting such as contemporary Hong Kong, communities are inevitably border-crossing.
Shared or common interests with a particular group/community are always partial.
(CONTINUED NO TEXT DELETED)
CSUS Debate Aff K ToolKit
TP
2008 - 2009

Perm Solves/Reform Key (2/2)


(CONTINUED NO TEXT DELETED)
Even in a given geographical locality, it is not easy to put different groups of persons together by assigning them a common identity, as
the interests of different ethnic, gender, income and age-groups are very diverse. Elaborating Iris Young’s critical notion of community,
Jeannie Martin (Martin and O’Loughlin 2002) nicely argues that the model of a small neighborhood that celebrates face-to-face relations
is inadequate to mediate among strangers and their unassimilated differences. Moreover, this model of community that privileges
commonness and sameness is blind to adverse political consequences such as exclusiveness and intolerance of difference. Hence, as
Martin argues, broader networks such as administrative, political, economic, cultural ones are crucial to communal projects in complex
societies, for without these networks the democratic and inclusive encountering of strangers will be impossible. That is why Martin
believes that community development should be understood largely as cultural work or cultural mediation that aims at constructively
handling “constellations of meanings, practices, identifications.” What Young and Martin proposed could be framed as the “community
of difference.” As Cameron and Gibson (2001: 17) suggest, “communities of difference” are nothing but “fluid
process [es] of moving between moments of sameness and difference, between being fixed and ‘in
place’ and becoming something new and ‘out of place.”’ This opens up a possi bility, though not easy to
realize, of reconciling the apparent contradiction between communal relationships and
independence/freedom of the individual.
CSUS Debate Aff K ToolKit
TP
2008 - 2009

Perm Solves- A2: State Bad


The perm is the only way to solve- all their reasons why the state prevents solvency justify
the perm. Revolutionary Russia replicated the problems of capitalism because it’s
movement wasn’t strong enough.
John Holloway Ph.D Political Science-University of Edinburgh and Alex Callinicos Ph.D Philosophy
University of Oxford, former Professor of Politics- University of York August 16, 2005
http://www.zmag.org/znet/viewArticle/5616

John said we have a transhistorical conception of the state which separates the state off from capitalist
relations of production. So let me say clearly that the state under which we live is an irredeemably
capitalist state. I don't want to be part of a movement whose aim is to take control of the existing capitalist state.
Nevertheless, this is not the only state that has existed in history. There have been many different forms of state in
the history of class society. What they have all had in common is organised and institutionalised class violence, the violence of
an exploiting minority over an exploited majority.
The question we are addressing now is, 'Can the working class as it organises itself collectively and socially to resist
exploitation by capital turn this situation round?' In other words, can
the working class create its own form of
organised class violence, distinctively working class in the way it is organised but which makes the
struggle against exploitation by capital more effective and also helps the working class build a new
society? As John knows, the answer to that question in the classic tradition of Marxism, in the writing of Marx
and Lenin, is yes. There is the idea of a workers' state, of workers' power, which is a temporary transitional form
through which the working class organises itself to get rid of capitalism and as part of the same
process democratically organises itself to create a new form of society.
I used the L word, I mentioned Lenin, and of course John will say this question was tried and proved to be
definitely wrong during the Russian Revolution of 1917 and particularly with the Stalinist
aftermath.
One of the contributions referred to the great debate between Marx and Bakunin at the time of the First International in the late 19th
century. He said that the experience of Stalinism proved that Bakunin's anti-state position had been proved to be right.
But how did that happen? If the idea is that state thinking was deep in Marx or Lenin's head and that led to
Stalinism, it is simply wrong. Marx said we need a revolution against the state in his critique of Bakunin. It was an idea Lenin
enthusiastically took up during the 1917 revolution.
So how did it happen? John talked about fissures. The
Russian Revolution of 1917 was a fissure. It tore a
great hole in the capitalist system, the biggest fissure so far in world history. But just to break a
hole in capitalism, even a hole as big as Russia, was not enoug h. There was a simple reason. The power of
capital is global and it can concentrate its forces massively to destroy any fissure that threatens it.
That is what they have been trying to do with Chávez in Venezuela. Whatever the problem is with his politics and so on, the US and its
allies have been trying to break the experiment taking place in Venezuela because it threatens to open up a fissure.
The power of capital is so great that usually they can close the fissures. Usually they do so by
overthrowing the revolutionary process and destroying its leaders and activists . There are many examples
of that. In the Russian case there was a particularly horrible way in which capital won, by creating
such pressures as to cause the revolutionary regime to transform itself into a barbarous replica of
the global system.
The reason that happened was not that Marx liked the state, but that there was not a powerful enough global
movement to break the power of capital globally. That doesn't have to be our fate. We are already
in the process collectively of creating the greatest global movement against capitalism in world
history. But we won't do that if we think that simply creating holes, fissures, in the existing system is enough to destroy it.
CSUS Debate Aff K ToolKit
TP
2008 - 2009

Reform Good (1/3)


Their Alt is worthless unless it can be attached to a specific political goal that expands
rights to the disenfranchised or reverses oppression
Stephen Eric Bronner, Professor of Political Science at Rutgers University, 2004, Reclaiming the Enlightenment:
Toward a Politics of Radical Engagement, p. 124-25

But the battles between the supporters of late critical theory and post-modernism and those of
Kierkegaard, Nietzsche, Heidegger, and the rest— when it comes to the subjectivity of the subject—are so
esoterically ferocious precisely because they reflect little more than what Freud termed “the
narcissism of small differences.” Other than for academic pedants, it is immaterial whether
subjectivity is secured through a fleeting moment of aesthetic-philosophic experience in
resisting the “totally administered society,” the experiential moment fueling the “eternal recurrence,” the
“insight into the essence” of reality, or the feeling of angst in the face of death. Whether the
culprit is “herd society” or “mass society” or the inherently mediocre “public” or the “culture
industry” is actually far less relevant than academic philosophers make it out to be. Strange
is how the left critique of Enlightenment, supposedly undertaken from the standpoint of
Enlightenment itself, should wind up harboring affinities with the thinking of right-wing
irrationalists and neo-romantics.24 But stranger still is how, using the original willingness of critical
theory to lump opposites together, it becomes evident that “the battle against positivism was common to all
the various spiritualist currents that put their stamp, positive or negative, on the culture of the time.. and that
criticism of positivism, whether it came from ‘noble’ culture or lower quarters, was always accompanied by
criticism of socialism, democracy, and political radicalism of all varieties.”25 Too much time has been
spent by fashionable philosophy on the evils of neo-positivism and positivism, which is
mistakenly identified with the philosophical spirit of Enlightenment, and too little with
whether an imperiled subjectivity is indeed the central problem of modernity: this has, in my
view, had a disastrous impact on the critical tradition.26 Plato had already recognized that politics
should not attempt to “care for the soul.” But that warning was ignored. Even Herbert Marcuse, whose
radicalism contradicted the politics of his more cautious friends in the Frankfurt School, showed little
awareness of the dangers in privileging subjectivity and the importance of fostering what Karl Popper termed
“the open society.” This, indeed, is where the genuinely new efforts in radical thinking should begin.
Reinvigorating critical theory calls for asserting its public aims, reconsidering its understanding
of subjectivity, and beginning the critique of those romantic and metaphysical preoccupations
that seek to present themselves as political.27 The problem of subjectivity has concretely—
that is to say historically and politically—had far less to do with some utopian transcendence of
the given order, or the potential integration of reforms, 28 than the demand for including the
excluded, extending the rule of law, civil rights, and economic justice to those suffering the
arbitrary exercise of power. The disempowered and the disenfranchised wished not to
cultivate their subjectivity like a hothouse plant, but exercise it in the public realm. They
wished their unheard voices to be heard and their ignored interests to be articulated: the
critique of the culture industry should begin where it contributes to the repression of these
voices, the misrepresentation of their interests, and the vulgarity of the life they live.
CSUS Debate Aff K ToolKit
TP
2008 - 2009

Reform Good (2/3)


Rejecting involvement with political structures discounts past positive advances and makes
reshaping structures like the state impossible
Stephen Eric Bronner, Professor of Political Science at Rutgers University, 2004, Reclaiming the Enlightenment:
Toward a Politics of Radical Engagement, p. x-xii

The preoccupations of the philosophes with social and institutional reform, and what Max Weber termed the
“elective affinity” between their values and progressive agents for change, now seem to receive scant
attention. This is all the more unfortunate since new transnational movements have come into existence,
often confused in terms of how they should respond to “globalization,” along with functioning transnational
political institutions that still suffer from a deficit of loyalty. New communications technologies are
providing new organizational possibilities for political resistance, expanding the range of available
experiences, and opening the way for new understandings of the most diverse cultures. New forms of
solidarity, reflected in the popular concern with “human rights,” have challenged imperialist
wars, outdated cultural norms, and authoritarian politics. The objective conditions for realizing
the unrealized hopes associated with internationalism, liberal democracy, and social justice are already there;
only the ideological willingness to embrace the assumptions underpinning these values is lacking. That is
what provides the Enlightenment with a new salience for our time. Humanity is not in the past, but rather in
the making. Conservatism may have set the agenda since the last quarter of the twentieth
century. But that does not justify the resignation and increasingly debilitating pessimism as-
sociated with so many current forms of “radical” thought. Genuinely progressive changes have
occurred: dictators have fallen and more citizens of the world have been enfranchised; battles for economic
justice have been won; racism and sexism are on the defensive; and there has been poetry— good poetry—
after Auschwitz. Easy to downplay the gains, suggest that they have now been “absorbed”; and embrace a
new version of the old and tired attitude known as “cultural pessimism.” Cynicism always comes cheap. The
real challenge lies in recognizing how the “system,” which was never as “totally administered” as many
would like to think has been changed for the better through social action inspired by
Enlightenment ideals. The closed society has become more open and—against the provincial,
religious, exploitative, and authoritarian sources of opposition—it has the potential of becoming more
open still. Deciding to enter the fray, however, becomes more difficult when relying on
philosophical perspectives that leave their supporters wandering about lost in Hegel’s night
in which all cows are black. It is necessary to distinguish between traditions not by making refer-
ence to metaphysics, but rather by looking at the political and ideological conflicts between
actual movements. Again, the radical democratic and egalitarian aspects of the Enlightenment have been
betrayed often enough. But this recognition presupposes that there was indeed something to betray. Which
promises made by the Enlightenment have been broken becomes apparent not from the
standpoint of “negative dialectics,” communitarian convictions, “pragmatism,” or ethical
relativism, but rather by taking seriously its universal understanding of liberty and
progress.
CSUS Debate Aff K ToolKit
TP
2008 - 2009

Reform Good (3/3)


Without reforming capitalism, their alternative fails
Paul Burrows, author and publisher from the SMAC lecture series New Colonist April 2001
http://www.newcolonist.com/altcap.html

Ithink that if we want to build a popular movement, and create an alternative to capitalism,
we need to start by asking such questions, and by articulating them in a language that’s
real. (Not many people are interested in the subtleties of the “dialectical relationship between base and superstructure.” Get real!)
From an organizing perspective alone, we need to recognize that the language we use, the mannerisms, style, and tone we adopt, is at
least as important as the substance of our message. We need to have a little humility —we
need to be a little less
attached to our conclusions, a little more questioning of our assumptions, a little less quick
with our judgements and dismissals. Instead of saying everyone else isn’t revolutionary
enough (while we sit on our ass waiting for the Revolution; “pure” but alone), we need to
look in the bloody mirror. We need to ask ourselves “What are we really doing to create a
welcoming movement, a culture of resistance; what are we really doing to foster solidarity;
when was the last time I reached out to someone who didn’t already share my politics;
when was the last time I actually had an impact on someone?” Instead of saying “those young anarchists
don’t know how to build institutions” (and then calling them “reformist” or “parochial” or “bourgeois” when they do), the Old Left
needs to recognize that all the same criticisms apply equally to themselves. In addition to saying “talk minus action equals zero,”
younger activists need to simultaneously pay more attention to history, theory, and the experiences of veteran activists. Talk minus
action is zero, but it’s also true that action minus well-thought-out ideas and principles can be less than zero. It can be damaging to
individual people, and it can hinder the growth of a radical movement. Ultimately, we need to be less concerned about the alleged
failings and ignorance of others, and more concerned about our own political relevance. The entire Left, progressive, activist community
(young and old, socialist or not) needs to build or expand upon its own institutions, and more importantly, the alternatives we create
must embody the values we profess to hold. Instead
of saying “Anything short of complete ‘Revolution’
is reformist” (and then going home to watch TV), we need to recognize that no revolution
begins with the overthrow of the State. The dismantling or seizure of the State is usually a reflection of a deep
revolution already occurring at the grassroots, community and workplace level. The Spanish Revolution of 1936-39 didn’t just happen
because the Spanish were more “radical” or “committed” than we are. It was the culmination of almost 70 years of organizing, making
mistakes, building a popular base. Pre-existing structures and worker organizations made possible a workers’ takeover of much of the
Spanish economy (especially in Catalonia). Participation in radical unions, factory committees, and collectives for decades, enabled
Spanish workers to develop knowledge of their enterprises, a sense of their own competence, and gave them direct experience with
collective organizational principles. The struggle of the Spanish anarchists and communists offers many lessons—not the least of which
is that revolution is a long-term agenda. Younger activists especially need to take this seriously, because they tend to think that militancy
alone (regardless of popular support) will bring about a fast demise of capitalism. Unrealistic
expectations are a fast
road to burnout and despair. At the same time, however, observing that the state-capitalist
system is powerful, and believing that revolution is a long-term agenda, is not an excuse to
stuff our nests, or avoid direct action. As Gramsci pointed out we need to maintain an optimism of will, even if we have a
pessimism of mind. In other words, we need to strike a balance between hope and reality—something that is absolutely necessary, if our
efforts are to be sustained beyond youthful idealism into the rest of our lives. We need to think hard about the meaning of solidarity.
Solidarity is NOT about supporting those who share your precise politics. It’s about supporting those who struggle against injustice—
even if their assumptions, methods, politics, and goals differ from our own. Any anarchist who says they won’t support Cuban solidarity
efforts, or could care less about the U.S. embargo, because the Cuban revolution is “Statist” and “authoritarian,” is in my opinion, full of
shit. (But this doesn’t imply that we should turn a blind eye to human rights violations in Cuba, just because they’re relatively non-
existent compared to the rest of Latin America (or Canada for that matter). It doesn’t imply that we should refrain from criticism of
Cuba’s economic system from a socialist and working-class perspective, simply because we’re worried about the declining number of
post-capitalist experiments to support.)
CSUS Debate Aff K ToolKit
TP
2008 - 2009

Working W/n the System Key (1/2)


Using structures of domination is politically necessary if it is the only way to end
oppression
Lawrence Grossburg, University of Illinois, We Gotta Get Outta This Place, 1992, p. 362-364

In their desire to renounce vanguardism, hierarchy and authoritarianism, too many intellectuals have
also renounced the value of intellectual and political authority. This renunciation of authority is predicated
on a theoretical crisis of representation in which the authority of any knowledge is suspect, since all knowledge is historically
determined and implicated in hierarchical relations of power. The political reflection of this suspicion is that structures and hierarchy are
equated with domination. Intellectuals cannot claim to speak the “truth” of the world, and they cannot speak for or in the name of other
people. There are only two strategies available to the critic. First, the ability to describe the reality of people’s experience or position in
the world can be given over entirely to the people who are the subjects of the analysis. They are “allowed” to speak for themselves
within the intellectual’s discourse. The critic merely inscribes the other’s own sense of their place within and relationship to specific
experiences and practices.” Second, the critic analyzes his or her own position self-reflexivly, and its consequences for his or her study
(i.e., my history and position have determined the inevitable failure of my authority) but without privileging that position. In either case,
it goes too far
there is little room for the critic’s own authority. While such a moment of intellectual suspicion is necessary,
when it assumes that all knowledge claims are unjustified and unjustifiable, leaving the
critic to celebrate difference and a radical and pluralist relativism. The fact of contextual determination
does not by itself mean that all knowledge claims are false, nor does it mean that all such claims are equally invalid or useless responses
The fact that specific discourses are articulated into
to a particular context. It need not entail relativism.
relations of power does not mean that these relations are necessary or guaranteed, nor that
all knowledges are equally bad—and to be opposed—for even if they are implicated with particular
structures of power, there as no reason to assume that all structures of power are equally bad.
Such an assumption would entail the futility of political struggle and the end of history. This is the conundrum of the intellectual Left,
for you can’t have knowledge without standards and authority. Similarly, although all structures of commonality, norrnality and the
This
sacred may be suspect, social existence itself is impossible without at least the imagination of such possibilities .
“intellectual’s crisis” of representation becomes particularly dangerous when it is projected
on everyday life and political struggle, when it is mistakenly identified with a very different crisis of authority. In the
post-Vietnam, post-Watergate, post-Three Mile Island, post-Challenger, post-Jimmy Bakker world, many if not all of the traditional
sources of moral, political and even intellectual authority (including those empowered by the postwar consensus) have collapsed or at
least lost a good deal of their aura. There is a deep seated public anxiety that America’s power (moral, political, economic, etc.) is on the
wane and that none of the traditional authorities is capable of protecting Americans from the many forces—natural and social—that
threaten them. Here we must assent to part of the new conservative argument: Structures of ironic cynicism have become increasingly
powerful and do represent a real cultural and political problem. Both ‘crises” involve a struggle to redefine cultural authority. For the
former it is a struggle to reestablish the political possibility of theory. For the latter it involves the need to construct politically effective
authorities, and to relocate the right of intellectuals to claim such authority without reproducing authoritarian relations. The intellectuals’
crisis is a reflexive and rather self-indulgent struggle against a pessimism which they have largely created for themselves. The conflation
of the two glosses over the increasing presence (even as popular figures) of new conservative intellectuals, and the threatening
implications of the power of a popular new conservatism. The new conservative alliance has quite intentionally addressed the crisis of
authority, often blaming it on the Left’s intellectual crisis of representation (e.g., the attacks on ‘political correctness”), as the occasion
Left intellectuals have
for their own efforts to set new authorities in place new positions, new criteria and new statements .
constructed their own irrelevance, not through their “elitist” language, but through their refusal to
find appropriate forms and sites of authority. Authority is not necessarily authoritarian; it need not
claim the privilege of an autonomous, sovereign and unified speaking subject. In the face
of real historical relations of domination and subordination, political intervention seems to
demand, as part of the political responsibility of those empowered to speak, that they speak to—
and sometimes for—others. And sometimes that speech must address questions about the
relative importance of different struggles and the relative value, even the enabling possibilities
of, different structures.
CSUS Debate Aff K Toolkit
TP
2008 - 2009

Working W/n the System Key (2/2)


We must use the institutions that exercise power to change them
Lawrence Grossburg, University of Illinois, We Gotta Get Outta This Place, 1992, p. 391-393

The Left needs institutions which can operate within the systems of governance,
understanding that such institutions are the mediating structures by which power is
actively realized. It is often by directing opposition against specific institutions that power
can be challenged. The Left has assumed from some time now that, since it has so little access
to the apparatuses of agency, its only alternative is to seek a public voice in the media through
tactical protests. The Left does in fact need more visibility, but it also needs greater access to
the entire range of apparatuses of decision making and power. Otherwise, the Left has
nothing but its own self-righteousness. It is not individuals who have produced starvation and
the other social disgraces of our world, although it is individuals who must take
responsibility for eliminating them. But to do so, they must act within organizations, and
within the system of organizations which in fact have the capacity (as well as the moral
responsibility) to fight them. Without such organizations, the only models of political commitment are self-
interest and charity. Charity suggests that we act on behalf of others who cannot act on their own behalf. But we are
all precariously caught in the circuits of global capitalism, and everyone’s position is increasingly precarious and
uncertain. It will not take much to change the position of any individual in the United States, as the experience of
many of the homeless, the elderly and the “fallen” middle class demonstrates. Nor are there any guarantees about the
future of any single nation. We can imagine ourselves involved in a politics where acting for another is always
acting for oneself as well, a politics in which everyone struggles with the resources they have to make their lives
(and the world) better, since the two are so intimately tied together! For example, we need to think of affirmation
action as in everyone’s best interests, because of the possibilities it opens. We need to think with what Axelos has
described as a “planetary thought” which “would be a coherent thought—but not a rationalizing and ‘rationalist’
inflection; it would be a fragmentary thought of the open totality—for what we can grasp are fragments unveiled on
the horizon of the totality. Such a politics will not begin by distinguishing between the local and the global (and
certainly not by valorizing one over the other) for the ways in which the former are incorporated into the latter
preclude the luxury of such choices. Resistance is always a local struggle, even when (as in parts of
the ecology movement) it is imagined to connect into its global structures of articulation:
Think globally, act locally. Opposition is predicated precisely on locating the points of articulation between them,
the points at which the global becomes local, and the local opens up onto the global. Since the meaning of these
terms has to be understood in the context of any particular struggle, one is always acting both globally and locally:
Think globally, act appropriately! Fight locally because that is the scene of action, but aim for the global because
that is the scene of agency. “Local struggles directly target national and international axioms, at the precise point of
their insertion into the field of immanence. This requires the imagination and construction of forms of unity,
commonality and social agency which do not deny differences. Without such commonality, politics is too easily
reduced to a question of individual rights (i.e., in the terms of classical utility theory); difference ends up “trumping”
politics, bringing it to an end. The struggle against the disciplined mobilization of everyday life can only be built on
affective commonalities, a shared “responsible yearning: a yearning out towards something more and something
better than this and this place now.” The Left, after all, is defined by its common commitment to principles of
justice, equality and democracy (although these might conflict) in economic, political and cultural life. It is based on
the hope, perhaps even the illusion, that such things are possible. The construction of an affective
commonality attempts to mobilize people in a common struggle, despite the fact that they
have no common identity or character, recognizing that they are the only force capable of
providing a new historical and oppositional agency. It strives to organize minorities into a
new majority

406/432
CSUS Debate Aff K Toolkit
TP
2008 - 2009

Total Rejection of Capitalism Bad (1/2)


Total rejection of capitalism fragments resistance – the perm solves best
J.K. Gibson-Graham, feminist economist, 1996, End of Capitalism

One of our goals as Marxists has been to produce a knowledge of capitalism. Yet as “that which is known,”
Capitalism has become the intimate enemy. We have uncloaked the ideologically-clothed,
obscure monster, but we have installed a naked and visible monster in its place. In return
for our labors of creation, the monster has robbed us of all force. We hear – and find it easy to
believe – that the left is in disarray. Part of what produces the disarray of the left is the vision of what the left
is arrayed against. When capitalism is represented as a unified system coextensive with the
nation or even the world, when it is portrayed as crowding out all other economic forms,
when it is allowed to define entire societies, it becomes something that can only be
defeated and replaced by a mass collective movement (or by a process of systemic dissolution that
such a movement might assist). The revolutionary task of replacing capitalism now seems
outmoded and unrealistic, yet we do not seem to have an alternative conception of class
transformation to take its place. The old political economic “systems” and “structures” that call forth a
vision of revolution as systemic replacement still seem to be dominant in the Marxist political imagination. The New World Order is
often represented as political fragmentation founded upon economic unification. In this vision the economy appears as the last
stronghold of unity and singularity in a world of diversity and plurality. But why can’t the economy be fragmented too? If we theorized
it as fragmented in the United States, we could being to see a huge state sector (incorporating a variety of forms of appropriation of
surplus labor), a very large sector of self-employed and family-based producers (most noncapitalist), a huge household sector (again,
quite various in terms of forms of exploitation, with some households moving towards communal or collective appropriation and others
If
operating in a traditional mode in which one adult appropriates surplus labor from another). None of these things is easy to see.
capitalism takes up the available social space, there’s no room for anything else. If
capitalism cannot coexist, there’s no possibility of anything else. If capitalism functions as
a unity, it cannot be partially or locally replaced. My intent is to help create the discursive
conception under which socialist or other noncapitalist construction becomes “realistic”
present activity rather than a ludicrous or utopian goal. To achieve this I must smash
Capitalism and see it in a thousand pieces. I must make its unity a fantasy, visible as a denial of
diversity and change.

407/432
CSUS Debate Aff K Toolkit
TP
2008 - 2009

Total Rejection of Capitalism Bad (2/2)


Their depiction of capitalism as everywhere and coopting everything is what makes
capitalism so powerful. This strategy makes any anti-capitalist alternative impossible.
J.K. Gibson-Graham, the pen name of Katherine Gibson, Senior Fellow of Human Geography at Australian
National University, and Julie Graham, professor of Geography at the University of Massachusetts, Amherst, 1996,
The End of Capitalism (As We Knew It), p. 1-3

Understanding capitalism has always been a project of the left, especially within the Marxian
tradition. There, where knowledges of “capitalism” arguably originated, theory is accorded an explicit social role. From
Marx to Lenin to the neo-Marxists of the post-World War II period, theorists have understood their work as
contributing — whether proximately or distantly — to anticapitalist projects of political action. In this
sense economic theory has related to politics as a subordinate and a servant: we understand the world in order to change it. Given the
avowed servitude of left theory to left political action it is ironic (though not surprising) that understandings
and images
of capitalism can quite readily be viewed as contributing to a crisis in left politics . Indeed, and
this is the argument we wish to make in this book, the project of understanding the beast has itself
produced a beast, or even a bestiary; and the process of producing knowledge in service to
politics has estranged rather than united understanding and action. Bringing these together again, or
allowing them to touch in different ways, is one of our motivating aspirations. “Capitalism” occupies a special
and privileged place in the language of social representation. References to “capitalist
society” are a commonplace of left and even mainstream social description, as are references — to the
market, to the global economy, to postindustrial society — in which an unnamed
capitalism is implicitly invoked as the defining and unifying moment of a complex
economic and social formation. Just as the economic system in eastern Europe used confidently to be described as communist or
socialist, so a general confidence in economic classification characterizes representations of an increasingly capitalist world system. But what might be seen
Why might it seem
as the grounds of this confidence, if we put aside notions of “reality” as the authentic origin of its representations ?
problematic to say that the United States is a Christian nation, or a heterosexual one, despite
the widespread belief that Christianity and heterosexuality are dominant or majority
practices in their respective domains, while at the same time it seems legitimate and indeed
“accurate” to say that the US is a capitalist country?1 What is it about the former expressions, and their critical history,
that makes them visible as “regulatory fictions,”2 ways of erasing or obscuring difference, while the latter is seen as accurate representation? Why,
moreover, have embracing and holistic expressions for social structure like patriarchy fallen into relative disuse among feminist theorists (see Pringle 1995;
Barrett and Phillips 1992) while similar conceptions of capitalism as a system or “structure of power” are still prevalent and resilient? These sorts of
this book.3 The End of Capitalism (As
questions, by virtue of their scarcity and scant claims to legitimacy, have provided us a motive for
We Knew It) problematizes “capitalism”. as an economic and social descriptor.4 Scrutinizing
what might be seen as throwaway uses of the term — passing references, for example, to the capitalist system
or to global capitalism — as well as systematic and deliberate attempts to represent capitalism as a central and
organizing feature of modern social experience, the book selectively traces the discursive origins of a
widespread understanding: that capitalism is the hegemonic. or even the only, present form
of economy and that it will continue to be so in the proximate future. It follows from this prevalent though not
ubiquitous view that noncapitalist economic sites, if they exist at all, must inhabit the social margins; and, as a
corollary, that deliberate attempts to develop noncapitalist economic practices and institutions must take place in
the social interstices, in the realm of experiment, or in a visionary space of revolutionary social
replacement. Representations of capitalism are a potent constituent of the anticapitalist
imagination, providing images of what is to be resisted and changed as well as intimations of the strategies,
techniques, and possibilities of changing it. For this reason, depictions of “capitalist hegemony” deserve a

Card continues

408/432
CSUS Debate Aff K Toolkit
TP
2008 - 2009

Card Continued
particularly skeptical reading. For in the vicinity of these representations, the very idea of a
noncapitalist economy takes the shape of an unlikelihood or even an impossibility. It becomes
difficult to entertain a vision of the prevalence and vitality of noncapitalist economic
forms, or of daily or partial replacements of capitalism by noncapitalist economic
practices, or of capitalist retreats and reversals. In this sense, “capitalist hegemony” operates
not only as a constituent of, but also as a brake upon, the anticapitalist imagination.5 What
difference might it make to release that brake and allow an anticapitalist economic imaginary to
develop unrestricted?6 If we were to dissolve the image that looms in the economic
foreground, what shadowy economic forms might come forward? In these questions we can identify the
broad outlines of our project: to discover or create a world of economic difference, and to populate that world with exotic creatures that
become, upon inspection, quite local and familiar (not to mention familiar beings that are not what they seem).

409/432
CSUS Debate Aff K Toolkit
TP
2008 - 2009

Capitalism Inevitable
Other forms of economics have been tried and they have failed- the industrial nature of
modern humanity means that capitalism will triumph everytime
Joseph Stromberg historian in residence at the Mises Institute July 9 2004 http://mises.org/article.aspx?Id=1562

In addition, he may have been the first to anticipate the way in which the
terms left and right would
eventually come to mean their precise opposite in the reforming economies of Eastern Europe.
He was fascinated but not entirely surprised by the events in old Yugoslavia, where a Stalinist system
had been forced to reform into a more market oriented economy. In fact, he noted that the trend had
begun in the 1960s, and extended all over Eastern Europe. What was essentially happening,
Rothbard wrote, was that socialism had been tried and failed and now these countries were
turning to market models. Keep in mind that this was 1973, when hardly anyone else believed these
countries capable of reform: "In Eastern Europe, then, I think that the prospects for the free market are
excellent--I think we’re getting free-market capitalism and that its triumph there is almost
inevitable." Ten years later, it was still fashionable to speak of authoritarian regimes that could
reform, as contrasted with socialist totalitarianism that could not be reform and presumably had to be
obliterated. Rothbard did not believe this, based on both theory and evidence. Rothbard saw that
all sectors in all countries moving either toward capitalism or toward socialism, which is to say,
toward freedom or toward control. In the US, the trends looked very bleak indeed but he found trends to cheer in the antiwar movement,
which he saw as a positive development against military central planning. “Both in Vietnam and in domestic government intervention, each
escalating step only creates more problems which confront the public with tile choice: either, press on further with more interventions, or
repeal them--in Vietnam, withdraw from the country." His conclusion must have sounded impossibly naive in 1973 but today we can see that
he saw further than any other "futurists" of his time: "the advent of industrialism and the
Industrial Revolution has irreversibly changed the prognosis for freedom and statism. In the
pre-industrial era, statism and despotism could peg along indefinitely, content to keep the
peasantry at subsistence levels and to live off their surplus. But industrialism has broken the
old tables; for it has become evident that socialism cannot run an industrial system, and it is
gradually becoming evident that neomercantilism, interventionism, in the long run cannot run
an industrial system either. Free-market capitalism, the victory of social power and the
economic means, is not only the only moral and by far the most productive system; it has
become the only viable system for mankind in the industrial era. Its eventual triumph is
therefore virtually inevitable

410/432
CSUS Debate Aff K Toolkit
TP
2008 - 2009

Capitalism Inevitable

The Left and Right both tug at what cannot be moved- the productive power of the market
versus dead end statist alternatives means capitalism is here to stay
Joseph Stromberg historian in residence at the Mises Institute July 9 2004 http://mises.org/article.aspx?Id=1562

In Rothbard's conception, it is not quite correct to characterize support for free markets as either
right or left. In 1973, he heard as many complaints about the supposed greed unleashed by markets from the
followers of Russell Kirk as he did from the new left socialists. The right, in fact, was afflicted with a serious
intellectual attachment to pre-capitalistic institutional forms of monopoly privilege, militarism, and the
unrelenting drive to war.This was what Rothbard saw the political establishment of 1973 bringing to the US: the
march of the partnership between government and business that is nothing but the reinvention
of political forms that pre-dated the capitalist revolution that began in the Italian city states of
the 16th century. The US conservatives were entirely complicit in this attempt to reverse the
classical liberal revolution in favor of free markets in order to fasten an old-world monopolist
system on society. In this, the conservatives resembled their supposed enemies, the socialists.
After all, socialism was, as Rothbard put it, "essentially a confused, middle-of-the-road
movement." Its supposed goal of liberty, peace, and prosperity was to be achieved through the
imposition of new forms of regimentation, mercantilism, and feudalism. Socialism seeks, in
Rothbard's words, "liberal ends by the use of conservative means." ("Left and Right: The Prospects for
Liberty," Left and Right, I, 1, Spring 1965). Conservatives could be counted on to support the means
but not the ends, and the result is something that approaches the current status quo in the US: a
mixed political system that combines the worst features of egalitarian ideology with corporate militarism— a
system that leaves enough of the private sector unhampered to permit impressive growth and
innovation. It was precisely the productive power of market, as versus the dead-end of statist
methods favored by both left and right, that led Rothbard to see that the gains of capitalism
could not finally be reversed.

411/432
CSUS Debate Aff K Toolkit
TP
2008 - 2009

Capitalism Inevitable- Markets


Capitalism will always exist because markets work- despite governmental tainting, success
it’s inevitable
Joseph Stromberg historian in residence at the Mises Institute July 9 2004 http://mises.org/article.aspx?Id=1562

Rothbard's optimism about the prospects for liberty is legendary but less well understood is the basis for it:
markets work and government do not. Left and right can define terms however much they
want, and they can rant and rave from the point of view of their own ideological
convictions, but what must achieve victory in the end is the remarkable influence of
millions and billions of mutually beneficial exchanges putting relentless pressure on the
designs of central planners to thwart their will. To be optimistic about the prospects for
capitalism requires only that we understand Mises's argument concerning the inability of
socialist means to produce rational outcomes, and to be hopeful about the triumph of choice over
coercion. Keynes notwithstanding, in the long run we are not all dead. Instead, the development of
markets—locally and globally—would, over the long haul, create the conditions for
liberty. Writing in the Southern Economic Journal in 1962, Rothbard expressed a key motif of his historical
vision. Replying to an economist who held, somewhat contradictorily, that advocates of laissez faire wanted
to return to an earlier status quo, the which status quo, however, had never existed, Rothbard wrote: "This
problem can, however, be resolved fairly simply. There was relatively less government intervention in the
nineteenth century than in the other eras—past or present–of human history, and in that sense there was at
least a significant shift in favor of laissez-faire. On the other hand, the shift was never completed, so that a
certain amount of government intervention still remained. Those economists and social
philosophers, few though they may be, who wish to change the present system to one of
laissez-faire, are therefore looking to the future, but as redeeming the partially-fulfilled
promise of the past."."

412/432
CSUS Debate Aff K Toolkit
TP
2008 - 2009

Collapse Inevitable- Credit Crisis


Neoliberalism is unsustainable-it has decreased worldwide growth and is kept afloat by
false US credit. Economic crises will only become more frequent and violent.
Minqi Li teaches political economy at the Department of Political Science of York University, Monthly Review
January 2004 http://www.monthlyreview.org/0104li.htm)

The advocates of neoliberalism promised that the neoliberal “reforms” or “structural


adjustments” would usher in an era of unprecedented economic growth, technological
progress, rising living standards, and material prosperity. In fact, the world economy has slowed
towards stagnation in the neoliberal era. The average annual growth rate of world GDP
declined from 4.9 percent between 1950 and 1973, to 3.0 percent between 1973 and 1992,
and to 2.7 percent between 1990 and 2001. Between 1980 and 1998, half of all the “developing
countries” (including the socalled “transition economies”) suffered from falling real per capita
GDP.3 The global economy has been kept afloat by the debt-financed U.S. economy.
Between 1995 and 2002, the U.S. economy accounted for 96 percent of the cumulative
growth in world GDP.4 The U.S. expansion has been financed by reducing domestic
savings, raising the private sector debts to historically unprecedented levels, and running
large and ever-rising current account deficits. The process is unsustainable. The enormous
imbalances have to be corrected one way or the other. If the United States cannot continue
to generate ever-rising current account deficits and none of the other large economies are
capable of functioning effectively as the autonomous driving force, the neoliberal global
economy will be under powerful downward .

413/432
CSUS Debate Aff K Toolkit
TP
2008 - 2009

Capitalism Inevitable/Good
Capitalism is the only, inevitable, solution
Gary Olson Dean of the College of Arts and Sciences at Illinois State University JAC 15.3, 1995 Proquest
Q. In Cultural Critique and elsewhere you propose that “the main problem in today’s society,” the “problem that overshadows all the
others,” is not the contemporary state but capital. In light of the turn to market economies by China, Russia, Eastern Block nations, and,
now, even Cuba to a certain extent, how do we resist capitalism and its corrosive effect on the social fabric? A.
Impossible. And we have no reason to resist because all these people are looking to capitalism as a solution to
their problems. I was in Petersburg last spring, and it was horrible to see all these people—very nice people—
without work, without money, and they are just waiting for capitalist investment in order to
make things supportable. There is obviously no other solution, except the ridiculous and dangerous
solution proposed by this crazy man, this neo-nazi, Zhirinovsky. Capitalism is the only solution. Obviously, the same is
true in China with a different way to manage the entrance into capitalism. No, no. This system has no competition, and
to resist it is not to make impediments against it, as in the old tradition. No, no. It’s to make the
people able to eat, to work, to sleep, to have a home, and so on. And in these conditions,
real resistance can appear.

414/432
CSUS Debate Aff K Toolkit
TP
2008 - 2009

Capitalism Not Root Cause


Capitalism isn’t the root cause of your impacts- the social environment we exist in is.
Flawed economics exist because of the nature of human behavior
Capitalism’s Gravediggers 22 January 2006 http://www.capitalisms-gravediggers.ws/index.php?
display=human.nature.faq

Think about the concept that “war is a part of human nature.” Does it mean that humans naturally
want to kill and maim people they have never met? That is what war means. It is not reasonable to assign war
to human nature. Humans do not have a biological need to kill other people independent of their
environment. Humans, as a species, do have a biological need to survive. But equating survival and
war is completely unreasonable. In fact, war undermines nature’s efforts for the survival of
the human species. War is unnatural.
By any reasonable definition of human nature, war cannot be considered part of human nature.
But war is a very prominent feature of human history. It could easily be argued that even though
war is not part of human nature, it is certainly a part of human behaviour, to date. Socialists are
not going to challenge that, because it is obviously true. What causes this deviant human
behaviour?
Nobody can talk to the first human who bashed in the skull of another human, to learn why. We can,
however, look at the world around us today, and at recorded history, to try to understand why humans war.
Even if you only consider the currently popular “reasons” for wars, they all relate to the
social environment in which we live. Religion, ethnic hatred, imperialism, oppression, and
economics, are all part of the social environment. Wars are caused by humans reacting to
their social environment. That is behaviour, it is not nature. The social environment is the
cause of war.

415/432
CSUS Debate Aff K Toolkit
TP
2008 - 2009

Capitalism Not Root Cause- No Free Markets


Capitalism isn’t causing destruction and poverty- it’s government involvment in
economics. Free markets would eliminate corruptive capitalism
Ron Paul speech before the US House of Representatives July 9, 2002 http://mises.org/story/2895

It is now commonplace and politically correct to blame what is referred to as the excesses of
capitalism for the economic problems we face, and especially for the Wall Street fraud that dominates the business news.
Politicians are having a field day with demagoguing the issue while, of course, failing to address the fraud and deceit
found in the budgetary shenanigans of the federal government – for which they are directly responsible. Instead, it
gives the Keynesian crowd that run the show a chance to attack free markets and ignore the issue of sound money.
So once again we hear the chant: "Capitalism has failed; we need more government controls over the entire financial
market." No one asks why the billions that have been spent and thousands of pages of regulations that have been written since the last major
attack on capitalism in the 1930s didn't prevent the fraud and deception of Enron, WorldCom, and Global Crossings. That failure
surely couldn't have come from a dearth of regulations. What is distinctively absent is any mention that all financial
bubbles are saturated with excesses in hype, speculation, debt, greed, fraud, gross errors in investment
judgment, carelessness on the part of analysts and investors, huge paper profits, conviction that
a new era economy has arrived and, above all else, pie-in-the-sky expectations . When the bubble is
inflating, there are no complaints. When it bursts, the blame game begins. This is especially true in the age of victimization, and is done on a
grand scale. It quickly becomes a philosophic, partisan, class, generational, and even a racial issue.
While avoiding the real cause, all the finger pointing makes it difficult to resolve the crisis and
further undermines the principles upon which freedom and prosperity rest . Nixon was right – once –
when he declared "We're all Keynesians now." All of Washington is in sync in declaring that too much
capitalism has brought us to where we are today. The only decision now before the central planners in Washington is
whose special interests will continue to benefit from the coming pretense at reform. The various special interests will be lobbying heavily
like the Wall Street investors, the corporations, the military-industrial complex, the banks, the workers, the unions, the farmers, the
politicians, and everybody else.
But what is not discussed is the actual cause and perpetration of the excesses now unraveling at
a frantic pace. This same response occurred in the 1930s in the United States as our policymakers responded to the very similar
excesses that developed and collapsed in 1929. Because of the failure to understand the problem then, the depression was prolonged.
These mistakes allowed our current problems to develop to a much greater degree . Consider the failure
to come to grips with the cause of the 1980s bubble, as Japan's economy continues to linger at no-growth and recession level, with their stock
market at approximately one-fourth of its peak 13 years ago. If we're not careful – and so far we've not been – we will make
the same errors that will prevent the correction needed before economic growth can be resumed.
In the 1930s, it was quite popular to condemn the greed of capitalism , the gold standard, lack of regulation, and a
lack government insurance on bank deposits for the disaster. Businessmen became the scapegoat. Changes were made as a
result, and the welfare/warfare state was institutionalized. Easy credit became the holy grail of monetary policy,
especially under Alan Greenspan, "the ultimate Maestro." Today, despite the presumed protection from these
government programs built into the system, we find ourselves in a bigger mess than ever before .
The bubble is bigger, the boom lasted longer, and the gold price has been deliberately undermined as an economic signal. Monetary inflation
continues at a rate never seen before in a frantic effort to prop up stock prices and continue the housing bubble, while avoiding the
consequences that inevitably come from easy credit. This is all done because we are unwilling to acknowledge
that current policy is only setting the stage for a huge drop in the value of the dollar . Everyone fears it,
but no one wants to deal with it. Ignorance, as well as disapproval for the natural restraints placed on
market excesses that capitalism and sound markets impose, cause our present leaders to reject
capitalism and blame it for all the problems we face. If this fallacy is not corrected and
capitalism is even further undermined, the prosperity that the free market generates will be
destroyed. Corruption and fraud in the accounting practices of many companies are coming to
light. There are those who would have us believe this is an integral part of free-market capitalism. If we did have free-market
capitalism, there would be no guarantees that some fraud wouldn't occur. When it did, it would then be
dealt with by local law-enforcement authority and not by the politicians in Congress, who had their chance to "prevent" such problems but
chose instead to politicize the issue, while using the opportunity to promote more Keynesian useless regulations.

416/432
CSUS Debate Aff K Toolkit
TP
2008 - 2009

Capitalism Not Root Cause/Reformable


Capitalism isn’t the root cause of societal oppression and is reformable
Richard Levin, president of Yale, 1998, The Minnesota Review, 48-49,
http://www.theminnesotareview.org/journal/ns48/levin.htm

As a result of this view of the world, many


people on the far right and far left are single-causers;
they believe not only that everything the demon does has bad effects in our society, but
also that everything bad in our society is caused by this demon. Right-wing extremists hold
feminism or secular humanism or ZOG responsible for drugs, crime, floridation, and the decline of "family
values," and many leftists—including some appearing in mr—claim that capitalism is the cause of
racism and sexism (Cotter 119-21, Lewis 97-98, Young 288-91). This, in turn, leads to the belief
that there's a single cure, and only this one cure, for all these social ills: the complete
extirpation of the demon that causes them and the complete transformation of society. Thus
extremists on both sides tend to be all-or-nothingists, to reject all reforms as "band-aids"
that are doomed to fail since they don't get at the source of our problems and so won't
further this radical transformation (Neilson/Meyerson 45: 268-69). Many are also millenarians who
believe the transformation will be brought about by an apocalyptic clash between the forces of good and evil
ending in the permanent defeat of the demon and the creation of a utopia(for fundamentalists this is a literal
Armageddon and Second Coming, for militias it's RaHoWa (Racial Holy War) or the uprising of true patriots
against our traitorous government foretold in The Turner Diaries with its Hitlerian "final solution," and for
Marxists it's the proletarian revolution that, their anthem tells us, will be "the final conflict." Another
consequence of their polarization is that partisans at both extremes try to eliminate the
intermediate positions between them, often by denying their differences. Neilson and
Meyerson say that "we should see liberalism and conservatism as flipsides" (45: 269) and argue that
Republicans and Democrats are really the same (47: 242), as does Tom Lewis at greater length (89-90).
Similarly, George Wallace, in his racist, third-party campaign, insisted that "there isn't a dime's worth of
difference between them." More sinister is their tendency to "disappear" these intermediate
positions by equating them with the opposite extreme. McCarthy and his followers attacked
Democrats and even liberal Republicans as "pinkos" and "fellow travelers," and Marxist regimes
condemned social democrats and even communists who deviated from the party line as
fascist counterrevolutionaries who must be liquidated. Some extremists on the academic
left employ this tactic against moderates and liberals, although with less lethal results. The
same Marxist critic who called me a "self-confessed liberal" also called me, in another essay published in the
same year, a "reactionary" ("Terminator" 64), and Donald Morton and Mas'ud Zavarzadeh consign Gerald
Graff, Stanley Fish, Richard Rorty, and Andrew Ross to the same camp as Rush Limbaugh (32-33). (Neilson
and Meyerson's attack on Bérubé is more restrained--the worst thing they call him is a "liberal pluralist" [45:
267, 47: 239, 245]; but they try to connect him, as I noted, to support of the far right in Central America.)
Such people need a simplistic division of the political world into two polar opposites with no awkward
alternatives (just as they need a simplistic explanation of the cause and cure of all our problems), because
they can't tolerate complexity or uncertainty. That mental set, I believe, is the most significant similarity (or
"equivalence") between the far right and far left.

417/432
CSUS Debate Aff K Toolkit
TP
2008 - 2009

Capitalism is Reformable
There is no dichotomy between liberalization and the state- we can reform the system of
capitalism
Mark Blyth associate Professor Dept. of Political Science Johns Hopkins University and Dr. Jonathan Hopkin
Lecturer in British and Comparative Politics American Political Science Association Sep 02, 200 4
http://www.allacademic.com/meta/p59770index.html

This dichotomous representation of the state-economy relationship builds on a long tradition, and finds its
most recent expression in the much-cited Hall and Soskice project. However, the political economy and
social policy literatures have also provided more nuanced categorizations of the
institutional arrangements of modern capitalism (Esping-Andersen 1990, Garrett 1998, Kitschelt et
al 1999, Schmidt 2002), and even a range of quite idiosyncratic national models, suggesting a
wide range of possible combinations of economic institutions. However, the problem with more
nuanced approaches is that what is gained in empirical nuance may be lost in analytic leverage. The more
attention is paid to institutional specificities, the more difficult it becomes to establish any
generalizable conclusions about the choices facing advanced political economies. This paper
seeks a middle way between the dichotomous approach and the more nuanced but less
analytic accounts of advanced capitalism. Building on the suggestive empirical findings of Hicks and
Kenworthy (2003), it suggests that advanced industrial nations can be usefully analyzed in
terms of a simple two-dimensional matrix which points to four possible answers to the
‘liberal-social’ dilemma. Advanced industrial economies are assessed in terms of both their welfare
effort (social outlays and income inequality) and their economic liberalism (microeconomic flexibility,
degree of structural reform, and macroeconomic orthodoxy). The data show that there is no simple
trade-off between liberalism and equality, and that different degrees of state intervention in
the productive economy can co-exist with both high and low levels of welfare effort, and
with varying degrees of success in combating inequality. This suggests that advance industrial
democracies are indeed constrained in their ability to fight inequality, but that the implications of
institutional variation for social cohesion may differ from the conventional ‘liberalization
-> inequality’ equation. This point is illustrated by short accounts of the ways in which three different
examples of Western European welfare capitalism have responded to the pressure for
reform.

418/432
CSUS Debate Aff K Toolkit
TP
2008 - 2009

Capitalism is Reformable

Capitalism is reformable
Mark Blyth associate Professor Dept. of Political Science Johns Hopkins University and Dr. Jonathan Hopkin
Lecturer in British and Comparative Politics American Political Science Association Sep 02, 200 4
http://www.allacademic.com/meta/p59770index.html

Second, the picture of wholesale structural reform painted above is in fact much more
complicated than the simple ‘liberalization -> inequality’ equation would allow. Reforms,
as noted above, have certainly occurred. Indeed, in the core areas of welfare transfers, public
pensions have put more emphasis on the link between contributions and benefits; private
spending on pensions is up; hospitals can and have been privatized; a voucher system has been introduced
into the secondary education system, and private school subsidies have been passed by the Riksdag. Yet
having said all this, it is less impressive than it sounds. By 1995 only 1 in 10 local authorities had
implemented an educational voucher system. Meanwhile, private school pupils accounted for only seven
percent of the eligible school population. 12 On pensions and unemployment benefits, while changes were
made to replacement rates, overall “the generosity of Swedish social security was on average the same in
1998 as in 1980.” 13 In fact “the unemployment benefit was [even] more generous [than formerly].” 14
Spending on private health and retirement certainly has increased, as has means tested benefits, which
implies more markets and less equality. However, this too is misleading since the proportion of the
population covered by such benefits has actually increased, in large part due to immigration. As
Lindbom argues, increased expenditure on social assistance is not the result of less
universalism and more liberalism. Rather, it is the opposite case where benefits cover more
people who need more assistance and who were not part of the older, narrower, regime

419/432
CSUS Debate Aff K toolkit
TP
2008 - 2009
Capitalism is Human Nature
Capitalism is Human nature
Nick Davis assistant Professor of English and Gender Studies at Northwestern University Libetarian International 19 89
http://www.libertarian.co.uk/lapubs/polin/polin038.pdf

Individual rights over one’s own mind are therefore derived from man’s nature. Capitalism is the
only socio-economic system which recognises a man’s right over his mind and the products of his
mind. Liberty in both economic and political spheres are therefore corollaries, second consequences, of
man’s nature - of holding man’s life to be an end in itself - the spectacle of a free and proud man
trading by choice with others. Human nature, and thus man, is heroic. I can do no better than summarise in the
words of Ayn Rand: “I am, I think, I will ... What must I say besides? These are the words. This is the answer. I stand
here on the summit of the mountain. I lift my head and I spread my arms. This - my body and spirit - this is the end of
the quest. I wished to know the meaning of things. I am the meaning. I wished to find a warrant for being. I need no
warrrant for being and no sanction on my being. I am the warrant and sanction.”

Deviance is human nature- it prevents us from being naturally socialist.


Capitalism’s Gravediggers 22 January 2006 http://www.capitalisms-gravediggers.ws/index.php?
display=human.nature.faq

It is not reasonable to assign only bad behaviour to human nature unless one takes the position that
any good behaviour is unnatural or deviant. It is somewhat bizarre to believe that desirable
behaviour is deviant. It is not necessarily inconsistent to hold that perspective, but it then seems incumbent
upon those holding that perspective to tell us what causes that deviance. That explanation does not seem
to be forthcoming, so until it is presented, reasonable people should reject the concept that only negative behaviour
should be assigned to our nature. Therefore, we will consider that concept that human nature is negative
enough to prevent a socialist society from functioning satisfactorily. When people speak of human
nature, they are almost always referring to human behaviour. For instance, when socialists mention that war
will not be necessary and therefore will not exist in a socialist society, we are told that is impossible,
because war is part of human nature.

420/432
CSUS Debate Aff K toolkit
TP
2008 - 2009
Cap Good – Financial Independence
Liberalization of markets has opened room for paid jobs for those unable to shoulder financial
responsibilities
Bretton Woods Project 14 June 2000 The World Bank And The State: A Recipe For Change?
http://www.brettonwoodsproject.org/art.shtml?x=16242 pg 15

The liberalisation of markets has also opened up economic opportunities to some groups for whom
they were not previously available. For many women, for instance, long excluded from many paid
jobs and thus economically dependent on husbands or fathers, paid employment (even if poorly paid)
has brought economic and social gains. The opportunity to gain financial independence, albeit
limited and possibly temporary, has helped to change some of the taboos and proscriptions on women’s
behaviour. The growing numbers of women in paid employment in Bangladesh, for instance, have
led to more young unchaperoned women travelling on buses, walking on the streets and going to
cinemas in Dhaka.87For women with children, part-time and home work can be a way of earning
money while still shouldering family responsibilities.

421/432
CSUS Debate Aff K toolkit
TP
2008 - 2009
***Environmental Security K***

422/432
CSUS Debate Aff K toolkit
TP
2008 - 2009
Affirmative Answers

The conflation of security and the environment is key to effective policy-making, the Clinton
Administration proves
Geoffrey Dabelko and PJ Simons, Environment and Security: Core Ideas and US Government
Initiatives, SAIS Review 17.1,
http://muse.jhu.edu.ezproxy.lib.utexas.edu/journals/sais_review/v017/17.1dabelko.html, 1997

Because this conception of environment and security is so broad, US policymakers have been
either unable or unwilling to agree on a cohesive, overarching policy or plan. Many agencies and
departments, however, have used environment and security arguments or terminology to explain
activities or raise the profile of international environmental concerns. For example, the Clinton
Administration's early decision to elevate environmental issues in policymaking led to the creation
of several high-level positions in more traditionally focused bureaucracies. These included the
establishment in 1993 of a directorate and Senior Director post for Global Environmental Affairs at the National
Security Council (NSC), an office of the Deputy Under Secretary of Defense for Environmental Security, an office of
the Under Secretary of State for Global Affairs, and the post of National Intelligence Officer for Global and
Multilateral Issues at the National Intelligence Council. 15
The first formal interagency mechanism of its kind was established in July 1996 through a Memorandum of
Understanding (MOU) on Environmental Security between the Departments of Defense and Energy and the
Environmental Protection Agency. The MOU states that "threats to environmental quality affect broad
national economic and security interests, as well as the health and well-being of individual
citizens." The MOU also pools departmental authorities and resources and proposes a framework
for strengthening the coordination of efforts on a broad range of scientific and technical topics. 16
In addition, the 1996 State Department initiative to integrate environmental concerns throughout
US foreign policy has been justified in part by environment and security arguments, although it
has been framed more broadly in terms of US national interests.

Alt can’t solve – behind closed doors elites like Bush will still speak the language of power you
cant change their minds
John J. Mearsheimer, the big cheese of realism, The Tragedy of Great Power Politics, 2001

Because Americans dislike realpolitik, public discourse about foreign policy in the United States is
usually couched in the language of liberalism. Hence the pronouncements of the policy elites are
heavily flavored with optimism and moralism. American academics are especially good at
promoting liberal thinking in the marketplace of idea. Behind closed doors, however, the elites
who make national security policy speak mostly the language of power not that of principle and
the United States acts in the international system according to the dictates of realist logic. In
essence, a discernible gap separates public rhetoric from the actual conduct of American foreign
policy

423/432
CSUS Debate Aff K toolkit
TP
2008 - 2009
Affirmative Answers
We must embrace environmental pragmatism- any other alternative is doomed to failure and fails
to prevent the impacts of the case
Joel Mintz, Prof. of Law at Nova Southeastern University and Scholar at Center for Progressive
Regulation, Some Thoughts on the Merits of Pragmatism as a Guide to Environmental Protection,
Boston College Environmental Affairs Law Review, 2004

What all of the environmental pragmatist approaches share, however, is a rejection of the view that
“adequate and workable environmental ethics must embrace non-anthropocentrism, holism, moral
monism, and, perhaps, a commitment to some form of intrinsic value.”39 [*PG7] For Kelly Parker, the
principal insight of environmental pragmatism is that “the human sphere is embedded at every
point in the broader natural sphere, that each inevitably affects the other in ways that are often
impossible to predict, and that values emerge in the ongoing transactions between humans and
environments.”40 Parker defines environment as “the field where experience occurs, where my life and the lives of
others arise and take place.”41 He believes that pragmatism commits us to treating all places where experience
unfolds, i.e., all environments, with “equal seriousness.”42 Moreover, under Parker’s pragmatic approach, people
are encouraged to “restructure our social institutions” so that the public is afforded “a real voice in
determining the kinds of environments we inhabit.”43 Like Parker, Sandra B. Rosenthal and Rogene A.
Buckholz also emphasize the organic unity of the individual embedded in his or her environment.44 To them, human
beings are biological creatures, part of, and continuous with, nature.45 In light of this, the
philosophical argument over anthropocentrism is meaningless since no real line may be drawn
between human and environmental well-being.46 Rosenthal and Buckholz see the “systematic focus” of
pragmatism as being on “science as method, or as lived through human activity, on what the
scientist does to gain knowledge.”47 Humans exist in the world as active experimenters who
create knowledge and formulate ethical values by integrating “potentially conflicting values and
viewpoints.”48 Another leading environmental pragmatist, Bryan G. Norton, also advocates a pluralistic
approach.49 In Norton’s opinion: The goal of seeking a unified, monistic theory of environmental ethics
represents a misguided mission, a mission that was formulated under a set of epistemological and
moral assumptions that harks back to Descartes and Newton. . . . The search for a “Holy Grail” of
unified
theory in environmental [*PG8] values has not progressed towards any consensus regarding what
inherent value in nature is, what objects have it, or what it means to have such a value.50 Norton’s
expressed preference is for the integration of multiple values on three “scales” of human concern and
valuation: (1) locally developed values that reflect the preferences of individuals; (2) community
values that protect and contribute to human and ecological communities; and (3) global values,
which express a hope for the long-term survival of our species.

424/432
CSUS Debate Aff K toolkit
TP
2008 - 2009
Affirmative Answers

The plan and the criticism are not mutually exclusive


Joel Mintz, Prof. of Law at Nova Southeastern University and Scholar at Center for Progressive
Regulation, Some Thoughts on the Merits of Pragmatism as a Guide to Environmental Protection,
Boston College Environmental Affairs Law Review, 2004

One particularly provocative aspect of environmental pragmatic thought is its desire for
compatibilism, i.e., a philosophical framework within which competing environmental theories
may be compatible in practice.53 Andrew Light is an advocate for this view.54 Light contrasts the views of
social ecologists and materialists, such as Murray Bookchin and Herbert Marcuse,55 who view environmental
degradation as presupposed by a capitalist economy, and ontologists, including “deep ecologists” like Arne Naess,56
whose focus is on reform of the self, and one’s relationship with the non-human world, as expressed in individual
identity.57 To harmonize these mutually antagonistic schools of environmental thought, Light
proposes a pragmatic “principle of tol[*PG9]erance.”58 Under it, theorists and practitioners are
required to communicate a “straightforward public position” that endorses the considerations on
which they agree, and the practices best suited to meeting their mutually desired goals, while
leaving some questions that divide them to private dispute.59 As Light sees it, “environmental
pragmatists are not wedded to any particular theoretical framework from which to evaluate
specific problems, but [they] can choose the avenue which best protects the long-term health and
stability of the environment, regardless of its theoretical origin.”60 For Light and other environmental
pragmatists, the “truth” of various environmental theories is thus not always important in
environmental practice.61 Instead, “the appropriateness of any one theory in a particular case is
contingent on historical, cultural, social and resource conditions.”62

425/432
CSUS Debate Aff K toolkit
TP
2008 - 2009
Affirmative Answers

State behavior is driven by security competition according to the dictates of offensive realism.
Critical theory does not have the ability to unseat realism as the dominant discourse of IR your
impact is inevitable
John J. Mearsheimer, the big cheese of realism, International Security, Vol. 20, No. 1, 1995

Realists believe that state behavior is largely shaped by the material structure of the international
system. The distribution of material capabilities among states is the key factor for understanding
world politics. For realists, some level of security competition among great powers is inevitable
because of the material structure of the international system. Individuals are free to adopt non-
realist discourses, but in the final analysis, the system forces states to behave according to the
dictates of realism, or risk destruction. Critical theorists, on the other hand, focus on the social structure of the international
system. They believe that "world politics is socially constructed," which is another way of saying that shared discourse, or how communities of
individuals think and talk about the world, largely shapes the world. Wendt recognizes that "material resources like gold and tanks exist," but he
argues that "such capabilities . . . only acquire meaning for human action through the structure of shared knowledge in which they are embedded."
Significantly for critical theorists, discourse can change, which means that realism is not forever, and that therefore it might be possible to move
beyond realism to a world where institutionalized norms cause states to behave in more communitarian and peaceful ways. The most revealing
aspect of Wendt's discussion is that he did not respond to the two main charges leveled against critical theory in "False Promise." The
first
problem with critical theory is that although the theory is deeply concerned with radically
changing state behavior, it says little about how change comes about. The theory does not tell us
why particular discourses become dominant, and others fall by the wayside. Specifically, Wendt
does not explain why realism has been the dominant discourse in world politics for well over a
thousand years, although I explicitly raised this question in "False Promise" (p. 42). Moreover, he
sheds no light on why the time is ripe for unseating realism, nor on why realism is likely to be
replaced by a more peaceful, communitarian discourse, although I explicitly raised both questions. Wendt's failure to
answer these questions has important ramifications for his own arguments. For example, he maintains that if it is possible to change international
political discourse and alter state behavior, "then it is irresponsible to pursue policies that perpetuate destructive old orders [i.e., realism], especially
if we care about the well-being of future generations." The clear implication here is that realists like me are irresponsible and do not care much
about the welfare of future generations. However, even if we change discourses and move beyond realism, a fundamental problem with Wendt's
argument remains: because his theory cannot predict the future, he cannot know whether the discourse
that ultimately replaces realism will be more benign than realism. He has no wayof knowing
whether a fascistic discourse more
violent than realism will emerge as the hegemonic discourse. For example, he obviously would like another
Gorbachev to come to power in Russia, but he cannot be sure we will not get a Zhirinovsky instead. So even from a critical theory
perspective, defending realism might very well be the more responsible policy choice. The second
major problem with critical theory is that its proponents have offered little empirical support for
their theory. For example, I noted in "False Promise" that critical theorists concede that realism has been the dominant discourse in
international politics from about 1300 to 1989, a remarkably long period of time. Wendt does not challenge this description of the historical record
by pointing to alternative discourses that influenced state behavior during this period. In fact, Wendt's discussion of history is obscure. I also noted
in "False Promise" that although critical theorists largely concede the past to realism, many believe that the end of the Cold War presents an
excellent opportunity to replace realism as the hegemonic discourse, and thus fundamentally change state behavior. I directly challenged this
assertion in my article, but Wendt responds with only a few vague words about this issue. Wendt writes in his response that "if critical theories fail,
this will be because they do not explain how the world works, not because of their values." I agree completely, but critical
theorists have
yet to provide evidence that their theory can explain very much. In fact, the distinguishing feature
of the critical theory literature, Wendt's work included, is its lack of empirical content. Possibly that
situation will change over time, but until it does, critical theory will not topple realism from its
commanding position in the international relations literature.

426/432
CSUS Debate Aff K toolkit
TP
2008 - 2009
Affirmative Answers

No form of knowledge can serve as the ultimate arbiter for thought and action, we must move between
the alternative and the plan action to prevent ourselves from being subverted by the will to order
Roland Bleiker, co-director of the Rotary Centre for International Studies at the University of
Queensland, Alternatives: Social Transformation and Humane Governance, vol. 23 issue 4, 1998

In the absence of authentic knowledge, the formulation of theoretical positions and practical action requires modesty.
Accepting difference and facilitating dialogue becomes more important than searching for the elusive Truth.
But dialogue is a process, an ideal, not an end point. Often there is no common discursive ground, no language that
can establish a link between the inside and the outside. The link has to be searched first.
But the celebration of difference is a process an ideal, not an end point. A call for tolerance and inclusion cannot be
void of power. Every social order, even the ones that are based on the acceptance of difference, excludes what does
not fit into their view of the world. Every form of thinking, some international theorists recognize, expresses a will to
power, a will that cannot but “privilege, oppress, and create in some manner.” There is no all-encompassing gaze.
Every process of revealing is at the same time a process of concealing. By opening up a particular perspective, no
matter how insightful it is, one conceals everything that is invisible from this vantage point. The enframing that occurs
by such processes of revealing, Martin Heidegger argues, runs the risk of making us forget that enframing is a claim, a
disciplinary act that “banishes man into that kind of revealing that is an ordering.” And where this ordering holds
sway, Heidegger continues, “it drives out every other possibility for revealing.” This is why one must move back and
forth between different, sometimes incommensurable forms of insight. Such an approach recognizes that the key to
circumventing the ordering mechanisms of revealing is to think in circles—not to rest too long at one point, but to pay
at least as much attention to linkages between than to contents of mental resting places.
Inclusiveness does not lie in the search for a utopian, all-encompassing worldview, but in the acceptance of the will to
power—in the recognition that we need to evaluate and judge, but that no form of knowledge can serve as the ultimate
arbiter for thought and action. As a critical practice, postmodernism must deal with its own will to power and to
subvert that of others. This is not to avoid accountability, but to take on responsibility in the form of bringing modesty
to a majority.

The alternative leads to worse harms than in the status quo,


Trombetta, No Date Given [Maria Julia, “The Securitization of the Environment and the Transformation of Security”,
http://archive.sgir.eu/uploads/Trombetta-the_securitization_of_the_environment_and_the_transformation_of_security.pdf]
Nevertheless, there are two major problems behind this suggestion. First, if securitization is normatively problematic,
desecuritization can be even more problematic. It can lead to a depoliticization and marginalisation of urgent and serious
issues, while leaving unchallenged the practices associated with security. In the case of the environment, many appeals to security are aimed at both
soliciting action and transforming what counts as security and the way of providing it. Second, within the School’s framework, desecuritization cannot be
possible. Securitization in fact can be inescapable, the unwanted result of discussing whether or not the environment is a security issue. As Huysmans has
noticed, the performative, constitutive approach suggested by the speech act theory implies that even talking and researching
about security can contribute to the securitization of an issue, even if that (and the practices associated with it) is not the
desired result. “The normative dilemma thus consists of how to write or speak about security when the security knowledge risks the production of what
one tries to avoid, what one criticizes: that is, the securitization of migration, drugs and so forth.”(Huysmans 2002: 43) When the understanding of security is
the problematic one described by the CopS, research itself can become a danger. This captures a paradox that characterizes the debate about
environmental security. As Jon Barnett has showed in The Meaning of Environmental Security (2001) the securitization of
the environment can have perverse effects and several attempts to transform environmental problems into security issues
have resulted in a spreading of the national security paradigm and the enemy logic, even if the intentions behind them were
different. Barnett has argued that “environmental security is not about the environment, it is about security; as a concept, it is
at its most meaningless and malign”(2001: 83) in this way, he seems to accept the ineluctability of the security mindset or logic evoked by
securitization. However, his suggestion of promoting a “human centered” understanding of security, in which environmental security is not about (national)
security but about people and their needs, within the securitization logic, cannot escape the trap he has described. Why, in fact, should the sort of his claim
be different from that of similar ones?

427/432
CSUS Debate Aff K toolkit
TP
2008 - 2009
Affirmative Answers
Environmental pragmatism solves
Joel Mintz, Prof. of Law at Nova Southeastern University and Scholar at Center for Progressive
Regulation, Some Thoughts on the Merits of Pragmatism as a Guide to Environmental Protection,
Boston College Environmental Affairs Law Review, 2004

Even though it is not a normative “theory for all seasons,” pragmatic thought has much to add to contemporary
discourse regarding environmental laws and policies. Pragmatism’s stress on concrete facts, flexibility,
experimentation, and practical, workable solutions to real-world problems, combined with its clear preference for
democratic consensus-building and social justice, appears to provide a sensible intellectual framework for innovation
and reform in environmental decision-making at all levels. Undoubtedly, pragmatism lacks universal intellectual
appeal. Some will believe that it is too cautious and modest a theory to be helpful in the rough and tumble of
environmental debate. Others are troubled by its non-dogmatic approach to “truth” and “ethics,” and/or its perceived
insensitivity to the importance of metaphysical issues and grand philosophical conversations. Nonetheless, as Farber’s
Eco-Pragmatism so marvelously illustrates, pragmatism has the potential to furnish a durable and useful set of
intellectual tools for analyzing [*PG26]knotty environmental policy issues. In the hands of a gifted legal scholar—like
Daniel Farber—those tools have already crafted a powerful, balanced, wise, and far-sighted set of proposed
environmental policies. Their potential for further good use, to similar laudable ends, is vast indeed.

Your criticism assumes a stoic interpretation of security. By recognizing the potential for human
security, which the plan does, we are forced to acknowledge our common welfare and work
towards freeing ourselves from fear itself.
Gregory D. Foster, Environmental Security: The Search for Strategic Legitimacy, Armed Forces &
Society, Vol. 27 Issue 3, 2001
In very much the same vein, the United Nations Development Program (UNDP) has invested a great deal of
intellectual capital in propounding the idea of human security, which, "though simple, is likely to revolutionize society
in the 21st century." Human security stands in clear distinction to traditional notions of national and global security,
where the focus was on such things as national interests, the defense of territory from external aggression and. in the
extreme, nuclear holocaust. Human security has two major dimensions: first, safety from such chronic threats as
hunger, disease, and repression: and second, protection from sudden and hurtful disruptions in the patterns of daily
life.
The UNDP identifies seven categories of threats to human security economic, food, health, environmental, personal,
community, and political. The environmental threats countries face today are a combination of the degradation of
local ecosystems and of the global system. While most forms of environmental degradation have their severest impact
locally, other effects migrate beyond national frontiers and thereby represent global challenges to human security.
(n12) By focusing on security at the human level, we are forced to acknowledge two things:
• In its fullest sense, security is not simply about providing for the common defense but also tending to those other
aims enunciated in the Preamble to the U.S. Constitution: forming and preserving a more perfect union, establishing
justice, ensuring domestic tranquility, promoting the general welfare, and securing the blessings of liberty.
• In its most fundamental sense, security means freedom not just from threat and intimidation, harm and danger, but
no less from doubt and fear, need and want.

428/432
CSUS Debate Aff K toolkit
TP
2008 - 2009
Affirmative Answers
Security and safety is a basic human need, thus environmental security is key to human rights,
turning the criticism
Gregory D. Foster, Environmental Security: The Search for Strategic Legitimacy, Armed Forces &
Society, Vol. 27 Issue 3, 2001
Psychologist Abraham Maslow brought us to the realization that security and safety are fundamental human needs,
exceeded in their potency only by the more basic physiological needs for food, water, shelter, and the like. Because
the state of the environment is instrumental in determining whether and how both of these levels of basic human needs
are met, and because such needs translate into human rights, there is a clear link between environmental security and
human rights.
Barbara Rose Johnston makes the persuasive case that "environmental degradation and human rights abuse are
inextricably linked." The right to health, a decent existence, work, and occupational safety and health, she notes, along
with the right to an adequate standard of living, freedom from hunger, an adequate and wholesome diet, and decent
housing; the right to education, culture, equality and nondiscrimination, dignity, and harmonious development of the
personality; the right to security of person and family; the right to peace; and the right to development are all
established by existing United Nations covenants. (Table 1, accompanying, contains passages from selected
documents concerning the environment as a human right.)
Thus: Human rights are abused when political and economic institutions and processes wrest control over traditionally
held resources without negotiation or compensation. Human rights are abused when political and economic
institutions and processes degrade environmental settings, place individuals and populations at risk, withhold
information about that risk, and rationalize selective exposure on the basis of national security, national energy, and
national debt. And, even in the context of strong legal protection for human rights and environmental quality, human
rights are abused when cultural forces and economic greed co-opt and corrupt the implementation of legal structures.
(n13)

Environmental security key to prevent warfare


Gregory D. Foster, Environmental Security: The Search for Strategic Legitimacy, Armed Forces &
Society, Vol. 27 Issue 3, 2001
By implication, then, where the public is safe from environmental dangers, natural resource scarcities and
environmental degradation arc ameliorated, and a healthy environment maintained, social stability is likely to be
promoted and social disorder and conflict prevented. The result is a state of environmental security. Where these
conditions do not exist, the result is environmental insecurity.

429/432
CSUS Debate Aff K toolkit
TP
2008 - 2009
Affirmative Answers

The affirmative’s broader interpretation of security calls into question the very notions of state
power that the negative critiques
Gregory D. Foster, Environmental Security: The Search for Strategic Legitimacy, Armed Forces &
Society, Vol. 27 Issue 3, 2001
Viewing security in this fashion--as, at root, a human state or condition--argues for the recognition that security at the
national or global level is tied to, even a function of, that at the individual level. And identifying the health of the
environment as a human right brings into question the continuing relevance of long-held notions of (state) sovereignty
and territorial integrity; not only do environmental effects readily cross national borders (a form of external
aggression), but the desiderata for responding to the human consequences of such effects could well be seen as
comparable to those for any other form of humanitarian intervention (an increasingly likely and acceptable practice in
the post-Cold War world).
As one scholar has noted perceptively, a broader interpretation of security than the traditional one we are used to
brings into question the optimistic social contract assumption by which state (national) security translates, ipso facto,
into security for all its citizens. This more comprehensive view envisions the provision of broad-based (including
ecological) security to the largest possible component of humanity, not just to the administrative apparatus of the
state; in so doing, it thereby tends to erode state primacy and sovereignty.(n14)

Rejection fails- we cannot desecuritize in the status quo, the best we can do is work with
traditional forms of security rhetoric to better emphasize human rights and needs.
P. H. Liotta, Through the Looking Glass: Creeping Vulnerabilities and the Reordering of Security,
Security Dialogue, Volume 36 Number 1, http://ejournals.ebsco.com.ezproxy.lib.utexas.edu/direct.asp?
ArticleID=441A92DF1585B5B1A0DA, 2005
Although it seems attractive to focus on exclusionary concepts that insist on desecuritization, privileged referent
objects, and the ‘belief’ that threats and vulnerabilities are little more than social constructions (Grayson, 2003), all
these concepts work in theory but fail in practice. While it may be true that national security paradigms can, and likely
will, continue to dominate issues that involve human security vulnerabilities – and even in some instances mistakenly
confuse ‘vulnerabilities’ as ‘threats’ – there are distinct linkages between these security concepts and applications.
With regard to environmental security, for example, Myers (1986: 251) recognized these linkages nearly two decades
ago: National security is not just about fighting forces and weaponry. It relates to watersheds, croplands, forests,
genetic resources, climate and other factors that rarely figure in the minds of military experts and political leaders, but
increasingly deserve, in their collectivity, to rank alongside military approaches as crucial in a nation’s security.

430/432
CSUS Debate Aff K toolkit
TP
2008 - 2009
Affirmative Answers

The affirmative is the only way to solve the very real problems facing humanity
P. H. Liotta, Through the Looking Glass: Creeping Vulnerabilities and the Reordering of Security,
Security Dialogue, Volume 36 Number 1, http://ejournals.ebsco.com.ezproxy.lib.utexas.edu/direct.asp?
ArticleID=441A92DF1585B5B1A0DA, 2005
Recently, there have been a number of voices that have spoken out on what the International Commission on
Intervention and State Sovereignty has termed the ‘responsibility to protect’:10 the responsibility of some agency or
state (whether it be a superpower such as the United States or an institution such as the United Nations) to enforce the
principle of security that sovereign states owe to their citizens. Yet, the creation of a sense of urgency to act – even on
some issues that may not have some impact for years or even decades to come– is perhaps the only appropriate first
response. The real cost of not investing in the right way and early enough in the places where trends and effects are
accelerating in the wrong direction is likely to be decades and decades of economic and political frustration – and,
potentially, military engagement. Rather than justifying intervention (especially military), we ought to be justifying
investment. Simply addressing the immensities of these challenges is not enough. Radical improvements in public
infrastructure and support for better governance, particularly in states and municipalities (especially along the Lagos–
Cairo–Karachi–Jakarta arc), will both improve security and create the conditions for shrinking the gap between
expectations and opportunity.

431/432
CSUS Debate Aff K toolkit
TP
2008 - 2009
Affirmative Answers
Turn- doing nothing increases the likelihood that the US will intervene and take even more
militaristic approaches at securing us from external threats
P. H. Liotta, Through the Looking Glass: Creeping Vulnerabilities and the Reordering of Security,
Security Dialogue, Volume 36 Number 1, http://ejournals.ebsco.com.ezproxy.lib.utexas.edu/direct.asp?
ArticleID=441A92DF1585B5B1A0DA, 2005
Recent history suggests that military intervention as the first line of response to human security conditions
underscores a seriously flawed approach. Moreover, those who advocate that a state’s disconnectedness from
globalization is inversely proportional to the likelihood of military (read: US) intervention fail to recognize unfolding
realities (Barnett, 2003, 2004). Both middle-power and major-power states, as well as the international community,
must increasingly focus on long-term creeping vulnerabilities in order to avoid crisis responses to conditions of
extreme vulnerability. Admittedly, some human security proponents have recently soured on the viability of the
concept in the face of recent ‘either with us or against us’ power politics (Suhrke, 2004). At the same time, and in a bit
more positive light, some have clearly recognized the sheer impossibility of international power politics continuing to
feign indifference in the face of moral categories. As Burgess (2004: 278) notes, ‘for all its evils, one of the promises
of globalization is the unmasking of the intertwined nature of ethics and politics in the complex landscape of social,
economic, political and environmental security’.

A multi-dimensional approach to security that incorporates the immediate accountability of the


affirmative and the thoughtfulness of the negative is crucial
P. H. Liotta, Through the Looking Glass: Creeping Vulnerabilities and the Reordering of Security,
Security Dialogue, Volume 36 Number 1, http://ejournals.ebsco.com.ezproxy.lib.utexas.edu/direct.asp?
ArticleID=441A92DF1585B5B1A0DA, 2005
While it is still not feasible to establish a threshold definition for human security that neatly fits all concerns and
arguments (as suggested by Owen, 2004: 383), it would be a tragic mistake to assume that national, human, and
environmental security are mutually harmonious constructs rather than more often locked in conflictual and contested
opposition with each other. Moreover, aspects of security resident in each concept are indeed themselves embedded
with extraordinary contradictions. Human security, in particular, is not now,
nor should likely ever be, the mirror image of national security. Yet, these contradictions are not the crucial
recognition here. On the contrary, rather than focusing on the security issues themselves, we should be focusing on the
best multi-dimensional approaches to confronting and solving them. One approach, which might avoid the massive
tidal impact of creeping vulnerabilities, is to sharply make a rudder shift from constant crisis intervention toward
strategic planning, strategic investment, and strategic attention. Clearly, the time is now to reorder our entire approach
to how we address – or fail to address – security.

432/432

También podría gustarte